Vous êtes sur la page 1sur 358

Luis Piscoya Hermoza es Doctor en

Educacin y Doctor en Filosofa, grados


acadmicos conferidos por la UNMSM.
Asimismo, es Doctor honoris causa por la
Universidad de Trujillo y Profesor honorario
.
de la Universidad Ricardo Palma En San
Marcos ha ejercido, entre otros, los cargos
,
de Director de la Escuela de Postgrado Jefe
del ex Departamento Acadmico de
Humanidades y Director de la revista Letras.
Actualmente es Profesor Principal en los
programas de Maestra en Epistemologa y
en el Doctorado en Filosofa, donde dirige
seminarios como el de Lgica Matemtica y
Epistemologa de las ciencias naturales y
formales. Profesor investigador visitante del
Instituto de Filosofa, Epistemologa y Lgica
de la Universidad de Ludwig Maximiliam de
Mnich (1999-2000), del Departamento de
Filosofa de la Universidad de Dortmund
(1992), Alemania, y del Departamento de
Filosofa de la Universidad de Castelln
(1997), Espaa, entre otras. Hizo estudios de
,
Postgrado en la Universidad de Kansas en la
Universidad de Stanford, California, y en la
Luis Piscoya Hermoza Universidad de Chile. Entre sus libros
mencionamos: Investigacin cientfica y
educacional, Metapedagoga, Tpicos en
Epistemologa, Lgica general, Filosofa y
Lgica para Educacin Secundaria, Filosofa
para el Bachillerato peruano, Filosofa: gua
del profesor y Perfil de la formacin docente
en el Per. Actualmente es Consultor del
Instituto Internacional para la Educacin
Superior en Amrica Latina y el Caribe de la
Ediciones del Vicerrectorado UNESCO y miembro del Consejo Nacional
Acadmico UNMSM para la Educacin.
Luis Piscoya Hermoza es Doctor en
Educacin y Doctor en Filosofa, grados
acadmicos conferidos por la UNMSM.
Asimismo, es Doctor honoris causa por la
Universidad de Trujillo y Profesor honorario
.
de la Universidad Ricardo Palma En San
Marcos ha ejercido, entre otros, los cargos
,
de Director de la Escuela de Postgrado Jefe
del ex Departamento Acadmico de
Humanidades y Director de la revista Letras.
Actualmente es Profesor Principal en los
programas de Maestra en Epistemologa y
en el Doctorado en Filosofa, donde dirige
seminarios como el de Lgica Matemtica y
Epistemologa de las ciencias naturales y
formales. Profesor investigador visitante del
Instituto de Filosofa, Epistemologa y Lgica
de la Universidad de Ludwig Maximiliam de
Mnich (1999-2000), del Departamento de
Filosofa de la Universidad de Dortmund
(1992), Alemania, y del Departamento de
Filosofa de la Universidad de Castelln
(1997), Espaa, entre otras. Hizo estudios de
,
Postgrado en la Universidad de Kansas en la
Universidad de Stanford, California, y en la
Luis Piscoya Hermoza Universidad de Chile. Entre sus libros
mencionamos: Investigacin cientfica y
educacional, Metapedagoga, Tpicos en
Epistemologa, Lgica general, Filosofa y
Lgica para Educacin Secundaria, Filosofa
para el Bachillerato peruano, Filosofa: gua
del profesor y Perfil de la formacin docente
en el Per. Actualmente es Consultor del
Instituto Internacional para la Educacin
Superior en Amrica Latina y el Caribe de la
Ediciones del Vicerrectorado UNESCO y miembro del Consejo Nacional
Acadmico UNMSM para la Educacin.
Luis Piscoya Hermoza

Lgica
general

Ediciones del
Vicerrectorado Acadmico
UNMSM
ISBN: 9972-46-374-7
Hecho el Depsito Legal en la
Biblioteca Nacional del Per N.: 2007-13522

Primera edicin: Lima, 1997


Segunda edicin: Lima, 2001
Tercera edicin: Lima, diciembre de 2007

Luis Piscoya Hermoza


Fondo Editorial de la UNMSM

Tiraje: 500 ejemplares

La universidad es lo que publica

Cartula: Derechos reservados

C e n tr o de P r o d u c c i n F o n d o E d it o r ia l

U n iv e r s id a d N a c i o n a l M a y o r d e S a n M a r c o s

Calle Germn Amzaga s /n Pabelln de la Biblioteca Central -


4." piso - Ciudad Universitaria,
Lima-Per
Correo electrnico: fondoedit@unmsm.edu.pe
Pgina web: http://www.unmsm.edu.pe/fondoeditorial/

Director / Jos Carlos Bailn Vargas

P r o d u c c i n

Editor / Odn R. Del Pozo O.


Diagramador / Gino Becerra Flores

V e n t a s y D is t r ib u c i n
Adolfo Franklin Winkelried Salazar
619-7000 (anexo 7530)

A d m in is t r a c i n

Erminia Prez Vsquez


Telefax: 619-7000 (anexo 7529)

D if u s i n

Miriam Castro Castaeda


619-7000 (anexo 7529)

Impreso en Lima-Per
Queda prohibida la reproduccin total o pardal sin
permiso escrito de la casa editora.
Contenido

Reconocimientos 15

Introduccin 17

Metodologa 23

I
PROPOSICIONES

CUESTIONARIO 1: Proposiciones 29
LECCIN 1. Proposiciones 31
1.1. Definicin 31
1.2. No son proposiciones 33
1.3. Proposiciones elpticas y descripciones definidas 34
1.4. Metalenguaje y lenguaje objeto 34

II
EL LENGUAJE DE LA LGICA PROPOSICIONAL

CUESTIONARIO 2: El lenguaje de la lgica proposicional 39


LECCIN 2. El lenguaje de la lgica proposicional 41
2.1. El lenguaje de PM 41
2.2. La conjuncin 42
2.3. Tablas de verdad 44
2.4. Conjuncin lgica vs conjuncin en el lenguaje natural. 47

7
III
D IS Y U N C I N Y N E G A C I N

CUESTIONARIO 3: Disyuncin y negacin 51


LECCIN 3. Disyuncin y Negacin 53
3.1. Las disyunciones inclusiva y exclusiva 53
3.2. La negacin 55
3.3. Negacin de una conjuncin y de una disyuncin 57
3.4. Doble negacin 59

IV
EL CONDICIONAL Y LA IMPLICACIN

CUESTIONARIO 4: El condicional y la implicacin 63


LECCIN 4. El condicional y la implicacin 65
4.1. Delimitacin conceptual 65
4.2. El condicional 66
4.3. Condicional contrafctico 67
4.4. Relacin de atingencia 69
4.5. Condicional vs. Lenguaje Natural 69
4.6. Implicacin 70
4.7. Implicacin estricta 71
4.8. Condicin necesaria vs condicin suficiente 71

V
BICONDICIONAL, FUNCIONES DE VERDAD
Y EQUIVALENCIA

CUESTIONARIO 5: Bicondicional, funciones de verdad y


equivalencia 77
LECCIN 5. Bicondicional, funciones de verdad y
equivalencia 79
5.1. El bicondicional 79
5.2. Las conectivas como funciones de verdad 80
5.3. Dominio y rango 82
5.4. Proposiciones atmicas y moleculares 83

8
5.5. Proposiciones lgicamente equivalentes 84
5.6. Traduccin de la proposicin bicondicional a una
conjuncin de dos condicionales. 85
5.7. Bicondicional y definicin 86

VI
JERARQUA DE LAS FRMULAS DEL LENGUAJE
DE LA LGICA PROPOSICIONAL

CUESTIONARIO 6: Jerarqua de las frmulas del lenguaje


de la lgica proposicional 91
LECCION 6. Jerarqua de las frmulas del lenguaje de
la lgica proposicional 93
6.1. Lenguaje natural versus lenguaje formalizado 93
6.2. Reglas de formacin de frmulas 94
6.3. La jerarqua en el lenguaje PM 95
6.5. Ocurrencias de una variable proposicional 98
6.6. Reglas auxiliares sobre jerarqua 99
6.7. Los puntos como signos de jerarqua 101
6.8. Tabla de verdad de las proposiciones con ms de dos
variables 102

VII
TAUTOLOGA, PRINCIPIOS LGICOS Y VALIDEZ

Cuestionario 7: Tautologas principios lgicos y validez 107


Leccin 7. Tautologas, principios lgicos y validez 111
7.1. Frmulas tautolgicas, consistentes y contradictorias 111
7.2. Frmulas tautolgicas vs. proposiciones tautolgicas 111
7.3. Limitaciones en la transformacin de proposiciones
tautolgicas 113
7.4. Los principios lgicos clsicos 114
7.5. La validez lgica 116
7.6. Tautologas vs. frmula lgicamente vlida 117
7.7. Tautologa vs. contenido informativo 118

9
VIII
F O R M A L IZ A C I N

Cuestionario 8: Formalizacin 123


Leccin 8. Formalizacin 127
8.1. Formalizacin mediante un lenguaje proposicional 127
8.2. Qu significa 'postular la verdad de una proposicin? 129
8.3. Inferencia 130
8.4. Formulacin y anlisis de la validez de argumentos
mediante el lenguaje de la lgica proposicional 131
8.5. Formalizacin y validez de argumentos complejos 134
8.6. Formas conocidas de argumento 136
8.7. Mtodo indirecto 138
8.8. Reglas adicionales de abreviacin 142

IX
DEDUCCIN NATURAL

Cuestionario 9: Deduccin natural 147


Leccin 9. Deduccin Natural 153
9.1. La deduccin de Gentzen 153
9.2. Transferencia de la verdad 153
9,3* Deduccin e implicacin 154
9.4. Esquemas de frmulas 155
9.5. Reglas de deduccin natural para un lenguaje
proposicional 156
9.6. Aplicacin de las reglas RDN 158
9.7. Las RDN no constituyen un algoritmo 160
9.8. Prueba condicional 161
9.9. Demostracin por reduccin al absurdo 163

X
SIMPLIFICACIN DEL LENGUAJE PROPOSICIONAL PM

CUESTIONARIO 10: Simplificacin del lenguaje


proposicional PM 167

10
LECCIN 10. Simplificacin del lenguaje proposicional PM 169
10.1. Las 16 funciones de verdad posibles 169
10.2. Lenguaje de Nicod 170
10.3. Algoritmo de Traduccin de Post 171
10.4. Traduccin algortmica al lenguaje de Nicod 173
10.5. El Ideal de Simplicidad 177
10.6. La Navaja de Occam 178
10.7. Traduccin del sistema Hilbert-Ackerman 180
10.8. Lenguaje de Nicod y Tecnologa 181

XI
BASES LGICAS DE LA INTELIGENCIA ARTIFICIAL

CUESTIONARIO 11: Bases lgicas de la inteligencia artificial 185


LECCIN 11. Bases lgicas de la inteligencia artificial 187
11.1. Sistemas expertos y robots 187
11.2. Hardware y Software 188
11.4. Diseo de circuitos elctricos para computadoras 190
11.5. Circuitos lgicos a compuertas 195
11.6. Circuitos lgicos a compuertas para frmulas negadas 200
11.7. Compuertas NAND y OR 201
11.8. Tablas de verdad vs. tablas aritmticas 202

XII
LGICA CLSICA

CUESTIONARIO 12: Lgica clsica 207


LECCIN 12. Lgica clsica 209
12.1 Criterio de demarcacin 209
12.2 El silogismo clsico 211
12.3 Las cuatro proposiciones predicativo - categricas clsicas 215
12.4 El cuadro de Boecio 216
12.5 Modos y Figuras silogistas 218
12.6 Tipos de generalidad 219
12.7 Los nombres propios 221

11
xm
DIAGRAMAS DE VENN

CUESTIONARIO 13: Los diagramas de Venn 225


LECCIN 13. Los diagramas de Venn 235
13.1 El mtodo de los diagramas de Venn 235
13.2 Aplicacin de los diagramas de Venn a la decisin de
la validez de silogismos. 241
13.3 Silogismo en los que se establecen condiciones necesarias 248
13.4 Inferencias inmediatas 249
13.5 Falacias Lgicas y Retricas 250

XIV
EL LENGUAJE PREDICATIVO

CUESTIONARIO 14: EL LENGUAJE PREDICATIVO PMP 255


LECCIN 14. Un lenguaje predicativo PMP 257
14.1 Predicados lgicos 257
14.2 Proposiciones en el lenguaje PMP 259
14.3 Trminos y frmulas 260
14.4 Cuantifcadores 261
14.5 Frmulas cerradas 262
14.6 Alcance de un cuantificador 263
14.7 Forma normal prenex 264
14.8 Formalizacin del cuadro de Boecio en el lenguaje PMP 266
14.9 Formalizacin de proposiciones con predicados de grado 2 267
14.10 Reglas de equivalencia entre cuantifcadores 268
14.11 Reglas de eliminacin y reintroduccin de
cuantifcadores 269
14.12 Aplicacin de las reglas RDNP a la deduccin
silogstica 272
14.13 Deduccin con predicados relacinales 273
14.14 Mecanismo de refutacin de hiptesis 275

12
XV
L A IN D U C C I N C L S IC A

Cuestionario 15: La induccin clsica 279


LECCION 15. La induccin clsica 281
15.1 La induccin como inferencia amplificadora 282
15.2 El fundamento del silogismo 283
15.3 Las seudoinducciones 284
15.4 El principio de uniformidad de la naturaleza 286
15.5 Premisas mayor de un silogismo inductivo 287
15.6 Leyes de la naturaleza y sistema axiomtico 288
15.7 La ley de la causalidad 289
15.8 Aspectos de la causalidad 290
15.9 Los mtodos de Stuart Mili 292
15.10 La deduccin 299
15.11 Resultados de la induccin clsica 300

XVI
RESEA HISTRICA DE LA LGICA

CUESTIONARIO 16: Resea histrica de la lgica 305


LECCION 16. Resea histrica de la lgica 307
16.1 Aristteles y los orgenes de la lgica 307
16.2 Los precursores de la Lgica matemtica 309
16.3 La lgica matemtica contempornea 310
16.4 La lgica matemtica en Amrica Latina 313
16.5 La lgica matemtica y otras lgicas 313

BIBLIOGRAFIA BSICA 315

Glosario 317

SOLUCIONARIO 329

ACTIVIDAD: MAPAS CONCEPTUALES 354

13
Reconocimientos

Deseo expresar mi gratitud a las siguientes personas: A mi amigo el pro


fesor Isaac Canales Quevedo por haberme invitado a escribir un libro de
Lgica que satisfaga las necesidades deformacin humanstica e ins-
tnimental requeridas por una adecuada educacin universitaria, a la pro
fesara Julia Rubio Caldern que ha coordinado con dedicacin y eficien
cia al equipo de correctores y digitadores de este texto, a la seorita Li
lia Pizarro y al seor Oliver Oscco que han tenido la gentileza de elabo
rar la primera versin del solucionario, al seor Cristbal Surez por
haberme ayudado a un mejor tratamiento pedaggico del texto propo
niendo la redaccin de los objetivos de las lecciones y diseando los ma
pas conceptuales. Al profesor Alberto Vsquez Tasayco que ley la pri
mera versin y sugiri algunos cambios terminolgicos A la seorita
Victoria Santisteban y al seor Femando Varas que han hecho la com
posicin de texto, en diferentes etapas, con la paciencia y el cuidado que
impone la edicin de un libro que contiene muchos signos especiales. A
la seorita Carla Piscoya Salinas que ha corregido, sin concesiones, las
tres primeras pruebas de pgina a las que se ha sometido el cuerpo de
este texto. No me hubiera sido posible elaborar este manual, en la forma
final que tiene, sin el esfuerzo, la inteligencia y la generosidad de este
equipo sanmarquino de trabajo. Sin embargo, debo enfatizar que los erro
res que el lector encuentre son de mi estricta responsabilidad.

Ciudad Universitaria de San Marcos, octubre de 2001.


El autor

[15]
Introduccin

Iniciaremos este texto examinando algunas creencias generaliza


das que usualmente se tiene sobre lo que es o debera ser la lgi
ca. Ellas han merecido nuestra atencin porque son desorienta-
doras aunque, presumiblemente, sean parte de nuestras tradicio
nes culturales.
La lgica es una y slo una
Normalmente se cree que existe una nica manera de pensar
lgicamente que correspondera a la estructura profunda de la
mente, de la razn o del cerebro, segn sea el caso. Esto conduce
a suponer que la Lgica se descubre de manera anloga a como se
habra descubierto la estructura de la clula o del tomo. Por tan
to, la Lgica existira ya hecha en algn lugar y la tarea del profesor
ya sea a travs de las clases o de un libro consistira en ense
arnos a descubrirla. Sin embargo, el desarrollo no slo de la l
gica sino de lo que actualmente se conoce como ciencias cognitivas,
nos conduce a pensar que lo anterior es un error. En efecto, en
sentido estricto no existe, dentro de la comunidad cientfica y
filo s fic a , la L g ic a como una unidad sino un conjunto
diversificado de sistemas lgicos o, en trminos ms descripti
vos, de lenguajes lgicos que no siempre son equivalentes en
tre s. Y es de esta manera, porque todos los investigadores nota
bles lo que han hecho es crear lenguajes lgicos, que pueden ser
utilizados como reglas, para analizar la correccin de los argu
mentos cientficos y de los que se usan en la comunicacin coti

[17]
diana. Por ejemplo, uno es el lenguaje lgico creado por Aristteles,
y otros notablemente distintos entre s los creados por Russell y
Whitehead, por Brouwer y Heyting, y por Vasiliev entre otros.
De lo anterior se deduce que lo que se ensea propiamente en las
universidades y en otros centros educativos no es propiamente la
Lgica sino un determinado lenguaje lgico, presuntamente el ms
difundido o ms usual dentro de nuestros medios acadmicos.
La lgica permite establecer verdades fundamentales
Con frecuencia se acude a la Lgica cuando se quiere probar
que ciertos puntos de partida o afirmaciones bsicas son clara
mente verdaderos a la luz de los hechos o de nuestra experiencia.
Este afn es infructuoso porque los lenguajes lgicos vale de
cir, la lgica no estn diseados para probar la verdad de una
afirmacin bsica que constituye el punto de partida de una
argumentacin cientfica, filosfica o cotidiana. La lgica est
diseada estrictamente para transferir o transmitir la verdad de
unas afirmaciones a otras una vez que sta ya ha sido estableci
da, por medios no lgicos. Haciendo una analoga con el compu
tador, que hoy es parte de la vida diaria, podemos afirmar que la
lgica debe ser entendida como un conjunto de comandos para
transferir la verdad pero no para producirla.
La lgica es difcil a causa de los signos que utiliza
En la medida que los cursos de lgica introducen al estudiante
en el manejo de lo que se denomina un lenguaje formalizado, cu
yos componentes se han tomado bsicamente del lenguaje mate
mtico, se cree usualmente que en el carcter poco natural o
artificial de estos signos radica la dificultad en el aprendizaje
de la lgica. Ello se debera a que se trata de un sistema de signos
muy alejado de la claridad y naturalidad que se experimenta cuan
do se usa el lenguaje cotidiano.
Empero, en lo fundamental el aprendizaje de la lgica con
siste, como siempre que se aprende una ciencia, en el aprendiza
je de un sistema de conceptos y los signos especiales slo cons
tituyen un medio para expresar con precisin tales conceptos.
Por tanto, la dificultad se produce cuando se olvida los conceptos
y se produce una enseanza mecanicista que convierte la clase de

18
lgica en un ejercicio que consiste en transformar unas manchas
de tinta en otras manchas de tinta sin que se comprenda el senti
do del simbolismo y el tipo de problemas que soluciona. Como se
deduce, el mismo riesgo corre la enseanza de la matemtica.
De afirmaciones falsas se deduce lgicamente slo falsedades
Es comn encontrar que personas, incluyendo muchas de las
que han aprobado un curso de lgica, crean que sta garantiza
que si partimos de afirmaciones falsas llegaremos a conclusiones
igualmente falsas. Se puede dar muchos ejemplos para probar lo
contrario, como lo haremos en la seccin correspondiente, pero el
argumento principal para poner en evidencia este error consiste
en precisar que los lenguajes lgicos son diseados estrictamen
te slo para transmitir la verdad y, por tanto, no brindan ningu
na garanta cuando las afirmaciones de partida son falsas. Por
tanto, hacer deducciones desde premisas falsas constituye un
mal uso de un sistema lgico y posibilita obtener cualquier con
clusin, inclusive una verdadera. Lo anterior hace comprensible
que los cientficos slo acudan al uso de un sistema lgico cuando
consideran que cuentan con al menos un punto de partida que
sus investigaciones prueban que es verdadero.
"Conclusiones verdaderas presuponen premisas verdaderas"
Esta es la creencia errnea complementaria o recproca res
pecto de la anterior. Parece tener analoga con la expresin que
sugiere que si los frutos son buenos la planta tambin debe ser
buena. Sin embargo, es importante sealar que la lgica no esta
blece relaciones de causa a efecto o de efecto a causa sino rela
ciones de deduccin, implicacin o de consecuencia que son de
otra naturaleza, pues no est prohibido por las reglas de los sis
temas lgicos en uso que la conclusin sea verdadera y las
premisas, sin embargo, falsas. En breve, la verdad de la conclu
sin no asegura la verdad de las premisas o puntos de partida.
Lo que s es correcto afirmar es que si la conclusin es falsa, en
tonces al menos una de las premisas es falsa. Por ello se ha dicho
que la lgica es la ciencia que transmite la verdad y retrotransmite
la falsedad.

19
"La lgica formal es un saber abstracto e intil"
Existe entre los planificadores y diseadores de los currculos
de los distintos niveles de nuestro sistema educativo, con alguna
frecuencia, la creencia de que la lgica formal es una disciplina
que consiste en la capacidad para manejar frmulas, parecidas a
las de la matemtica, pero que carecen de aplicaciones importan
tes y que no ayudan a mejorar sensiblemente ni el razonamiento
cotidiano ni el razonamiento cientfico. Por el contrario, se cree
que existen recursos retricos que no requieren del usuario el
manejo de frmulas pero que son muy tiles para construir argu
mentos que convencen o persuaden con eficiencia al interlocutor.
Segn los cultores de esta creencia, lo que debera incluirse en los
currculos para mejorar la capacidad argumentativa de los alum
nos es elementos de retrica o alguna forma de lgica intuitiva
que no exija el manejo de frmulas.
Respecto del prejuicio antes descrito podemos afirmar, sin
riesgo de inexactitud, que dentro de la comunidad acadmica in
ternacional todos los sistemas lgicos cientficamente reconoci
dos son estructuras simblicas que estn constituidas por conjun
tos de frmulas sometidas a reglas precisas de transformacin y
deduccin. En breve, dentro del mbito de la ciencia y de la tec
nologa lo nico que existe, desde hace algo ms de un siglo, para
decidir la validez de los argumentos y de las pruebas son los sis
temas de lgica matemtica denominados genricamente lgica
formal.
Asimismo, dichos sistemas, creados inicialmente por George
Boole y desarrollados posteriormente con diversidad, profundi
dad y complejidad crecientes, se han convertido en el sector del
conocimiento terico que ha dado lugar a las ms impresionantes
y eficientes aplicaciones tecnolgicas durante los ltimos sesenta
aos. A ello debe aadirse sus aplicaciones en la matemtica, en
el anlisis, construccin y reconstruccin de teoras cientficas, en
el diseo experimental de simuladores de las funciones del cere
bro y de la mente y en el conocimiento metodolgico, por men
cionar slo algunos ejemplos.
Refirindonos a las aplicaciones tecnolgicas, es suficiente1
destacar que tanto la arquitectura del computador electrnico

20
como los lenguajes de autmata son subproductos de las investi
gaciones en lgica-matemtica realizadas alrededor de 1935 por
A. Church, S. Kleene, A. Turing y C. Shannon, entre otros. Asi
mismo, los circuitos de todo computador electrnico, hasta la fe
cha, estn gobernados por las ecuaciones del lgebra de Boole
que es anterior a 1850. Es innecesario abundar aqu en datos para
mostrar cmo la tecnologa digital, la robtica y la informtica
han transformado, entre otros campos, las comunicaciones, la pro
duccin industrial y la medicina. En relacin con las aplicaciones
al desarrollo mismo de la ciencia, es relevante para los educado
res reparar en el hecho, por citar slo un ejemplo, de que las in
vestigaciones psicolgicas de Piaget en el campo del desarrollo
conceptual humano son prcticamente ininteligibles para un lec
tor que carece de conocimientos de lgica proposicional y de la
estructura algebraica de grupo.
Finalmente, pretender que la lgica puede ser sustituida por
la retrica o por la teora de la argumentacin equivale a perder
de vista el sentido de la filosofa y de la ciencia en su conjunto. El
conocimiento cientfico y filosfico que utiliza la lgica como ins
trumento de anlisis y de prueba no se propone la persuasin o el
convencimiento de un auditorio sino el establecimiento de un sa
ber verdadero. Histricamente, el convencimiento o persuasin
de un auditorio, con mucha frecuencia, no ha requerido de argu
mentos lgicos estrictos sino de imgenes o figuras que interesen
o agraden al interlocutor. Ello permite entender por qu durante
casi quince siglos de nuestra era las mayoras representadas por
los intelectuales oficiales pensaban y sostenan que la tierra era
plana y que no se mova. De esta manera hemos aportado argu
mentos suficientes para permitirnos advertir que lo que se pre
tende como sustituto til de la lgica ha sido histricamente un
obstculo para el desarrollo de la ciencia y para la bsqueda de la
verdad.

21
Metodologa

La forma como est estructurado este texto parte de algunas te


sis metodolgicas que se fundan en lo que consideramos ms pro
ductivo para justificar la enseanza de la lgica en los currculos
universitarios. La estrategia didctica aqu propuesta es conoci
da en los medios pedaggicos como enseanza por solucin de
problemas.

1. La enseanza de la lgica, como disciplina cientfica, debe


tomar en consideracin que los estudiantes ingresan al cur
so con una habilidad intuitiva para resolver correctamente
algunos problemas lgicos. Consecuentemente, el xito de
la enseanza de esta disciplina debe medirse por el incre
mento de la capacidad de los estudiantes para compren
der y solucionar problemas que antes de tomar el curso ex
cedan sus posibilidades.
2. La informacin que se proporciona en el texto de un curso
de lgica debe tener como referencia precisa el tipo de pro
blemas relevantes que permite resolver al usuario. Conse
cuentemente, parece ms productivo que la enseanza est
organizada de tal manera que se parta de los problemas a
la informacin y no viceversa. De esta suerte, el aprendi
zaje se convierte en un proceso de investigacin en el que
el estudiante se enfrenta primero con problemas que debe
examinar con atencin y luego busca la informacin que,
en funcin de sus necesidades, requiere para resolverlos

[23]
correctamente. En breve, consideramos que el orden ms
adecuado es el inverso al usado tradicionalmente en los
manuales que proporcionan primero informacin y luego
ejercicios. Tericamente, si un alumno estuviera en condi
ciones de resolver los problemas propuestos sin leer el tex
to, entonces debera ser exonerado del curso y tomar uno
ms avanzado.
3. Las consideraciones anteriores explican por qu en este tex
to comenzamos las lecciones con un cuestionario de pro
blemas o interrogantes y, luego, proporcionamos la infor
macin necesaria para su solucin. Pretendemos as que
el mtodo de enseanza reproduzca, en lo que es posible,
las inquietudes propias del mtodo de investigacin cien
tfica, cuya dinmica depende de la profundidad e intensi
dad con que el investigador percibe los problemas. *
4. Las fronteras entre lgica y matemtica son tan difusas
como discutibles. Hay especialistas que consideran a la l
gica como parte de la matemtica y otros que sostienen la
tesis inversa. Sin tomar partido en esta discusin, en este
manual no se abordar elementos de teora de las clases o
de los conjuntos porque actualmente ya se cuenta con tex
tos especializados en lengua castellana. Asimismo, la
formalizacin axiomtica de la teora de los conjuntos ex
cede el nivel elemental de este libro.
5. El sentido de este curso de lgica es formativo e instrumen
tal. Desde el punto de vista formativo, se pretende ejerci
tar funciones mentales como el conceptualizar, deducir, ar
gumentar y contraargumentar. Desde el punto de vista ins
trumental, se propone entrenar en el manejo de una frac
cin de un lenguaje standard de primer orden que puede
ser usada como herramienta, principalmente, en el mbito
del anlisis de la ciencia y de su metodologa de investiga- 1
cin. Por ello, concedemos un lugar importante al manejo
de variables predicativas y cuantificadores, que posibilitan
el anlisis de relaciones que usualmente escapan al alcan
ce de la mayor parte de los manuales en uso, an de los
buenos.

24
6. Existe la creencia infundada pero difundida de que las co
nocidas como falacias retricas son parte de la lgica. Des
de el punto de vista de los sistemas lgicos en uso, disea
dos principalmente para el anlisis de argumentos cient
ficos, estas falacias carecen de relevancia porque es claro
que su objetivo es la persuasin apelando a los sentimien
tos y no la prueba de la verdad de una afirmacin. Sin em
bargo, como el uso de tales falacias es frecuente, particu
larmente en el discurso poltico que no raras veces, preten
de hacerse pasar por riguroso, hemos considerado alguna
de ellas en vista de que los estudiantes tendrn poca opor
tunidad de abordar este tema en otro curso. Asimismo,
como ciertamente existen falacias propiamente lgicas, he
mos incluido una seccin especial (13.5) para abordar el
asunto de manera elemental.
7. Consideramos fundamental, desde el punto de vista forma-
tivo, que el estudiante logre una comprensin bsica de la
evolucin histrica de los conceptos cientficos. Ello permite
apreciar la medida en que el pasado condiciona el presen
te y el futuro. Empero, cmo podra comprenderse ade
cuadamente la evolucin de conceptos que, presumi
blemente, se ignora casi totalmente? La respuesta la hemos
dado situando la parte histrica al final de nuestro texto,
esto es, en la fase en la que el estudiante ha tenido ya con
tacto con los conceptos cuya evolucin nuestra breve re
sea histrica pretende hacer inteligible.
8. P a ra a p ro b a r el cu rso d el P ro g ra m a L E M M n o es in
d is p e n s a b le q u e e l a lu m n o c o n o z c a la s le c c io n e s 9 , 1 0 ,
1 4 y 1 5 . E lla s e s t n re s e r v a d a s p a ra q u ie n e s d e s e e n u n a
a lta c a lific a c i n . E n e s te c a s o , lo s in te r e s a d o s s o lic ita
r n a lo s tu to re s d e l P ro g ra m a , la e v a lu a c i n a d ic io n a l
c o rre s p o n d ie n te .

25
I
PROPOSICIONES

Objetivos:

Identificar proposiciones y distinguirlas de las expresiones no


proposicionales
Reconocer las proposiciones elpticas como proposiciones
abreviadas.
Distinguir las descripciones definidas de las proposiciones.
Distinguir el lenguaje objetivo del metalenguaje.
CUESTIONARIO 1: Proposiciones

Instrucciones.-

I. Entre las siguientes expresiones unas son proposiciones y otras


no. Escriba sobre la lnea que se encuentra frente a cada expre
sin la palabra S en caso de que sta sea proposicin y la pala
bra N o , en caso contrario. Para orientar adecuadamente su res
puesta, estudie cuidadosamente el contenido de la leccin 1.

1. El manco de Lepanto., 2 2 . Existe a lo ms un gato.

2 . El m anco es de Lepanto. 2 3 . La noche est estrellada y tiritan

3 . E l cuadrado de dos.____ azules los astros a lo lejo s ._____

4. Eureka!________________ 2 4 . E l dios de los Incas.

5. E l hijo de U lises. 2 5 . La B iblia es la palabra de Dios.

6. El teorema de Pitgoras. 2 6 . Am ado sea el nio, que cae y aun

7. No matars a tu prjimo. llora, y el hombre que ha cado y ya

8. El quinto dice no matars. no llora!.

9. L a m ujer del Csar.______ 2 7 . D os ms dos son cinco.

10. L a m ujer del C sar 2 8 . El ro que cruza P a rs._

debe ser honesta. 2 9 . El ro es caudaloso.

11. L a mujer del C sar 3 0 . E s el hijo de Hctor.

fue honesta. 3 1 . E s el h ijo de Hctor?_

12. Cm prame una novela.. 3 2 . El h ijo de H ctor y Luisa.


2
13. El teorema es de Pitgoras., 33. a es siempre par._______

14. L a espada de D am ocles.___ 3 4 . L a s Olimpiadas de Atenas._

15. E l solitario de Sayn.______ 35. Las Olimpiadas fueron en Atenas._

16. E l culpable de la crisis. 3 6 . O jal me am aras!_____________

17. L isto s... Fuego!______ 3 7 . Hay un universo sin objetos.,

18. Fuego!______________ 3 8 . G ato es un animal.

19. Tres al cubo. 39 . C arla es bisilbica.

2 0 . T res no es cbico. 4 0 . 2 x 3 = 6 es el nombre

2 1 . O jal hubiera m arcianos. de 2 x 3 = 6

4 1 . P erro es un mamfero

[29]
LECCIN 1
Proposiciones

1.1. Definicin

A continuacin escribiremos un conjunto de afirmaciones con


las que est familiarizado todo estudiante que ha concluido
secundaria.

a . El lapicero es rojo.
b. El protn tiene carga positiva
c . El teorema de Tales est demostrado en los Eletnentos
d. Todo nmero entero positivo elevado a la potencia cero es igual a uno.
e. 2 + 5 = 5 + 2
f. El sol es una estrella fija.
g . La tierra no se mueve.
h. Existe al menos un crculo con rea equivalente a tin cuadrado.

Es sencillo constatar que estas afirmaciones pertenecen a cam


pos distintos. La primera, por ejemplo, a la vida cotidiana de cual
quier escolar. Las otras, a la matemtica, a la fsica, etc. Asimis
mo, todas ellas estn expresadas en un lenguaje determinado que
en este caso es el espaol, con excepcin de la quinta que est
expresada en lo que llamaremos l e n g u a j e m a t e m t i c o . La afir
macin 2+5=5+2 puede figurar de la misma manera en un libro
en ingls, francs o cualquier otro idioma ya que es una expresin
matemtica que es parte de un lenguaje distinto a los indicados y
que se usa intemacionalmente. Adems, debido a que las afirma-

[31]
ciones anteriores estn expresadas o formuladas en un lenguaje,
todas estn constituidas por un conjunto de signos escritos que
respetan ciertas reglas. Por ejemplo, la que dice que el verbo
copulativo 'ser7 o 'estar' debe estar entre el sujeto y el predicado.
Hablando en trminos que describen mejor lo que observamos,
cada una de las afirmaciones est expresada por una sucesin o
secuencia finita de signos, cada uno de los cuales es una letra de
nuestro alfabeto.
De otra parte, sabemos que algunos de nuestros ejemplos son
afirmaciones verdaderas (los cinco primeros) y que otros estn
constituidos por afirmaciones falsas (los tres ltimos). Esto signi
fica que hay secuencias finitas de signos que con sentido pueden
ser calificadas de verdaderas y que hay secuencias finitas de sig
nos que pueden ser calificadas de falsas. Las secuencias finitas de
signos de esta clase nos interesan particularmente en un curso de
lgica y se denominan proposiciones. En armona con esto, pro
porcionaremos la siguiente definicin.

Definicin 1. Diremos que desde el punto de vista lgico son proposiciones:


i) Todas las secuencias finitas de signos que con sentido pueden ser calificadas de
verdaderas.
^ ii) Todas las secuenas finitas de signos que con sentido pueden ser calificadas de falsas. ^

De conformidad con la definicin anterior todas las afirma


ciones que hemos formulado al comenzar esta explicacin son
proposiciones. Las cinco primeras, por ser consecuencias finitas
de signos que podemos calificar de verdaderas, satisfacen la pri
mera condicin de la definicin. Las tres ltimas, por ser secuen
cias finitas de signos que podemos calificar de falsas, satisfacen la
segunda condicin de la definicin. Podram os definir
abreviadamente 'proposicin' indicando que es toda secuencia
finita de signos que con sentido puede ser calificada de verda
dera o de falsa. Esta definicin es correcta a condicin de que se
entienda que esto no significa que una misma secuencia finita de
signos pueda ser verdadera y falsa a la vez.
Es importante puntualizar que as como a las personas y a las
cosas les asignamos nombres que nos permiten hablar sobre ellas,
de la misma manera podemos darle nombres a las proposiciones,

32
los mismos, que pueden ser muy breves para ganar simplicidad.
De este modo, podemos convenir que en nuestro listado inicial
cada una de las letras es el nombre de la proposicin a la cual
antecede. La primera proposicin tiene como nombre a, la segun
da tiene como nombre b, y as, sucesivamente, hasta llegar a la
octava cuyo nombre es h. En adelante, para abreviar, llamaremos
a tales proposiciones por su nombre.
Hay proposiciones que son oraciones gramaticales como es
el caso de todas las de nuestro listado excepto e. Estas oraciones
estn formuladas en un lenguaje que en nuestra situacin concre
ta es el espaol, pero bien podra serlo el ingls, el francs, el ale
mn o cualquier otra lengua que se use en la vida diaria o cotidia
na. A tales lenguas se les denomina lenguajes naturales o
vernculos, y a las proposiciones que son oraciones dadas en es
tos lenguajes se les llama proposiciones en lenguaje natural. En
cambio, la proposicin e est formulada usando signos especiales
que no son los que usamos en la comunicacin familiar o social
sino cuando trabajamos en Matemtica. Esta proposicin est dada
en un lenguaje especializado, el matemtico. Y a los anlogos a
ste les llamaremos lenguajes formalizados, los mismos que no
son usados en la comunicacin cotidiana sino principalmente en
la actividad cientfica.

1.2. No son p roposiciones

En los lenguajes naturales hay oraciones que no son proposiciones.


Tal es el caso de las oraciones interrogativas como Qu hora es?
u oraciones imperativas, como Vete a dormir! que son ciertamen
te secuencias finitas de signos pero que no pueden ser calificadas
comq verdaderas o como falsas. Lo mismo ocurre con exclamacio
nes como Gracias a Dios! u Ojal ganaras! Las oraciones de nues
tro listado inicial son de naturaleza especial, pues a ellas s sin di
ficultad las podemos calificar de verdaderas o de falsas debido a
que todas afirman o describen algo. Cuando ese algo es el caso,
entonces decimos que son verdaderas y cuando ese algo no es
el caso, entonces decimos que son falsas. Por esta razn estas ora
ciones se llaman gramaticalmente aseverativas y podemos decir
que toda oracin aseverativa es una proposicin.

33
Sin embargo, no podemos sostener que toda proposicin es
una oracin aseverativa, pues la proposicin e, que est escrita en
lenguaje formalizado, no es exactamente una oracin aseverativa
sino es ms propiamente un tipo de frmula matemtica.

1.3. Proposiciones elpticas y descripciones definidas

Hay expresiones exclamativas como Oro!, o Fuego!, por citar slo


dos ejemplos, que podran ser interpretadas como proposiciones
en el sentido de que ellas pueden traducirse por 'En mi mina hay
oro' y 'All hay fuego, respectivamente. De esta suerte, las anterio
res exclamaciones resultan proposiciones abreviadas o elpticas. La
interpretacin anterior en general es correcta y podemos decir que
cuando una exclamacin puede ser expresada de modo ms deta
llado, mediante una oracin aseve-rativa, entonces tal exclamacin
puede ser considerada una proposicin elptica o abreviada.
De otra parte, es importante advertir que hay un cierto tipo
de frases que a menudo originan dificultades en los estudiantes
que tienden a confundirlas con proposiciones. Por ejemplo, las
frases 'El autor del Quijote7, 'El hijo de la Reina Madre', 'El cua
drado de dos7, y las de su tipo, no son proposiciones porque no
aseveran nada. Estas frases se reducen a ser meros artificios para
sustituir nombres, pues la primera puede ser sustituida por 'Mi
guel de Cervantes7, la segunda por 'Luis Miguel7, y la tercera por
'47. En efecto, un nombre no es una proposicin sino slo un com
ponente de ella. Consecuentemente, cuando nos encontramos ante
una secuencia finita de signos que puede ser sustituida por un
nombre, con toda seguridad tal secuencia no es una proposicin
y larlamaremos descripcin definida.
El uso de la palabra 'proposicin 7 es muy difundido entre los
especialistas, pero algunos prefieren usar palabras como 'enun
ciado7, 'sentencia7/ oracin', etc. para referirse a lo que, en este
texto, denotamos con 'proposicin'.

1.4. Metalenguaje y lenguaje objeto

En la seccin 1. hemos dicho que a, b, c, etc. son nombres de pro


posiciones. As el nombre de la proposicin '2+5=5+27 es e. Esto

34
significa que e no es nombre de lo que se llama un objeto mate
rial, por ejemplo, una mesa, sino el nombre de una proposicin
que es una secuencia de signos que denominamos objeto lings
tico. Por esta razn e es un nombre metalingstico y forma par
te de un metalenguaje que se define como un lenguaje que se
usa para describir otro lenguaje llamado lenguaje objeto. Esta
distincin se refiere a dos funciones distintas y no a dos idiomas
distintos. As las afirmaciones que hacemos en este texto sobre las
propiedades de las proposiciones son metalingsticas y las pro
posiciones mismas que usamos como ejemplos son parte del len
guaje objeto. Una manera sencilla de construir el nombre de una
proposicin, que no excluye otras, consiste en escribirla entre co
millas simples. De este modo '2+5=5+2' es el nombre de 2+5=5+2.
La primera expresin pertenece al metalenguaje de este texto y
la segunda a su lenguaje objeto. Anlogamente se puede construir
nombres de nombres, de predicados, etc. Por ejemplo, 'perro 7 es
el nombre de la palabra perro y 'audaz7 es el nombre de la pala
bra audaz. Y no puedo decir que 'perro7 es un animal pero s qu e
'perro7 es bisilbica, pues en este caso no estoy hablando de un
animal sino de la palabra misma. Afirmaciones como 'gato' es un
animal son malas construcciones y no se consideran proposicio
nes sino sin sentidos.

35
II
EL LENGUAJE DE LA LGICA PROPOSICIONAL

Objetivos:

Aprender la lgica como la ciencia dedicada a la construccin


de lenguajes formales.
Identificar las caractersticas de lenguaje standard de PM.
Definir la conjuncin como conectiva proposicional.
Reconocer y aplicar el concepto de variable proposicional en la
construccin de frmulas conjuntivas.
Definir las condiciones de verdad de una proposicin conjuntiva.
Construir algortmicamente tablas de valores.
CUESTIONARIO 2:
El lenguaje de la lgica proposicional

Instrucciones

I. Escriba las proposiciones componentes de las siguientes expre


siones, reemplace cada proposicin componente por una varia
ble proposicional y luego construya una frmula conjuntiva.

1. ]uan y Pedro viajarn al Japn,


2. El profesor de historia es amistoso pero estricto.
3. El hijo de Ulises fu e paciente pero no buen arquero.
4. El problema de la cuadratura del crculo ha tenido solucin, aunque
fue difcil encontrarla.
5. Gustavo tiene dificultades con su investigacin, sin embargo perseve
rar en su empeo.
6 . El nmero ocho es una potencia par y Lima es una ciudad grande.
7. 2" es una potencia par pero 3nes una potencia impar.
8 . Pedrito predica caridad; sin embargo vive con mucho lujo.
9. Francisco es a la vez juez y parte.
10. Fortunato cobra dinero a pesar de que no trabaja.
11. Sam, el defensor de los torturados, es tambin empresario.
12. Pan y circo destruyeron a los romanos.
13. Ftbol y circo distraen a las multitudes.
14. La integral de Nezuton era correcta pero la de Riemann era ms
sencilla.

[39]
II. Sealar cules en el siguiente listado son afirmaciones falsas:

1. El signo especial llamado conectiva de conjuncin forma parte del len


guaje natural.
2. Las variables proposicionales son parte de nuestro lenguaje objeto.
3. Como el alfabeto es finito nosotros podemos obtener nuevas variables
proposicionales, en caso de necesitarlas, escribiendo la misma letra
con diversos subndices y tendremos p 1........ p
4. El margen de una tabla para una proposicin con cuatro variables
proposicionales tiene 12 arreglos.
5. La matriz de la conjuncin es verdadera para el tercer arreglo.
6. Para que una proposicin conjuntiva sea falsa es necesario que sus dos
componentes sean falsos.

III. Responder a las siguientes preguntas:

1. Cmo sera la matriz de la conjuncin si en lugar de comenzar


el margen con un arreglo constituido por dos valores verdade
ros lo comenzarmos con un par de valores falsos?
2. Cmo se define algoritmo?
3. Son las cuatro operaciones elementales de la aritmtica
algortmicas?
4. Por qu la matriz principal de una conjuncin de dos variables
proposicionales no puede tener dos valores verdaderos?
5. Qu relacin hay entre el nmero de valores que puede tomar
una variable y la definicin de proposicin?
6 . Podemos entender el primer arreglo del margen como un par
de proposiciones verdaderas?
7. Quedes lo importante en la tabla de verdad, el valor de las pro
posiciones o su significado?

40
LECCIN 2
El lenguaje de la lgica proposicional

2.1. El lenguaje de PM

La lgica actual, tambin denominada Lgica matemtica, pue


de ser definida, en una primera aproximacin, como una ciencia
dedicada a la construccin de lenguajes especiales, llamados len
guajes formales o artificiales, adecuados para el anlisis de la es
tructura y contenido de las teoras cientficas. Por extensin, los
lenguajes lgicos tambin son muy productivos en el anlisis del
mtodo de investigacin cientfica y de estructuras argumentati
vas filosficas, morales y jurdicas, por citar slo algunos ejem
plos. La variante que es necesario enfatizar, desde el inicio, es que
cuando un lenguaje lgico se usa para analizar una teora mate
mtica o fsica, entonces ocurre que se usa un lenguaje formal para
examinar otro lenguaje formal, pero cuando se lo usa en el anli
sis de una argumentacin moral o poltica, sucede que se usa un
lenguaje formal para analizar lo que se llama el lenguaje natural,
que es el que se usa en la comunicacin cotidiana, por lo que tam
bin se le llama lenguaje ordinario.
Los lenguajes lgicos tienen diferentes usos y niveles de com
plejidad. A un lenguaje que rene los requisitos mnimos como
para ser adecuado para el anlisis de discusiones cientficas y fi
losficas se lo conoce como un lenguaje de primer orden. Asimis
mo, hay variantes importantes dentro de los lenguajes de primer
orden y en el desarrollo de un curso hay que elegir el que goza de
mayor aceptabilidad dentro de la comunidad cientfica intema-

[41]
cional. Consideramos que el lenguaje de primer orden ms ade
cuado para nuestros objetivos es el que se construye a partir de la
obra seera de B. Russell y A. Whitehead, titulada Principia
Mathematica, cuya primera edicin se public en Londres en 1910.
Nos referiremos a l con la sigla PM y en lo que sigue de este
texto, que es muy elemental, desarrollaremos slo algunos frag
mentos de este lenguaje, siendo el primero el que corresponde a
lo que se denomina un lenguaje proposicional.
Procederemos a explicar la funcin de una clase especial de
trm inos que los lgicos su elen denom inar co n ectiv a s
proposicionales porque, en general, cumplen la funcin de co
nectar o enlazar a las proposiciones entre s. En algunos libros de
lgica se les denomina conectores; en otros, trminos de enlace u
operadores proposicionales constantes lgicas. Los autores que
los denominan operadores proposicionales desean enfatizar que
estos trminos adems de enlazar proposiciones establecen ope
raciones entre ellas, que son anlogas a cualquier operacin ma
temtica. Nosotros hemos preferido usar la denominacin
conectivas proposicionales porque es la que con mayor frecuen
cia se usa en los textos de lgica que hay en lengua espaola.

2.2. La co n ju n ci n

Para explicar esta conectiva tomemos como punto de partida las


dos siguientes proposiciones:

a. Kant fue un notable filsofo


b. Gdelfue un lgico checoslovaco.

Sobre la base de estas dos proposiciones, enlazndolas me


diante la partcula 'y' nosotros podemos construir una nueva. De
este modo tenemos:

c. Kant fue un notable filsofo y Gdel fu e un lgico checoslovaco.

Como puede observarse, la proposicin c tiene como compo


nentes a las proposiciones a y b, las mismas que se encuentran
ligadas por la partcula 'y', a la cual llamaremos conjuncin. Asi

42
mismo, a la nueva proposicin c la denominaremos proposicin
conjuntiva y quien la afirma dice la verdad solamente en el caso
que la proposicin 'Kant fue un notable filsofo' sea verdadera y
la proposicin 'Godel fue un lgico checoslovaco' sea verdadera.
Vale decir, las dos proposiciones componentes deben ser verda
deras para que una proposicin conjuntiva sea verdadera.
Siguiendo el mismo procedimiento nosotros podemos cons
truir un nmero ilimitado de proposiciones conjuntivas. Todo lo
que necesitamos hacer es elegir pares de proposiciones y luego
ligarlas mediante la conectiva 'y'. De esta manera tendremos tan
tas proposiciones conjuntivas como deseemos y todas ellas ten
drn en comn una forma lgica o estructura que puede ser re
presentada as

y
En este esquema los puntos suspensivos que estn hacia la iz
quierda de la 'Y representan el lugar que ocupara la primera pro
posicin, y los puntos suspensivos hacia la derecha de la 'Y repre
sentan el lugar que ocupara la segunda proposicin. Este recurso
tambin nos muestra que para entender la estructura lgica de una
proposicin conjuntiva, no es indispensable recurrir a ejemplos
concretos sino que los puntos suspensivos son suficientes para in
dicamos que los lugares a la derecha y a la izquierda de la 'y 7 pue
den ser ocupados por cualquier par de proposiciones.
Debido a lo anterior es posible que en lugar de los puntos
suspensivos utilicemos las ltimas letras de nuestro alfabeto ( p,
q, r, s, e tc ) para representar proposiciones sin necesidad de inte
resamos en especificarlas al detalle. A estas letras se les denomi
na variables proposicionales por analoga con las variables
algebraicas de expresiones tales como ' 2 x + c', en las que la varia
ble algebraica V representa a cualquier nmero no especificado.
De modo semejante, por ejemplo, la variable proposicional/? re
presenta a cualquier proposicin. En armona con lo anterior pro
porcionaremos la siguiente definicin:

D efinicin 2. Son variables proposicionales las le/ras p , q, r, etc., que tienen la


Juncin de representar a cualquier proposicin no especificadas.

43
Empleando variables proposicionales, la estructura lgica de
cualquier proposicin conjuntiva puede ser formulada as:

p y q

donde la variable proposicional p representa la primera propo


sicin que elijamos y la variable proposicional q representa la
segunda. Pero adems, para incrementar este nuevo lenguaje que
estamos construyendo, podemos reemplazar la 'y' por el signo
especial a al que llamaremos conectiva de conjuncin y lo tra
' '

duciremos al castellano por 'y'. De esta manera, utilizando va


riables proposicionales y el signo a podemos representar, de
' '

la siguiente manera la estructura lgica de cualquier proposicin


conjuntiva.

p a q

La expresin anterior constituye en sentido estricto una fr


mula lgica que no est dada en idioma espaol sino en un len
guaje lgico formalizado que iremos incrementando progresiva
mente. Debido al nivel elemental de este curso, a la frmula ante
rior tambin la llamaremos proposicin conjuntiva sin entrar
en otras distinciones propias de niveles ms avanzados. El mis
mo procedimiento ser adoptado en adelante en casos similares.
El vocabulario lgico introducido anteriormente nos permite
ahora dar una definicin que establezca las condiciones de ver
dad de una proposicin conjuntiva.
---------f -----------------------------------;------------------------------------------------ v
Definicin S. Si las variables proposicionales p y q representan cualquier par de
proposiciones, luego la proposicin conjuntiva de la forma p a q es verdadera
solamente en el caso que p sea verdadera y q tambin sea verdadera. En cualquier
otro caso la proposicin p a q es falsa.
\________________________________________________________ /

2.3. Tablas de verdad

Para determinar cmo funciona la definicin anterior en la prc


tica, es necesario recurrir a un artificio lgico llamado tabla de

44
verdad, el cual parece que fue conocido de manera rudimentaria
desde la antigedad. La presentacin que hoy da tiene es la que
us el filsofo Ludwig Wittgenstein en su libro Tractatus Logico-
Philosophicus. La tabla de verdad es necesaria debido a que por
definicin de variable proposicional es posible que p y q repre
senten en unos casos a proposiciones verdaderas y en otros a pro
posiciones falsas, lo que nos da varias posibilidades de com
binar sus valores. Sin embargo, todo lo que puede ocurrir debido
a esto es que p sea en unos casos verdadera y en otros casos falsa
y lo mismo con q. Consecuentemente, p puede asumir o tomar
dos posibles valores (Verdadero - Falso) y la variable q tambin.
Luego, la tabla de verdad debe presentar en orden todas las com
binaciones posibles de los valores de las variables p y q para lue
go aplicar la correspondiente definicin y establecer la verdad de
la proposicin conjuntiva. El proceso de construccin de la tabla
de verdad puede hacerse siguiendo las siguientes reglas.

2.3.1. Algoritmo para la construccin de tablas de verdad

R l. Dibjese una tabla, denominada de doble entrada, como la


que sigue, de tal manera que para cada variable proposicional
exista una correspondiente columna debajo de ella y los valores
que asuma la proposicin conjuntiva, por aplicacin de la Defi
nicin 3, puedan ser escritos, paralelamente y en corresponden
cia, con los valores de las variables proposicionales. Al sector de
la tabla donde deben estar las columnas de valores de las varia
bles se les llama margen.

p q p A q

MARGEN

R2, Escrbase en columnas todas las combinaciones posibles, de


los posibles valores de las variables p y q de tal manera que stos
aparezcan ordenados por pares. (Usese para el valor verdadero

45
la abreviatura V y para el falso F). A cada uno de dichos pares se
le denomina arreglo. El nmero de valores que van a constituir
cada columna se calcula aplicando la frmula: N. de valores de
cada columna = 2n. En esta frmula la letra 'n' es una variable
numrica cuyo valor depende del nmero de variables proposi
cionales que tenga la proposicin que vamos a tabular. En nues
tro caso, dado que nuestra proposicin a tabular contiene sola
mente las variables p y q, entonces n = 2 y, consecuentemente,
2n = 4. El nmero de arreglos coincide con el nmero de valo
res que constituyen cada columna.
Efectuado lo dicho en R 2, la tabla queda as:

P q p a q

Primer arreglo V Y
Segundo arreglo V F
Tercer arreglo F V
Cuarto arreglo F F

Es recomendable escribir en la primera columna como apa


rece aqu, la mitad de valores verdaderos y la mitad de valores
falsos. En la segunda columna un cuarto de valores verdaderos y
un cuarto de valores falsos; en la tercera, cuando hay tres varia
bles, un octavo y as sucesivamente. Estas sucesivas particiones
de las columnas de valores alticos son siempre posibles debido a
que todo nmero que se obtiene aplicando la frmula 2 es siem
pre par.'
R$. Inspeccinese cada uno de los arreglos y escrbase deba
jo de la conectiva de la proposicin conjuntiva el valor que les
corresponde de acuerdo a lo establecido por la Definicin 3. Por
ejemplo, en este caso la proposicin conjuntiva es verdadera sola
mente en el primer arreglo, pues es el nico en el que se cumple
que ambas variables proposicionales son verdaderas, como lo pres
cribe la Definicin 3. En todos los dems arreglos le asignaremos
a la proposicin conjuntiva el valor falso porque al menos una de
las dos variables proposicionales es falsa. As obtenemos una nue
va columna de valores que llamaremos matriz de la conjuncin.
La tabla queda finalmente de esta manera:

46
p q p A q

V V V
V F F
F V F
F F F MATRIZ DE LA CONJUNCIN
Tabla 1

Este proceso de construccin de una tabla de verdad puede


parecer largo y tedioso a un principiante. Sin embargo, con muy
poca prctica, se encuentra que es muy sencillo y rpido. Lo que
ocurre es que la explicacin ha sido lo ms detallada posible para
que queden claros todos los aspectos que deben tenerse en cuen
ta, para lograr una comprensin lcida que permita luego cons
truirla mecnicamente. En efecto, la construccin de una tabla de
verdad es un procedimiento mecnico que adecuadamente apli
cado conduce necesariamente al resultado buscado y a los proce
dimientos de esta clase se les denomina en lgica algoritmos.
Es importante aclarar, antes de seguir adelante, que algunas
palabras que en el lenguaje natural no tienen exactamente el mis
mo uso que 'y', deben ser traducidas al lenguaje lgico por la
conectiva de conjuncin. Es el caso de palabras como 'pero' 'sin
embargo', 'aunque', 'empero', que desde el punto de vista lgico
son equivalentes a 'y'. Si asumimos que la variable proposicional
p representa a la proposicin 'Carlita ir al cine' y que la variable
q representa a la proposicin 'Carlita no tiene dinero', luego p a cj
es la traduccin lgica de las siguientes proposiciones:

Carlita ir al cine pero no tiene dinero.


Carlita ir al cine, sin embargo no tiene dinero.
Carlita ir al cine aunque no tenga dinero.

2.4. Conjuncin lgica vs conjuncin en el lenguaje natural.

De otra parte, la conjuncin en lgica es conmutativa, mientras


que en el lenguaje natural no ocurre siempre as. Por ejemplo, en
lenguaje natural la proposicin 'Gustavo dispar y mat al vena
do' tiene distinto sentido que 'Gustavo mat al venado y dispa-

47
r\ La diferencia radica en que la primera sugiere claramente una
relacin de causalidad que se desvirta en la segunda que ya no
expresa claramente a qu dispar Gustavo. Sin embargo, la co
nectiva de conjuncin no establece ningn tipo de nexo causal o
de orden. En armona con ello, las siguientes proposiciones son
lgicamente equivalentes aunque en el lenguaje natural no sea as:
Luis abraz a su novia y se fu e a la China.
Luis se fu e a la China y abraz a su novia.

48
III
DISYUNCIN Y NEGACIN

Objetivos:

D efinir la disyuncin y la negacin como conectivas


proposicionales.
Esclarecer el carcter inclusivo y exclusivo de la disyuncin.
Diferenciar entre la negacin de una variable proporcional y la
negacin de una conectiva u operador de conjuncin y de
disyuncin
Definir las condiciones de verdad de la disyuncin inclusiva,
disyuncin exclusiva y de la negacin
Expresar en el lenguaje lgico las proposiciones disyuntivas y
negativas del lenguaje natural.
Distinguir los usos de la doble negacin en el lenguaje formal y
en el lenguaje natural.
CUESTIONARIO 3: Disyuncin y Negacin

Instrucciones

I. Usando el lenguaje lgico expresar las siguientes proposiciones,


distinguiendo las disyunciones inclusivas de las exclusivas:
1 .Jaime es pimponista o tenista.
I.Este polgono es un tringulo o un cuadrado.
3. Volvers con el escudo o sobre el escudo.
4. Se presentarn al Jurado los que tengan libreta electoral o sean mayo
res de 18 aos.
5. Ingresarn a la Escuela Politcnica los que aprueben el examen de
ingreso o los que estn exonerados de l.
6. El soldado sobrevivir o perecer en combate.
7. El libro es voluminoso o interesante.
8. Borges es cuentista o novelista.
9 .0 aceptas el aumento o vas a la crcel.
10. Recibirs el dinero o la casa pero no ambas cosas.
I I . Perico es alto o bajo.
12. Perico es alto o Mara es estudiosa.
13. Csar conquista las Galias o Cleopatra no es reina de Egipto.
14. Perico no es piadoso o Jaime no es belicoso.
15. Perico nunca ganar el premio o ser feliz.

11. Usando el lenguaje lgico expresar las siguientes proposicio


nes (no debe usarse la disyuncin exclusiva):

[51]
1. No es el caso que seis sea impar o que existan agujeros negros en el
cosmos.
2. No es el caso que seis sea impar, o que existan gatos.
3. No es el caso que el acusado sea inocente y que sea sentenciado.
4. No es el caso que un nmero sea divisible entre dos y que no sea par.
5. No es el caso que una persona obligatoriamente sea atea o inmoral
6 . Es el caso que Lina es estudiosa o no es aplicada.
7. No es el caso que no te diviertas o seas infeliz.
8. No tengo nada o soy muy rico.

III. Indicar cules en el siguiente listado son afirmaciones falsas:

1. La matriz de la disyuncin exclusiva tiene ms valores verdaderos que


la matriz de la inclusiva.
2. Si a la negacin de p la negamos,, nuevamente obtenemos una matriz
igual a los valores de p.
3. Basta que una variable sea verdadera para que la matriz de la disyun
cin inclusiva sea verdadera.
4. La disyuncin exclusiva es verdadera en el nico caso en que la inclusiva
es falsa.

IV. Responder a las siguientes preguntas:

1. Por qu la negacin es un mecanismo inversor?


2. En qu se diferencia la negacin lgica de la castellana?
3. Existe en espaol un signo especial para la disyuncin inclusiva y
otro para la exclusiva?

52
LECCIN 3
Disyuncin y negacin

3-1. Las d isyu nciones in clu siv a y exclusiva

A menudo nosotros nos encontramos con proposiciones tales


como las siguientes:

a. Carlos es un buen jugador de ajedrez o un buen lector.


b. El marco de una pintura es deforma rectangular o deforma circular.

Lo que tienen en comn las proposiciones a y b es que ellas


han sido construidas sobre la base de otras proposiciones que han
sido enlazadas mediante la partcula 'o' que en lgica se denomi
na conectiva de disyuncin. De manera detallada, las proposi
ciones que constituyen la proposicin cuyo hombre es a son 'Car
los es un buen jugador de ajedrez7 y'Carlos es un buen lector'. En
nuestro ejemplo aparecen de manera abreviada que evita redun
dancias no elegantes en idioma castellano. La proposicin disyun
tiva que forman estas proposiciones es de carcter inclusivo en
el sentido de que, aunque se dice que Carlos tiene una entre dos
propiedades, no excluye la posibilidad de que pueda poseer
ambas. Es completamente posible que alguien pueda ser al mis
mo tiempo un buen jugador de ajedrez y un buen lector.
Distinta es la situacin del ejemplo nombrado por b. La ra
zn de ello es que las proposiciones componentes 'El marco de
una pintura es de forma rectangular' y 'El marco de una pintura
es de forma circular' no pueden ser ambas verdaderas. Si se da el
caso de que el marco es rectangular, entonces ya no puede ser
circular y si se da el caso de que es circular, entonces ya no puede

[531
ser rectangular, porque tales propiedades son excluyentes entre
s. Vale decir, en nuestro segundo ejemplo, la verdad de una de
las proposiciones componentes excluye la verdad de la otra. Por
eso se dice que se trata de una disyuncin en sentido exclusivo.
Aunque en espaol en ambos casos se usa la misma letra V , en
lgica la disyuncin exclusiva se denota por el signo V para dis
tinguirla de la inclusiva que se denota por el signo V 7.
Utilizando las variables proposicionales que representan en
cada caso a las proposiciones componentes, la estructura lgica
de la disyuncin inclusiva es mostrada por la siguiente frmula:

pvq

y la de la disyuncin exclusiva por

p * q

Debemos anotar que la distincin anterior es importante, pero


no indispensable. Se puede prescindir de la disyuncin exclusiva
con relativa facilidad en lgica, razn por la que muchos autores
no la mencionan y la mayor parte de los ejercicios de este texto,
slo requiere, para su ejecucin, de la disyuncin inclusiva.

f D. efinicin 4. La proposicin disyuntiva inclusiva de la forma p v q es verdadera \


siempre que p sea verdadera o que q sea verdadera o que ambas variables
prop osicion ales sean verdaderas. Es fa ls a slo cuando am bas variables
^proposicionales son falsas._____________________________________________

Definicin 5. La proposicin disyuntiva exclusiva de la forma p ^ q e s verdadera si


una, y solamente una de las variables proposicionales, es verdadera. En cualquier
oti& caso es falsa.

Con el auxilio de estas definiciones nos encontramos en con


diciones de construir la tabla de verdad de la disyuncin inclusiva
y de la disyuncin exclusiva. Para ello es necesario proceder exac
tamente de la misma manera como procedimos en el caso de la
conjuncin, hasta la aplicacin de la regla R2.
En el momento de realizar el paso correspondiente a la regla
R3, entonces la variante consistir en que aplicaremos la Defini
cin 4, para obtener la matriz de la disyuncin inclusiva y la De-

54
finicin 5, para obtener la matriz de la disyuncin exclusiva. La
tabla de verdad de la disyuncin inclusiva es como a continua
cin se grafica:

MATRIZ DE LA
DISYUNCIN
INCLUSIVA
T a b la 2 1 ____

Asimismo, la tabla de verdad de la disyuncin exclusiva que


da graficada del siguiente modo:

p q p*q
V V F
V F V
F V V MATRIZ DE LA
F F DISYUNCIN
F
EXCLUSIVA
T a b la 3 t ____

Es evidente que la disyuncin inclusiva slo es falsa en el


cuarto arreglo debido a que es el nico en que ambas variables
son falsas. La disyuncin exclusiva es verdadera en los arreglos
segundo y tercero porque slo en ellos una y slo una de las va
riables es verdadera. Remarcaremos que la diferencia entre am
bas tablas se encuentra en el valor correspondiente al primer arre
glo que es verdadero en la disyuncin inclusiva, y falso en la ex
clusiva que no admite que dos proposiciones sean verdaderas.

3.2. La negacin

La negacin es una conectiva especial porque no enlaza proposi


ciones sino que se aplica directamente a slo una proposicin. Esto
lo comprenderemos muy fcilmente usando ejemplos. Tengamos
las proposiciones:

55
c. El cuaderno es rojo.
d. El nmero seis es par.

En efecto, a partir de ella es posible construir nuevas propo


siciones que sean sus negaciones, introduciendo la partcula 'no7.
As tenemos:

e. El cuaderno no es rojo.
f. El nmero seis no es par.

Y este procedimiento podemos aplicarlo tanto como quera


mos, pues, dada una proposicin, siguiendo un mecanismo se
mejante a ste, siempre es posible construir una nueva que sea su
negacin, a la que se denomina proposicin negativa. Sin embar
go, el uso de la partcula 'no 7 en lgica no se hace dentro de la
oracin como en los casos anteriores, en los que se respeta la gra
mtica espaola usual. Los lgicos prefieren construir la nega
cin de una proposicin anteponindole la partcula 'no7. Siguien
do este criterio, las negaciones lgicas de a y b son:

el. no - (El cuaderno es rojo)


f l. no - (El nmero seis es par)

Esto nos permite comprender que la estructura lgica de una


proposicin negativa cualquiera puede ser grafcada como sigue:

n - ( ................................... )

SiAisamos variables proposicionales y el signo ' ~ 7 que se usa


en lgica para simbolizar la partcula 'no7, entonces tenemos que
una proposicin negativa se escribe en lenguaje lgico as:

~P

Es claro que 'e l 7 y 'f l 7 no son afirmaciones elegantes en espaol


aunque sean lgicamente correctas. Para salvar este detalle ellas
pueden ser traducidas por 'No es el caso que el cuaderno sea rojo7 y
por JNo es el caso que el nmero seis sea par'. Asimismo, resulta muy

56
intuitivo que cuando una proposicin es verdadera su correspon
diente proposicin negativa debe ser falsa y viceversa.

Definicin 6. La proposicin negativa de laform a ~p es verdadera solamente cuando


la variable p es fa lsa y es fa lsa solamente cuando la variable p es verdadera.

Con ayuda de esta definicin y de las reglas R1 y R2 pode


mos construir fcilmente la tabla de verdad para una proposicin
negativa. La variante en este caso ser, adems de la aplicacin de
la Definicin 6 , que hay una sola variable proposicional, por lo que
en el margen habr slo una columna. Asimismo, la columna ten
dr solamente dos valores debido a que en este caso n = 1 y 2 = 2
porque hay una sola variable proposicional.

V
F

T a b la 4

Debe observarse que los valores de la matriz de ~ p se escri


ben en columna debajo de la conectiva para no confundirlos
con los valores de la variable p. Asimismo, la disposicin de la
tabla nos permite comprender ahora que, es muy conveniente
colocar la negacin delante de la variable para poder escribir
los valores de la matriz sin dar lugar a confusiones. Anotamos
tambin que la conectiva de negacin funciona como un artificio
inversor que transforma el valor verdadero en falso y viceversa.

3.3. Negacin de una conjuncin y de una disyuncin

Hemos dicho que la conectiva de negacin se aplica a una pro


posicin. Los ejemplos e y f muestran ello con proposiciones sim
ples pero una proposicin tambin puede ser compuesta como es
el caso de las conjuntivas y disyuntivas. Por ejemplo, podemos
examinar las siguientes proposiciones:

57
g. No es el caso que Juan sea honesto y tenga una conducta inmoral.
h. No es el caso que Hugo sea piloto o cocinero.

En los ejemplos anteriores no se niega proposiciones simples


sino se niega operaciones lgicas con proposiciones simples: en el
ejemplo g se niega una conjuncin y en el ejemplo h una disyun
cin. Como en este caso no se niega proposiciones aisladas sino la
conexin entre proposiciones, los esquemas que corresponden
a g y h, respectivamente son:

~ ( ..... A ......)
~ ( ..............v ............... )

De este modo las frmulas del lenguaje proposicional que


corresponden a g y h, en el mismo orden, son:

gl. ~ ( p A q )
h l. ~ ( p v q )

As queda claro que es verdad que la conectiva de negacin


siempre se aplica a una sola proposicin, la misma que puede ser
simple o compuesta. En este segundo caso, para hacer visible la
unidad de la proposicin compuesta, hemos procedido a ence
rrarla entre parntesis y de este modo se logra que se perciba con
claridad que el alcance de la conectiva llega hasta el corres
pondiente parntesis de cierre.
Aunque posteriormente trataremos con detalle la jerarqua
de las conectivas, podemos adelantar que en g l y h l la conectiva
de negacin es la de mayor jerarqua, pues en el primer caso nie
ga la matriz de la conjuncin y en el segundo la matriz de la disyun
cin inclusiva. Por ello a las reglas R l, R2 y R3 hay que aadirles,
en este caso, la regla que establece que la matriz principal de una
frmula es la que corresponde a su conectiva de ms alta jerar
qua. Como se aprecia en las tablas de verdad, los valores verda-
dero-falso de las proposiciones simples no estn negadas.

58
p q ~ ( P Aq) P q ~ ( P V q)
V V F V V V F V
V F V F V F F V
F V V F F V F V
F F V F F F V F
i
MATRIZ PRINCIPAL DE LA MATRIZ PRINCIPAL DE LA
NEGACIN ALTERNATIVA NEGACIN CONJUNTA
El lgico norteamericano W. O. Quine denomina negacin
alternativa a la negacin de la conjuncin y negacin conjunta a
la negacin de la disyuncin. Para entender el sentido de estas
denominaciones es necesario conocer las frmulas de De Morgan
que presentaremos en una leccin posterior. Asimismo, la nega
cin alternativa la utilizaremos para definir la regla de construc
cin de la matriz de la conectiva que en este libro llamamos ba
rra de Nicod.

3.4. Doble negacin

En los lenguajes lgicos, como el lenguaje PM, es frecuente el uso


de la doble negacin para construir las frmulas que correspon
den a expresiones como:

i. No es el caso que el nmero cuatro no sea par.

La frmula correspondiente a i se obtiene a partir de un es


quema como el siguiente:

No (n o ( el nmero cuatro es par) )

que traducido a frmula permite obtener ~ ( ~ ( p ) ) o simple


mente, prescindiendo de los parntesis, se tiene p. La tabla de
verdad respectiva es:

V F
MATRIZ PRINCIPAL DE
F V LA DOBLE NEGACIN

59
Esto permite constatar que los valores de p coinciden con los
de la matriz de p. Por ello se dice que en lgica la doble negacin
de una proposicin siempre equivale a su afirmacin. Esto no ocu
rre igualmente en castellano. Una persona en lugar de decir Yo no
tengo dinero para negar que posee dinero, dice coloquialmente:
Yo no tengo nada. Si procedemos a anteponer la negacin, como
sealan las reglas de PM, obtenemos el esquema

No ( Yo tengo nada )

el mismo que podemos interpretar como No es verdad que yo


tenga nada que permite entender que Yo tengo algo, lo que
contradice la intencin del hablante cuando dice Yo no tengo
nada. En este sentido, si contamos "nada7 como una segunda ne
gacin, debemos aceptar que en castellano frecuentemente la do
ble negacin sigue siendo negacin.
Un ejemplo adicional y frecuente lo proporciona la expre
sin coloquial No hay nadie, usada en castellano para expresar
la completa ausencia de personas en un recinto determinado. Si
interpretamos el segmento 'hay nadie 7 en trminos de no existe
en el recinto al menos una persona, encontramos que la mencio
nada locucin da lugar al siguiente esquema:

No ( no existe en el recinto al menos una persona )

El esquema anterior como se aprecia, contiene claramente


una doble negacin que da lugar a que, desde el punto de vista
lgico, se lo pueda interpretar como equivalente a Existe en el
recintb al menos una persona, lo que contradice la intencin del
hablante que dice No hay nadie, normalmente, en sentido ne
gativo. Esta peculiaridad del castellano, que consiste en admitir
usos que transgreden la regla de equivalencia lgica entre una
proposicin afirmada y su doble negacin, no es compartida por
- otros idiomas como el ingls o el alemn cuyas oraciones negati
vas usan una sola vez la negacin. En castellano esta discrepancia
entre regla lgica y uso se puede subsanar si en lugar de decir
No hay nadie, decimos No hay persona alguna. Esta ltima
expresin conserva el sentido negativo del lenguaje coloquial y
usa slo una negacin.

60
IV
EL CONDICIONAL Y LA IMPLICACIN

Objetivos:

Definir el condicional como una conectiva proposicional.


Distinguir entre los conceptos de condicional, implicacin, im
plicacin estricta y condicional contrafctico.
Reconocer las condiciones de verdad de un condicional.
Distinguir la verdad de una proposicin condicional de la rela
cin de atingencia entre antecedente y consecuente.
Definir lgicamente los conceptos de condicin necesaria y con
dicin suficiente.
CUESTIONARIO 4
El condicional y la implicacin

Instrucciones

I. Subrayar el antecedente y el consecuente de cada una de las


siguientes proposiciones.

1. Si vas a la Iglesia, entonces eres creyente.


2. Si comes alimentos, entonces no adelgazars.
3. Si lloras, entonces no demostrars valor.
4. Si Arstides no es honrado, entonces nadie es honrado en Atenas.
5. Si Pancho no es deshonesto, entonces nadie es deshonesto.
6. Si los precios suben por la crisis, entonces hay quien est ganando
dinero con la crisis.
7. Si la Aritmtica es consistente, luego la Geometra tambin lo es.
8. Si hay problemas sociales, entonces seremos muy cautos con las
distracciones.

II. Expresar en el lenguaje lgico las siguientes proposiciones.

1. Ir al cine solamente si tengo dinero.


2. Las crisis se producen porque alguien toma malas decisiones.
3. Un nmero es par si es divisible por 2.
4. Una figura es un tringulo siempre que tenga exactamente 3 lados.
5. La Geometra de Riemannfue posible porque existi la de Euclides.
6. Los vendedores de armas ganan dinero solamente si hay guerra.

[63]
7. Si alguien gana dinero con la crisis, entonces hay alguien que tiene
inters en mantener la crisis.
8 . No es posible gastar en distracciones porque no hay dinero para la
solucin de las necesidades primarias.

III. Realizar las siguientes tareas:

1. Construir cinco ejemplos de condicionales contrafcticos.


2. Construir cinco ejemplos de implicaciones.
3. Construir tres ejemplos de implicaciones estrictas.

IV. Responder a las siguientes preguntas:

1. A qu se llama relacin de atingencia entre el antecedente y el


consecuente?
2. Es necesario que haya relacin de atingencia entre el antecedente y el
consecuente para que una proposicin condicional sea lgicamente
correcta?

V. Construir frmulas y tablas para:

1. Si p, luego no es el caso que no p .


2. Si no es el caso que no p, luego p.
3. Si p o n o p , entonces no es el caso que no p.
4. Si p, luego no p.
5. p si no es el caso que no p.

(En este grupo de ejercicios debe usarse de manera explcita la doble


negacin.)

64
LECCIN 4
El condicional y la implicacin

4.1. D elim itaci n concep tu al

En esta seccin abordaremos probablemente la conectiva ms dis


cutida por los especialistas y de mayor relevancia en la compren
sin de lo que es el razonamiento lgico denominado por Piaget
pensamiento hipottico deductivo: el condicional o implicacin
material. Al respecto los trabajos especializados distinguen entre
el condicional o implicacin material, la implicacin y la impli
cacin estricta desarrollada por C. I. Lewis. Las diferencias son
finas e importantes pero en este texto, bastante introductorio, no
podemos detallarlas. Por ahora slo sealaremos que las tres im
plicaciones antes mencionadas tienen en comn corresponder en
castellano a la expresin esquemtica Si..., entonces.... que a su
vez corresponde a nuestras oraciones hipotticas del tipo 'Si me
sacara la lotera, me comprara un Mercedes Benz'. En este caso, no
estoy afirmando sin condiciones que me voy a comprar un Mer
cedes Benz sino que lo har si tuviera lugar la hipottica situa
cin de que me sacara la lotera. Normalmente no se me conside
rar mentiroso si no me compro un Mercedes Benz mientras no
me saque la lotera, pero s en el caso de que me la saque y no lo
compre. En buena cuenta, podemos entender que lo que quiere
decir la anterior oracin hipottica es: JNo es posible que me saque
la lotera y que no me compre un Mercedes Ben. Esta expresin tra
ducida al lenguaje de PM corresponde a la frmula: ~ ( p a - q )

[65]
En esta seccin trataremos el condicional o implicacin ma
terial entendindolo como un operador (as tambin se suele lla
mar a las conectivas) que equivale a la negacin de una conjun
cin cuya primera variable proposicional est afirmada y cuya
segunda variable proposicional est negada. Asimismo, preferi
mos usar en esta expo-sicin inicialmente la palabra 'condicional'
para luego introducir con mayor precisin el uso de la palabra
'implicacin7.

4.2. E l cond icion al

Iniciaremos esta explicacin a partir de algunos ejemplos muy cer


canos a nuestra experiencia:
a. Si son dados el par de puntos A y B, entonces se puede trazar una
recta que los una.
b. Si Ricardo Palma ha nacido en Lima, entonces es peruano.
c. Si todos los gatos son negros, entonces algunos gatos son negros.
d. Si Tpac Ama.ru hubiera atacado el Cusco, entonces su revolucin
habra triunfado.
e. Si la Luna se ve blanca, entonces la Luna es de queso.

Todas estas proposiciones, llamadas proposiciones condicio


nales, tienen la caracterstica comn de tener una estructura del
tipo 'Si...entonces...'. Lo que las diferencia es que los componentes
que ocupan los lugares que corresponden a los puntos suspensivos
son en cada ejemplo distintos. Como en los casos anteriores, a la
lgica le interesa fundamentalmente el aspecto estructural. Por
eso la expresin 'S i ... entonces .../ se interpreta como la conectiva
denominada condicional cuyo signo lgico es
A la proposicin que se encuentra entre 'Si7 y 'entonces7 se
denomina antecedente y a la que se encuentra despus de 'en
tonces7 se le denomina consecuente. Por ejemplo, de manera de
tallada, en la proposicin a el antecedente es 'son dados el par de
puntos A y B' y el consecuente e s 'se puede trazar una recta que una a
los puntos A y B7. Este consecuente no coincide exactamente con el
que aparece en a pero es estrictamente equivalente a l. Lo que
ocurre es que a como las otras proposiciones de nuestro listado,
contiene algunas abreviaciones propias del idioma espaol que

66
nosotros hacemos completamente explcitas cuando damos la
versin detallada.
De manera intuitiva y sin mayor discusin, se puede conce
der que las cuatro primeras proposiciones del listado son correc
tas. La proposicin a porque es un postulado conocido de la Geo
metra. La proposicin b porque establece una condicin que sa
tisfacen todos los limeos que, por ser tales, son necesariamente
peruanos. Aceptamos la correcin de c porque esta proposicin
no afirma a secas Todos los gatos son negros'. Ms bien, nos dice
que si ste fuera el caso, si aceptamos esta hipottica situacin en
la que todos los gatos son negros, entonces algunos de ellos (una
parte) tienen que ser negros.

4.3. C o n d icio n al contrafctico

La proposicin d es usualmente reclamada como correcta por los


historiadores, pues no se afirma que Tpac Amaru atac el Cus
co, sino que si se hubiera producido esa hipottica situacin, en
tonces habra triunfado su revolucin. En esta condicin, noso
tros aceptamos que d expresa un razonamiento correcto a pesar
de que sabemos que el antecedente aisladamente no es una pro
posicin verdadera ni el consecuente tampoco. A las proposicio
nes como d se las llama condicionales contrafcticos, porque se
acepta su correcin a pesar de que sus antecedentes y sus conse
cuentes van contra los hechos. Otro ejemplo de este tipo es 'Si
Dante Alighieri hubiera nacido en Lima, entonces sera paisano
de Jos Santos Chocano', cuya correcin lgica es irreprochable.
Una situacin distinta nos plantea la proposicin e. Todos
diramos que es falsa. Lo importante en este caso es precisar cul
es la variante que presenta esta proposicin condicional respecto
a las anteriores que nos obliga a calificarla de falsa. Una primera
inspeccin nos muestra que el antecedente 'La Luna se ve blcmcd es
verdadero mientras que el consecuente 'La Lima es de queso' es
decididamente falso. Esta es ciertamente una situacin nueva,
como lo demostraremos.
En efecto, examinando el ejemplo a ocurre que su anteceden
te es una proposicin verdadera y su consecuente tambin es una
proposicin verdadera. Lo mismo puede decirse de b. En el caso

67
de la proposicin condicional c encontramos que el antecedente
es una proposicin falsa y el consecuente una proposicin verda
dera. Examinando la proposicin condicional d es claro que el
antecedente es una proposicin falsa y el consecuente tambin es
una proposicin falsa. Sin embargo, estos distintos valores de los
antecedentes y consecuentes de estas cuatro proposiciones condi
cionales parecen no alterar fundamentalmente la situacin gene
ral, pues ocurre que en tanto cada una de ellas expresa una co
nexin razonable entre antecedente y consecuente, las calificamos
como verdaderas sin mayor dificultad. En cambio, cuando el an
tecedente es verdadero y el consecuente es falso la situacin ge
neral s cambia, pues la proposicin condicional resulta inacepta
ble racionalmente y por ello la calificamos de falsa, como es el
caso de e. Este anlisis puede ser esquematizado como sigue, usan
do la abreviacin V para verdadero y la abreviacin F para falso.

EJEMPLO ANTECEDENTE CONSECUENTE PROPOSICIN


CONDICIONAL
a V V V
b V V V
c F V V
d F F V
e V F F
Esto muestra claramente que las proposiciones cuya estruc
tura lgica est dada por 'Si ... entonces solamente son falsas
cuando el antecedente es verdadero y el consecuente es falso.
Asimismo, si usamos variables proposicionales para el antecedente
y el consecuente, y el signo entonces tenemos que la frmula
lgica de una proposicin condicional es:

p ->q
defin icin 7. La proposicin condicional de la form a p q, que tiene como
antecedente a p y como consecuente a q, es fa lsa solamente cuando p es verdadera
y q es falsa. En cualquier otro caso es verdadera. _____ j

Con ayuda de esta definicin y siguiendo las reglas que ya


conocemos, construiremos a continuacin la tabla de verdad de
una proposicin condicional.

68
La tabla anterior permite entender que el condicional puede
ser interpretado como una prohibicin que dice que no es posible
que el antecedente sea verdadero y el consecuente falso. Si esto
sucede, entonces la proposicin condicional es falsa. Igualmente
nos asegura que una proposicin condicional con antecedente
verdadero slo es verdadera cuando su consecuente tambin es
verdadero.

4.4. R elaci n de atingencia

Es necesario aclarar, adems, que la verdad de una proposicin


condicional es completamente independiente de las relaciones
que puedan existir o no entre los significados del antecedente
y del consecuente. En los ejemplos de nuestro listado existe rela
cin entre lo que afirman los antecedentes y los consecuentes; ha
blan de lo mismo, por decirlo as. Cuando esto ocurre, entonces,
hay una relacin de atingencia entre el antecedente y el conse
cuente. Sin embargo, pueden encontrarse muchos ejemplos de
proposiciones condicionales verdaderas en las que no se da una
relacin de atingencia, pues lo que dice el antecedente es com
pletamente diferente de lo que dice el consecuente. As, tenemos
la proposicin 'Si 2 + 2 = 4, entonces el Per est en Sudamricd es
verdadera a pesar de que no existe relacin entre los significados
de sus proposiciones componentes, porque el antecedente ' 2 + 2 =
4' es verdadero y el consecuente 'el Per est en Sudamrica' tam
bin es verdadero. Sin embargo, es importante aclarar que los con
dicionales interesantes para la ciencia y la filosofa general
mente son atingentes.

4.5. C ond icional vs. Lenguaje N atural

Asimismo, la frmula, en lenguaje lgico, p > q n o slo sirve para


expresar proposiciones de la forma 'Si p, entonces q', sino tam
bin proposiciones frecuentes en espaol de las formas:
p solamente si q
qsip
q porque p
q siempre que p

69
Para ilustrar lo anterior daremos algunos ejemplos. Supon
gamos que la variable p representa la proposicin 'Gustavo com
pra un reloj' y que la variable q representa la proposicin 'Gusta
vo recibe una propina de su padre'. Luego la proposicin p - q
es la expresin lgica de las siguientes proposiciones formuladas
en lenguaje natural:

a. P Gustavo compra un reloj solamente si recibe una propina de su padre


tJ
b. q Gustavo recibe una propina de su padre es p ara comprar un reloj
U
c. q Gustavo recibe una propina de su padre porque comprar un reloj

Gustavo recibe una propina de su padre | siempre que compre un reloj


-u
Como se aprecia, cuando se usa /... solamente si el orden
dado en el lenguaje natural se conserva en el mismo sentido en la
expresin lgica. Pero cuando se usan las partculas'... s i ...', '...
porque ...' y /... siempre que ...' el orden de las proposiciones se in
vierte cuando se pasa a la expresin lgica.

4.6. Im p licacin

El antecedente de un condicional puede ser cualquier proposicin


compusta y de la misma manera el consecuente. Por ejemplo, Si
Irene no va a misa y Juan no es catlico, entonces Juan se casar reli
giosamente o no es catlico. En este ejemplo el antecedente es una
conjuncin y el consecuente una disyuncin. Si usamos parnte-
sisTpara separar claramente el antecedente del consecuente, en
tonces obtendremos la siguiente frmula y su respectiva tabla de
verdad (construiremos ocho arreglos debido a que la traduccin
requiere tres variables proposicionales).

70
p q r (~ p a ~ q )-> (r v ~ q)

V V V F F F V V F
V V F F F F V F F
V F V F F V V Y ,v
V F F F F V V V V
F V V V F F V V F
F V F V F F V F F
F F V V V V V V V
F F F V V V V V V

MATRIZ DEL MATRIZ MATRIZ DEL


ANTECEDENTE PRINCIPAL CONSECUENTE

Observando la matriz principal se encuentra que esta propo


sicin condicional es siempre verdadera (por ello se le denomina
tautologa). Ello nos permite definir una implicacin como un
condicional cuya matriz principal es siempre verdadera. Tam
bin se califican los condicionales de este tipo de lgicamente
vlidos. Esta peculiaridad diferencia al condicional anterior d e p - > q ,
cuya matriz principal no es siempre verdadera.

4.7. Implicacin estricta

En este caso daremos directamente un ejemplo y procederemos


a construir su correspondiente tabla de verdad.
Si una proposicin es verdadera o no lo es, entonces no es posible
qne sea verdadera y al mismo tiempo no lo sea.
Como se observa, es una implicacin pero se diferencia de la
anterior en que la matriz de su antecedente es siempre verdade
ra. As definiremos una implicacin estricta como una implica
cin cuyo antecedente es siempre verdadero.

4.8. Condicin necesaria vs condicin suficiente

De otra parte, en el vocabulario lgico se dice que cuando se tie


ne una proposicin condicional de la forma p q, entonces el

71
consecuente es condicin necesaria del antecedente y el antece
dente es condicin suficiente del consecuente.
Cabe precisar que tradicionalmente a la condicin necesaria
se le ha conocido como condicin sine qua non, lo que en caste
llano significa condicin sin la cual no se produce un cierto he
cho, acontecimiento o fenmeno. Por ejemplo, en condiciones
usuales, el oxgeno es condicin necesaria para la combustin.
Por tanto, nosotros podemos armar el siguiente condicional 'Si
se produce combustin en la habitacin Z en to n ces existe oxgeno en la
habitacin Z, con la seguridad de que siempre que el antecedente
es verdadero el consecuente tambin, inevitablemente, lo es. En
cambio el condicional recproco 'Si existe oxgeno en la habita
cin Z, entonces se produce combustin en la habitacin Z' es
falso porque es posible que exista oxgeno en cualquier habita
cin, por ejemplo en la Z, y, sin embargo, no se produzca com
bustin, como ocurre todos los das en las habitaciones de nues
tras viviendas. De este anlisis se deduce que si reconocemos un
condicional como verdadero, entonces lo afirmado en el conse
cuente es condicin necesaria para lo afirmado en el antecedente,
y lo afirmado en el antecedente es condicin suficiente para lo
afirmado en el consecuente. En nuestro ejemplo, la existencia de
combustin en la habitacin Z basta para afirmar la existencia de
oxgeno en dicha habitacin. Sin embargo, no es condicin nece
saria porque es completamente factible cambiarla. Podemos de
ducir el mismo consecuente desde otro antecedente. Por ejemplo:
'Si existen personas respirando satisfactoriamente en la habitacin Z,
entonces existe oxgeno en Zf es un condicional tambin verdadero.
En cambio, la condicin necesaria no la podemos cambiar, pues
tod^s sabemos que 'No es posible que se produzca combustin y no
que haya oxgeno'. Esta ltima afirmacin formalizada correspon
de a ~ ( p A ~ q ) , que pone en evidencia que para que se cumpla
'p' la presencia del oxgeno es inexcusable. Asimismo, si se hace
la tabla de verdad de dicha frmula, se ver que coincide con la
de p -> q.
Existe la tendencia errnea a creer que la condicin necesaria
debera aparecer primero, vale decir, como antecedente. Espera
mos que ello est suficientemente aclarado en esta seccin. Igual
mente es un error intentar definir estos conceptos sin una refe

72
rencia especfica. Ninguna afirmacin expresa en s misma una
condicin necesaria o una suficiente. Son conceptos relacinales.
Debido a ello siempre un A, por ejemplo, es condicin necesaria
para algn B, pero podra no serlo para un C. La manera efectiva
de evitar imprecisiones es construir condicionales verdaderos cada
vez que estos conceptos estn en discusin.

73
V
BICONDICIONAL, FUNCIONES
DE VERDAD Y EQUIVALENCIA

Objetivos:

Definir el bicondicional como una conectiva proposicional.


Reconocer las condiciones de verdad del bicondicional.
Relacionar el concepto de funcin matemtica y el concepto de
funcin de verdad.
Distinguir las proposiciones atmicas de las moleculares.
Determinar cundo las frmulas son lgicamente equivalentes.
Usar las frmulas equivalentes como definiciones que permi
ten reemplazar una frmula por otra.
Traducir las proposiciones bicondicionales en condicionales.
CUESTIONARIO 5
Bicondicional, funciones de verdad y equivalencia

Instru cciones

I. Sabiendo que las letras p, q, etc. representan proposiciones, ex


presar completamente en lenguaje lgico las siguientes afirma
ciones.

1. p, si y solamente si p
2. No p si y solamente si no p
3. Si no p, entonces no q
4. Si no q, entonces p
5. No p si y solamente si q

II. Traducir al lenguaje lgico las siguientes afirmaciones.

1. Un nmero es par si y solamente si es divisible por 2


2. Ir ajuicio si y slo si estoy seguro de ganar.
3. Ganars dinero solamente si trabajas.
4. Juan campeonar si gana la pelea.
5. El postulado V es verdadero si y slosi el espacio es recto.

III. Dados los bicondicionales p <-> q y r <- s, Cules de las si


guientes proposiciones son equivalentes a uno de ellos ?

1. p - q y q - r 4. r - s y s - r
2. p - q y r - p 5. s - r y r - s
3. p - q y q - p 6 . r q y p r

[77]
IV. Indicar cules del siguiente listado son afirmaciones falsas.

1. El bicondicional es verdadero all y slo all donde la disyuncin ex-


chisiva es falsa.
2. La conectiva de negacin puede definirse com of( p ) : W 2
3. Dos condicionales con cuatro variables distintas son equivalentes a un
bicondicional con dos variables distintas.

V. Indicar usando variables proposicionales cuntas proposiciones


atmicas distintas hay en cada uno de los siguientes razonamien
tos. Se recomienda hacer un listado de proposiciones atmicas y
poner a un costado las variables que las representan.

1 . Si est lloviendo o nevando entonces est corriendo viento.


corrie?ido viento entonces no est nevando.
Y
si no est

2. Usted se casar o se convertir en una actriz. Si usted no se convierte


en una actriz, usted no ser famosa. Usted ser famosa o no se casar.
Por tanto, usted no se convertir en una actriz.
3. Si la gente no piensa en la crisis; entonces alguien la distrae. Si el
fitbol es una distraccin de masas, entonces puede ser propiciado por
los beneficiados por la crisis.

VI. Resolver:

1. Es ~ ( p a q ) lgicamente equivalente a pv ~ q )l
2. Es ( p a q ) lgicamente equivalente a ~ ( ~ p v ~ 7 ) ?
3. Es ( p -> q ) lgicamente equivalente a ~ ( p a ~ q ) l

4. Es ( p - q ) lgicamente equivalente a (~ p v q ) ?
5. 4 s ( p v q ) lgicamente equivalente a ~ (~ p a ~ q ) ?

78
LECCIN 5
Bicondicional, funciones de verdad y equivalencia

5.1. El bicondicional

El bicondicional es una conectiva que en algunos libros es llama


da equivalencia. En el lenguaje natural su sentido est dado por
la expresin/... si y solamente s i que en el lenguaje lgico se de
nota mediante el signo que es una flecha en ambas direccio
nes. Las proposiciones bicondicionales se encuentran especialmen
te en la matemtica. Por ejemplo, 'Un nmero es par si y solamente
si es divisible por 2'. En el lenguaje natural tambin encontramos
proposiciones bicondicionales tales como 'Matas viajar a Jauja
si y solamente si toma el tren. Lo que caracteriza esencialmente
a los ejemplos anteriores es que establecen las siguientes propo
siciones, cada una de las cuales est constituida por dos proposi
ciones condicionales de sentido inverso.

a. Si un nmero es par, entonces es divisible por dos y si un nmero es


divisible por dos, entonces es par.
b . Si Matas viaja a Jauja, entonces toma el tren y si Matas toma el tren,
entonces viaja a Jauja.

Como puede apreciarse, las proposiciones bicondicionales


sancionan relaciones ms exigentes que las puramente condicio
nales. Establecen que si el antecedente es verdadero, entonces el
consecuente tiene que ser verdadero pero, adems, que si el con
secuente es verdadero, entonces el antecedente tambin tiene que

[79]
serlo. En otras palabras, la verdad o falsedad de una proposicin
exige necesariamente la verdad o falsedad de la otra.

Definicin 8. La proposicin bicondicional de la forma p<-> q es verdadera cuando


las variables p y q tienen e l mismo valor, esto es, cuando am bas son verdaderas y
cuando am bas son falsas. En cualquier otro caso es falsa.

Con el auxilio de esta definicin y de reglas que ya no necesi


tamos repetir, construiremos la tabla de verdad de una proposi
cin bicondicional de la siguiente manera.

p q p q
V V V
b rT7 MATRIZ DE LA
pr pr
V
\VT PROPOSICIN
BICONDICIONAL
F F V /
A
Tabla 6
Observando la tabla anterior se encontrar que la conectiva
bicondicional puede ser interpretada como inversa de la disyun
cin exclusiva, en el sentido de que es verdadera en los arreglos
en los que la disyuncin exclusiva es falsa y es falsa en los arre
glos en los que la disyuncin exclusiva es verdadera.
Asimismo, en la proposicin bicondicional p <-> q se dice que
p es condicin necesaria y suficiente de q y que q es condicin
necesaria y suficiente de p.

5.2. Ls conectivas como funciones de verdad

El concepto de funcin es uno de los ms fundamentales de la


matemtica y por ello desde los cursos introductorios de aritm
tica y lgebra todo estudiante lo conoce. Como es sabido, un ejem
plo de funcin aritmtica es ' y = 2x 7 que se comporta de la si
guiente manera:

80
Cuando 'x' es igual a 1, entonces 'y ' es igual a 2.
Cuando V es igual a 2, entonces 'y ' es igual a 4.
Cuando 'x' es igual a 3, entonces 'y' es igual
Y as sucesivamente...

Puede apreciarse fcilmente que el mecanismo de la funcin


consiste en que a un determinado valor de la variable 'x' le co
rresponde un nico valor de la variable 'y'. Y no es posible que
dos valores distintos de la variable 'y' correspondan al mismo
valor de la variable 'x\ Este tipo de correspondencia, que va de
los valores de 'x' a los valores de 'y', es lo que da lugar a que los
valores de 'y sean determinados nica y exclusivamente por los
valores de 'x .
De otra parte, analizando cualquiera de las tablas de verdad
que hemos construido, por ejemplo, la de la conjuncin, encon
tramos que en ella se establece una correspondencia de tal mane
ra que a cada arreglo ( a cada par de valores de p y de q ) le co
rresponde solamente un valor en la matriz, y los valores de sta
son determinados nica y exclusivamente por los valores de las
variables proposicionales. Vemoslo en la tabla.

P q p Aq

1. a rre g lo (V , V) V
2. a rre g lo (V , F ) - ------ F
3. a rre g lo (F , V ---- F
) -
4. a rre g lo (F , * H ------- F
Debido a la existencia de este tipo de correspondencia, que
va de los valores de las variables proposicionales a los valores de
la matriz, es que, por analoga con la matemtica, se ha llamado a
las conectivas funciones. Pero como en este caso las variables no
asumen valores numricos sino slo el valor verdadero y el valor
falso, entonces para tipificarlas se les denomina funciones de
verdad. El lgico que inici el estudio de las funciones de verdad
fue Gottlob Frege en su libro escrito en alemn bajo el ttulo de
Begriffsschrift, publicado en 1879.

81
5.3. Dominio y rango

Recurriendo al concepto matemtico de funcin, sabemos que


toda funcin f ( x ) se define tomando como referencia un con
junto dominio y un conjunto rango. La funcin f es una regla que
asocia cada valor que toma la variable x en el conjunto dominio
con un nico valor en el rango. Si A es el dominio y B el rango,
la funcin f ( x ) se define mediante el esquema:

f ( x ): A => B

En el caso de las funciones lgicas o funciones de verdad


el conjunto base es W = { V, F } que tiene solamente dos objetos
distintos. Si, por ejemplo, la conjuncin la representamos como
funcin, escribimos f l ( p, q ) y podemos definirla as:

fl(p,q):WxW=> W

Hemos tomado como dominio el producto cartesiano de W x


W (W2) debido a que f 1 es una funcin que tiene dos variables y,
por tanto, asocia, cada vez, un par ordenado (arreglo) con un ni
co valor del rango. Esta definicin puede aplicarse a todas las
conectivas de dos variables. La negacin, que podramos repre
sentarla como la funcin f 0 ( p ) sera la nica funcin, que por
tener una sola variable, puede definirse simplemente de W en W
como a continuacin:

f 0 ( p ) : W => W

Los grficos conjuntistas de f l y fO se representan as:


W2 W W W

A f 1 se le llama funcin suryectiva y a fO se le llama funcin


biyectiva.

82
Definicin 9. Las frmulas conjuntivas, disyuntivas, negativas, condicionales y
bicondicionales son funciones de verdad definidas tomando como base el conjunto
W= { V,F} debido a que los valores de sus matrices son determinados de manera
nica y exclusiva por los valores de sus variables proposicionales componentes.

5.4. P rop osicion es atm icas y m olecu lares

Las conectivas lgicas que hemos introducido en las secciones pre


cedentes constituyen, junto con las variables proposicionales, los
elementos bsicos del lenguaje de la lgica proposicional desarro
llada en este curso. A partir de ellos construiremos proposicio
nes ms complejas, analizaremos la correccin lgica de argumen
tos o razonamientos y esbozaremos alguna aplicacin de la lgi
ca a la solucin de problemas tecnolgicos.
Antes de continuar, es necesario que sobre la base de lo hasta
aqu dicho demos una clasificacin muy sencilla de las proposi
ciones que las divide en atmicas y moleculares. A las primeras
tambin se les denomina simples y a las segundas compuestas.

Definicin 10. Se dice que una proposicin es atmica cuando no contiene entre
sus signos a ninguna conectiva proposicional y puede ser representada slo por una
variable proposicional p.
V________________________________________________________ /
Definicin 11. Se dice que una proposicin es molecular cuando entre sus signos
contiene al menos una conectiva proposicional.

En armona con las definiciones anteriores, todas las propo


siciones para las cuales hemos construido tablas de verdad son
moleculares, pues ellas contienen necesariamente una conectiva
proposicional. En cambio, las proposiciones siguientes, debido a
que todas carecen de conectivas proposicionales, son atmicas.

c. La pizarra es verde.
d. ( a + b )2 = a2 + 2ab + b2
e. (a + b ) + c = (a + b ) + c
f. Juan es hermano de Enrique.
g. Luna estu entre Tr'ujulo cu.

83
Existe la creencia errnea de considerar que una proposicin
atmica debe tener un slo sujeto. Los ejemplos g y f demuestran
lo contrario a nivel del lenguaje natural, en el sentido de que, des
de el punto de vista lgico, Lima, Trujillo e lea son tres nombres,
que tienen la misma jerarqua en una relacin, en este caso, tridica.
Lgicamente, ninguno es predicado. En estos ejemplos los predi-
cados son '...entre ...'y y hermano de ...'.
Asimismo, a modo de ejemplo, daremos un razonamiento
para luego sealar las proposiciones atmicas que lo constituyen
y las variables proposicionales que pueden representar a cada una
de ellas.
O los pjaros estn trinando o la beb est llorando. Si lar beb no
est llorando, el viento no est soplando. O los pjaros no trinan o el
viento no sopla. En consecuencia, si la beb no est llorando, los pjaros
no estn trinando.
Las proposiciones atmicas que constituyen este razonamien
to son:

Los pjaros estn trinando ( la representamos por p ).


La beb est llorando ( la representamos por q ).
El viento est soplando ( la representamos por r ).

Evidentem ente, toda proposicin negativa es tam bin


molecular de acuerdo con la Definicin 11. Las proposiciones
moleculares pueden ser claramente distinguidas porque contie
nen conectivas y estn separadas por punto y seguido.

5.5. P rop osicion es l g icam en te equ iv alen tes

Para exponer el concepto de proposiciones lgicamente equiva


lentes o de frmulas lgicamente equivalentes es aconsejable
pasar al examen directo de una tabla de verdad que facilite el ac
ceso a definiciones generales. Tomaremos como ejemplo la tabla
de verdad de una proposicin bicondicional cuyos componentes
son proposiciones moleculares.
En Id tabla anterior el primer miembro dei bicondicional es
una conjuncin con los componentes negados y el segundo miem

84
bro es una disyuncin negada. La matriz principal, que corres
ponde a la conectiva bicondicional, es siempre verdadera (tauto
loga) debido a que la matriz del primer miembro asigna a cada
arreglo del margen los mismos valores que la matriz del segundo
miembro. Esta igualdad de valores en sus matrices significa que
la frmula del primer miembro es lgicamente equivalente a la
frmula del segundo miembro. Por tanto, la tabla prueba que la
frmula ( ~ p a ~ q ) es lgicamente equivalente a ~ ( p v 7 ). Otra
manera de expresar el mismo concepto es decir: si una frmula
bicondicional es verdadera en todos sus arreglos (tautologa), en
tonces sus dos miembros son entre s proposiciones o frmulas
lgicamente equivalentes.
Probar que dos frmulas son lgicamente equivalentes es
muy importante dentro del estudio de la lgica y cada que lo
hacemos, como en la tabla anterior, tenemos lo que en breve se
denomina una equivalencia. Esta permite que podamos reem
plazar una frmula por la otra cada vez que lo consideremos
necesario. En la prctica, una equivalencia es una frmula
bicondicional tautolgica que funciona como una regla que
autoriza a transformar una frmula en otra. Como referencia
histrica, sealamos que el ejemplo de nuestra tabla se conoce
como equivalencia o tautologa de De Morgan (DM) en home
naje a Augusto De Morgan, uno de los fundadores de la Lgica
matemtica.

5.6. T ra d u cci n de la p ro p o sici n b ic o n d ic io n a l a u na


co n ju n ci n de dos c o n d ic io n a le s .

Para facilitar el manejo del lenguaje de la lgica proposicional de


PM se usa con frecuencia una equivalencia que posibilita el re
emplazo de una frmula bicondicional ( p <- q ) por la conjun
cin de dos condicionales ( p - q ) a ( q ~ > p ).
Este procedimiento, dentro del lenguaje natural o coloquial,
se usa en la seccin 5.1 de esta leccin con los ejemplos a y b. En
este caso lo haremos dentro del lenguaje formalizado y para ello
construiremos una tabla de verdad que nos permite demostrar
que la equivalencia antes mencionada existe.

85
p q ( P <-> q ) <-> ( ( p - > q ) A q - > p ) )

V V V V V V
V F V F F V
F V V V F F
F F V V V V
T 4
MATRIZ DEL MIEMBRO MATRIZ MATRIZ DEL MIEMBRO
IZQUIERDO PRINCIPAL DERECHO

Como se observa, la frmula tabulada muestra una matriz


principal siempre verdadera lo que prueba que sus miembros son
frmulas lgicamente equivalentes y pueden reemplazarse mu
tuamente. Resulta pertinente destacar que el primer miembro de
esta frmula es a su vez una frmula bicondicional pero no una
equivalencia en tanto que es claro que ( p ) no es una tautolo
ga. Lo que es una equivalencia es la frmula total.

5.7. Bicondicional y definicin

La conectiva bicondicional, en la medida que posibilita construir


equivalencias, es utilizada para construir reglas de traduccin de
un lenguaje lgico a otro, como ocurre con los diccionarios bilin
ges. Como miembro izquierdo o entrada se escribe la frmula
que queremos definir y que pertenece a un lenguaje L que no es
el nuestro. Como miembro derecho se escribe la frmula equiva
lente a la primera y que pertenece a nuestro lenguaje L1. De esta
manera se establece una regla que permite traducir una frmula
de L a otra de L1. El bicondicional en su conjunto es propiamen
te una definicin. Al miembro izquierdo se lo denomina defi-
niendum y al miembro derecho definiens.
Si suponemos, verbi gracia, que L es el lenguaje de PM y que
nuestro lenguaje L es una versin, reducida en el nmero de
conectivas, que contiene solamente las de conjuncin ' A ' y nega
cin entonces las definiciones que son reglas de traduccin de
las frmulas de L a las de L son:

86
( p v q) f > ~ ( ~ p A ~ q )
(P ~ > q) * * ~ ( P A ~ q )
( p * q) 0 ~ ( p A q ) A ~ ( ~ p A ~ q )
( p o q ) f>~(pA~q)A~(~pAq)

(Ejercicio: construir las tablas correspondientes a las cuatro


equivalencias anteriores).

87
VI
JERARQUA DE LAS FRMULAS
DEL LENGUAJE DE LA LGICA
PROPOSICIONAL

Objetivos:
Conocer y aplicar las reglas de formacin de frmulas.
Definir y aplicar en el lenguaje formalizado el concepto de
Jerarqua.
Reconocer el carcter convencional de los sistemas de jerarqua.
Aplicar notaciones alternativas para jerarquizar frmulas
Establecer el orden de operaciones en la resolucin de tablas de
verdad.
CUESTIONARIO 6
Jerarqua de las frmulas
del lenguaje de la lgica proposicional

Instrucciones

I. Cules de las siguientes expresiones son frmulas de PM ?

I . (p-> (q a r ) ) ~ ( ~ q v ~ ( ~ r ) )
2- ( P -> q -> ) P
3. ~ p ( q v r ) p
4- ( P v q ) ->(p A q )
5- ( ( P A <1 ) Ar) A ~ S A p
6. p ( ~ ( ~ ( ~ q )))

II. Las siguientes frmulas estn ordenadas o jerarquizadas por el


mtodo de los puntos. Ordnalas manteniendo las mismas je
rarquas por el mtodo de los parntesis.

1 .p o q /v p e q

2. p o q-qv~p

3. p - q v r - s

4. p -> q - > r

5. p > q > r v p > r v q

6. p - q -q-r-p-r

[91]
7 p-~q^q -~p

8. p -> q a r . A . p - q v p - r
III. Ordenar por el mtodo de los puntos las siguientes proposicio
nes de tal manera que la jerarqua no se altere.

1. ( p v q ) A ( r v p )
2. ( p v q ) A ( r v - p )
3. p > ( q a ( t a - s ))
4. ( p - > q ) v ( ( r - > s ) a ( s v p ))
5- ( ( p - > q ) v s ) A ~ r

IV. Construir tablas de verdad para las proposiciones de los gru


pos II y III.

V. Usando el mtodo de los parntesis proceder a negar las si


guientes proposiciones:

1. p - > q
2. p * q
3. p v ~ q
4 . p > ~ q
5. ( p q ) > ( q p )
6. ( p - > q ) > ( ( p - > q ) v r )

VI. Responda a las siguientes preguntas:

1. Por qu es necesario establecer una jerarqua entre las


conectivas?
2. Son equivalentes dos proposiciones que tienen las mismas
variables componentes, las mismas conectivas, pero diferente
jerarqua?
3. En qu consiste el mtodo mixto de jerarquizacin?
4. Existen slo los mtodos de jerarquizacin aqu estudiados o
cualquier persona podra proponer y usar otro?
5. Qu significa decir que un mtodo es convencional?

92
LECCIN 6
Jerarqua de las frmulas del lenguaje de
la lgica proposicional

6.1. Lenguaje natural versus lenguaje formalizado

Tomando como punto de partida nuestro conocimiento del len


guaje natural, comenzaremos esta seccin enfatizando lo impor
tante que es el uso de los signos de puntuacin, para precisar el
sentido de aquello que deseamos comunicar. Son conocidos ejem
plos tales como:

a. Mientras dorman, los centinelas vigilaron el campamento


b. Mientras dorman los centinelas, vigilaron el campamento.

Como se aprecia entre los ejemplos a y b hay una diferencia


sustantiva de sentido. En el primer caso, se entiende que fueron
los centinelas los que realizaron la vigilancia. En cambio, en el
segundo, fueron otras personas las que presuntamente vigilaron
mientras los centinelas descansaban. Es importante percibir que
a y b son conjuntos de palabras que tienen exactamente los mis
mos elementos pero que, sin embargo, dicen cosas distintas debi
do a que estn ordenadas o jerarquizadas de manera distinta por
la coma. Esto significa que para entender castellano no es sufi
ciente conocer el uso de las palabras. Se necesita tambin conocer
las reglas que rigen su ordenamiento, las mismas que definen
una jerarqua. Entre tales reglas son muy importantes ias refe
rentes al uso de los signos de puntuacin.

[93]
Una diferencia notable entre los lenguajes lgicos y los len
guajes naturales es que estos tienen una cantidad, en trminos
comparativos, muy grande de palabras en relacin con los len
guajes formalizados. Un lenguaje formalizado, como el tipo PM
que estamos desarrollando, cuenta propiamente con slo seis fr
mulas bsicas. Ellas son las frmulas de negacin, conjuncin,
disyuncin inclusiva, disyuncin exclusiva, condicional y
bicondicional. Asimismo, hay muchos lenguajes para la lgica
proposicional que usan slo dos frmulas bsicas. Entre ellos el
lenguaje de Post usa solamente las frmula de conjuncin y nega
cin. En el caso lmite, hay lenguajes que usan slo una frmulas
bsica. Un ejemplo lo brinda el lenguaje de Nicod que usa slo la
frmula de incompatibilidad.
La otra diferencia es que los lenguajes formalizados, propia
mente, no jerarquizan sus frmulas mediante signos de puntua
cin sino a travs de signos de agrupacin, que establecen el or
den en que debe ejecutarse las operaciones que la frmula define.
Como veremos los signos de agrupacin ms usuales en los len
guajes lgicos son los parntesis, sin embargo ello no obsta para
usar otras convenciones, que bien pueden ser puntos, que tratn
dose de frmulas lgicas tambin funcionan como signos de agru
pacin que definen un orden de operaciones.

6.2. Reglas de formacin de frmulas

En los lenguajes lgicos las frmulas, a diferencia de las oracio


nes en los lenguajes naturales, se construyen en base a reglas de
formacin muy precisas y todas ellas pueden ser interpretadas
corno proposiciones o afirmaciones.
Ahora es oportuno hacer una aclaracin. Hasta el momento
hemos usado de manera casi indistinta los trminos 'frmula' y
'proposicin'. En un curso introductorio ello no entraa mayores
dificultades, pero es importante sealar que en el trabajo especia
lizado, existe una diferencia sustantiva entre ambos conceptos.
Por ahora, basta sealar que las proposiciones, que hemos usado
en este libro, generalmente han sido expresadas en un lenguaje
natural o en lenguaje matemtico, en cambio las frmulas sola
mente en el lenguaje PM. Las frmulas estn sujetas al concepto

94
de prueba o deduccin, mientras las proposiciones al concepto
de verdad. Asimismo, todas las frmulas que hemos usado hasta
el momento pueden ser interpretadas como proposiciones, pero
una cosa es una frmula y otra su interpretacin. Por ahora, deja
mos anotado que los sistemas lgicos frecuentemente se desarro
llan como conjuntos de frmulas sin interpretar, lo que no impide
que a nivel elemental sigamos hablando indistintamente de fr
mulas y proposiciones.
Si mantenemos la convencin de llamar al lenguaje pro-
posicional que estamos exponiendo, lenguaje PM, las reglas de
formacin o construccin de frmulas son las siguientes:

rl. Si p es una variable proposicional entonces es una frmula


de PM.
r2. Si p es una frmula de PM, entonces ~ p es tambin
una frmula de PM.
r3. Si p y q son frmulas de PM, entonces tambin p a q,
p v qf p ^ *?/P *7 Y P * * q sonfrmulas de PM.
r4. (Regla de cierre) Solamente son frmulas de PM aquellas
expresiones construidas por la aplicacin de r l, r2 o r3.

Las reglas anteriores son metalingsticas y definen a PM


como un lenguaje de estructura predeterminada y cerrada. Lo
primero debido a que las reglas preven la estructura de toda po
sible frmula de PM y lo segundo porque se excluye que cual
quier otra frmula, obtenible por la aplicacin de reglas distin
tas, sea considerada como elemento o miembro de PM.
Por otro lado rl, r2, r3 y r4 constituyen lo que se conoce como
una definicin recursiva de 'frmula' en el lenguaje PM y, a su
vez, son un algoritmo que permite decidir mecnicamente si una
expresin cualquiera es o no una frmula d PM.

6.3. La jerarqua en el lenguaje P M

La existencia de dos o ms conectivas plantea la necesidad de es-


lauiCLCi una jc ia iq u ia c ilc caacjo y p a ia cuu caolcl ic^iab que: nu

son estrictamente novedosas porque han sido tomadas de la ma


temtica. Consecuentemente, esta explicacin partir de la expe-

95
rienca que gana un estudiante en cualquiera de sus cursos de
matemtica, por ejemplo, el de lgebra elemental.
Supongamos que alguien escribiera en la pizarra la siguiente
expresin

3x + 2 - 5 . 4 - 7x = 8

y luego nos pidiera que la calculemos. La respuesta inmediata es


que no se puede hacer ningn clculo en tales condiciones, pues
no se ha establecido ninguna jerarqua entre las operaciones;
de tal modo que no est claramente determinado por cul co
menzar y por cul terminar. Esta es la razn por la que un pro
fesor de matemtica, cuando quiere que se haga un clculo pre
ciso, presenta la expresin anterior, por ejemplo, de la siguiente
manera:

3 (x + 2 ) - 5 ( 4 - 7 x ) = 8

Entonces ahora s est unvocamente establecido que la ope


racin de mayor jerarqua es la resta que est al centro del pri
mer miembro de la igualdad y que la suma y la resta que estn
dentro de los parntesis tienen menos jerarqua que los pro-ductos
establecidos fuera de los parntesis. As podemos decir que el
primer miembro de la igualdad es una diferencia de dos produc
tos y ningn estudiante dira que 5 ( 4 - 7x ) es una resta sino un
producto, porque sabe que toda operacin exterior a los parn
tesis tiene ms jerarqua que toda operacin interior a los pa
rntesis y que la operacin de mayor jerarqua es la que da nom
bre a una expresin.

6.4. Jerarqua y tablas de verdad

En el lenguaje lgico puede producirse una situacin similar a la


anterior. Valindose del vocabulario que hemos introducido, es
posible escribir una expresin tal como:

p v p > q

96
que no puede ser tabulada porque hay dos conectivas y rio se sabe
cul es la de mayor jerarqua para establecer un orden en la cons
truccin de la tabla. Valindonos del mtodo de los parntesis,
usado en la matemtica, podemos darle la siguiente presentacin
a la operacin anterior:

p v (p q)

De esta manera hemos establecido una jerarqua o un orden


que, siguiendo la regla general de uso de parntesis, nos indica
que la conectiva principal es la disyuncin porque est fuera del
parntesis y que la de menor jerarqua es el condicional porque es
interno al parntesis. sto significa adems que si confecciona
mos la tabla de verdad habr dos matrices: una secundaria, que
ir debajo de la conectiva de menor jerarqua, y una principal,
que ir debajo de la conectiva de mayor jerarqua y que tiene la
condicin de ser el resultado final de la tabla de verdad.
Como el acto de escribir una matriz debajo de la conectiva
puede ser entendido como una operacin, entonces podemos de
cir sin dificultades que cada una de las conectivas estudiadas de
termina una operacin lgica. En este mismo sentido, podemos
decir que la jerarqua que establecen los parntesis en una propo
sicin determina de manera inequvoca un orden de operaciones
que va desde las operaciones de menor jerarqua hasta finalizar
en la de mayor jerarqua. Antes de construir la tabla de verdad,
de la proposicin que hemos ordenado o jerarquizado mediante
parntesis, sealaremos a travs de la siguiente figura el orden de
operacin a seguir:
Observando el orden de operaciones anterior, la tabla resul
ta as:

pv(p-^q) MATRIZ
SECUNDARIA
ra.

2 da<_ MATRIZ PRINCIPAL

97
6.5. Ocurrencias de una variable proposicional

La matriz secundaria es el resultado de confrontar los valores de


p y de q de acuerdo a la definicin que corresponde al condicio
nal. Despus se ha confeccionado la matriz principal que es el re
sultado de confrontar los valores de p con los valores de la ma-*
triz secundaria, de acuerdo a la definicin que le corresponde a
la disyuncin inclusiva. Es claro que el margen tiene slo cuatro
arreglos porque la proposicin tabulada tiene slo dos variables
proposicionales distintas que son p y q. Lo que sucede es que p
tiene dos apariciones que en lgica se llaman ocurrencias de una
variable proposicional, pero sigue siendo la misma variable. Es
ms, una misma variable puede tener muchas ocurrencias en la
misma proposicin pero se la cuenta siempre slo como una.
Antes de dar otro ejemplo aadiremos como una regla que la
conectiva de negacin, cuando est delante de una variable
proposicional, es siempre el operador de menor jerarqua. Po-
dentos verlo en la siguiente proposicin:

( p v ~ q ) > p

en la que la negacin tiene menor jerarqua que la disyuncin in


clusiva y, consecuentemente, al hacer la tabla, la primera matriz
que se calcula es la de la negacin. La tabla es como sigue :

98
p q (p. v ~ q ) -> P
V V V F V 3.a MATRIZ
V F V V V < (Principal o final)
F V F F V
F F V V F 1.a MATRIZ
t 1
t 2.a MATRIZ
Tabla 8
En este caso el orden de operacin ha sido el siguiente. Pri
mero se obtuvo la matriz de la negacin, aplicando la Definicin 6
a los valores de q. Luego se obtuvo la segunda matriz, aplicando
la Definicin 4 a los valores de p y a los valores que constituyen la
primera matriz.
Es evidente que el primer valor de p se coteja con el primer
valor de la primera matriz y as sucesivamente. Finalmente, la
matriz principal se obtuvo aplicando la Definicin 7 a los valores
de la segunda matriz y a los valores de p.

Definicin 12. Una proposicin dada en el lenguaje lgico est bien escrita si y slo
si existe una jerarqua claramente establecida entre sus conectivas. A esta jerarqua
tambin se le llama un orden. Asimismo, una proposicin mal escrita carece de sentido.

D efinicin 13. En una proposicin dada en lenguaje lgico, si se ha establecido


un orden mediante parntesis, entonces toda conectiva interna a los parntesis es
de menor jerarqu a que toda conectiva externa a los parntesis. La conectiva que
se encuentra ms externa a los parntesis es la de mayorjerarqu a y la que le da
nombre a la proposicin.

De acuerdo con estas definiciones, las dos proposiciones an


teriores estn bien escritas y, teniendo en cuenta cules son sus
operadores ms externos, ocurre que la primera se llama proposi
cin disyuntiva y la segunda proposicin condicional.

6.6. Reglas auxiliares sobre jerarqua

se deduce de la Definicin13,
La primera regla adicional se refiere a la negacin. Aunque ella
sin embargo,, nosotros la haremos
explcita para facilitar el aprendizaje.

99
R4. Cuando se desea negar una proposicin compuesta o
molecular se la encierra dentro de parntesis y se le antepone el
signo En este caso la conectiva de negacin no niega los valo
res de una variable proposicional sino los valores de la matriz
que est debajo de la conectiva de mayor jerarqua que se encuen
tra dentro del parntesis. Asimismo, de acuerdo con la Definicin
13, estas proposiciones tomadas en su conjunto tienen como
conectiva principal a la negacin y son proposiciones negativas.
Por ejemplo, si se desea negar la proposicin p <-> q, entonces la
respectiva proposicin negativa es ~ ( p q ). De acuerdo a lo
anterior, la tabla de verdad de esta proposicin es la que aparece
en esta pgina como Tabla 9.

p q ~ (p <H>q)
V V F V
V F V F
F V V F MATRIZ PRINCIPAL
F F F
t V Tokio O
1

R5. En ausencia de parntesis y de todo signo de jerarqua,


puede considerarse por convencin que la conectiva de mayor
jerarqua es el bicondicional; en segundo lugar el condicional; en
tercer lugar la disyuncin exclusiva; en cuarto lugar la disyun
cin inclusiva; en quinto lugar la conjuncin; y en sexto lugar la
negacin. Esto significa lo siguiente:

p q -> r se lee como si fuera p <->( q -> r)


p q ^ r se lee como si fuera p ( q * r )
p v q ^ r se lee como si fuera ( p v q ) j = r
p a q v r se lee como si fuera ( p a q ) v r

Consecuentemente, la proposicin siguiente, bastante ms


compleja que las anteriores,

pAq^r-r^pvq

100
se le e c o m o s i fu e r a

( ( p Aq ) - M - ) < - > ( r - > ( p v q ) )

A pesar de que esta regla es una convencin bastante usa


da por los lgicos, nosotros en este texto la usaremos lo menos
posible.

6.7. Los p u ntos com o signos de jerarq u a

R 6 . Se puede determinar la jerarqua de las conectivas reempla


zando algunos o todos los parntesis por puntos. La conectiva que
tiene el mayor nmero de puntos es la de mayor jerarqua. Si se
usan puntos y parntesis, entonces se cumple que toda conectiva
interna a los parntesis, aunque tenga puntos, es de menor jerar
qua que otra externa a los parntesis aunque no tenga puntos.
La proposicin anterior, ordenada por el mtodo de los pun
tos, queda de la siguiente manera:

p A q ^ ro r^ p v q
Si usamos tantos puntos como parntesis, entonces tenemos:

( P a q -> r ) ( r -> p v q )

En este caso, aunque el bicondicional no tiene puntos es de


mayor jerarqua que los dos condicionales que tienen puntos por
que estn internos en los parntesis.
El mtodo mixto de usar puntos y parntesis tiene su origen
en la obra de Whitehead y Russell titulada Principia Mathematica
y es empleado por muchos autores. Nosotros lo indicamos para
que el estudiante lo conozca, pero slo usaremos los parntesis y
cuando sea necesario las convenciones establecidas por la regla
R5.
Aclaramos que los mtodos de ordenar proposiciones, as
como muchos otros aspectos relacionados con los signos lgicos,
son completamente convencionales. Dcp Uj. L.C1Li.d C\_lid dj C.CiUD
especialistas. Nosotros hemos elegido uno que nos parece ade
cuado, pero con igual xito puede seguirse otro.

101
6.8. Tabla de verdad de las proposiciones con ms de dos
variables

En este caso la mayor variante est en el margen, pues al haber tres


proposiciones entonces la frmula '2n' para 'n=3' da lugar a 8 arre
glos que es el valor de dos al cubo. A continuacin daremos un
ejemplo sealando previamente el orden de operaciones estableci
do por los parntesis para luego construir la tabla de verdad.

(( P AJ I ) V ( p A r ) ) <-> ~ p
3 .a matriz 2 .a matriz 1 .amatriz

4.a matriz
i-------------v-------------
5.a matriz (principal)

La tabla de verdad es como sigue:

p q r ((P a q) v (p a r )) <-> ~ P
V V V V V V T F
V V F V V F F F
V F V F V V F F
V F F F F F V F
F V V F F F F V
F V F F F F F V
F F V F F F F V
F F F F F F F V
T
Tabla 10 MATRIZ PRINCIPAL

Otro ejemplo de tabla de verdad deuna proposicin con tres


variables es:

102
p q r ( ( p v ~ p ) - > ( q A ~ q ) ) A r

V V V V F F F F F
V V F V F F F F F
V F V V F F F V F
V F F V F F F V F
F V V V V F F F F
F V F V V F F F F
F F V V V F F V F
F F F V V F F V F
4
MATRIZ
Tabla 11 PRINCIPAL
VII
TAUTOLOGA, PRINCIPIOS LGICOS
Y VALIDEZ

Objetivos:

Reconocer frmulas tautolgicas consistentes y contradictorias.


Diferenciar las frmulas de las proposiciones tautolgicas.
Identificar los principios lgicos clsicos.
Distinguir el concepto de frmula lgicamente vlida del de
frmula tautolgia.
Diferenciar la validez lgica del contenido informativo.
CUESTIONARIO 7
Tautologas principios lgicos y validez

Instru cciones

I. Cules de las siguientes frmulas son tautologas, cules con


sistentes y cules contradictorias?

1. ( pAq) ->p
2. (p v q ) -> ( p a q )
3 ( p - > q ) - > ( ( p - > q ) v (r->p))
4. ( p -> q ) ( ~p v q )
5-~((p->q)v(q->p))A(r->p)
M ( p - > q ) Aq ) - > p

II. Determine a qu frmulas tautolgicas corresponden los siguien


tes razonamientos:

1. Si existe un libro, entonces existe un lpiz o un libro.


2. Dadas dos proposiciones distintas, ocurre que la segunda se deduce de
la primera o la primera de la segunda.
3. Una proposicin implica que cualquier otra proposicin la implica.
4. Si desde una proposicin se deduce una contradiccin, entonces esta
proposicin debe ser rechazada.
5. Una proposicin de la que se deduce su propia negacin debe ser
rechazada.

III. Determine cules de las siguientes afirmaciones verdaderas no


son tautolgicas y cules son atmicas.

[107]
1. Si n es un nmero natural, entonces n tiene un sucesor.
2. Todo nmero natural es par o impar.
3. Todo nmero es igual a s mismo.
4. El nmero tres es mayor que cero.
5. No es posible que el mismo nmero sea par e impar.
6. No existe un nmero entero positivo que sea mayor que todos los otros.

IV.Determinar si las siguientes proposiciones son equivalentes.

1. pv~q y ~ ( p A q)
2- (p q) y (~ pv q)
3. ( p v ( q Ar)) y ( ( P v q ) A ( P v r ))
4- ( P - ~ q ) y ( q - > p)
-Q

5. (p e q ) l

l
&
>

<

<
y

V. En lo que sigue usamos las abreviaturas V para verdadero y F


para falso. Completar correctamente las siguientes afirmacio
nes usando las mismas abreviaturas.

1. Si un condicional tiene el antecedente igual a F, luego, sin importar el


valor del consecuente, el condicional es necesariamente....
2 . Si un condicional tiene su consecuente igual a V, entonces el condicio
nal completo es igual a.....
3. Si la primera variable de una conjuncin es F, entonces la conjuncin
es.......
4. Un condicional con consecuente F para ser verdadero debe tener como
antecedente......
5. La negacin del principio de no contradiccin da una matriz con slo
valores........
6. Una disyuncin inclusiva que tiene una variable V es necesariamen
te.....
7. Si se pone una disyuncin exclusiva como antecedente de un condicio
nal y una inclusiva como consecuente, luego l condicional en el pri
mer arreglo ser.....
8 . Si p es V, luego la negacin de la negacin de p es...
9. Si ~ p es F, luego la negacin de ~p es.....
10. Si la negacin de la negacin de p es F, luego p es....

108
VI. Responder las siguientes preguntas:

1. Cuntos arreglos tendr la tabla de verdad de una proposicin


con cinco variables?
2. Qu es una implicacin?
3. A qu se denomina lgica polivalente?
4. Cmo se define el espacio lgico?
5. Qu diferencia existe entre sinonimias y equivalencias lgicas?
6. Qu significa afirmar que las tautologas abren el espacio lgico?
7. Por qu las contradicciones cierran el espacio lgico?
8. Por qu las tautologas, no transmiten informacin especfica
alguna?

109
LECCIN 7
Tautologas, principios lgicos y validez

7.1. F rm ulas tau tolgicas, consisten tes y contradictorias

Las tablas de verdad que hemos construido hasta ahora son sufi
cientes para permitimos comprender que los valores de la ma
triz principal de una frmula se encuentran necesariamente en
una de las tres siguientes condiciones: 1) todos sus valores son
verdaderos, 2) algunos de sus valores son verdaderos y algunos
son falsos; y 3) todos sus valores son falsos. Las frmulas cuya
matriz principal se encuentra en la primera condicin se llaman
tautologas. (Ver tabla 7). Las frmulas que poseen una matriz
principal que se encuentran en la segunda condicin se denomi
nan consistentes. Entre otros ejemplos pueden verse las tablas 8,
9 y 10. Las frmulas cuya matriz principal se encuentra en la ter
cera condicin se denominan contradicciones. Un ejemplo nos lo
proporciona la frmula de la tabla 11.

7.2. F rm ulas tau tolgicas vs. p roposiciones tautolgicas

Hasta esta seccin hemos hablado indistintamente de proposicio


nes tautolgicas y de frmulas tautolgicas, aunque hemos ad
vertido que no es el mismo concepto. El tema que ahora tratare
mos requiere que precisemos un criterio de diferenciacin y lo
haremos a partir de ejemplos.

[111]
a. p v ~ p
b. Lucas es casado o no es casado
c. Mercedes es silenciosa o no es silenciosa

El ejemplo a es claramente una frmula siempre verdadera o


tautolgica como puede establecerse, fcilmente, construyendo
la correspondiente tabla de verdad. Y decimos que a es una fr
mula porque el significado de la proposicin que est represen
tada por la variable proposicional p est indeterminado. Terica
mente, cualquier proposicin puede ocupar el lugar de p, lo que
equivale a decir, ninguna en especial.
Los ejemplos b y c si son genuinas proposiciones tautolgicas.
Son proposiciones porque, claramente, tiene cada una un signifi
cado especfico que es un prerrequisito para poder calificar a una
proposicin de verdadera, pues no podemos hablar de la verdad
de una proposicin cuyo significado ignoramos porque ello equi
valdra a hablar de la verdad de lo que desconocemos, y son
tautolgicas porque tienen la forma de la frmula tautolgica
p v ~ p , sin embargo b y c son dos proposiciones semnticamente
distintas. En efecto, una cosa es hablar del estado civil de Lucas y
otra del temperamento o carcter de Mercedes.
Una frmula como la del ejemplo a es fundamentalmente una
forma o estructura que admite reemplazos en sus lugares en blan
co, que son los ocupados por p; y que tiene la propiedad de que
todos sus reem plazos posibles dan lugar a proposiciones
disyuntivas verdaderas. Como esto es as, abreviadamente, se
acostumbra a decir que la frmula p v ~ p es siempre verdadera,
pero lo riguroso sera afirmar que todos sus reemplazos son ver-
daeros. Es ms, en la medida que toda la funcin que cumple la
variable p se reduce a la de representar un espacio a ser ocupado
por una proposicin, podemos usar otro artificio que cumpla la
misma funcin. Por ejemplo, si usamos puntos suspensivos y pa
rntesis que representan la nocin de espacio en blanco, la fr
mula a puede ser reemplazada, sin prdida alguna, por;

( ................... ) v - ( ....................... )

112
7.3. L im itacion es en la tran sform aci n de proposiciones
tau tolgicas

Una ventaja sustancial del ejemplo a sobre los otros dos es que
todas las transformaciones que hagamos sobre l, aplicando equi
valencias lgicas, dan lugar a frmulas que son, igualmente, siem
pre verdaderas (tautologas). En cambio, si trabajamos con el ejem
plo b y hacemos reemplazos usando lo que seran equivalencias
en castellano, encontraremos que algunas transformaciones po
dran ser falsas. Si definimos 'casado7 como 'no soltero7, y hace
mos el reemplazo correspondiente en el segundo componente de
la disyuncin b, obtendremos lo siguiente:

b'. Lucas es casado o no no soltero.

Y ocurre que V puede ser una afirmacin falsa si sucede que


Lucas es, por ejemplo, viudo. Bajo tal situacin la proposicin
'Lucas es casado' ser falsa y tambin la proposicin 'Lucas es no
no soltero' pues, conociendo que en PM la doble negacin equi
vale a una afirmacin, esta ltima proposicin equivale a 'Lucas
es soltero7 que, obviamente, no puede ser verdadera si Lucas es
viudo. Como sabemos, una disyuncin, en este caso b', con am
bos componentes falsos es falsa. Consecuentemente, la transfor
macin de b a b ' es lgicamente inadmisible porque obtiene a
partir de una proposicin verdadera una falsa. Vale decir, no se
cumple el sentido fundamental de la lgica que es la transmisin
de la verdad.
Sin embargo, lo anterior no se debe a deficiencias lgicas sino
a deficiencias en el castellano. Nosotros hemos supuesto que 'ca
sado7y 'no soltero7son sinnimos y eso no es correcto. Ello debi
do a que en los lenguajes naturales la sinonimia es siempre muy
imprecisa. Por tanto, b ' no ha sido obtenida desde b por aplica
cin de reglas lgicas sino por aplicacin de sinonimias castella
nas que, por ser imprecisas, conducen a error.
De este modo queda aclarado que las proposiciones tauto
lgicas formuladas en un lenguaje natural tienen la propiedad de
no poder ser transformadas, sin restricciones, por sus supuestas
equivalentes. En cambio, las frmulas tautolgicas pueden ser

113
reemplazadas, sin limitacin alguna, por sus equivalentes. El no
conocer esta distincin conduce al error de atribuir las mismas
propiedades a las frmulas tautolgicas y a las proposiciones
tautolgicas. Por ello, en esta seccin el lector deber tener pre
sente cundo nos referimos a unas y cundo a las otras.
En lo que sigue consideraremos a las proposiciones tauto
lgicas como un caso particular o un ejemplo, en un lenguaje na
tural, de la estructura o forma que muestra la correspondiente
frmula tautolgica. Asimismo, todas las reglas lgicas proporcio
nadas en este libro se refieren, propiamente, a frmulas de PM.

7.4 Los principios lgicos clsicos

Es importante sealar que los lgicos desde Aristteles hasta el


siglo pasado acostumbraron a hablar de tres principios lgicds
fundamentales: el de identidad, el de no-contradiccin y el del
tercio excluido. Las proposiciones que en lenguaje natural corres
ponden a cada uno de estos principios, en el mismo orden en que
han sido nombrados, son las siguientes:

c. Toda proposicin es verdadera si y slo si ella misma es verdadera.


d. No es posible que una proposicin sea verdadera y falsa al mismo
tiempo.
e. Toda proposicin es necesariamente verdadera o necesariamente falsa.
No'-existe una posibilidad intermedia.

Estas tres proposiciones, introduciendo variables propo-


sicionales, podemos expresarlas as:

c'. p es verdadera si y slo si p es verdadera.


d'. No es posible que p sea verdadera y falsa a la vez.
e'. O p es verdadera o p es falsa.

Luego introduciendo las conectivas que conocemos y acep


tando que escribir la variable p sin negacin equivale a decir p es
verdadera y con negacin, a decir p es falsa, nosotros podemos tra
ducir las proposiciones, c, d y e dadas en lenguaje natural, a las
siguientes formuladas en el lenguaje lgico.

114
c". p <-> p ( Principio de identidad)
d". ~ ( P A ~ p) ( Principio de no-contradiccin)
e". p v ~ p ( Principio del tercio excluido)

Mediante el mtodo de las tablas probaremos que estas pro


posiciones son tautologas.

P P <> P
V V
F V Tabla 12

Los filsofos tradicionales magnificaron las tres proposiciones


anteriores y consideraron que eran las que tenan la jerarqua mxi
ma dentro de la lgica. Igualmente, pensaron que eran los princi
pios fundamentales de la realidad, razn por la que fueron conside
radas las proposiciones fundamentales de la Ontologa. Hoy da cual
quier aprendiz de lgica sabe que rigen el comportamiento de los
lenguajes lgicos standard, tipo PM, pero que admiten usos diferen
ciados en los lenguajes no clsicos, como los que se usan para cons
truir los sistemas polivalentes de Lukasiewicz, el sistema intuicionista
de Heyting o los sistemas paraconsistentes de Jaskowskiy Da Costa,
entre otros. El error de los filsofos y lgicos tradicionales radic en
sostener que estas proposiciones eran privilegiadas por ser las ni
cas necesariamente verdaderas de manera evidente.
La investigacin de este siglo ha probado fehacientemente que,
por ejemplo, el principio del tercio excluido no es umversalmente
verdadero. Esto se desprende de la lgica del polaco Lukasiewicz,
que admite, adems de los valores verdadero-falso, un tercer valor.
A esta lgica se le denomina polivalente para oponerla a la que es
tudiamos en este texto que se llama bivalente, porque sus variables
pueden asumir slo dos valores (verdadero, falso). Los trabajos de
Lukasiewicz son un argumento entre muchos contra la lgica tradi
cional y contra el no menos tradicional criterio de evidencia.
Es pertinente aclarar que el principio de identidad tambin
puede ser lgicamente expresado mediante un condicional que
tiene como antecedente y consecuente a la misma variable.

115
Es muy sencillo probar que esta frmula es una tautologa.
(Dejamos como ejercicio para el estudiante la construccin de la
tabla de verdad correspondiente).

7.5. La validez lgica

Las tautologas son frmulas que han llamado la atencin de los


filsofos y de los lgicos por tener la peculiaridad de ser siempre
verdaderas. Por esta razn se les denomina frmulas lgicamen
te verdaderas o lgicamente vlidas.
En ellas no resulta decisivo el valor concreto que asuman las
variables proposicionales componentes, pues el valor verdadero
corresponde de la misma manera a todos los arreglos. De esta
suerte, las tautologas, originan una situacin muy similar a la de
las funciones constantes en matemtica. Por ejemplo, la funcin
'y = x ' siempre da lugar a 'y = V para todos los valores enteros
de V , pues es conocido que una potencia de grado cero es igual
a 1, cualquiera que sea el nmero entero de la base. De manera
anloga, una tautologa es una funcin de verdad constante que
siempre toma el valor verdadero sea cuales fueren los valores de
sus variables componentes. El estudio de esta clase de funciones
de verdad constituye la tarea fundamental de los sistemas de l
gica proposicional, en tanto ellas son un instrumento poderoso
para el anlisis de los argumentos dados en el lenguaje natural y
en los lenguajes formalizados, como es el caso de la matemtica.

Definicin 14. Si una proposicin formulada en el lenguaje de lgica proposicional


es-una tautologa, entonces se dice que es una proposicin lgicamente verdadera
o lgicamente vlida,
y

Algunos autores llaman a las proposiciones lgicamente v


lidas, simplemente, vlidas. Otros ponen nfasis en el hecho de
que el mtodo de las tablas es puramente formal y se realiza con
total independencia de la observacin de los hechos, razn por la
que a las tautologas las llaman proposiciones analticamente
verdaderas o verdaderas a priori.

116
7.6. Tautologas vs. frmula lgicamente vlida

Durante las dcadas de 1920 y 1930 muchos filsofos y lgicos, en


tre ellos L. Wittgenstein, usaron los conceptos de afirmacin lgi
camente vlida y de tautologa como sinnimos. Sin embargo, este
uso gener dificultades cuando se us el lenguaje de PM para for
malizar teoras matemticas que presuponen la existencia de con
juntos infinitos actuales, las mismas que resultaron ser la mayora.
Una solucin fue denominar tautologas solamente a las frmulas
lgicamente vlidas del lenguaje proposicional y a las de los len
guajes predicativos cuyas frmulas sean traducibles a las del len
guaje proposicional. Pero a las frmulas de los lenguajes predica
tivos que son siempre verdaderas y que no son reducibles a fr
mulas del lenguaje proposicional de PM se les denomin slo l
gicamente vlidas o lgicamente verdaderas. Rudolf Camap, no
table lgico de nuestro siglo, us la expresin afirm acin L-verda-
dera para expresar esta calificacin.
La distincin anterior significa que en el uso actual el con
cepto de frmula lgicamente vlida tiene mayor generalidad
que el de tautologa. Es decir, toda tautologa es una frmula
lgicamente vlida pero no toda frmula lgicamente vlida es
una tautologa. En breve, el conjunto de las tautologas es un
subconjunto propio del conjunto de las frmulas lgicamente
vlidas.
Para ilustrar lo antes afirmado ser suficiente dar un ejemplo
intuitivo, en lenguaje natural, de una afirmacin que siempre es
verdadera pero que no es reducible a una frmula proposicional.
Es el caso de la afirmacin T ara todo objeto x, se cumple x = x'
(Todo objeto es igual a s mismo ). En efecto se trata de una afirma
cin atmica, lgicamente vlida, que no puede ser representada
simplemente por p, pues de hacerlo tendramos que aceptar que
puede tomar el valor verdadero o el valor falso. Y nuestra afirma
cin en ningn caso es falsa. Por tanto, no es expresable en el len
guaje proposicional de PM.

117
7.7. Tautologa vs. contenido informativo

En la tabla de una frmula de dos variables proposicionales, el


margen siempre contiene cuatro arreglos que son los pares orde
nados ( V, V ), ( V, F ), ( F, V ) y ( F, F ). En general, para una
frmula de n variables proposicionales, el margen contendr 2n
arreglos y cada uno de ellos ser una n-ada ordenada tal que la
primera tendr n valores verdaderos y la ltima n valores falsos.
El esquema es como sigue:

Cada arreglo del margen del esquema anterior ha sido inter


pretado como un 'mundo posible', concepto acuado por el fil
sofo Leibniz. Esto quiere decir que el primer arreglo representa
un mundo cuyos hechos convertirn en verdaderas a todas las
variables de la frmula dada y el ltimo arreglo representa un
mundo cuyos hechos convertiran en falsas a todas sus variables.
Los arreglos intermedios, que slo se sugieren, representaran
diversos mundos que convertiran en verdaderas a unas varia-
blesy en falsas a otras. Segn el filsofo Wittgenstein, el conjunto
dlos mundos posibles (arreglos) constituye el 'espacio lgico'.
En la medida que cualquier tautologa es una funcin de ver
d ad / ( pt , , p n ) siempre verdadera, significa que es igualmen
te verdadera en todos los mundos posibles sin excepcin. De esto
se deduce que una tautologa no describe especficamente mun
do alguno y, por tanto, no nos dice nada especial del llamado
mundo real en el que vivimos. Por tanto, una tautologa sera una
manera de decirnos que todos los mundos son igualmente posi
bles, el nuestro y cualquier otro racionalmente pensable. Debido
a ello Wittgenstein afirm que las tautologas "abren el espacio

118
lgico" mientras las contradicciones lo cierran al ser siempre fal
sas, lo que equivale a afirmar que ningn mundo es posible.
Lo dicho significa que las tautologas son lgicamente vli
das pero vacas de contenido informativo. Por ello no nos sirven
como afirmaciones cientficas sobre la realidad sino como re
glas de deduccin que transfieren la verdad establecida previa
mente por la investigacin cientfica. Son poderosas para orga
nizar lgicamente el conocimiento pero no para producirlo.

119
VIII
FORMALIZACIN

Objetivos:

* Definir y aplicar el concepto de formalizacin de argumentos


expresados en un lenguaje natural en funcin de frmulas de
PM.
* Comprender el concepto de postulacin.
* Definir la nocin de inferencia.
* Analizar la validez o invalidez de argumentos formulados en el
lenguaje natural.
* Conocer y aplicar el mtodo indirecto al anlisis de la vlidez
lgica de argumentos.
CUESTIONARIO 8
Formalizacin

Instrucciones

I. Responder de manera precisa a las siguientes preguntas:

1. Es el concepto de verdad sinnimo del de deduccin?


2. Es verdad que a la lgica no le interesa la semntica?
3. Cules han sido los aportes de Tarski y Kurt Godel a la lgica?
4. Cmo se define el proceso de formalizacin lgica?
5. Es el lenguaje proposicional de PM adecuado para la forma
lizacin de argumentos cientficos?
6. Cmo se denomina la forma de argumento (frmulas) que es
tablece que si el antecedente es verdadero el consecuente tam
bin lo es?
7. Cmo se denomina la forma de argumento (frmulas) que es
tablece que si una conjuncin es verdadera, entonces cualquie
ra de sus componentes tambin lo es?
8. Cmo se denomina la forma de argumento (frmulas) que es
tablece que la relacin de deduccin es transitiva?

II. Decidir por el mtodo indirecto la validez lgica de las siguien


tes frmulas.

1. ( ( p A q ) a i ) - > ( s > q )

2- ((pvq)vr) ( p A s )
3-((p-q)A ( r - s ) ) - ( ~ s - ~ p )

[123]
4. ( p - > q ) - > ( r v ( p - > q ) )
5.(p-^(qvr))-> ((q A r)A s)

III. Determinar si los siguientes razonamientos son vlidos. (Slo


debe usarse en la formalizacin la disyuncin inclusiva).

1. Si el postulado V es independiente, entonces no puede ser un teorema.


El postulado V es independiente, luego no es un teorema.
V
2. Si la geometra de Lobaclievski es correcta, entonces el postulado es
independiente. Si el postidado V no es independiente, entonces no es

V
un teorema. Por tanto; si la geometra de Lobaclievski es correcta, en
tonces el postulado no es un teorema.
3. Un nmero es inductivo o transfinito. Pero no es inductivo. Luego es
transfnito.
4. Si Gdel hubiera estado joven, entonces habra demostrado la hipte
sis del continuo. Pero Gdel no demostr la hiptesis del continuo. En
consecuencia, Gdel no estaba joven.
5. Si un pas no puede cubrir las necesidades primaras de. su gente, en
tonces no puede gastar dinero en distracciones como el ftbol. Pero si
gasta en distracciones como el ftbol, entonces los aficionados olvida
rn sus problemas mientras dure el partido. Luego, si un pas no pue
de cubrir las necesidades primaras de su gente, entonces los aficiona
dos olvidarn sus necesidades mientras dure el partido.
6. Si las leyes son justas, entonces todos son tratados de manera igual. Si
hay privilegiados, entonces no todos son tratados igual O todos son
tratados de manera igual o no lo son. En consecuencia, o las leyes no
son justas o no hay privilegiados.
7. No es posible que una persona sea privilegiada y defensora de lo justo.
Pero es privilegiada, entonces no es defensora de lo justo.
8. Si la matemtica es correcta, entonces es eficaz. La matemtica es efi
caz. Por tanto, la matemtica es correcta.
9. Si alguien impone su voluntad por la fiierza, entonces est recurrien
do a la ley del ms fuerte. Si alguien recurre a la ley del ms fuerte,
entonces se est portando como si fuera una fiera en la selva. Por tan
to, si alguien no se comporta como una fiera de la selva, entonces no
impone su voluntad por la fuerza.
10. Hitler orden la quema de libros porque crey que las ideas podan ser
quemadas. Si Hitler no tuvo xito, entonces los que hoy queman libros

124
corrern la misma suerte. O bien los que queman libros no corren la
misma suerte. O bien Hitler no orden la quema de libros. En conse
cuencia, o Hitler tuvo xito o las ideas no pueden ser quemadas.

IV. Responda a las siguientes preguntas:

1. En qu consiste la postulacin ?
2. De acuerdo con la Lgica y a la Matemtica moderna hay al
guna diferencia entre axioma y postulado?
3. Cmo se define la inferencia?
4. Cundo un argumento est adecuadamente formalizado?
5. Qu se hace para que los argumentos muestren su estructura
lgica?
6. Cundo es un argumento lgicamente vlido?
7. Asegura la validez lgica la verdad de las premisas de un ar
gumento?
8. Qu es lo que esencialmente prohibe la validez lgica de un
argumento?

V. Complete correctamente las siguientes afirmaciones:

1. Un condicional cuyo antecedente es una proposicin contradictoria es


necesariamente..........................
2. En todo argumento se postula la verdad d e ..............
3. Un argumento carece de fiabilidad lgica cuando se encuentra que las
premisas son...................................
4. Un argumento no es lgicamente vlido si hay un arreglo en el que las
premisas son..............y la conclusin e s ...........

125
LECCIN 8
Formalizacin

8.1. Formalizacin mediante un lenguaje proposicional

Iniciamos esta seccin definiendo el concepto de formalizacin,


como el proceso de traduccin de un conjunto A de afirmacio
nes, formuladas en un lenguaje L, a frmulas escritas en un len
guaje lgico, en este caso, el lenguaje PM. Este proceso a su vez
puede definirse como una funcin matemtica f que toma como
dominio al conjunto de afirmaciones A y como rango al conjun
to B, el mismo que tiene como miembros a cada una de las fr
mulas de PM . La funcin f se comporta de tal manera que a cada
una de las afirmaciones de A le asigna una nica frmula de B.
El proceso de formalizacin es el mecanismo ms conocido
para aplicar un lenguaje proposicional al anlisis del lenguaje
cientfico y del lenguaje natural. Sin embargo, el lenguaje
proposicional PM y todos sus similares son muy pobres para
analizar el lenguaje cientfico, razn por la que sus aplicaciones
ms usuales se realizan con el lenguaje natural. Ello explica que
numerosos textos de lgica elemental, se dediquen a examinar
razonamientos que carecen de toda relevancia cientfica y que
inclusive son perogrullescos.
Es verdad que en la medida que la lgica trata propiamente
con frmulas; el contenido de las afirmaciones que se usa como
ejemplos es irrelevante para la comprensin de las propiedades
lgicas de las frmulas. Pero esto no ocurre desde la perspectiva
de la aplicacin. En efecto, la lgica moderna o matemtica no se

[127]
invent para analizar razonamientos cotidianos sino para abor
dar con rigor los procesos deductivos que realizan los matemti
cos, y los cientficos que usan la matemtica como medio expresi
vo de sus teoras. Por ejemplo, una preocupacin central de Frege,
uno de los fundadores de la lgica actual, fue la fundamentacin
lgica de la definicin del concepto de nmero natural y de la
aritmtica.
Nosotros asumimos como criterio, en la elaboracin de este
texto, que la enseanza de la lgica debe tender a usar ejemplos
que permitan visualizar su relevancia en el desarrollo de la cien
cia y de la tecnologa. Para ello el lenguaje proposicional es muy
pobre porque carece de medios para analizar conceptos cientfi
cos debido a que carece de predicados, relaciones y cuantifica-
dores. Ello da lugar a que no permita hablar de la verdad de pro
posicin alguna respecto de un conjunto de objetos y de relacio
nes definidas entre los mismos, lo que constituye el meollo de las
proposiciones cientficas significativas.
La formalizacin tericamente interesante se realiza mediante
la construccin de modelos para las teoras cientficas. Este es un
proceso a travs del cual se dota a un sistema de frmulas lgicas
de significado especfico respecto de las propiedades de los obje
tos que son miembros del conjunto de base del modelo. De esta
manera se asigna a los sistemas lgicos una semntica que da
lugar a que la lgica no sea una disciplina puramente sintctica o
formal. Ciertamente, el concepto de verdad es el ms importante
de la semntica lgica y el de deduccin o prueba de la sintaxis.
Por algn tiempo se pens que los conceptos anteriores tenan la
misma extensin. Se conjetur que las afirmaciones' Si A es dedu-
cibldentro de la teora T, entonces A es verdadera en un modelo de Y y
' Si A es verdadera en un modelo de T, entonces A es deducible en T
eran, ambas, verdaderas. El investigador Kurt Gdel derrumb
esa esperanza en 1931 cuando demostr que la segunda afirma
cin era falsa. Prob que en toda teora matemtica interesante
existe al menos una proposicin que es verdadera en el modelo
principal pero que es indeducible. Este resultado, conocido como
el segundo teorema de Gdel sobre proposiciones indecidibles,
es, tal vez, el ms famoso del siglo XX en lgica matemtica por-

128
que establece que el concepto de verdad tiene un significado ms
rico que el de deduccin o prueba.
El lgico polaco Alfredo Tarski es reconocido como el instau-
rador de la Teora lgica de modelos en la dcada de 1940. Se
mencionan a su lado los trabajos de Henkin y Robinson. Destaca
mos este hecho porque todos lo trabajos serios en formalizacin
lgica estn enmarcados dentro de la teora de modelos y noso
tros en este curso introductorio no estudiaremos estos conceptos.
Sin embargo, daremos algunos instrumentos conceptuales y sim
blicos para que nuestros ejercicios de formalizacin no se limi
ten al puro anlisis del lenguaje natural. Asimismo, presentare
mos los elementos bsicos para comprender la aplicacin de la
lgica al diseo de circuitos para computadoras digitales, para lo
cual s es suficiente un lenguaje proposicional reducido.

8.2. Qu significa 'postular' la verdad de una proposicin?


Para comprender adecuadamente cualquier anlisis lgico de in
ferencias, llamadas tambin argumentos, es indispensable tener
una idea muy clara del significado del concepto de postulacin.
De la misma manera, la comprensin de este concepto es clave
para la captacin de la naturaleza de la demostracin matemti
ca y de las hiptesis de las ciencias empricas.
Se postulan esencialmente proposiciones y hacerlo equivale
exactamente a suponer que ellas son verdaderas. Consecuente
mente, el que postula la verdad de una proposicin no afirma
que esa proposicin es efectivamente verdadera sino que l
adopta la estrategia de suponer o ponerse en el caso de que sea
verdadera. Si la proposicin postulada resulta falsa entonces slo
cabe retirarla y postular otra en su lugar.
Las proposiciones fundamentales o primitivas de la mate
mtica y de la lgica son esencialmente postulaciones, motivo por
el cual muchos especialistas contem-porneos prefieren hablar de
postulados en lugar de axiomas. Sin embargo, cuando un mate
mtico contemporneo o un lgico todava habla de axiomas, no
lo hace pensando en proposiciones evidentes por s mismas, como
tradicionalmente se pens desde Euclides, sino en proposiciones
cuya verdad se supone. Consecuentemente, los axiomas de cual

129
quier sistema matemtico son postulaciones que se hacen con in
dependencia de que su verdad sea o no evidente. Asimismo, los
conceptos de axioma y de postulado son hoy da, en los medios
especializados, completamente equivalentes.

8.3. In feren cia

Tradicionalmente se da como ejemplo de inferencia razonamien


tos como los siguientes:

(I) a. Si estoy viendo una pelcula entonces estoy en el cine.


b. Estoy viendo ima pelcula.
Por tanto, estoy en el cine.
(II) c. Juan es buen alumno o buen hijo.
d. Juan no es buen alumno.

Por tanto, Juan es buen hijo.


El sentido de estos dos razonamientos es afirmar la verdad
de las proposiciones llamadas conclusiones (las que se encuen
tran debajo de las rayas) a partir de la verdad de las proposicio
nes llamadas premisas (las que se encuentran sobre las rayas). Lo
importante aqu es reconocer que la verdad de las premisas es
postulada. Slo si se admite la suposicin de su verdad, la infe
rencia tiene sentido. De ser falsas las premisas, la inferencia no es
incorrecta sino sin relevancia desde el punto de vista lgico.
rDefinicin 15. Una inferencia o deduccin es una operacin lgica p or la que a
partir de la postulacin de la verdad de ciertas proposiciones llam adas premisas, se
deriva la verdad de otra proposicin llam ada conclusin.

Los ejemplos de inferencia dados anteriormente se encuen


tran formulados en lenguaje natural. Podemos dar otro usando
esta vez proposiciones de la matemtica.
(III) e. Si 2x =8, entonces x = 4
f. 2x = 8___________________________
Por tanto x = 4

130
Para diferenciar las inferencias de la matemtica, que gene
ralmente son llamadas demostraciones, de las formuladas en el
lenguaje natural, los autores usualmente llaman a las segundas
razonamientos o argumentos. Nosotros utilizaremos esta ltima
denominacin.

8.4. Formulacin y anlisis de la validez de argumentos


mediante el lenguaje de la lgica proposicional
En efecto, los ejemplos I y II son tpicamente argumentos porque
estn dados completamente en lenguaje natural. Estos argumen
tos constituyen inferencias pero todava no sabemos si son, ade
ms, lgicamente vlidas. Dar un mtodo que nos permita esta
blecer o decidir si esto ltimo ocurre es lo que nos preocupar en
adelante.
Lo primero que hay que hacer para poder decidir la validez
lgica de un argumento es determinar su estructura o forma.
Nosotros estamos en condiciones de hacerlo, por ejemplo, con los
argumentos I y II, porque estamos en condiciones de traducirlos
al lenguaje lgico. A este proceso de traduccin de un argumento
dado en lenguaje natural al lenguaje lgico es lo que se denomina
la formalizacin lgica de un argumento. Por extensin, cuando
una demostracin matemtica es traducida al lenguaje lgico tam
bin se dice que ha sido formalizada.
Un argumento est adecuadamente formalizado cuando cada
una de sus distintas proposiciones atmicas ha sido sustituida
por distintas variables proposicionales y las palabras tales como
y, o, Si... entonces ..., etc., han sido sustituidas por los correspon
dientes signos lgicos, denominados conectivas. Esto le da al len
guaje lgico capacidad para mostrar la estructura de los argu
mentos sin considerar su contenido o significado.
Examinando el ejemplo I y representando por p la proposi
cin Estoy viendo una pelcula y por q la proposicin Estoy en el
cine, encontramos fcilmente que la traduccin de tal argumento
al lenguaje lgico es la siguiente:

131
p > q \ < Premisas
P J
q} < Conclusin

En el caso del argumento II, si ahora p representa a la propo


sicin Juan es buen alumno y q a la proposicin Juan es buen hijo,
encontramos que la forma lgica de tal argumento es:

p v q \ < Premisas
~P J
q } < Conclusin

En el caso de la inferencia matemtica que es el ejemplo III, si


representamos por p la proposicin '2x = 8 ' y por q la proposi
cin 'x = 4' entonces encontramos que la forma lgica de esta infe
rencia matemtica es la misma que la que corresponde al primer
argumento. Esto pone en evidencia que puede haber inferencias
que tengan el ms diverso contenido pero que compartan la mis
ma forma lgica. Consecuentemente, dichas inferencias desde el
punto de vista lgico pierden su diversidad para reducirse a un
slo tipo de forma o estructura lgica.
Como segundo paso para decidir la validez de un argumen
to, podemos construir un condicional que tenga como anteceden
te a las premisas unidas por la conectiva de conjuncin y como
consecuente a la conclusin. Si efectuada la tabla de verdad de
este condicional encontramos que se trata de una tautologa,
entonces el argumento es lgicamente vlido y se afirma que
las premisas implican a la conclusin.
En el caso de los argumentos I y II los respectivos condicio
nales son:

( ( p -> q ) a p ) -> q
( ( p v q ) A - p ) -> q

El uso de los parntesis es necesario para que la jerarqua


corresponda de manera inequvoca a la estructura condicional del
argumento. De manera intuitiva la estructura lgica de cualquier
argumento puede ser representada as:

132
Prem isas Conclusin

Los puntos suspensivos indican los lugares de las premisas


y de la conclusin.
Ahora vamos a construir la tabla de verdad que corresponde
a la estructura lgica del primer argumento.

p q ((p q )Ap) q

V v V V V
V F F F V
F V V F V
F F V F V

Como puede verse, se trata de una tautologa y, consecuen


temente, el argumento I es lgicamente vlido. Pero como ade
ms, la inferencia matemtica III tambin posee esta estructura,
ella tambin es lgicamente vlida.
El segundo condicional tambin es una tautologa. Dejamos
al lector comprobarlo mediante la construccin de la tabla corres
pondiente.

D efinicin 16. Un argumento A es lgicamente vlido si y slo si, al ser traducido


al lenguaje lgico, el condicional resultante que tiene como antecedente a la traduccin
de las prem isas de A y como consecuente a la traduccin de la conclusin de A es
una tautologa. Si no es tautologa, el argumento es-lgicamente invlido.

Es fcil darse cuenta que lo que prohbe la lgica es que un


argumento vlido pueda tener todas las premisas verdaderas y la
conclusin falsa, porque en este caso el condicional correspon
diente sera falso para, al menos, un arreglo y no sera una tauto
loga, Consecuentemente, la exigencia lgica esencial es que en
todo argumento vlido si todas las premisas son verdaderas la
conclusin es necesariamente verdadera. Esto es todo lo que
garantiza la validez lgica de un argumento.
8.5. Formalizacin y validez de argumentos complejos

Determinar la validez de argumentos ms complejos que los an


teriores no es en principio ms difcil sino ms laborioso. Se si
gue el procedimiento anterior, pero hay que considerar mayores
detalles, algunos de los cuales vamos a exponer luego.
R7. Es necesario considerar que cuando hay tres premisas,
entonces es necesario utilizar dos conjunciones para unirlas y, en
general, si hay un nmero n de premisas se necesitar un nmero
n - 1 de conjunciones para enlazarlas de tal manera que todo el
antecedente sea una proposicin conjuntiva. Esta situacin para
un argumento que tiene tres premisas puede ser representada as:

( ( ........................) A ( .......................... ) A ( .......................) ) - * ( .........................)

En este caso, se han necesitado dos conjunciones y, debido a


que ambas estn igualmente libres de parntesis, la matriz final
del antecedente puede ser escrita debajo de cualquiera de ellas.
R8. Hay que considerar, asimismo, que palabras como 'pero",
'sin embargo7, son desde el punto de vista lgico conjunciones y
que esta conectiva es conmutativa porque en una proposicin
conjuntiva los componentes pueden intercambiar lugares. En bre
ve, las proposiciones:

p/vq y qa p

son lgicamente equivalentes.

R9. Proposiciones como No es posible que corra y duerma se


tradcen al lenguaje lgico como la negacin de la conjuncin y
no de las proposiciones componentes. As, el ejemplo anterior se
traduce por la proposicin negativa:

~ (P a q)

RIO. La conclusin de un argumento se reconoce porque,


generalmente, se escribe despus de palabras como 'consecuen
temente7, 'por tanto7, 'luego7, 'entonces7, 'en consecuencia7, 'por

134
consiguiente', etc. En algunos argumentos que no estn lgica
mente ordenados, la conclusin aparece antes que las premisas.
En tales casos la conclusin aparece antes de palabras como 'por
que', 'debido a', etc.
Tomando en consideracin las reglas anteriores, podemos
pasar a determinar la validez del siguiente argumento.
Habr un concierto si y solamente si hay una conferencia. Si no
hay un baile entonces no habr conferencia. Sin embargo, no es el caso
que haya baile y concierto pero habr uno de los dos. Consecuentemente,
no habr conferencia.
La primera tarea consiste en formalizar el argumento. Para
ello hay que representar sus distintas proposiciones atmicas por
distintas variables proposicionales. Esto lo podemos conseguir de
la siguiente manera:

Sea p la proposicin Habr un concierto.


Sea c la proposicin Hay una conferencia.
Sea r la proposicin Hay un baile.

No hay ms proposiciones distintas, pues el argumento est


constituido por la combinacin repetida de las anteriores.
La primera premisa Habr un concierto si y solamente si hay una
conferencia es claramente bicondicional.

p<-> q

La segunda premisa Si no hay un baile entonces no habr conferen


cia es un condicional con el antecedente y el consecuente negados.

~r ~q

Lo que sigue hasta antes de la palabra 'Consecuentemente'


son realmente dos premisas ms, con la variante de que la ltima
est escrita de modo abreviado y nos corresponde hacerla com
pletamente explcita. La primera parte es la premisa No es el caso
que haya baile y concierto y la segunda parte es, de manera explci
ta, la premisa O hay baile o hay concierto. Estas dos premisas se
formalizan en el mismo orden, como sigue:

135
~ ( r AP )
rvp

Lo que viene despus de 'Consecuentemente' es la conclu


sin No habr conferencia que se expresa por

~q

De acuerdo con lo anterior, la estructura lgica del argumen


to examinado es la siguiente:

p <- q
^ r ~ q
^(r a p)
r v p
~q
Y para decidir su validez lgica es necesario determinar si el
siguiente condicional es una tautologa.

( ( P<->q)A ( ~ r - ~ q ) A ~ ( r A p ) A ( r v p ) ) - > ~ q
Nosotros adelantamos al estudiante que este condicional es
una implicacin lgica, lo que significa que es necesariamente
tautolgico. Por consiguiente, el argumento es lgicamente vli
do. Dejamos como ejercicio la construccin de la correspondiente
tabla de verdad.

8.6, Form as conocidas de argum ento

A continuacin daremos una lista de estructuras o formas de ar


gumento que son lgicamente vlidas, lo cual se puede verificar
formulando el condicional correspondiente y efectuando la tabla.
Estas formas de argumento son conocidas debido a que a ellas
recurre a menudo el razonamiento matemtico y tambin el for
mulado en lenguaje natural.

136
F l. p q Modus Ponens (MP)
P
q

F2. p -> q Modus Tollens (MT)


~ q
~ p

F3. p v q Silogismo Disyuntivo (SD)


~ p
q

F4. p ->q Silogismo Hipottico (SH.)


q - r
p - r

F5. P Adicin (Ad.)


pv q

F 6. p aq , p aq Simplificacin (Simp.)

P q

El nombre de una forma de argumento es tambin el de la


tautologa correspondiente. Dejamos al lector, como ejercicio, la
construccin de las respectivas tablas de verdad.
Hacemos notar que los ejemplos que dimos antes como I y III
corresponden a la forma de argumento del Modus Ponens. El
ejemplo II, en cambio, corresponde a la forma de argumento del
Silogismo Disyuntivo. Un ejemplo que tiene la forma del Modus
Tollens es el siguiente:

Si TpacAmaru hubiera atacado el Cusco, entonces su revolucin


habra triunfado. Pero su revolucin no triunf, en consecuencia, Tpac
Amaru no atac el Cusco.

137
Un ejemplo de argumento de la forma del Silogismo Hipo
ttico es como sigue:

Si errar es humano, entonces nadie es infalible. Y si nadie es infali


ble, entonces Pedrito se equivoca. En consecuencia, si errar es humano,
Pedrito se equivoca.

Un ejemplo de argumento en forma de Adicin es:

Si vas a misa, entonces vas a misa o lees.

Un ejemplo de Sim plificacin es:

Si el 8 es nmero cbico y el 4 es cuadrado, entonces el 8 es


nmero cbico.

8.7, Mtodo indirecto

Existe un procedimiento que permite decidir si una frmula de


la lgica proposicional es lgicamente vlida sin necesidad de
recurrir a la construccin completa de una tabla de verdad. Este
mtodo ha sido divulgado en nuestro medio, con el nombre de
mtodo abreviado, a travs del libro de Irving Copi titulado In
troduccin a la lgica. Su utilizacin es recomendable cuando la
frmula, cuya validez se desea decidir, tiene numerosas variables
proposicionales, lo que no excluye que, por razones pedaggicas
o de comodidad , se lo aplique a casos simples. Puede conside
rarse que a partir de tres variables proposicionales la confeccin
d una tabla puede ser laboriosa. Asimismo, aunque este proce
dimiento puede aplicarse a cualquier frmula de la lgica pro
posicional, su empleo est indicado cuando se trata de frmulas
condicionales, que son las que resultan cuando se formaliza ar
gumentos.
La aplicacin de este mtodo se funda en las siguientes pre
suposiciones:

P l. Toda frmula lgicamente vlida es una tautologa y, por


tanto, contiene en su matriz principal slo valores verdaderos.

138
P2. Si una frmula es una tautologa, entonces la hiptesis H
que supone que su matriz principal contiene al menos un valor falso
debe conducir a contradiccin y, por tanto, debe rechazarse H.
P3. Si la hiptesis H que supone que la matriz principal de una
frmula tiene al menos un valor falso permite construir de manera
coherente el arreglo correspondiente ( sin contradiccin), entonces
H debe ser aceptada y la frmula dada no es una tautologa.
Las presuposiciones P2 y P3 se basan en la regla de deduc
cin por reduccin al absurdo (RAA), cuya forma lgica presen
taremos ms adelante. Esta regla establece que toda hiptesis que
conduce a contradiccin debe ser rechazada. En la medida que en
la larga tradicin matemtica las demostraciones por reduccin
al absurdo se conocen como demostraciones indirectas, hemos
decidido denominar al procedimiento que presentamos mtodo
indirecto. Las reglas son como sigue:
rl. Dada una frmula condicional W debe formularse la hi
ptesis H que afirma W no es tina tautologa porque su matriz prin
cipal contiene al menos un valor falso.
rl. En armona con las hiptesis H debe asignarse al antece
dente de W el valor verdadero y al consecuente el valor falso.
r3. Inmediatamente, debe asignrsele a las letras variables del
consecuente exactamente los valores que se ajusten a la suposicin
de su falsedad. Por ejemplo, si el consecuente est constituido por
una frmula atmica, se asignar el valor falso a dicha frmula. Si
el consecuente es de la forma ~A, se asignar a A el valor verdade
ro. Si el consecuente tiene la forma A v B, entonces debe asignrsele
el valor falso a ambos componentes. Si tiene la forma A a B es sufi
ciente con asignar el valor falso a uno de los componentes. Si tiene
la forma A -> B debe asignarse necesariamente el valor verdadero
a A y el falso a B. Si tiene la forma A B debe asignrsele distinto
valor a cada componente. Si tiene la forma A * B , debe asignarse
el mismo valor a ambos componentes.
r4. Debe asignarse a cada una de las letras variables del ante
cedente, que tambin aparecen en el consecuente, exactamente
los mismos valores que ya se les asign por efecto de la aplicacin
de la regla r3. Si esto es suficiente para convertir al antecedente,
en su conjunto, en verdadero, entonces la hiptesis H debe ser
aceptada y W no es una tautologa. Pero, si esta asignacin de

139
valores entra en contradiccin con la suposicin de que el antece
dente es verdadero, entonces la hiptesis H debe ser rechazada y
W es una tautologa.
r5. Si la aplicacin de r4 no permite tomar una decisin debi
do a que en el antecedente existen letras variables que no han
recibido asignacin de valores porque no aparecen en el conse
cuente, entonces debe asignarse a estas letras variables los valo
res adecuados para convertir al antecedente en verdadero. Si esto
es posible, la hiptesis H debe ser aceptada y W no es tautologa,
pero si esta asignacin de valores entra en contradiccin con la
suposicin de que el antecedente es verdadero, entonces H debe
ser rechazada y W es tautologa.
Por ejemplo, supongamos que debemos decidir si es o no tau
tologa la frmula:

(p-> q) (p a q)

Aplicando rl, la hiptesis H que dice que la frmula anterior


tiene al menos un arreglo falso la escribiremos as:

F((p->q) -> ( p A q ) )

La consecuencia inmediata de H que asigna al antecedente el


valor verdadero y al consecuente el valor falso (regla r2) es:

F ( V ( p -> q ) -> F ( p a q ) )

Para mantener la hiptesis de que el consecuente es falso, es


sficiente, por ejemplo, aplicando r3, asumir que la variable q es
falsa. As tenemos:

F ( V ( p q ) - F (p a F ( q ) ) )

Aplicando r4 al antecedente tenemos:

F(V (p-F(q))-> F( p a F( q) ) )

140
En este caso la decisin final depende del valor que le asigne a
p . Como el procedimiento consiste en hacer todo lo lgicamente
permitido para confirmar la hiptesis H, entonces, por r5, asigno a
p el valor falso y de esta manera convierto al antecedente en verda
dero. Ello me permite construir justamente el arreglo en el que W es
falsa y, por tanto, debo aceptar la hiptesis H . En breve y con algn
sentido de observacin, se puede decir que la frmula propuesta no
es tautologa porque su matriz principal es falsa al menos en el cuar
to arreglo. La asignacin completa de valores es como sigue:

F( V( F ( p ) F ( q ) ) -> F ( F ( p ) a F ( q ) ) )

Examinemos un segundo ejemplo. Se trata de decidir si la


siguiente frmula es una tautologa.

( ( P A q ) A r ) -> ( p v s )

Aplicando r l, la hiptesis H que dice que la frmula anterior


tiene al menos un valor falso en su matriz principal, la escribire
mos as:

F ( ( (p A q ) A r ) -> ( p v s ) )

Y la suposicin derivada de la anterior, que afirma que el


antecedente es verdadero y el consecuente falso ( aplicando r 2 ),
la escribiremos como sigue:

F ( v ( (p A q ) A r ) - > F ( pvs))

Por aplicacin de r3, la nica manera de mantener la suposi


cin de la falsedad del consecuente es asignar el valor falso a ambos
componentes. As tenemos:

F ( V ( ( p a q) a r) F( F (p )v F(s) ) )

Aplicando r4, escribimos el valor de la nica variable del


antecedente, en este caso p, que ya tiene valor asignado en el con
secuente y obtenemos:

141
F ( V ( ( F ( p ) A q ) A r ) -> F (F (p )v F (s)))

Esta asignacin de valores es suficiente para tomar una deci


sin. En efecto, como p es tambin falsa en el antecedente, enton
ces la conjuncin p a q ser falsa y, consecuentemente, la con
juncin ( p a q ) a r, del mismo modo, ser falsa, lo que hace
imposible construir el arreglo que convertira al antecedente en
verdadero manteniendo el consecuente falso, rechazndose as la
hiptesis H. Por tanto W es una tautologa.

8.8. Reglas adicionales de abreviacin

El uso del mtodo indirecto puede ser adecuadamente comple


mentado con la aplicacin de algunas reglas de abreviacin que
permiten simplificar el proceso.
ral. En la medida que la conjuncin y la disyuncin son ope
radores conmutativos y asociativos, en los casos pertinentes, pue
den escribirse como conjunciones o disyunciones de n componen
tes. As en lugar de las frmulas:

( ( p A q ) A T ) A S ) A t ) A...,)
( ( p v q ) v r ) v s ) v t ) v ...,)

puede escribirse:

(p 1 a p a / /a p n)
( p V p V , ... , V p )
z

1 2
2

donde el subndice n puede tomar un valor tan grande como


s quiera.
ra2. Es suficiente que en una conjuncin de n componentes
exista una letra variable falsa para que la conjuncin, en su totali
dad, sea falsa.
ra3. Es suficiente que en una disyuncin de n componentes
exista una letra variable verdadera para que la disyuncin, en su
totalidad, sea verdadera.
ra4. La manera abreviada de escribir una frmula siempre
verdadera es ( pv~p) y la de escribir una frmula siempre falsa es
(p a ~P).

142
ra5. Es suficiente que el consecuente de un condicional sea
verdadero para que el condicional, en su totalidad, sea verdade
ro. Y tambin es suficiente que un condicional tenga el antece
dente falso para que , en su totalidad, sea verdadero.
ra6. Si usamos 'l'e n lugar del valor verdadero, '0' en lugar
del valor falso y la expresin 'se reduce a para precisar que una
expresin puede ser reemplazada por la otra y viceversa, se pue
de establecer las siguientes abreviaciones:

ra6.1. P v ~ P se reduce a 1
ra6.2. P A ~ P se reduce a 0
ra6,3. P A 1 se reduce a p
ra6.4. P A 0 se reduce a 0
ra6.5. P V 1 se reduce a 1
ra6.6. P V 0 se reduce a p
ra6.7. P - 1 se reduce a 1
ra6.8. 0 - P se reduce a 1

Aplicaremos las reglas anteriores a algunos ejemplos de sim-


plificacin de frmulas.

A. ( p A q A ~ q ) - > s
( p A 0 ) - s ( por ra6.2)
0 -> s (por ra6.4)
1 ( por ra6.8)
p v~p ( por ra6.1)

B. p - > ( q v r v ~ r )
p - > ( q v 1) ( por ra6.1)
p-l ( por ra6.5)
1 ( por ra6.7)
X

( por ra6.1)
<

C. ( p A q A ( s v ~ s ) ) - > ( q v ( P A P ) )
((P A q )A l)-> (q v O ) ( por ra6.1 y
( P A q ) > q ( por ra6.3 y

La ventaja de las reglas ra6.1 - ra6.8 parece obvia. Es claro


que la frmula simplificada es equivalente, en cada caso, a la
inicial.

143
IX
DEDUCCIN NATURAL

O b je tiv o s.-

Comprender y usar las reglas de deduccin natural (RDN) como


aquellas que permiten transferir la verdad de unas proposicio
nes a otras.
Ejecutar deducciones directas y por reduccin al absurdo (RAA)
adquiriendo destreza en el manejo de 21 reglas de Gentzen.
Manejar las RDN como procedimiento no algortmico de carc
ter heurstico.
Resolver problemas cuya solucin requiere del ejercicio del pen
samiento hipottico deductivo.
Cuestionario 9
Deduccin natural

Instru cciones

I. Justifique las siguientes deducciones propuestas por Irving Copi


en su libro Symbolic Logic

1.(1) (O - > ~ P ) a ( ~ Q - > R )


(2) ( S - T ) a ( ~ U - ~ Z )
(3) ( ~ P - > S ) a ( R - > ~ U )
(4) ( T v ~ Z ) ( W a X )
(5) O v ~ Q / \ W a X
6. ~ P v R
7. S v ~ U
8. T v ~ Z
9. W a X

2.(1) [(Av ~ B )V C ]-> [D (E


(2) ( A v ~ B ) [ ( F * G ) - > H]
(3) A [ ( E <-> F ) - ( F G )]
(4) A / \ D H
5. Av ~B
6. (Av~ B )v C
7. D ( E * F )
8. ( E * F ) - > ( F <- >G)
9. D ( F <H> G )
10. (F <->G)->H
11. D H

[147]
3.(1) A - B
(2) C -> D
(3) ~ Bv~ D
(4) A
(5) ( E a F ) - C / \ ~ ( E a F)
6. (A->B) a (C->D)
7. ~A v ~C
8. ~C
9. ~ (EaF)

4.(1) ( G - H ) - ( I <- J )
(2) K v ~ ( L -> M )
(3) ( G -> H ) v ~ K
(4) N -> ( L -> M )
(5) ~ (I o j ) / \~N
6. ~ ( G -> H )
7. ~K
8. ~ ( L -> M )
9. ~N

5.(1) H -> ( I - J )
(2) K - ( I - > J )
(3) ( ~ H a ~ K ) - > ( ~ L v ~ M )
(4) ( ~ L - ~ N ) a ( ~ M ~O)
(5) ( P - > N ) a ( Q - O )
(6).~(I-> J ) / \ ~ P v ~ Q
7. ~ H
8. ~ K
~H a ~K
10. ~ L v ~ M
11. ~ N v O
12. ~ P v ~ Q

II. Ejecute las siguientes deducciones o pruebas formales. Los ejer


cicios 4 y 5 deben ser resueltos por reduccin al absurdo

1.(1) F v ( G v H )
(2) ( G - > I ) a ( H - > J )

148
(3) ( I v J ) - ( F v H )
(4) ~ F / H

2.(1) K - L
(2) M -> N
(3) ( O - N ) J ( P - L )
(4) (~Nv~L)a(~M v~0) / (~Ov~P)a(~ M v ~K)

3.(1) Q -> ( R - > S )


(2) ( R - S ) -> T
(3) (S aU)->~P
(4) ~P ->(R ~W )
(5) ~ T v ~ ( R < - > ~ W) / ~ Q v ~ ( S a U)

4.(1) ( 0 - ~ P ) a ( P - Q )
(2) Q - > O
(3) ~ R -> P / R

5.(1) X - (Y -> Z )
(2) X - ( A - B )
(3) X a (Y v A)
(4) ~ Z / .. B

6 .( 1) C - ( D - > ~ C )
(2) D/ ~ C a ~D

7.(1) J v ( ~ K v J )
(2) K v ( ~ J v K ) / (JaK )v (~ J a~ K )

8.(1) ( L v M ) v ( N a O )
(2) ( ~ L a O ) a ~ ( ~L a M ) / ~La N

III. Resolver aplicando la RDN, de prueba condicional, las siguien


tes deducciones

1.(1) ( A v B ) - > ( C a D)
(2) ( D v E ) - F / A - F

149
2.(1) (EvF)-> G
(2) ( J - > ~ G ) a ~ H
(3) J v K / /. E K

3.(1) Q - > P
(2) T v S
(3) Q v ~ S / ~(Pv R)-> T

4.(1) A - > ( B - > C )


(2) B - ( C -> D ) / /. A > ( B > D )

IV. A continuacin se presenta un conjunto de premisas y un con


junto constituido por conclusiones, algunas de las cuales se si
guen lgicamente desde las premisas y otras no. En otras pala
bras, algunas de las conclusiones propuestas completan las *
premisas de manera tal que configuran con ellas un razonamien
to o argumento lgicamente vlido, mientras que otras no se
comportan as. El ejercicio consiste en decidir mediante tablas
de verdad, en cada caso, si la conclusin propuesta se sigue
lgicamente o no desde el conjunto dado de premisas. Adems,
debe construirse una deduccin slo para los casos en los que el
argumento es lgicamente vlido.

a)Premisas

1) La lgica es difcil o no le gusta a muchos estudiantes.


2) Si la matemtica es fcil, luego la lgica no es difcil.

Conclusiones propuestas Se sigue lgicamente?


C . La matemtica no es fcil si a muchos
estudiantes les gusta la lgica.____________________________

C2. A muchos estudiantes no les


gusta la lgica si la matemtica
no es fcil._______________________________________________

C3. La lgica no es difcil o la


matemtica es fcil._______________________________________

150
Conclusiones propuestas Se sigue lgicamente?
C.La matemtica no es fcil
o la lgica es difcil._______________________________________

C5. La lgica no es difcil o la matemtica


no es fcil.________________________________________________

C6. La lgica es difcil o la matemtica


no es fcil._______________________________________________

Cr Si a muchos estudiantes les gusta la lgica,


Luego la matemtica no es fcil o la lgica
no es difcil.______________________________________________

151
LECCIN 9
Deduccin natural

9.1. La deduccin de Gentzen


Las reglas conocidas como de Deduccin Natural fueron propues
tas en 1934 por el investigador Gerhard Gentzen. Desde entonces
se conocen diversas variantes de ellas que algunos textos de lgica
presentan como reglas para construir deducciones o pruebas for
males. A nosotros nos parece ms adecuado respetar su denomi
nacin original de Reglas de Deduccin Natural, aunque la versin
que presentamos ya no sea la de Gentzen sino una versin ms in
tuitiva y pedaggica. Esto al propio tiempo quiere decir, desde el
punto de vista lgico, un conjunto ms recargado de reglas puesto
que la facilidad pedaggica hace recomendable que se usen como
si fueran reglas necesarias, algunas que realmente son omitibles por
aplicacin reiterada de otras reglas bsicas. Sin embargo, si obvia-
ramos las reglas que no son bsicas, las deducciones seran ms sim
ples si se considera que se utilizara un nmero menor de reglas,
pero seran menos pedaggicas y ms laboriosas si se considera la
mayor longitud de las deducciones resultantes.

9.2. Transferencia de la verdad


Para entender el sentido fundamental de las Reglas de Deduccin
Natural (RDN) es necesario tener presente que la funcin esen
cial, de cualquiera de estas reglas, es la transferencia de la ver
dad de unas proposiciones a otras. A las proposiciones que se usa

[1531
como condiciones iniciales para este proceso de transferencia se
les conoce como premisas y a las proposiciones que reciben la ver
dad transferida o que heredan la verdad, por as decirlo, se les
denomina conclusiones o consecuencias lgicas. Esta idea se pue
de graficar (Fig. 1) a travs de un modelo de caja negra que con
cibe a las reglas de deduccin como una mquina, la que cada
que entra informacin verdadera en sus unidades de procesamien
to, emite o produce como salida, necesariamente, informacin ver
dadera. Adicionalmente, la mquina no proporciona garanta al
guna cuando la informacin que ingresa es falsa, pues no est di
seada para procesar entradas falsas.
Recprocamente, la mquina s est diseada para asegurar
nos que si la salida emite informacin falsa, entonces est ingre
sando, necesariamente, informacin falsa. Esto se puede graficar
(Fig. 2) mediante el mismo modelo de caja negra aadiendo un
circuito de feed back o de retroalimentacin.

Fig. 1 V V
Entrada Salida

Entrada Salida F
Fig. 2
F
Feed back.
------------- <-----------

9.3. Deduccin e im plicacin


La siguiente cuestin a tenerse en cuenta es que si desde un con
junto de premisas Px, P2, P n hemos deducido, aplicando correc
tamente las Reglas de Deduccin Natural, la conclusin o conse
cuencia lgica C, entonces podemos decir que las referidas pre
misas implican a la conclusin y que, por tanto, el condicional

( P j A P j a , .........., a P ) C

es lgicamente vlido. Ahora, si las premisas y la conclusin es


tn escritas en el lenguaje de la Lgica proposicional, de PM, por
citar un ejemplo, entonces el esquema anterior debe ser una tau
tologa.

154
Esto ltimo pued suscitar en el aprendiz la siguiente pre
gunta: Y para qu necesitamos las RDN si con la sola aplicacin
de tablas de verdad podemos decidir si las premisas implican o
no a la conclusin. La respuesta es que, por un lado, cuando las
frmulas que constituyen las premisas exhiben cinco variables
proposicionales o ms, resulta mucho ms breve y elegante apli
car las RDN, pero, por otro lado, existen muchos casos que exce
den a los lenguajes proposicionales en los que la construccin de
una tabla de verdad para decidir si las premisas implican a la
conclusin, no es posible, pues el mtodo de las tablas de verdad
slo tiene lugar en los casos elementales. Por ejemplo, cuando se
parte de premisas que contienen predicados didicos, usuales en
aritmtica, como ...mayor que..., ...menor que..., ..igual a...
la construccin de tablas de verdad es, en principio, imposible
debido a la existencia de pares de cuantificadores universales y
existenciales variando sobre un conjunto infinito.

9.4. Esquemas de frmulas

Escribiremos las RDN numerndolas y en forma de quebrado, de


tal manera que en el lugar del numerador aparecen las premisas
y en lugar del denominador aparece la conclusin. En este caso,
las premisas y la conclusin no son frmulas, en sentido estricto,
de la lgica proposicional, sino esquemas de frmulas que perte
necen al metalenguaje de la lgica proposicional porque su fun
cin es prescribir lo que est permitido hacer con las frmulas.
Estos esquemas proporcionan la estructura comn a una multi
tud de frmulas y el hecho de estar escritas en forma de quebra
do nos dice que las frmulas que aparecen en el numerador im
plican a las que aparecen en el denominador. Por ejemplo, las fr
mulas de la lgica proposicional siguientes:

pv^q
(p A q ) v ~ ( r v s )
pv ~ ( q a t )

tienen en comn el ser de la forma A v ~ B, o en otras palabras, el


corresponder a dicho esquema.

155
En el caso de las reglas, que establecen equivalencias o dobles
implicaciones, hemos preferido escribirlas horizontalmente ligndo
las por la abreviatura sss que corresponde a la expresin 'si y slo si7
que a su vez corresponde a la lectura castellana del bicondicional.
En el caso de las reglas, que no conducen a una nica conclu
sin, hemos escrito al costado derecho el mismo esquema acom
paado de la conclusin alternativa. Asimismo, es conveniente
enfatizar que las reglas que son equivalencias funcionan propia
mente como reglas de reemplazo, pues autorizan a reemplazar
una frmula de cierta forma por su equivalente, cualquiera que
fuera el lugar donde sta aparezca.
Tambin es oportuno sealar que el dar las reglas en forma de
esquemas y el construir deducciones con letras esquemticas ( A, B,
P, Q,...) tiene la ventaja que de este modo se hace alusin a conjuntos
infinitos de casos particulares que son vlidos por simple inspeccin
en el caso de que su estructura corresponda a alguna de las 2 1 RDN
a continuacin proporcionadas. En la versin que presentamos in
cluimos a la regla de la Prueba condicional y a la de Demostracin
por reduccin al absurdo, las mismas que a menudo son omitidas y
tratadas como si slo fueran estrategias deductivas. Lo real es que lo
son, pero su legitimidad reposa en las reglas 20 y 21.

9.5. R eglas de d ed u cci n n atu ral p ara u n len g u aje


proposicional

1. A - B
A Modus Ponens (MP)
B
2. A - B
~B Modus Tollens (MT)
.'.-A
3. A -> B
B - C Silogismo Hipottico (SH)
..A ->C
4. A v B
-A Silogismo Disyuntivo (SD)
B

156
5. A B
C -> D Dilema
A vC Constructivo (DC)
B vD
6. A-> B
C -> 1) Dilema
-B v ~D Destructivo (DD)
-A v~ C
7. P a Q y tambin P a Q Simplificacin
.P Q (Simp.)
8. P p
Q y tambin Q Conjuncin
PaQ .Q a P (Conj.)
9. P V tambin P Adicin (Ad.)
..P v Q Qv P
10. ~ ( P a Q) sss ~P v ~ Q Regla de
-(P v Q ) sss ~P a ~ Q De Morgan
(DM)
11. ( P a Q) sss ( Q a P) Conmutatividad
(P v Q ) sss (Q v P ) (Conm.)
12. P A (Q A R) sss (PaQ)aR Asociatividad (As.)
P V (Q V R) sss (P v Q )v R
13. P a (Q v R) sss (PaQ)v(PaR )Distributividad
(Dist.)
P v (Q a R) sss (Pv Q ) a (Pv R)

14. P sss P Doble negacin (DN)


15. A ->B sss ~B > ~A Transposicin
(Trans.)
16. A ^ -B sss ~A v B Definicin de 4 por v

157
17. A o B sss (A B) a (B A) Definicin
d e 'o '

18. A<-B sss (A a B ) v (-A a ^B) Definicin


de ' o '
(A a B)-^ C sss A -^(B^C) Exportacin
(Exp.)

19. A sss A aA Idempotencia (Idemp.)


A sss Av A

20. 1.A
Prueba condicional
n. B
n + 1. A - > B

21. A > ( B a ~ B ) Reduccin al


~A absurdo (RAA)

9.6. Aplicacin de las reglas RDN


En lo que sigue presentamos ejemplos de deduccin natural que
prueban que una frmula denominada conclusin se deduce de
otras frmulas que son, en cada caso, las premisas. Es importan
te enfatizar que las frmulas en s mismas no son ni conclusiones
ni premisas y que estas denominaciones hacen referencia a la fun
cin especfica que cumplen en una situacin determinada. Di
cha funcin podra ser muy distinta en otro contexto. Al respec
to, mencionamos que hay otro tipo de deducciones, denomina
das axiomticas, y semiaxiomticas que no abordaremos en esta
seccin.
Cada deduccin est constituida por una secuencia de lneas
numeradas, en cada una de las cuales est escrita una frmula.
Las lneas con los nmeros entre parntesis son las premisas y las
Otras son las consecuencias lgicas de las premisas obtenidas por
la aplicacin de las RDN. La ltima lnea debe ser necesariamen
te la conclusin que se pretende deducir.
En trminos descriptivos, una deduccin o prueba es una
secuencia finita de lneas que por aplicacin de los RDN transfor

158
ma unas frmulas denominadas premisas, en otras frmulas hasta
obtener una deseada, denominada conclusin.
A la derecha de cada lnea, por razones pedaggicas, hemos
escrito unas abreviaturas que corresponden al nombre de la regla
que nos ha permitido obtener la respectiva frmula. Tambin apa
recen, unos nmeros que corresponden a las lneas anteriores a
las que hemos aplicado las reglas, para obtener la transformacin
deseada. A estas inscripciones o anotaciones se les conoce como
justificaciones de las lneas de deduccin.
Una presuposicin vlida y comprensible es que la forma
lizacin de algn argumento que no hace falta especificar, ha dado
lugar a la construccin de las premisas y de la conclusin pro
puesta. El sentido de la prueba es establecer que desde las premisas
se deduce la conclusin, ms no que las premisas sean, en efecto,
verdaderas, cuestin que como sabemos, no concierne a las re
glas de deduccin lgica.
A continuacin proporcionaremos un ejemplo explicativo del
mecanismo de construccin de una deduccin, conocida tambin
como prueba formal. Nos valdremos nuevamente de un ejemplo
tomado del libro de Irving Copi titulado Symbolic Logic.

1. (1) ( A a B ) -> [ A - ( D a E ) ]
(2)( A a B ) a C / .*. D v E
3. A a B Simp. (2)
4. A - > ( D a E) MP (1), 3
5. A Simp. 3
6. D a E MP 4, 5
7. D Simp. 6
8. D v E Ad. 7

Este ejercicio est constituido por ocho lneas de demostracin


o prueba. Las dos primeras (1) y (2) son premisas y desde la lnea 3.
a la nmero 8. tenemos seis lneas deducidas desde las premisas
aplicando las RDN. Al costado de la lnea (2) y despus de un seg
mento diagonal, sucedido por tres puntitos, se encuentra escrita la
conclusin que se pretende obtener o probar. Por tanto, la parte
que est a la derecha de los tres puntitos no es parte de la deduc-

159
cin sino slo una ayuda para que el aprendiz tenga presente a
dnde quiere llegar con los pasos demostrativos.
Las lneas (1) y (2) no requieren justificacin porque la verdad
de las premisas se presupone o postula. La lnea 3. ha sido deduci
da de la linea (2) por la RDN N. 7, de simplificacin, debido a que
( A a B ) a C tiene la forma del esquema de frmula P y\Q. La lnea
4. ha sido obtenida por aplicacin de la RDN N.l (MP) a las lneas
(1) y 3, debido a que ( A a B ) > [ A > ( D a E ) ] tiene la forma de
A -> B , de tal manera que ( A a B ) es A. La lnea 5. se ha obtenido
por una nueva aplicacin de la regla de simplificacin a la lnea 3.
La lnea 6. se ha obtenido por una nueva aplicacin de la regla 1.,
MP, a las lneas 4. y 5. La lnea 7. se ha obtenido por aplicacin, una
vez ms, de la regla de simplificacin a la lnea 6. Finalmente, la
lnea 8. se ha obtenido por aplicacin de la RDN N. 9, de Adicin,
a la lnea 7. De este modo se ha deducido la conclusin propuesta
desde las premisas numeradas con (1) y (2).
Es claro que en cada deduccin, cada lnea tiene slo dos
posibilidades: O es una premisa o es consecuencia de una o ms
premisas obtenida por algunas de las RDN. Asimismo, es com
pletamente lcito aplicar la misma regla tantas veces como se juz
gue necesario. El ejercicio que hemos realizado se denomina de
justificacin de las lneas de deduccin.

9.7. Las RD N no constituyen un algoritmo


A continuacin desarrollamos un ejemplo, tambin de I. Copi,
muy similar al anterior. Este tiene la peculiaridad de contener 7
letras variables distintas. Esto significa, que si alguien deseara pro
ba? por el mtodo de las tablas que la conclusin se deduce de
las premisas, tendra que construir una tabla con un margen de
128 arreglos que es el valor de 27. Asimismo, el uso del mtodo
indirecto para decidir si una frmula es una tautologa tambin
sera laborioso. Por ello, una deduccin de 9 lneas resulta un pro
cedimiento breve y elegante que muestra las virtudes de las RDN.
La limitacin de las reglas que estamos presentando es que no
constituyen un algoritmo porque no garantizan que, en un nme
ro finito de pasos, decidiremos si la conclusin se sigue o no de
las premisas. Podra ocurrir que la decisin debiera ser positiva

160
pero que carezcamos del ingenio suficiente como para construir
la deduccin adecuada. Pero podra ser que la decisin debiera
ser negativa y que carezcamos de ingenio para probar que la con
clusin no se deduce de las premisas. Slo tenemos garantas ple
nas en un caso: cuando somos capaces de construir la deduccin.
Si es as, sabemos que la conclusin se deduce de las premisas
como en el ejemplo que sigue.

(1)(~Xv ~ Y )-> [A -> (P a ~Q)]


(2)(~Xa ~ R )-> [(P a ~Q)->Z]
(3)(~Xa ~ R ) a ( ~ Z v A) / .*. A - ^ - Z
4. ~ X a ~ R Simp. (3)
5. ( P a ~ Q ) - > Z MP (2), 4
6. - X Simp. 4
7. ~ X v - Y A d.6
8. A - > ( P a ~Q) MP (1), 7
9. A - * Z SH 8 ,5

Es importante puntualizar que los problemas lgicos y mate


mticos tratables algortmicamente son los ms elementales. En
lgica todos ellos se reducen, en ltima instancia, a la construccin
de tablas de verdad o a su equivalente. En matemtica, slo son
algoritmizables las funciones que se reducen a sumas. Empero los
problemas relevantes en lgica y matemtica plantean la creacin
de deducciones, lo que excede en mucho el nivel algortmico.

9.8. Prueba condicional


Ahora introduciremos un ejemplo que ilustre el manejo de la RDN
20, conocida como regla de la prueba condicional. Tericamente,
esta regla es muy importante porque expresa con claridad la idea
de consecuencia lgica y es, adems, irreductible a las diecinue
ve anteriores. Destacaremos, inmediatamente, este segundo aspec
to sealando que de las diecinueve reglas anteriores, diecisiete son
redundantes en el sentido de que son omitibles al costo de hacer
mucho ms laboriosas las deducciones. Se trata de reglas que son
derivables con ayuda de las no-omitibles que son el Modus Po
nens, la definicin que figura como RDN 16, la Prueba condicio

161
nal y la RDN 21, que es la regla de deduccin o demostracin por
reduccin al absurdo. Por tanto, las reglas omitibles cumplen la
funcin de abreviaciones deductivas, las mismas que tienen un
obvio inters desde el punto de vista pedaggico.
Para comprender de qu manera esta regla da expresin al
concepto de consecuencia lgica, podemos referir el hecho de que
hay deducciones, lgicamente vlidas, que slo son posibles si se
usa la regla de la prueba condicional (PC). Por ejemplo, la frmu
la A > ( A a B ) se deduce desde la frmula ( A -> B ). Sin embar
go, la demostracin correspondiente slo se puede realizar, den
tro del sistema de Gentzen, usando PC. La razn de ello, es que es
la nica regla que nos permite incrementar nuevas premisas para
ayudar a las que ya disponemos, las mismas que pueden ser
insuficientes. Sin embargo, esas premisas adicionales son una es
pecie de muletas que una vez que nos permiten lograr el objeti
vo deseado, podemos desecharlas, eventualidad que en el papel
no se puede representar de manera muy intuitiva porque la refe
rida muleta est escrita de la misma manera que las premisas
propiamente dichas. A causa de ello se introduce una simbologa
especial prescrita por las reglas que a continuacin especificamos.

rcl. La regla PC est indicada en las deducciones en las que la


conclusin a deducir tiene la forma condicional A > B.
rc2. Debe aadirse a las premisas dadas, como una premisa adi
cional, la frmula que sea antecedente de la conclusin busca-
da,y escribirse a su derecha Pr. ai., que es la abreviatura de
Premisa adicional.
rc3. sando las premisas dadas y la premisa adicional, aplicando
fes RDN, deben hacerse las transformaciones que permitan de
ducir la frmula que figure como consecuente de la conclusin,
lo cual se alcanza en el paso n de la deduccin, al que lo llama
remos lnea Ln.
rc4. En la lnea siguiente, en Ln + 1 , constryase una frmula con
dicional que tenga como antecedente a la premisa adicional y
como consecuente a la frmula obtenida en la lnea Ln. Escrba
se a la derecha de la lnea Ln + 1 , como justificacin, la abrevia
tura PC, con el nmero correspondiente a la lnea de la premisa
adicional y el nmero correspondiente a Ln.

162
rc5. Despus de aplicada la regla anterior, trcese una flecha en L de
tal manera que nazca al costado izquierdo del nmero de la lnea
de la premisa adicional y termine como si subrayara la frmula de
Ln. El sentido de esta flecha es precisar que la frmula de Ln + 1 ,
que es la conclusin buscada, no depende de la premisa adicional
sino solamente de las inicialmente dadas. Es el paso que marca el
abandono de las muletas y es de uso muy importante cuando se
aborda deducciones no muy elementales, las que escapan al alcan
ce de este manual. Sin embargo, sera un error omitir esta regla.

Nuestro ejemplo estar constituido por las frmulas que ci


tamos antes, por ser particularmente ilustrativas.

(1) A - > B / A > (A a B )


2. A Pr. ad. Por rc2
3. B MP en (1) y 2 . Segn rc3
4. A a B Conj. en 2 y 3 .Segn rc3
------------------------ >
5. A ^ ( A a B) PC en 2, - 4 segn rc4

Un ejemplo adicional lo proporciona el siguiente ejemplo,


tomado con algunas adaptaciones, del libro de Suppes titulado
Introduccin a la lgica matemtica.

(1) S a ( ~ P v M)
(2) M - > ( Q v R ) / :. P -> (~ Q -> R)
3. P Pr. ad.
4. ~ P v M Simp. en (1)
5. P DN en 3
6. M SD en 4, 5
7. Q v R MP en 2, 6
8. Q v R DN en 7
9. ~ Q ^ R ^ RDN 16 en 8
10. P - > ( ~ Q - > R ) PC en 3, 9

9.9. Demostracin por reduccin al absurdo

La regla RDN 21 (RAA) es conocida desde la antigedad en ma


temtica y la us Euclides en su famosa obra Elementos. Consiste
en postular o suponer que la conclusin no se deduce de las pre-
misas, para luego rechazar esta postulacin debido a que condu
ce a una contradiccin de la forma A a ^ A.
La aplicacin de la deduccin por reduccin al absurdo es un
caso particular de la deduccin por PC. La variante radica en que
se aade como premisa adicional la negacin de la conclusin
buscada. Luego, a partir de las premisas dadas y de la adicional,
se deduce una frmula contradictoria, se aplica PC y se niega (re
chaza) la premisa adicional aplicando RDN 21. Finalmente por
RD N 14, doble negacin, se demuestra que la negacin de la pre
misa adicional es la conclusin buscada. En breve, la hiptesis
bsica de esta estrategia demostrativa es que toda postulacin
que conduce a contradiccin es absurda y, por tanto, debe ser
rechazada.
Un ejemplo dar operatividad a esta explicacin.

( 1 ) A - > ( B a C)
(2) ( B v D ) -> E
(3) D v A /.-. E
4. ~E Pr. Ad.
5. ~( B v D ) MT en (2), 4
6. ~B a ~ D DM en 5
7. ~D Simp. en 6
8. A SD en (3), 7
9. B a C MP en (1), 8
10. B Simp. en 9
11. ~B Simp. en 6
12. B a ~B Conj. en 10, 11
13,. ~ E ( B a ~ B ) PC en 4, - 12
14. E RAAen 13
1'5. E DN en 14
Los textos frecuentemente dan por concluida la deduccin
en la lnea 12, cuando se deduce la contradiccin. Eso es una abre
viacin. La idea completa es como se muestra en la deduccin
nterior.

164
X
SIMPLIFICACIN DEL LENGUAJE
PROPOSICION AL PM

Objetivos:

Identificar el lenguaje Nicod como un lenguaje capaz de expre


sar de manera completa la lgica proposicional standard me
diante el uso de un solo operador.
Conocer y manejar las reglas del algoritmo de Post, para la tra
duccin de una frmula cualquiera en el lenguaje PM.
Comprender la importancia del lenguaje de Nicod en la tecno
loga del diseo de los circuitos que constituyen la arquitectura
del computador electrnico.
CUESTIONARIO 10
Simplificacin del lenguaje proposicional PM

Instrucciones

I. Responder a las siguientes preguntas:

1. Hasta cuntas conectivas de dos argumentos puede definirse?


2. Puede seleccionarse slo dos conectivas para construir un len
guaje proposicional adecuado?
3. Es posible enumerar las funciones de verdad didicas que no
tienen uso en el lenguaje PM?, cules son?
4. Existe alguna funcin de verdad didica equivalente a la ne
gacin de la variable p y otra equivalente a la negacin de la
variable q l
5. Entre las funciones de verdad de dos argumentos cuntas son
funciones de valor constante?
6. Si a la negacin de una funcin de verdad f la denominamos
funcin inversa de f , es verdad que cada funcin de verdad de
dos argumentos tiene su funcin inversa?
7. Si consideramos funciones de verdad de tres argumentos o va
riables proposicionales y las denominamos funciones tridicas
cuntas funciones de verdad tridicas puede definirse?
8. Cmo se prueba que toda frmula de PM es traducible al siste
ma de Nicod?
9. Es el lenguaje lgico ms simple, a la vez, el de manejo ms
sencillo?

[167]
10. Por qu puede afirmarse que el sistema de Nicod expresa el
ideal de la Navaja de Occam?
11. Podra el lenguaje construido por Nicod reemplazar al de PM?
12. Es verdad que una frmula del lenguaje PM admite una nica
traduccin en el lenguaje de Nicod?

II.Traducir al lenguaje de Nicod las siguientes frmulas (en este


ejercicio usaremos letras maysculas):

1. ~ ( ~ ( A a B ) a ~ ( D a E ) )
2. ( ~ A a ~ B ) v ( ~ B a C )
3. ~ ( A v B ) v ~ ( C v D )
4. ~ ( ~ ( ~ ( C a D ) a ~ E ) a ~ ( ~ A a B ) )
5. ( A a B ) v ( C a D )
6. ~ ( ( ~ A v ~ B ) a ( ~ C v ~ D ) )
7. ~ ( ( A - > B ) a ( B - > A ) )
8. ~ (( A v B) a ( A B ))
9 . ~ ( ~ ( A B ) a ( C v D))
10. (( A B ) a A ) B
11. ( A v B ) -> ( B v A)
12. A -> ( B v A )

168
LECCIN 10
Simplificacin del lenguaje proposicional PM

10.1. Las 16 funciones de verdad posibles

Es importante sealar que las conectivas que utiliza el lenguaje


PM son las que parecen ms adecuadas para la formalizacin del
lenguaje cientfico y del lenguaje natural. Sin embargo, ellas son
slo una parte del conjunto de las funciones de verdad de dos
argumentos. Anteriormente, hemos definido cada conectiva di-
dica como una funcin f : W2 -> W, donde W = { V, F }, lo que
equivale a afirmar que las matrices de la conjuncin, disyuncin,
etc., son elementos de un conjunto de 16 funciones de verdad di-
dicas posibles, como lo demostraremos a travs de la siguiente
tabla.

p q f f f f f5 f f f f f1 f 2 f3 f4 f5 f6

V V V V V V V V V V F F F F F F F F
V F V V V V F F F F V V V V F F F F
F V V V F F V V F F V V F F V V F F
F F V F V F V F V F V F V F V F V F

La tabla anterior muestra que su margen, que est constitui


do por los cuatro pares ordenados del producto cartesiano W x
W (W 2), puede ser asociado con 16 matrices posibles. Es visible
que la funcin f2 corresponde a la disyuncin inclusiva, f5 al con
dicional, f 7 al bicondicional, f8 a la conjuncin y flO a la disyun
cin exclusiva. Las otras once matrices no tienen normalmente
uso en PM, sin embargo, algunas de ellas podran ser especial

[169]
mente tiles, lo que de hecho ocurre con f9 y f l5 que se conocen
como operadores de Sheffer. El primero se denomina operador
de incompatibilidad y el segundo de negacin conjunta. Aunque
ambos se traducen de manera poco comn al lenguaje ordinario,
son especialmente productivos en los usos tericos y tecnolgicos
de la lgica proposicional. En esta seccin dedicaremos especial
atencin al operador de incompatibilidad, el mismo que sirvi a
Nicod para crear un sistema lgico particularmente interesante
por sus aplicaciones a la tecnologa.
Para representar la funcin f9 como conectiva usaremos el
signo"/', conocido tambin como barra de Nicod. Su tabla de ver
dad es:

p q p/q
v v V
V V V
F V V
F F F

Una inspeccin mostrar que la matriz de una frmula de


incompatibilidad es equivalente a la de ~ ( p a q ). Esto nos
permitir definirla explcitamente en los trminos siguientes:
/ _ \
Definicin 17. Unafrm ula de incompatibilidad p / q e s fa lsa s i , y slo si, sus dos
variables proposicionales p, q son verdaderas. En cualquier otro caso el valor de
su matriz ser verdadero.

10.2. Lenguaje de Nicod

Se conoce como lenguaje de Nicod a un lenguaje capaz de expre


sar de manera completa la lgica proposicional standard a tra
vs del uso exclusivo del operador , denominado barra de Ni
cod, y de signos de agrupacin ordinarios como parntesis, cor
chetes, etc.
En trminos operacionales la frmula 'p/ q' se define como la
negacin de la conjuncin ordinaria, lo que claramente establece
la validez de la equivalencia '( p / ^ ) o ~ ( p a ^ Desde el
punto de vista operacional, la traduccin de las frmulas de la
lgica proposicional, escritas usualmente en el lenguaje de Prin

170
cipia Mathematica (PM), al lenguaje de Nicod se facilita mucho
cuando se utiliza como intermediario al lenguaje de Post, el mis
mo que se limita a usar exclusivamente los operadores de ' a ' y de
ms los usuales signos de agrupacin.
Las traducciones ordinarias del lenguaje de PM al lenguaje
de Post son las que a continuacin escribimos:

D I. (pvq) = Df~(~pA~q)
D2. (p q ) = Df ~ ( p a ~ q )
D3. ( p< - q) = D f ~ ( p A ~ q ) A ~ ( ~ p A q )
D4. (p*q) = Df~(pAq)A~(~pA~q)

Las traducciones anteriores son operacionalmente equivalen


cias en el sentido de que la tabla de verdad que corresponde a la
frmula de la izquierda tiene la misma matriz final que la tabla
de verdad de la frmula que se encuentra a la derecha de la defi
nicin. A la frmula de la izquierda se le conoce como D efi-
niendum, y a la de la derecha como Definiens. Esto significa que
las cuatro definiciones anteriores pueden ser entendidas como una
mquina muy simple que tiene la propiedad que consiste en que
cada vez que introducimos en ella un Definiendum, en el len
guaje de PM, emite com o salida u n D e fin ie n s en el lenguaje
de Post. La nica excepcin a las definiciones anteriores est
dada por las frm ulas que son tautologas. Sin em bargo,
ello se subsana con sencillez cuando la traduccin al len
guaje de P ost no se realiza intuitivam ente, sino se la regula
a travs de u n conjunto de reglas de operacin de eficacia
inevitable que se denom ina algoritm o.

10.3. Algoritmo de Traduccin de Post

Consideremos que toda frmula de la lgica proposicional se es


cribe utilizando las letras p, q, r,..., etc. como variables proposi
cionales distintas. Sin embargo, tambin se pueden escribir tan
tas variables proposicionales distintas como se desee, utilizando
slo una letra a la cual se le aade cada vez que es necesario un
subndice distinto. As, por ejemplo, asumimos que P , P2,..., Pn
son n variables proposicionales distintas. Evidentemente, en cada

171
caso concreto el valor especfico de n depender del nmero de
variables distintas que necesitamos.
Asimismo, tomando como referencia la tabla de verdad de
una frmula de PM cualquiera, estableceremos que con la expre
sin Ai podemos referirnos a cualquiera de los 2n arreglos de
valores verdadero-falso que existen en el margen de la Tabla y
con la expresin Fi a cualquiera de los valores falsos que apare
cen en la matriz final de dicha tabla. Consecuentemente, diremos
que un valor falso Fi corresponde al arreglo Ai. En el caso de una
tabla de verdad para una frmula que tiene slo las variables
proposicionales P l, P2, P3, es claro que el valor de n es igual a 3 y
el nmero de arreglos verdadero-falso es 8. Si la matriz de dicha
frmula fuera falsa en el primer y en el ltimo arreglo, entonces
Fi se transformara en F l en el primer caso y en F8 en el segundo
caso. A F l le correspondera el arreglo V W y a F8 le correspon
dera el arreglo FFF.
Asumiendo las convenciones anteriores, las reglas del algo
ritmo de Post, para traducir una frmula W, dada en el lenguaje
de PM, a otra WO en el lenguaje de Post, debe procederse de la
siguiente manera:
RP1. Se identifica las n letras variables proposicionales que
aparecen en W y se procede a construir su tabla de verdad para
sus 2n arreglos de valores verdadero-falso. Existen slo dos posi
bilidades relevantes para W segn las caractersticas de su matriz
final. O la frmula es una tautologa porque posee slo valores
verdaderos para sus 2n arreglos, o no es una tautologa y tiene un
nmero n de valores falsos tal que n es siempre mayor que cero.
^RP2. Si W no es una tautologa, se procede a construir un
nmero n de espacios entre parntesis; se antepone a cada uno de
dichos espacios el signo de negacin y entre cada par de espacios
entre parntesis se escribe el signo de conjuncin. De esta mane
ra, cada uno de los espacios entre parntesis corresponde a cada
uno de los valores Fi y a cada uno de los arreglos Ai en los que W
resulta falsa en la tabla de verdad. Por ejemplo, si una frmula
tiene en su matriz principal 3 valores falsos ( n = 3) se escribirn
tres espacios entre parntesis as

( .......................... ) A ~ ( .................................) a ~ ( .............................)

172
RP3. Escribir dentro de cada espacio entre parntesis corres
pondiente a cada Fl las variables proposicionales P l, P2,..., Pn de
tal manera que si en el respectivo arreglo Ai una letra Pi (1 < Pi< n ),
tiene asignado el valor verdadero, entonces se la escribe afirmada
y si tiene asignado el valor falso, se la escribe negada. Entre cada
par de letras cualquiera, escrbase el signo de conjuncin.
RP4. Si W es tautologa, entonces escrbase slo un espacio
entre parntesis, antepngasele el signo de negacin, escrbase
dentro de l las P l, P2,...,Pn letras variables proposicionales y
adase a ellas ~Pn. Entre cualquier par de letras escrbase el
operador de conjuncin.
Las reglas anteriores dan lugar a que toda frmula de la
lgica proposicional en el lenguaje de Post tenga la forma de ~
( . . . a . . . a . . . a ...) o de conjunciones cuyos componentes re
producen el esquema anterior. Cuando una frmula rene es
tos requisitos se afirma que constituye una Forma Normal de
Post y por ello a las cuatro reglas anteriores se les llama algo
ritmo de normalizacin de Post. Como ejercicio elemental de
aplicacin, el lector puede aplicar el algoritmo de Post a las
frmulas de las definiciones D I, D2, D3 y D4. Haciendo la ta
bla de verdad de cada D efiniendum , obtendr el D efiniens,
mecnicamente, por el algoritmo de Post.

10.4. Traduccin algortmica al lenguaje de Nicod

Ahora, volviendo a nuestro objetivo inicial, ocurre que debido a


que el operador de Nicod, llamado tambin de incompatibilidad,
se define mediante la negacin de la conjuncin y a que la forma
normal de Post consiste de una o ms conjunciones negadas, en
tonces resulta sencillo establecer un algoritmo para traducir fr
mulas que contienen a lo ms dos variables proposicionales dis
tintas, y que estn escritas en el lenguaje de Post, a frmulas en
el lenguaje de Nicod. Las reglas son:

rl. Toda frmula de la forma ~ ( p A q ) se traduce por la frmu


la de la forma p / q .
r2. Toda frmula de la forma ~p se traduce por p /p .
r3. Toda frmula de la forma p A q se traduce por ( p / q ) / ( p /q ) .

173
Es importante advertir que cuando hablamos de una fr
mula de la forma... es necesario tomar esta expresin con mucho
cuidado. Por ejemplo, una frmula de la forma ~ A puede ser ~q
o tambin una que anteponga la negacin a un parntesis, tal como
~ ( V A *7 ) Lo que define la forma en este caso es que la frmula
en cuestin tenga como operador de ms alta jerarqua al de ne
gacin. Anlogamente, una frmula de la forma A a B es cual
quier frmula que tiene el operador de conjuncin como el de
ms alta jerarqua. Esto significa que las formas especficas del
componente p y del componente q quedan sin determinacin al
guna. Puede ocurrir que a su vez tanto p como q sean de la forma
del esquema ~ ( .... ), aunque esto ltimo no tiene que ser necesa
riamente as.
En el caso de frmulas en el lenguaje de Post, que tienen ms
de dos variables proposicionales, es suficiente dar una regla adi
cional que se aplica igualmente a las expresiones que estn den
tro de los parntesis como a la jerarquizacin de las conjunciones
externas a los parntesis.
r4. Si se tiene una conjuncin de la forma P1 a P2 a ... Pn debern
usarse parntesis, tantos como sean necesarios para que la frmu
la adquiera la forma A a B. Realizado este proceso se aplica r3.
Consecuentemente, mediante la aplicacin mecnica y mo
ntona de las reglas RP1, RP2, RP3, RP4 y r l, r2, r3, r4 podemos
transformar toda frmula de la lgica proposicional del lenguaje
PM, lenguaje standard, en una frmula en el lenguaje de Nicod.
A continuacin desarrollaremos un ejemplo completo.
Sea W una frmula dada en el lenguaje de PM, tal que W es:

( ( p - > q ) A ( q - > r ) ) - ( r - > p )

Para construir su correspondiente frmula WO, conocida como


forma normal de Post, aplicamos RP1 construyendo su tradicio
nal tabla de verdad.

174
pqr ((p->q)A(q->r))->(r->p)
l.V VV V V V V V
2.V VF V F F V V
3.V FV F F V V V
4.V F F F F V V V
5.F VV V V V F F
6.F VF V F F V V
7.F FV V V V F F
8.F F F V V V V V
(1) (3) (2) (5) (4)

En este caso W no es tautologa y el nmero n de valores


falsos en su matriz principal es 2. En la quinta lnea y en la spti
ma, esto es lo que denotamos como F5 y F7 que corresponden al
arreglo A5 que es F V V y al arreglo A7 que es F F V, respec
tivamente. De esta manera construimos W aplicando RP2 y RP3,
lo que da como resultado la conjuncin de los dos siguientes pa
rntesis negados, uno por cada Fi.

W: ~ ( ~ p a q A r) a ~ ( ~ p A ~ q A r)

A esta frmula, que hemos denominado WO, le aplicaremos


las reglas r l, r2, r3, r4 para obtener una nueva frmula W00 que
ser la traduccin de WO, luego por transitividad W00 ser tra
duccin de W.
Por razones de comodidad transformaremos primero el pri
mer componente de la conjuncin, el mismo que por tener ms de
dos variables proposicionales, aplicando r4, lo reescribiremos as:

~ ( ( ~ p a q) a r)

aplicando r l tendremos:

(~PAq)/r

175
lo que aplicando r2 resulta en (( p / p ) a q ) / r, donde el compo
nente '({p / p ) A q ) ' es de la forma A a B. Debido a ello aplican
do r3 obtenemos:

'(((p/p)/q)/((p/p)/q))/r',

que denominaremos abreviadamente D. Aplicando el mismo pro


cedimiento al segundo componente conjuntivo de W obtenemos:

' ( ( ( p / p ) / ( q / q ) ) / ( ( p / p ) / ( q / q ) ))/*',

que denominaremos abreviadamente E.


De lo anterior se deduce que la frmula W ha quedado trans
formada por ahora en el esquema que abreviadamente se escribe
D a E. Pero como este esquema mantiene todava el signo de ma
yor jerarqua de W, que es el operador externo de conjuncin, en
tonces debemos aplicarle r3 y as obtenemos el esquema:

W "(D/E)/(D/E)

que es el esquema de la nueva frmula en el lenguaje de Nicod, a


la que hemos llamado W . Ella se escribe explcitamente, reem
plazando D y E por las frmulas que representan, de la siguiente
manera:

(((p/p)/q)/((p/p)/q))/r./.(((p/p)/(q/q))/((p/p)/
(q / q ) ) )/*:/: ( ( ( p /p ) /q ) /( ( p /p ) /q ) ) /r ./.( ( ( p /p )
/ ( q / q ))/
(fp/p)/(q/q)))/r

Dada la longitud de la expresin, para no abundar en parnte


sis, que dificultan la lectura, hemos recurrido a un mtodo de Russell
para establecer la jerarqua de los operadores usando puntos, con
la variante de que Russell tiende a prescindir de los parntesis, lo
que nosotros no hacemos, pues hemos optado por una alternativa
mixta. La regla de jerarqua prescribe que entre cualquier par de
operadores (en este caso barras de Nicod), igualmente externos a
los parntesis, tiene mayor jerarqua el operador que posee en sus

176
flancos el mayor nmero de puntos. En nuestro ejemplo, la barra
punteada :/: es el operador de mayor jerarqua y divide la expre
sin en dos compo-nentes iguales. A su vez cada componente est
subdividido de manera semejante en dos segmentos por la barra
punteada /. que obviamente tiene mayor jerarqua que una barra
sin puntos. Como se comprender, el esquema ( D / E ) / ( D / E )
tiene en esta exposicin una funcin pedaggica y no lgica. Su
escritura es omitible si se tiene prctica en el manejo de frmulas
de longitud mayor que la de una lnea.

10.5. El Ideal de Simplicidad

Es importante advertir que si bien el procedimiento algortmico


que hemos propuesto nos conduce necesariamente desde una fr
mula W de PM a otra W00 en el lenguaje de Nicod, sin embargo,
una misma frmula de PM tiene traducciones alternativas en el
lenguaje de Nicod, igualmente vlidas pero con algunas propie
dades distintas, como la de simplicidad. Por ejemplo si a 'p <- cf
de D3 le aplicamos las reglas anteriores, obtendremos:

'((p/(q/q))/((p/p)/q))/((p/(q/q))/((p/p)/q))'

Pero si a 'p<-> q* se le aplica la conocida equivalencia'(p a q )


v (~ P A ~ Y y se le transforma por la regla de De Morgan, se
llega a una frmula muy parecida a la forma normal de Post que
permite obtener directamente: '(p / q ) / ( ( p / p ) / ( q / q ) ) ' .
Esta frmula es una traduccin igualmente vlida que la anterior
pero ms breve y, por tanto, de ms fcil manejo. Sin embargo, el
argumento de la brevedad es importante pero no decisivo. Ocu
rre que hay frmulas que tienen mayor productividad deductiva
que sus equivalentes ms breves. Consecuentemente, cuando se
trata, por ejemplo, de elegir axiomas, lo ms importante es la pro
ductividad deductiva de las frmulas que se eligen y en segundo
lugar la brevedad.
De otra parte, la simplicidad no siempre se entiende en tr
minos de brevedad. Por ejemplo, la construccin de lenguajes
como el de Nicod est animada por un ideal de simplicidad ms
que de brevedad. En efecto, se considera que un lenguaje lgico

177
es ms simple que otro cuando usa un nmero menor de opera
dores. En este sentido el lenguaje de Post es ms simple que el de
PM, pues las presentaciones ordinarias de PM contienen 6 opera
dores, mientras que el de Post, como hemos visto, slo 2. El len
guaje de Nicod va an ms lejos porque utiliza un solo operador
para expresar cualquier frmula de la lgica proposicional. Este
operador fue tambin conocido por Sheffer pero fue Nicod el que
construy una axiomatizacin completa de la lgica proposicional
que podemos considerar la ms simple posible porque utiliza un
solo operador. Sin embargo no es la ms breve u operativa para
los seres humanos porque las frmulas resultan bastante largas.
Existe otro lenguaje igualmente simple que el de Nicod construi
do mediante la daga de Sheffer, la misma que se escribe as: '4/.
Empero, las frmulas obtenidas por este medio no han sido igual
mente interesantes para las aplicaciones de la lgica, especialmente
al diseo de circuito para computadoras.

10.6. La Navaja de Occam

El ideal de simplicidad o de economa es de vieja data en el pen


samiento occidental desde que Guillermo de Occam formulara el
famoso enunciado Entia non sunt m ultiplicanda p raeter necesi~
tatem (No multiplicar innecesariamente las entidades) conocido
como la Navaja de Occam. En este caso tomara la forma No
multiplicar innecesariamente los operadores lgicos.
Pero Nicod no slo se propuso construir un sistema lgico
con el mnimo posible de operadores sino con el mnimo posible
de axiomas. En su tiempo el sistema PM tena cinco axiomas para
laf lgica proposicional; la simplificacin de Hilbert-Ackerman
redujo los axiomas a cuatro; Lukasiewicz propuso una axioma-
tizacin con slo tres axiomas, pero todo ello fue excesivo en rela
cin con el ideal de Nicod de no multiplicar innecesariamente los
axiomas. De este modo propuso, utilizando el lenguaje ms sim
ple posible, la axiomatizacin de la lgica proposicional con el
mnimo nmero de axiomas, esto es, uno. Asimismo, utiliz una
nica regla de deduccin diferente del tradicional Modus Ponens
utilizado en PM y sistemas afines. El axioma de Nicod para la
lgica proposicional es el siguiente:

178
( P/ ( Q/ R)) / ( ( T/ ( T/T )) / ( ( S / Q) / ( ( P/S ) / ( P/S ) ) ) )
La regla de deduccin es:

Desde P y P / ( R / Q ) se deduce Q

Advertimos que hemos utilizado letras maysculas porque el


anterior, aunque puede ser entendido como axioma, es propiamente
un esquema axiomtico, concepto que no se utiliz en PM, sistema
que requiri una regla adicional de sustitucin de variables
proposicionales que es innecesaria cuando se usa el concepto de
esquema axiomtico. Los axiomas tradicionales, tipo PM, son res
pecto a los esquemas axiomticos ejemplos concretos, dados en el
lenguaje de PM, de una estructura general o de un conjunto de
estructuras generales que no pertenecen a PM sino al metalenguaje
de PM. Con este mismo criterio qued establecido que las reglas de
deduccin no pertenecen al sistema axiomtico sino a su metalen-
guaje. De este modo, los sistemas axiomticos lgicos contempor
neos se diferencian de la versin original de PM en que demues
tran, ms propiamente, metateoremas en lugar de teoremas.
La potencia deductiva del sistema de Nicod (SN) se aprecia
si se considera que dentro de l son deducibles como teoremas los
axiomas del sistema Hilbert-Ackerman (HA) y, por ende, los de
PM. Esto ha llevado a una mejor comprensin del concepto tradi
cional de axioma que ha sido definido por su autoevidencia y su
carcter absoluto. Lo errada que era la concepcin tradicional se
comprueba con el hecho de que el axioma de Nicod, por ejemplo,
no es evidente, pues la alta redundancia del operador' / ' obliga a
un anlisis para establecer su validez. Asimismo, el hecho de que
los axiomas de HA y PM aparezcan en SN como frmulas deriva
das o teoremas, permite comprender claramente que el concepto
de axioma es sostenible solamente si se lo define en relacin a una
teora. Consecuentemente, no se puede hablar rigurosamente de
axioma 'A' en general sino del axioma 'A' de la teora axiomtica
T. Pues es perfectamente posible que en otra teora, por ejemplo
T , 'A' sea teorema y no axioma.

179
10.7. Traduccin del sistema Hilbert-Ackerman

Los esquemas axiomticos de HA son:

e l. ~ ( P v P ) v P
e2. P v ~ ( P v Q )
e3.-(PvQ)v(QvP)
e4. ~ ( ~ P v Q ) v ( ~ ( R v P ) v ( R v Q ) )

La traduccin de estos axiomas requiere de una regla de tra


duccin de frmulas de la forma A v B al lenguaje de Nicod. Si se
toma la forma normal de Post, el procedimiento antes expuesto
conduce a la forma ( A /A ) / ( B/B ) en el lenguaje de Nicod. Esto
conduce a un proceso altamente redundante que para el esquema
el., da lugar a:

( e l* ) ( ( ( ( P / P ) ) / ( ( P / P ) / ( P / P ) ) ) / ( ( ( P / P ) / ( P / P ) ) / ( (
p/p ) / ( p/p ))))/( p/p )

La frmula anterior es ms simple que e l. en el sentido de


que est expresada a travs de un slo operador, pero es menos
breve y menos intuitiva. Es ms simple en el sentido de ms ho
mognea, pero por este mismo hecho es altamente reiterativa y
montona. A los seres humanos las operaciones montonamente
repetids nos agotan rpidamente y llevan a errores mecnicos,
ya sea por fallas en la coordinacin visomotriz o por cansancio de
nuestros rganos sensoriales. Sin embargo, las mquinas no es
tn igualmente expuestas a estas debilidades. Por ello el esquema
E puede ser fcilmente ledo por una computadora que est capa
citada para repetir la misma operacin millones de veces por se
gundo o por fracciones de segundo. Bajo tales condiciones la fr
mula anterior resulta breve y operativa. Lo difcil para la compu
tadora es variar sus rutinas. Cuanto menos variaciones hay en la
rutina, el proceso es ms rpido. Esto explica por qu el lenguaje
de Nicod es ms interesante que el de PM y sistemas afines cuan
do se trata de disear un computador. Permite reducir los dife
rentes operadores de la lgica proposicional standard a una sola
rutina. As resulta irrelevante que para demostrar e l en SN se

180
requieran diecisis largos teoremas previos, pues procesados a la
velocidad, de un computador los mes de pasos previos se cum
plen en lapsos brevsimos.

10.8. Lenguaje de Nicod y Tecnologa

Y lo anterior es posible porque el lenguaje de Nicod ha facilitado


el diseo de los circuitos internos del computador que se han con
vertido en millones de artificios llamados compuertas (en ingls
Gates), que realizan elctricamente el comportamiento de la fr
mula 'p/ q'. La compuerta conocida como NAND (abreviacin de
NO-AND), repetida tantas veces como se desee, permite cumplir
el trabajo de cualquier frmula de la lgica proposicional. Si a ello
se aade el hecho de que los tradicionales valores verdadero-fal-
so pueden traducirse sin dificultad a los dgitos 0,1, entonces las
combinaciones de valores verdadero-falso de las tablas de verdad
se convierten en nmeros escritos en el sistema binario y las ope
raciones lgicas, adecuadamente combinadas, se convierten en
operaciones aritmticas. Con un par de compuertas NAND se
puede construir, por ejemplo, un artificio conocido como flip-flop
(FF) el mismo que puede ser utilizado para construir unidades
de memoria y para cumplir procesos de correccin mediante sus
feed-backs.

NAND

D NAND

FLIP-FLOP
Las frmulas que gobiernan este flip-flop son:

D = A/B; B = C/D; D=A/(<7D); B = C/(A/B)

Adems de ellas, para comprender su funcionamiento, es


necesario considerar el sentido de la secuencia temporal y tener

181
informacin sobre el estado anterior. A simple vista se advierte
que este flip-flop permite el almacenamiento de la salida A /B en
la segunda compuerta y el retorno de la informacin que trans
porta la salida C /D a la primera compuerta, lo cual puede permi
tir recuperar informacin o corregir. En este simple ejemplo esta
mos operando con nmeros binarios que no exceden los tres
dgitos, pero en la medida que podemos prolongar el flip-flop a
voluntad, aadiendo nuevas compuertas NAND, entonces pode
mos operar con cifras de magnitud considerable.
De este modo hemos podido tener una visin suficiente de la
importancia terica y tecnolgica del lenguaje de Nicod. Si no se
le visualiza desde esta perspectiva, parecera el fruto de una pa
sin intil por la simplicidad. Ello, afortunadamente, no es as,
pues SN posee potencia deductiva y productividad tecnolgica,
a pesar de que la idea tiene algo ms de setenta aos. Ciertamente
es muy anterior a la construccin del primer computador electr
nico con circuitos lgicos, tarea que cumpli Shannon. Sin embar
go, Shannon utiliz los clsicos circuitos en lnea-paralelo que
constituyen una etapa previa al uso de las computadoras. Cuan
do esto ocurri, el montono y redundante lenguaje de Nicod re
sult el ms adecuado a las caractersticas rutinarias del compu
tador, para el cual la longitud de los algoritmos carece de rele
vancia. El problema de fondo se presenta cuando estamos ante
un problema lgico, matemtico o real, el cual no es reducible a
una rutina conocida. En ese caso el computador tiene poco que
hacer. Pero la situacin es ms radical cuando estamos ante con
juntos de frmulas que se demuestra que no es posible someter
las a rutina alguna. Esto es, que no admiten solucin en trminos
dereiteradas operaciones montonas. En este caso se dice que tal
conjunto de frmulas no admiten solucin algortmica y, conse
cuentemente, el computador all no tiene lugar alguno. Un ejem
plo de conjunto de frmulas de este tipo lo proporciona la aritm
tica de Peano formalizada axiomticamente en el lenguaje de PM
y otro una teora lgica de primer orden con predicados n-arios.

182
XI
BASES LGICAS
DE LA INTELIGENCIA ARTIFICIAL

Objetivos:

Entender la manera como el lenguaje P.M. puede aplicarse al


diseo de circuitos que constituyen la arquitectura de los compu
tadores electrnicos (hardware).
Aplicar el lenguaje P.M. a la. construccin de circuitos en serie
y en paralelo.
Aplicar el lenguaje P.M. a la construccin de circuitos a com
puertas.
Entender los mecanismos que permiten convertir a una tabla
de verdad en una tabla aritmtica.
Entender los mecanismos que dan lugar a que un computador
ejecute operaciones aritmticas y simule la conducta inteligente.
CUESTIONARIO 11
Bases lgicas de la inteligencia artificial

Instrucciones

I. Responda a las siguientes preguntas.


1. Cmo se define un sistema experto?
2. Qu diferencia puede establecerse entre un sistema experto y
un robot?
3. De qu trata la teora de autmata?
4. Qu diferencia existe entre cuestiones de software y cuestiones
de hardware ?
5. Cul es el mecanismo mediante el cual las tablas de verdad
permiten realizar operaciones aritmticas?
6. En qu consisti el aporte de Shannon al diseo de
computadoras?
7. En qu consisti el aporte de Hao Wang?
8. Es posible construir un circuito a compuertas para cualquier
frmula del lenguaje PM?
9. Existe para todo circuito en lnea y paralelo un correspondien
te circuito a compuertas?
10. Cul es el tipo de compuerta ms compatible con la Forma
normal de Post?
11. Qu diferencia existe entre autmata fsico y autmata ideal?
12. En qu consiste el problema del halting1
13. Puede describirse una tarea que est fuera del alcance de cual
quier computador?
14. Qu es una mquina de Turing?

[185]
15. Se ha elaborado la teora de autmata examinando computa
dores reales o fsicos?
16. Cuntos dgitos binarios se necesitan para escribir 64 y 127?

II. Construir circuitos para:


1. ( p A q ) v ( p v q )
2. ( p v ( q A r ) ) A ( r v s ) )
3. ( p a q ) v ( ~ q a r ) v ( ~ P a ~ s )
4 . ( p A q A r ) v ( p A q A - r ) v ( ~ P A~ q )
5. Cul es a frmula del circuito siguiente?

III. Construir un diagrama de circuito a compuertas para cada una


de las frmulas del ejercicio II y convertir el plano de circuito
de II.5. tambin a un circuito a compuertas.

IV. Cul es la frmula que corresponde al siguiente circuito a com


puertas?

186
LECCIN 11
Bases lgicas de la inteligencia artificial

11.1. Sistemas expertos y robots

A partir de 1960 se ha desarrollado intensamente una nueva dis


ciplina conocida con el nombre de Inteligencia artificial, a la mis
ma que usualmente se hace referencia con la abreviacin IA. Den
tro de este campo de investigacin se emplea como herramienta
principal de trabajo el computador digital.
La idea central de la IA es la construccin de programas que
ordenen a un computador adecuado que simule lo que normal
mente se reconoce como una conducta inteligente. Por tanto, los
investigadores en IA, propiamente, no se proponen la construc
cin de artefactos inteligentes sino de simuladores de la con
ducta inteligente. Se considera iniciadores de esta disciplina a J.
McCarthy, M. Minsky, Papert,R. Schank, T. Winograd y D. Marr,
entre otros
Dentro de este contexto se considera, que la conducta de un
mdico auscultando a un paciente para hacer un diagnstico, la
de un cajero de un banco atendiendo a un cliente en la ventanilla
o la de un abogado resolviendo el curso ms adecuado que debe
darse a un expediente judicial, son ejemplos de las conductas de
un experto en un determinado campo. Uno de los temas ya clsi
cos en IA es elaborar programas que ordenen a una computadora
que simule la conducta de un cierto experto. A los programas de
esta clase se los denomina Sistemas expertos.

[187]
En otros casos no se trata de simular conductas sino, por
ejemplo, el funcionamiento del cerebro. Estas investigaciones
comenzaron con los esfuerzos de McCullochy Pitts que justamente
utlizaron el lenguaje proposicional de PM para construir un mo
delo que simule el funcionamiento de las neuronas. Con objetivos
semejantes F. Rosenblatt cre una mquina llamada Perceptron
que se propona la simulacin de la funcin retiniana en el acto
de la visin. Esta lnea de investigacin no se propone solamente
crear programas sino robots con una estructura fsica que les per
mita ejecutar tareas especficas.

11.2. Hardware y Software

El lenguaje de la lgica proposicional, en la versin que desarro


llamos en este libro o en la que lo presenta como un algebra de
Boole, ha sido decisivo para el diseo de los circuitos elctricos
que constituyen la estructura fsica o la arquitectura de un com
putador digital. De esta manera se ha contado con un mtodo al
gebraico que, en lo principal, permite calcular, por decirlo as,
dado un trabajo determinado, cul es el circuito ptimo que pue
de realizarlo. Consecuentemente, el lenguaje proposicional PM es
conocido, dentro de la jerga especializada, como la base terica
ms fuerte del hardware del computador debido a que se aplica
al diseo de su estructura fsica.
Empero, el hardware es solamente un aspecto dentro de la
problemtica que plantean los computadores. El otro, es el rela
cionado con el software y se refiere a la creacin de los lenguajes
especiales que se requiere para dar rdenes al computador y las
formas como debe representrselas. Estos lenguajes, conocidos
tambin como lenguajes de mquina, requieren de una gramti
ca, la misma que hoy da se ha diversificado en varios tipos. El
lenguaje lgico tambin constituye la base terica principal para
el desarrollo de las gramticas de los lenguajes de mquina. Y el
estudio de las relaciones entre las gramticas y las computadoras
constituye la Teora de autmata.
El lenguaje PM desarrollado en este manual es insuficiente
para aplicarlo a las gramticas de los lenguajes formales o de
mquina, razn por la que en esta seccin nos limitaremos a brin

188
dar los conocimientos ms elementales que permitan entender la
manera como el lenguaje PM, aqu desarrollado, puede aplicarse
al diseo lgico de circuitos, el mismo que es un tema de hardware.

11.3. Mquina de Turing

A las computadoras tambin se les denomina autmatas y es ne


cesario distinguir entre un autmata terico o ideal y otro real o
fsico. Lo ms inmediato es precisar que un autmata real o fsi
co es lo que constituye propiamente la materia de los problemas
de hardware y es un artefacto sujeto a un conjunto grande de con
tingencias. Por ejemplo, puede dejar de funcionar por falta de flui
do elctrico, entregar resultados incorrectos por desgaste de una
pieza, incendiarse, etc. En cambio, un autmata ideal es una des
cripcin lgica de un artificio infalible que no est sujeto a nin
guna contingencia material. Es ms, desde esta perspectiva tam
poco interesa si existen los recursos para construir un artefacto
que sea capaz de asumir los estados y realizar los cmputos pro
puestos en la descripcin del autmata ideal. Y es que ste no es
un artefacto sino, esencialmente, un teorema demostrado dentro
de un formalismo adecuado.
Se conoce como Mquina de Turing (abreviadamente Tm),
con el problema del halting resuelto, a un autmata ideal que no
es otra cosa que la descripcin ms general posible de un artificio
capaz de realizar cualquier cmputo pensable de manera infali
ble. Esto es posible debido a que cualquier estado que asuma la
Tm determina de manera necesaria el siguiente. La transicin de
un estado a otro se realiza mediante reglas preestablecidas que,
adecuadamente aplicadas, conducen inevitablemente a un resul
tado. Tales reglas constituyen un algoritmo y establecen un pro
cedimiento denominado recursivo. Las Tms son mltiples y no
todas tienen resuelto el problema del halting, lo que significa que
no siempre se puede demostrar que la mquina har alto, condi
cin indispensable para que entregue un resultado.
La Tm es la base terica de cualquier tipo de computador
digital, pues si se trata de una artefacto capaz de entregar resulta
dos mediante su unidad de salida, entonces es necesariamente la
materializacin de algn tipo de Tm ideal con el problema del

189
halting resuelto. Asimismo, todo computador material o real slo
puede hacer los cmputos previstos en alguna Tm.
Los resultados reseados antes fueron establecidos, por el l
gico y matemtico ingls Alan Turing, en un famoso articulo publi
cado en 1936, con el ttulo On computable numbers, with an applicatio7i
to the Entscheidungsprblem ( Sobre nmeros computables, una apli
cacin al problema de la decisin), el mismo que fue presentado a
la Sociedad Matemtica de Londres al menos cinco aos antes de
que exista computador electrnico alguno. Este trabajo tiene la enor
me virtud de sealar los alcances y lmites de una Tm y, por tanto,
de cualquier computador real o posible. En el se establece clara
mente que una Tm es un artificio que slo puede hacer cmputos
que requieran un nmero finito de pasos o estados de la mquina.
S i , por ejemplo, se le ordena a una Tm, por poderosa y moderna
que sea fsicamente, que imprima todos los nmeros reales que
existen entre 1 y 0, no podr ejecutar esta orden porque en ese in
tervalo hay un conjunto infinito de nmeros reales que, adems,
no pueden ser contados o enumerados. Y no slo no existe sino que
, lgicamente, no puede existir una Tm capaz de hacer de manera
efectiva un nmero infinito no enumerable de cmputos. De esta
suerte queda claro que la teora lgica permite demostrar no slo lo
que un computador fsico, en el mejor de los casos, puede hacer,
sino tambin lo que sera absurdo esperar de l.

11.4. Diseo de circuitos elctricos para computadoras

La aplicacin de la lgica proposicional a la construccin de cir


cuitos elctricos se debe mayormente al aporte del investigador
Cladio Shannon, quien es uno de los diseadores de las moder
nas computadoras. En lo que sigue explicaremos las nociones ms
elementales de la contribucin de Shannon.
Consideramos que una llave elctrica de las que todos cono
cemos puede ser representada mediante una palanquita; como la
de la figura 1, que solamente tiene dos posiciones posibles: o deja
pasar la corriente cuando se la baja y entonces est en la posicin
de cerrada*, o no deja pasar la corriente cuando est levantada y

* Los trminos abierto y cerrado se usan en sentido inverso en otras


publicaciones.

190
est en la posicin de abierta. Nosotros podemos denominar a la
llave o conmutador con la variable proposicional p. Cuando el
conmutador p est en la posicin de cerrado, diremos que p toma
el valor de V; y cuando el conmutador p est en la posicin de
abierto, diremos que toma el valor de F. Asimismo, con la II llave
p podemos construir un circuito, como el de la figura 1 que tiene
un pedazo de cable de entrada y otro de salida. Se aprecia clara
mente que slo hay una corriente de salida si la llave p est cerra
da, pues en otro caso la corriente elctrica no pasa.
De la misma manera, cuando hay impulso elctrico de salida
y se enciende un foco, diremos que la salida es igual a V; cuando
por ausencia de impulso elctrico el foco no se enciende diremos
que la salida es igual a F.

Foco encendido = V
Foco apagado = F
____ E
Entrada Salida
Figura 1

Ahora, usando los conmutadores p y q construyamos un cir


cuito en lnea como lo muestra la figura 2. La observacin nos
indica claramente que para que el foquito se encienda es necesa
rio que las dos llaves estn cerradas, pues basta que una est abierta
para que no haya corriente de salida y el foco no se encienda. En
otras palabras, para que la salida sea igual a V es absolutamente
necesario que p sea V y que q sea V, pues en cualquier otra posi
cin de las llaves, la salida ser F. Esto significa que un circuito
en lnea se comporta exactamente igual que una conjuncin.
Por tanto, en el lenguaje lgico un circuito en lnea se representa
por una conjuncin p a q cuando hay slo dos llaves. Pero si es un
circuito en lnea con tres llaves o ms se representa por una con
juncin con tres o ms variables proposicionales, una por cada
llave. Por ejemplo, la expresin lgica del circuito de la figura 3 es
p a q a r. Como indicamos anteriormente, en este caso no es nece
sario usar signos de jerarqua.

191
Ahora procedemos a construir un circuito en paralelo como
se muestra en la figura 4. Este circuito tiene a los conmutadores p
y q ubicados uno frente al otro. Como la observacin claramente
lo revela, es suficiente que uno de los conmutadores est en la
posicin de cerrado para que haya impulso elctrico de salida y
se encienda el foquito. Asimismo, para que no haya corriente de
salida es absolutamente necesario que los dos conmutadores es
tn en la posicin de abierto, pues solamente en este caso la co
rriente no pasa al sector del circuito denominado de salida. Con
secuentemente, el circuito en paralelo se comporta como una
disyuncin inclusiva debido a que la salida es igual a V cuando
p es V o cuando q e s V, y la salida es F solamente cuando ambos
conmutadores toman el valor F, lo cual coincide exactamente con
la definicin dada para construir la tabla de verdad de la disyun
cin inclusiva.

Figura 4. Circuito en paralelo para p v q


Tambin presentaremos un conmutador llamado inversor,
que es la versin electrnica de la negacin lgica. Al conmuta-
doj inversor lo denominaremos con la variable p a la que aadi
remos una comilla simple: p '. De esta manera, p r representa el
inversor o negacin de p. La propiedad esencial de p' es que se
encuentra en la posicin de cerrado si y slo si p se encuentra en
la posicin de abierto, y se encuentra en la posicin de abierto si y
slo si p se encuentra en la posicin de cerrado. Vale decir, p' es V
cuando p es F y p' es F cuando p es V. En la figura 5 representa
mos un conmutador inversor.

192
______ 5 ^ _________
Figura 5. Conmutador inversor que representa p

Con los conocimientos anteriores se pueden construir circui


tos ms complejos para expresiones de la lgica proposicional ms
complicadas y que estn constituidas por disyunciones inclusivas,
conjunciones y negaciones. La jerarqua de la expresin lgica se
conserva en el circuito, de tal manera, que si la disyuncin es la
conectiva de mayor jerarqua, entonces el circuito de mayor jerar
qua ser en paralelo y si la conjuncin es la conectiva de mayor
jerarqua, entonces el circuito principal ser en lnea. Por ejem
plo, dada la frmula de la lgica proposicional ( p a q ) v r, su
circuito correspondiente ser uno en paralelo cuyo primer sub-
circuito estar en lnea, como se ve en la figura 6.

--------- ir
b o------ ------------cr o-----------
o------------------o

Entrada Salida

Figura 6

Como fcilmente se entiende, cada vez que se enciende el


foquito equivale a un valor V en la matriz principal de la tabla de
verdad, de la correspondiente expresin lgica. Y cada vez que
no enciende el foquito equivale a un valor F en la misma matriz.
Las diferentes posiciones en que podemos colocar las llaves co
rresponden a los diferentes arreglos del margen de la tabla de
verdad. Consecuentemente, en este caso slo podemos mover las
tres llaves hasta ocho posiciones posibles que coinciden con el
nmero de arreglos de una tabla de verdad para una frmula con
tres variables. Por tanto, este circuito de la figura 6 es exactamen
te el plano de una maquinita que con las veces que se enciende y
las que no se enciende su foquito nos indica cul es la matriz prin
cipal de ( p a q ) v y.
El circuito para ( p v ^ ) A ( r v s ) ser uno en lnea con dos
subcircuitos en paralelo, como se muestra en la figura 7.

193
Figura 7

Asimismo, de acuerdo a lo anterior, a una tautologa debe


corresponderle un circuito que para todas las posibles posiciones
de sus llaves siempre encienda el foquito. Por ejemplo, a la tauto
loga p ~ p le corresponde un circuito en paralelo, que en un
lado tiene una llave y en el otro su correspondiente inversor, de
tal manera que la corriente siempre pasa, puesto que si p estuvie
ra abierto, entonces p f debe estar necesariamente en la posicin
de cerrado y por este lado pasara la corriente. Asimismo, si p r
estuviera abierto, entonces p necesariamente estara en la posi- '
cin de cerrado y dejara pasar la corriente. El plano de este cir
cuito se puede ver en la figura 8.

P
':V
&I
*3?
Entrada Salida

Figura 8. Circuito para p v - p

A continuacin veremos dos ejemplos de construccin de cir


cuitos para dos frmulas lgicas ms complicadas. En cada caso
lo decisivo es mantener la jerarqua de las conectivas en el diseo
del circuito.

a. ( p A ( f q A r ) v s ) ) v ( ~ p a ( ( ~ q ~ r ) v s ))
a

b. (~pAq)v((pv(~pAq))Ar)v(pA(qvrvs))

194
Salida

Figura 9. Circuito para a.


11.5. Circuitos lgicos a compuertas

Los circuitos, que hemos presentado en la seccin anterior, pro


porcionan una idea precisa sobre la forma como la lgica propo
sicional puede ser aplicada al diseo de computadoras electrni
cas. Dichos circuitos que son conocidos como circuitos a conmu
tadores, llaves o switches, han sido, sin embargo, reemplazados
por dispositivos ms giles, conocidos como circuitos lgicos a
compuertas, que son ms acordes con las exigencias de la tecno
loga contempornea.
La necesidad de disear compuertas est ligada al hecho de
que la computadora actual se encuentra en la prctica muy aleja
da de las llaves o switches, a los que ha sustituido gradualmente
por relays, transistores y circuitos integrados (chips). Pero esto
no debe llevar a la creencia de que las compuertas complican el
manejo lgico de los circuitos, pues la situacin es exactamente al
revs. Lo facilitan y permiten visualizar mejor la aplicacin de las
frmulas lgicas.
Una compuerta es un artefacto que en general tiene entra
das y una salida, las mismas que se representan con lneas. El
artefacto mismo se representa convencionalmente por una me
dia luna o por un triangulito y su funcin es dejar o no pasar un
tipo de impulso elctrico, bajo ciertas condiciones.
La figura 11 facilitar considerablemente la comprensin de
lo dicho a travs de un ejemplo que ser el de la compuerta de
conjuncin.

195
A

Figura 11. Compuerta de conjuncin o compuerta


del producto lgico.
Los alambres A y B son las entradas y el alambre A . B es la
salida. Debemos aclarar que A . B significa lo mismo que A B
pero hemos preferido usar el punto en este caso porque se aproxi
ma ms a la idea de producto y la conjuncin es el producto
lgico. Consecuentemente, al diagrama de la figura 11 llamare-
mos compuerta de conjuncin o del producto lgico, o simple
mente, compuerta Y. Su regla de funcionamiento es la siguiente:
R ll. Una compuerta Y, de conjuncin, emite un impulso elc
trico de salida si y slo si todas sus entradas, en este caso A y B,
permiten el ingreso de impulsos elctricos. Al impulso elctrico
de salida lo llamamos abreviadamente impulso A . B
Sin embargo, las compuertas en la prctica no se utilizan para
distinguir entre la presencia de impulso elctrico y su ausencia,
como los conmutadores o switches de los circuitos en lnea y en
paralel. Para que la lgica sea aplicable es suficiente que se distin
ga claramente dos estados que no tienen que ser del tipo todo-nada
como los switches, sino que perfectamente pueden ser del tipo im
pulso elctrico de alto potencial versus impulso elctrico de bajo
potencial; en breve, la dicotoma impulso alto-bajo. De acuerdo a
esto debemos reajustar la Regla 11 en la siguiente forma:
R ll . reajustada. Una compuerta Y emite un impulso de sali
da alto si y slo si todas sus entradas, en este caso A y B, permiten
el ingreso de impulsos altos. En otro caso, vale decir, si al menos
una de sus entradas permite el ingreso de un impulso bajo, enton
ces no se produce el impulso alto de salida A . B.
El diagrama de compuerta de disyuncin o suma es muy
parecido al de la conjuncin. Para distinguirlos se ha convenido
que en este caso la media luna est cortada horizontalmente por

196
la linea que representa al alambre de salida. A esta compuerta se
tiende a llamarla de suma porque la disyuncin es la suma lgi
ca y, abreviadamente, se la denomina compuerta O. La represen
tacin de las entradas y de la salida es la misma que en el caso de
la compuerta Y.

R12. Una compuerta O, de disyuncin o suma, emite un im


pulso alto de salida cuando al menos una de sus entradas, en este
caso A y B, permite el ingreso de un impulso alto. Al impulso alto
de salida lo llamamos abreviadamente A + B, y, evidentemente,
si ms de una entrada permite el ingreso de un impulso alto tam
bin se producir el impulso de salida A + B. En el nico caso en
el que no se emite impulso de salida A + B es cuando ninguna de
las entradas permite el ingreso de un impulso alto.
La tercera compuerta que es necesario diagramar es la del
tipo inversor que corresponde a la negacin lgica. Un ligero an
lisis muestra que en la frmula ~ p el operador de negacin fun
ciona como un inversor de valores, pues cuando p = V se obtiene
~ p = F y cuando p = F se obtiene ~ p = V. En correspondencia con
esta propiedad de la negacin lgica, una compuerta inversora
transforma un impulso alto en un impulso bajo, y viceversa. Su
representacin convencional es un triangulito con un circulito a
la salida.

NO

Figura 13. Compuerta inversora o compuerta No.

197
R13. Una compuerta inversora o compuerta No (se le llama
abreviadamente inversor) emite un impulso de salida alto si la
entrada admite un impulso bajo y emite un impulso de salida
bajo si la entrada admite un impulso alto. Al impulso de salida se
le llama A que significa lo mismo que ~A. El cambio de notacin
se debe a que en el diseo de circuitos se usa un simbolismo
algebraico en lugar del de la lgica proposicional. Sin embargo,
como hemos visto, las equivalencias son muy sencillas.
Generalmente, se prefiere no usar los valores verdadero-fal-
so; En este texto por razones de comodidad a los impulsos altos
se los representa por el cero y a los impulsos bajos por el uno.
Asimismo, si cambiamos en el margen el orden en que escribimos
los arreglos y comenzamos con el arreglo 0,0 las tablas de verdad
de la conjuncin, disyuncin y negacin se transforman de la si
guiente manera:

A B A .B A B A +B A A
0 0 0 0 0 0 0 1
0 1 0 0 1 1 1 0
1 0 0 1 0 1
1 1 1 1 1 1

En base a estas tablas, las reglas anteriores se pueden abre


viar en los trminos siguientes. En una compuerta Y se tiene A.B =
1 si y solamente s iA = l y B = l ; e n otro caso A.B = 0. En una
compuerta O se tiene A + B = 0 si y slo si A = 0 y B = 0; en
otro caso A + B = 1. En una compuerta No se tiene A = 1 si A = 0 y
A 7 0 si A = 1.
R14. Jerarqua de las compuertas. La jerarqua de las com
puertas corresponde a la jerarqua de las conectivas de las frmu
las cuyo circuito se est diagramando. Asimismo, una compuerta
cualquiera W tiene ms jerarqua que cualquier otra Z si la salida
de Z funciona como una entrada de W.
Para explicar mejor el contenido de la regla anterior recurri
remos a un ejemplo. Construiremos el circuito lgico a compuer
tas de la frmula ( p a q ) v r que podemos escribirla, usando
notacin algebraica, de la siguiente manera ( A . B ) + R. En la

198
frmula la disyuncin o suma es de mayor jerarqua que la con
juncin o producto; por consiguiente en el diagrama de circuito,
la compuerta o debe ser de mayor jerarqua que la compuerta Y,
lo que significa que la salida de la compuerta Y debe ser entrada
de la compuerta o.

El circuito de la figura 14 ejecuta elctricamente la tabla de la


frmula ( A . B ) + R que a continuacin se detalla. El uso de 'V y
'O', dgitos binarios, facilita, cuando se necesita, la adaptacin de
los circuitos para que realicen operaciones aritmticas que es su
objetivo principal.

ABR A.B) + R
1. arreglo 0 0 0 0 0
2. arreglo 0 0 1 0 L
3. arreglo 0 1 0 0 0
4. arreglo 0 1 1 0 1 MATRIZ PRINCIPAL
5. arreglo 1 0 0 0 0
6. arreglo 1 0 1 0 1
7. arreglo 1 1 0 1 1
8. arreglo 1 1 1 1

Como se puede apreciar, el nico caso en el que la tabla an


terior no coincide con la suma y el producto de la aritmtica es en
el octavo arreglo, pues como A = 1, B = 1 y R = 1, se tiene en
trminos aritmticos (1 x 1 ) + 1 que es igual a 2 y no a 1 como dice
la matriz principal de la tabla. Todos los dems casos coinciden
con las operaciones arit-mticas, as el sptimo arreglo, en el que
A = 1, B = 1 y R = 0, da lugar a (1 x 1 ) + 0 que es igual a 1, lo que
coincide con el valor que la matriz principal otorga a ese arreglo.
Tomando otro caso, el tercer arreglo da lugar a (0 x 1 ) + 0 que es
igual a 0, coincidiendo nuevamente con la tabla. El anlisis de los

199
otros arreglos puede dejarse como ejercicio. Tngase presente
siempre que la frmula en trminos aritmticos establece la suma
de un producto ms un dgito.
El caso discrepante, el octavo arreglo, se corrige con un circui
to adicional que nos permite representar el nmero dos en dgitos
binarios, artificio sencillo pero que por ahora debe esperar.

11.6. Circuitos lgicos a compuertas para frmulas negadas

Una de las ventajas de los circuitos a compuertas es que permi


ten diagramar fcilmente circuitos para frmulas negadas como
es el caso de las de De Morgan. Este autor seal las siguientes
equivalencias que las escribiremos en lenguaje algebraico.
Es necesario aclarar que la frmula (A + B) equivale a la
~ ( A v B ) dla lgica proposicional, y la ( A . B ) a la ~ (A a B ).
Se trata de las negaciones de la disyuncin y de la conjuncin
respectivamente.
El ejemplo i afirma que los dos circuitos siguientes son equi
valentes.

A A+B
B
El ejemplo ii afirma que los dos siguientes circuitos tambin
son equivalentes.
Asimismo, valindonos de lo aprendido hasta ahora pode
mos construir el circuito de la frmula ( A .B ) + (C + D ) d e l a
siguiente manera.

Antes de terminar con esta exposicin contestaremos una


pregunta que debe preocupar al lector. Es posible construir cir
cuitos para proposiciones que contienen otras conectivas adems
de la disyuncin inclusiva, la conjuncin y la negacin?. La res
puesta es afirmativa y el mtodo para lograrlo es sencillo. Es ms,
casi todos los problemas de lgica que pueden plantearse con los
conocimientos dados en este texto, pueden ser resueltos con gran
facilidad por una computadora construida mediante un nmero
muy grande de circuitos como los que hemos mostrado. El lgico
chino Hao Wang en el ao 1960, usando una computadora IBM -
704, demostr 220 teoremas de la lgica proposicional en slo tres
minutos. Como se puede comprender, las computadoras pueden
demostrar teoremas de la Lgica Matemtica, porque sus circui
tos han sido diseados en correspondencia con las propiedades
de las conectivas.

11.7. Compuertas NAND y OR

En el diseo de circuitos es frecuente utilizar algunas abreviacio


nes tanto en el vocabulario como en los diagramas. As a la se
gunda compuerta del ejemplo i) se le dice abreviadamente com
puerta OR por que es la negacin de una disyuncin en ingls
(Not - or). Anlogamente a la primera compuerta del ejemplo ii)
se le llama compuerta NAND (Not - and). El operador de incom
patibilidad o barra de Nicod se representa usualmente por una
compuerta NAND debido que es equivalente a la negacin de una
conjuncin. Usando este artificio es posible construir un circuito

201
para cualquier frmula de PM aunque contenga operadores dis
tintos a la negacin, disyuncin y conjuncin. Un recurso es ob
tener para la frmula dada su Forma normal de Post siguiendo
el algoritmo que desarrollamos en la leccin anterior. El otro es
aplicar las equivalencias que sean adecuadas, tomando como re
ferencia las que dimos como Reglas de deduccin natural.
Por ejemplo, dada la frmula ~ ( A <-> B ) su circuito usando
compuertas NAND es el siguiente:

La estructura del circuito se debe a que la Forma normal de Post


de A <-> B es ~( A a ~B ) a ~( ~ A . B ) y, consecuentemente, su
negacin tiene como frmula equivalente ~(~(A a ~ B ) a ~(~A a B )).
Utilizando una compuerta NAND y dos compuertas OR, el
circuito de la frmula ~(~ ( A - B ) A ~ ( B - A ) ) e s e l siguiente:

La forma de este diagrama se debe a que la frmula: ~ (~ (A >


B) a ~(B A ) ) puede ser, aplicando la RDN16, transformada
fcilmente en A v B ) a ~(~B v A )).

11.8. Tablas de verdad vs. tablas aritmticas

En esta seccin sugeriremos el mecanismo a travs del cual una


tabla de verdad puede ser interpretada como una tabla aritmti-

202
ca. La idea central se funda en el hecho de que si en lugar de F
escribimos 0 y en lugar de V escribimos 1, entonces si comenza
mos primero con los valores falsos y pensamos en una tabla para
una frmula con tres letras variables p, q, rf el primer arreglo se
escribir 000, lo que corresponder al primer nmero entero 0,
pues los ceros a la izquierda no aaden valor a la cifra. El segun
do arreglo se escribir 001, que corresponde al nmero entero po
sitivo 1, pues, reiteramos, los ceros a la izquierda no aaden va
lor a la cifra. El tercer arreglo se escribir 010, que corresponde
al nmero dos del sistema decimal. Y como este tercer caso no es
de comprensin obvia como los anteriores, describiremos breve
mente el principio que regula la construccin de sistemas num
ricos con cifras de valor posicional.
En principio todo sistema de numeracin usa un determina
do nmero de signos. El decimal, que es cotidiano, usa diez: 0,1,
2 ,3 ,4 , 5, 6, 7, 8, 9. Y el binario, que es el que resulta de las tablas
de verdad, usa slo dos signos 0, 1. La base de un sistema de
numeracin est dada por el nmero de signos que usa. En este
caso la base de nuestro sistema es 2. Una caracterstica inherente
a los sistemas de numeracin posicional es que el valor de la base
no se puede escribir, dentro del sistema, usando un slo signo.
Una muestra de ello es que diez es el primer nmero que se escri
be con dos signos dentro del sistema decimal. Anlogamente, dos
no se puede escribir con un slo signo dentro del lenguaje binario.
En efecto se necesita dos signos para escribir dicho valor: 10. En el
prrafo anterior aparece con un 0 adelante, pero prescindimos de
l porque est a la izquierda.
En lo que sigue hablaremos genricamente de cifra binaria y
diremos que cualquier cifra binaria es una n-ada de dgitos 0,1
cuyo valor es igual a la suma del valor posicional de sus compo
nentes. El valor posicional de cada componente se determina
multiplicndolo por la potencia correspondiente de una n-ada de
potencias en base 2 crecientes hacia la izquierda. Esta n-ada de
potencias tiene la forma: 2n, 2 n -l,..., 2o. Por razones pedaggicas
hemos confeccionado una tabla de doble entrada cuyas columnas
precisan el valor posicional de cada digito binario.

203
23 23 23 23 V alor d ecim al
1 0 1 1 11
1 1 0 0 12
0 0 1 0 02

La cifra binaria 1011 corresponde al once de nuestro sistema


decimal. Esto se explica por qu el primer 1 de la derecha vale
uno debido a que se multiplica por 20que siempre es igual a uno.
El siguiente 1 se multiplica por 21 y , consecuentemente vale dos.
El 0 , obviamente tiene valor cero y e l l del extremo izquierdo,
por las razones dadas, vale ocho, pues se multiplica por 23. Su
mando los valores anteriores 1 + 2 + 0 + 8 = 1 1 , obtenemos el
valor de la cifra binaria examinada.
Aplicando el criterio anterior se constatar que la cifra 1100
corresponde al valor doce y no hace falta pormenorizar por qu a
10 le corresponde el valor dos. De igual manera se comprobar
que el margen de cualquier tabla de verdad, con las sustituciones
propuestas y comenzando con el valor F, en este caso 0, puede
leerse como una sucesin creciente de nmeros escritos en cdigo
binario. Cuando la tabla sea para tres variables, el ltimo arreglo
corresponder al valor siete. Si es para cuatro letras variables, el
lti-mo arreglo corresponder al valor 15. Esto demuestra, que
aumentando el nmero de arreglos, podemos obtener una cifra
binaria para un nmero tan grande como se desee.
Todos los clculos que realiza un computador los efecta en
dgitos binarios y luego los traduce al sistema decimal. En efecto,
sus Circuitos estn gobernados por frmulas lgicas que se com
putan como tablas de verdad, las mismas que son equivalentes a
tablas aritmticas en cdigo binario. Asimismo, la estructura in
terna del hardware son circuitos y estos en en ltimo trmino pue
den distinguir entre dos cosas reducibles al hecho elemental de
que la corriente pase o no pase.

204
XII
LGICA CLSICA

Objetivos.-

Reconocer proposiciones categricas tpicas en el cuadro de


Boecio.
Distinguir en un silogismo las premisas, la conclusin y sus tres
trminos.
Definir y construir silogismos categricos y reconocer en ellos
el modo y la figura a la que pertenecen.
Distinguir entre contradiccin y oposicin entre proposiciones.
Distinguir a las proposiciones por su grado de generalidad.
Identificar nombres propios en lgica.
CUESTIONARIO 12
Lgica clsica.

Instrucciones

I. Responde a las siguientes preguntas


1. En qu sentido Principia Mathematica es una obra que se inscri
be dentro de la lgica clsica?
2. Porqu se considera a la lgica intuicionista no-clsica?
3. Cul es la caracterstica de los sistemas llamados paraconsis-
tentes?
4. Excluye Principia Mathematica al silogismo aristotlico?
5. Porqu el lenguaje proposicional PM no permite establecer re
glas para decidir la validez de silogismos?
6. Por qu 'perro7, 'g a to ',' hombre' y otros sustantivos comunes
no son nombres desde el punto de vista lgico?
7. Cmo se decide cul es la premisa mayor y cul es la premisa
menor de un silogismo?
8. Si los silogismos tuvieran tres premisas y una conclusin Cun
tos modos habra?
9. Es correcto afirmar que la negacin de una proposicin gene
ral universal es a veces tambin universal?
10. Considerando que los trminos S y P tienen un significado es
tablecido cuntas
contradicciones pueden construirse en el cuadro de Boecio?
11. Es correcto afirmar que dos silogismos tienen la misma es
tructura si coinciden en la figura pero no en el modo?
12. Por qu las preposiciones existenciales tambin son generales?

[207]
13. Por qu una descripcin individual no corresponde a ningu
no de los esquemas del cuadro de Boecio?
14. Cmo se define una constante individual y cmo se represen
ta en caso de que necesitemos millones de ellas?
15. Porqu la expresin' proposicin particular' es desorientadora?

II. Dados los siguientes silogismos, determinar su modo y figura.

1. Ningn cuadrpedo sabe silbar.


Algunos gatos son cuadrpedos.__________________________
Luego, algunos animales que saben silbar no son gatos.
2. Ningn borrego es terrible.
Algunos sueos son terribles.
Luego, algunos sueos no son borregos.
3. Ninguna criatura razonables espera imposibles.
Todo bogavante es razonable.
Luego, ningn bogavante espera imposibles.
4. Todas las criaturas hoscas son mal acogidas.
Todas las avispas son hoscas.______________________________
Luego, todas las avispas son mal acogidas.
5. Ningn canario que se siente melanclico canta con potencia.
Todos los canarios bien nutridos cantan con potencia.________
Luego, ningn canario bien nutrido se siente melanclico.
6. Ninguna experiencia desagradable se busca con avidez.
Toda pesadilla es desagradable
Luego, ninguna pesadilla se busca con avidez.
7. Todos los mamferos respiran a travs de pulmones.
Algunos mamferos son animales acuticos.
Lego, algunos animales acuticos respiran a travs de pulmones.
8. Todas las serpientes marinas son animales acuticos
Todas las serpientes marinas son serpientes.
Luego, todas las serpientes son animales acuticos.
9. Todos los no-fumadores ahorran dinero
Todos los vegetarianos son no-fumadores.
Luego, todos los vegetarianos ahorran dinero.
10.Todos los no-fumadores ahorran dinero
Ningn vegetariano es fumador.
Luego, ningn vegetariano es gastador.

208
LECCIN 12
Lgica clsica

12.1 Criterio de demarcacin

En los llamados textos de lgica moderna se asume, de manera


explcita o implcita que la lgica clsica o aristotlica est cons
tituida por la teora de silogismo y el manejo del cuadro de Boe
cio. A ello se aade que la lgica moderna, simblica o matem
tica, surge a mediados del siglo pasado con el Algebra de Boole
y con el tratamiento que hizo, posteriormente, Gottlob Frege de
la lgica proposicional, en su famosa obra Begriffsschrift.
Sin embargo, este esquema que traza una lnea de demarca
cin entre la lgica clsica y la lgica matemtica se ha ido modifi
cacin paulatinamente. En principio, se ha establecido con clari
dad que los lgicos megricos de la antigua Grecia, entre ellos
Crisipo, Filn y Diodoro Cronos, tuvieron ideas interesantes sobre
la lgica proposicional, sector considerado inicialmente cono pa
trimonio exclusivo de la lgica moderna. Asimismo, se ha encon
trado que muchos desarrollos contemporneos de la lgica mate
mtica, a pesar de ser deductivamente mucho ms poderosos que
el silogismo de Aristteles comparten con l algunas propiedades
sustanciales que los inscriben dentro de la misma concepcin de lo
que est permitido y excluido por las reglas lgicas.
Para precisar este anlisis sealamos que Principia Mathematica
(PM), de Whitehead y Russell, es el primer trabajo sistemtico que
se reconoce como expresin completa del nuevo sistema de lgi
ca, que marca una diferencia radical con el Organon de Aristteles,

[209]
el mismo que no queda excluido de la lgica sino convertido, des
pus de haberlo sido todo por ms de 2000 aos, en una pequea
subestructura del nuevo edificio de la lgica-matemtica. Sin
embargo, esta diferencia tan notoria entre lo nuevo y lo antiguo
no exclua el hecho de que tanto el silogismo como el sistema PM
compartan los mismos principios lgicos y, por tanto, las mis
mas estrategias deductivas. Por ejemplo, tanto Aristteles cono
Russell y Whitehead admitan la estrategia de demostracin o
deduccin por reduccin al absurdo, que hemos examinado cuan
do estudiamos las reglas RDN de Gentzen.
Sin embargo, los lgicos y matemticos intuicionistas de la
primera mitad de nuestro siglo, como Brouwer y Heyting, objeta
ron duramente la validez del mtodo de demostracin de reduc
cin al absurdo, porque presupona la plena vigencia del princi
pio lgico clsico del tercio excluido, que ellos consideraban que
deba ser omitido para rigorizar la matemtica a travs de mto
dos constructivos. En ese sentido, los intuicionistas consideraban
que en lo fundamental el sistema de PM era tan clsico como el de
Aristteles y que ellos eran los primeros en crear sistemas no
aristotlicos.
Francisco Mir Quesada, investigador peruano, a partir de
este hecho ha propuesto un criterio para calificar a un sistema
lgico de clsico o no. Un sistema lgico S es clsico si admite
como vlidos los tres principios lgicos de identidad, no-contra
diccin y del tercio excluido. Y un sistema S no es clsico, en ma
yor o menor medida, si desconoce la validez plena de uno, dos o
todos los principios clsicos. En este sentido, que recoge un punto
de vista extendido en la comunidad acadmica internacional, el
sistema de PM es clsico pero no antiguo y los sistemas de Heyting
y Broouwer son no-clsicos. Existen otros sistemas que debilitan
el uso del principio de no-contradiccin, a los que el mismo Mir
Quesada ha denominado paraconsistentes. Entre ellos se cuentan
los de Vassiliev, Jaskowski, da Costa y Routley. Consecuentemen
te, hablar de lgica clsica no es necesariamente hacer referencia
a la lgica aristotlica, pues tanto PM como todos los sistemas
que se deriven de PM, que son los ms usados por la ciencia con
tempornea, son sistemas clsicos en el sentido aqu definido. Por
otra parte, hablar de lgica no-clsica es hacer referencia a un con-

210
junto de sistemas lgicos dismiles entre s. Por ejemplo, los cl
culos Cn de da Costa admiten la demostracin por reduccin al
absurdo que excluyen los sistemas intuicionistas.
En esta seccin desarrollaremos algunos elementos de lgica
clsica, en el sentido de lgica aristotlica o de la antigua Grecia.
Este aporte ha estado vigente por ms de dos mil aos a tal extre
mo que filsofos notables como Kant consideraron que a lo hecho
por Aristteles no era pertinente quitarle o aadirle algo. El siste
ma del silogismo era toda la lgica y pareca cosa acabada en su
perfeccin. Respecto a los sistemas no-clsicos, hay que decir que
conservan vitalidad; pero en este texto slo podemos mencionar
los con motivo de este necesario esclarecimiento terminolgico.

12.2 El silogismo clsico

Como se conoce, Aristteles invent la lgica como disciplina sis


temtica y estudi con detalle el silogismo que es una estructura
deductiva histricamente anterior a la lgica proposicional pre
sentada en las pginas precedentes. En esta unidad se estudia al
silogismo pero sin recurrir a las reglas aristotlicas clsicas, cuyo
estudio podra ser muy tedioso para el estudiante que se inicia.
Por este motivo hemos preferido un tratamiento ms moderno,
en base al lgebra de Boole, que tiene la ventaja de ser ms sim
ple y de no adolecer de algunos importantes errores del genial
estagirita.
Para introducirnos en los conceptos de esta parte de la lgica
comenzaremos reflexionando sobre un ejemplo de argumento o
inferencia que tiene particular inters para nosotros.

(I) Ningn m am fero es insecto.


Algunos anim ales dom sticos son mamferos.
Luego, algunos anim ales dom sticos no son insectos.

Es fcil darse cuenta de que el argumento anterior es lgica


mente vlido, pues la verdad de la conclusin se deduce clara
mente de la verdad de las premisas. No se viola la exigencia lgi
ca que prohbe que las premisas sean verdaderas y la conclusin
falsa, porque positivamente sabemos que las premisas son verda-

211
deras y que la conclusin tambin lo es. Sin embargo, si forma
lizramos el argumento precedente usando el lenguaje de la lgi
ca proposicional, encontraramos que resultara lgicamente in
vlido porque si representamos las premisas con las variables
p y q, y la conclusin con r, ocurre que el condicional:

(P a q) -> r

no es una tautologa , los cual puede verificar el lector cons


truyendo la tabla de verdad correspondiente.
Este hallazgo planteara una especie de contradiccin entre lo
que intuitivamente vemos con claridad y lo que establece la lgi
ca. Pero afortunadamente sta no es una situacin de esa clase. En
efecto, el anterior argumento es lgicamente vlido y lo que ocu
rre es que la lgica proposicional no es un instrumento lo suficien
temente potente para mostrar su estructura y decidir su validez.
En este caso es inadecuado proceder a representar directamente
las proposiciones componentes por variables proposicionales, de
bido a que es necesario analizar los componentes de cada proposi
cin para que se nos revele su estructura interna y podamos pro
nunciarnos sobre la validez lgica del argumento.
Avanzando en nuestro estudio podemos esquematizar con
sencillez nuestro argumento. Si por el trmino 'mamfero7 usa
mos la abreviacin M, por 'insecto' la abreviacin P y por 'ani
males domsticos' la letra S, tenemos:

(E l) Ningn M es P
Algunos S son M
Luego, algunos S no son P

Este esquema es una aproximacin a la estructura lgica del


argumento examinado y adems pone en evidencia que pode
mos construir otros, del ms variado contenido/ que posean la
misma forma. Por ejemplo:

(II) Ningn p eru an o es chilen o.


Algunos fu tbolistas son peruanos.
L u ego, algunos fu tbolistas no son chilenos.

212
Representando los trminos 'peruano7, 'chileno7 y 'futbolis
ta7 por las letras maysculas, M, P y S, respectivamente, obten
dremos el esquema E l. De otra parte, tambin es posible cons
truir argumentos similares como el que a continuacin sigue, in
troduciendo algunas variantes.

(III) Todos los m am feros son vertebrados.


Todos los gatos son mamferos.
Luego, todos los g atos son vertebrados.

Esta vez el esquema es as:

(E2) Todos los M son P


Todos los S son M
Luego, todos los S son P
Los argumentos anteriores se llaman silogismos y son los
que ocuparon preferentemente la atencin del filsofo griego
Aristteles.
Como lo sugiere la observacin de los esquemas que hemos
formulado, el estudio del silogismo requiere que se ponga aten
cin en los elementos componentes de las proposiciones, que son
palabras. En ellos se representan con signos especiales (letras
maysculas) las palabras que aparecen como sujetos y predica
dos de las proposiciones, las que en lgica se denominan trmi
nos. De esta suerte cada proposicin posee un trmino predica
do y un trmino sujeto.
Es importante enfatizar que las denominaciones precedentes
se refieren a la funcin que desempea un trmino en la inferen
cia y no al trmino en s mismo. Por ejemplo, en los silogismos I, II
y III se cumple que el trmino que es sujeto en la primera premi
sa, representado en los esquemas por M, desempea la funcin
de predicado en la segunda.
Los trminos sujeto y predicado de la lgica no deben ser iden
tificados con sus correspondientes gramaticales. Los sujetos gra
maticales normales son denotados por nombres propios como 'juan
Hinojosa7 y por nombres comunes como 'perro7. Nosotros en esta
parte del curso slo trabajaremos con sustantivos comunes, que
desde el punto de vista lgico son predicados porque ellos hacen

213
referencia a propiedades y no a un individuo en concreto, como la
hacen los nombres propios. As, en nuestros ejemplos, las propie
dades de las que se habla son la de "ser mamfero, la 7ser insecto", la
de 'ser animar, la de 'ser peruano', etc. Consecuentemente, aun
que trminos como los anteriores pueden desempear la funcin
de sujetos en las proposiciones, ellos no son nombres en este con
texto sino predicados lgicos porque denotan propiedades. Por esta
razn, a la parte de la lgica que se ocupa del estudio de tales
trminos se le llama lgica de los predicados.
En lo que sigue, en base a la observacin de los ejemplos que
hemos dado, definiremos el silogismo y proporcionaremos algu
nos detalles adicionales.

Definicin 18. Un silogismo es un tipo de inferencia o argumento que consta de


tres proposiciones de las cuales dos son premisas y una conclusin. Tiene tres
trminos y cada unos se repite dos veces. Hay uno que aparece en ambas premisas
y se omite en la conclusin.

Examinando el ejemplo II, o cualquiera de los dados, se cons


tata que en efecto tienen dos premisas y una concusin. En el in
dicado ejemplo los trminos son 'peruano', 'chileno' y 'futbolis
ta', y cada uno se repite dos veces. El trmino comn a ambas
premisas es 'peruano'.
Aunque no es indispensable, daremos la nomenclatura que
tradicionalmente se utiliza para los silogismos, debido a que to
dava muchos autores recurren a ella y el estudiante podra des
orientarse, por un detalle irrelevante, al leer otro libro.
Al trmino comn a ambas premisas, que se omite en la con
clusin, se le llama trmino medio y se lo simboliza con la letra
mayscula M. El predicado de la conclusin recibe el nombre de
trmino mayor y se lo representa por la letra mayscula P. El
sujeto de la conclusin es el trmino menor y se lo simboliza
mediante la letra mayscula S. A la premisa que contiene al tr
mino mayor se le llama premisa mayor y a la que contiene al tr
mino menor se le denomina premisa menor. En el ejemplo I, el
trmino medio es 'mamfero', el trmino mayor 'insecto' y el tr
mino menor 'animales domsticos'. La premisa mayor es la pro
posicin 'Ningn mamfero es insecto' y la premisa menor la pro
posicin 'Algunos animales domsticos son mamferos'.

214
El mismo Aristteles tambin llam al silogismo inferencia
mediata, porque tiene ms de dos proposiciones. Asimismo, al si
logismo cuyo estudio hemos iniciado se le dice categrico para di
ferenciarlo del hipottico, que es una forma de argumento que per
tenece a la lgica proposicional y que no es de origen aristotlico.

12.3 Las cuatro proposiciones predicativo - categricas


clsicas
Todo silogismo se construye sobre la base de proposiciones, cada
una de las cuales corresponde a uno de los cuatro esquemas
siguientes:

A. Todos los S son P


E. Ningn S es P
I. Algunos S son P
O. Algunos S no son P

Las letras que aparecen a la izquierda son los nombres con


que se conoce a estas proposiciones desde que los medievales
enseaban el famoso cuadro de Boecio. El esquema A correspon
de a proposiciones como Todos los pjaros tienen alas. El esquema E
a proposiciones como Ningn gato es perro. El Esquema I a propo
siciones como Algunos sabios son fsicos. Y el esquema O a proposi
ciones como Algunos lgicos no son griegos.
Puede afirmarse que la lgica aristotlica se redujo al conoci
miento de estas cuatro proposiciones y a las inferencias que se
puede construir con ellas. Hoy da es un sector muy pequeo y
dbil de la lgica, pues en l no se pueden expresar los problemas
matemticos y cientficos relevantes.
En el esquema A el sentido de la palabra 'todos' es muy im
portante, pues hay proposiciones que no contienen dicha palabra
pero que corresponden al mencionado esquema. Por ejemplo, las
proposiciones 'Cada hombre tiene una cabeza', 'Cualquier nmero par
es divisible por 2', deben ser traducidas a las proposiciones 'Todo
hombre tiene una cabeza' y 'Todo nmero par es divisible por 2' que
mantienen su sentido y corresponden a la forma cannica que
hemos establecido como esquema A.

215
Asimismo, hay proposiciones que son generales pero que no
contienen explcitamente una palabra que denote cantidad, como
'Los hombres son mortales', cuya expresin ms precisa es 'Todos los
hombres son mortales'. El esquema A corresponde a las proposicio
nes universales afirmativas. El esquema E corresponde a las pro
posiciones universales negativas y en l la palabra 'ninguno' debe
entenderse en el sentido de 'Ni siquiera uno' o de 'No existe aunque
sea uno'. Por tanto, afirmar 'Ningn felino es cetceo' equivale a decir
'Ni siquiera un felino es cetceo' o 'No existe aunque sea un felino que
sea cetceo'.
El esquema I corresponde a las proposiciones particulares
afirmativas y el esquema O a las particulares negativas. En am
bos esquemas aparece la palabra 'algunos' y su sentido es el de 'al
menos uno'. Esto significa que cuando en general uno o varios ob
jetos, pero no todos, tienen una propiedad se puede decir que
algunos la tienen. Por ejemplo, sabemos que muchos futbolistas
son veloces, pero no todos lo son. Luego podemos decir 'Algunos
futbolistas son veloces'. Asimismo, slo podemos decir 'Algunas es
trellas son conocidas' porque no tenemos la seguridad de conocer
todas, aunque tenemos informacin sobre millones de ellas.
Otra variante para 'algunos', muy usada por los lgicos, es
'hay al menos uno'. Si recurrimos a ella nuestros ejemplos anterio
res son traducidos a 'Hay al menos un futbolista veloz' y 'Hay al
menos una estrella conocida'.
Es importante puntualizar que las llamadas inferencias in
mediata clsicas tambin se construyen con proposiciones que
corresponden a uno de los cuatro esquemas expuestos. Nosotros
noclas abordamos en detalle pero el mtodo que emplearemos
para determinar la validez de los silogismos se usa tambin para
decidir la validez de tales inferencias que constituyen argumen
tos ms sencillos.

12,4 El cuadro de B o ecio

Tradicionalmente se atribuye al lgico Ancius Boecio, que vivi


entre los aos 480-524 de nuestra era, una manera muy especial
de clasificar y presentar las cuatro proposiciones predicativo - ca
tegricas clsicas.

216
Las proposiciones de las formas A, O son recprocamente con
tradictorias. Asimismo, lo son las formas E, I. Esto significa que A es
equivalente a la negacin de O y, recprocamente, O es equivalente a
la negacin de A. Tambin que E es equivalente a la negacin de I, e
I es equivalente a la negacin de E. La relacin anterior entre las
proposiciones puede verse con ms claridad a travs de ejemplos.
La proposicin 'Algunos hombres no son altos' es de la forma
O. Su negacin 'No es el caso que algunos hombres no sean altos'
equivale exactamente a 'Todos los hombres son altos' que es de la
forma A. Ahora, la negacin de esta ltima proposicin es 'No es
el caso que todos los hombres sean altos', que equivale a algunos hom
bres no son altos. Consecuentemente, ~ O <- A y ~ A <- O.
La proposicin 'Algunos gatos son blancos7 es de forma I. Su
negacin es 'No es el caso que al menos un gato sea blanco' que equi
vale a 'Ningn gato es blanco' que es una E. Asimismo, la negacin
de esta ltima proposicin es 'No es caso que ningn gato sea blanco'
que equivale a 'Al menos un gato es blanco' que es de la forma I.
Consecuentemente, ~ I <-> E ~ E <-> I.
Nosotros por ahora no podemos dar una prueba formal de la
validez de los bicondicionales o equivalencias, anteriores. Los
ejemplos son un recurso intuitivo pero no de plena validez lgi
ca. Sin embargo, ms adelante brindaremos una mejor aproxima
cin a la validez de dichos bicondicionales mediante grficos.
A las proposiciones del par A, E se les llama recprocamente
contrarias. A las del par I, O se les denomina recprocamente
subcontrarias. Y a los pares A, I y E, O, exactamente en este or
den, se les llama subalternas.
Boecio, segn narra la tradicin, representada la anterior cla
sificando mediante el siguiente grfico conocido como cuadro de
oposicin o, simplemente, Cuadro de Boecio.

Ningn S es P (E)

Algunos Sno son


P(O)

Las flechas dobles sealan los casos en que las relaciones son recprocas; y las
simples, los casos en que la relacin se cumple en un solo sentido.

217
12.5 Modos y Figuras silogistas

Por definicin, todo silogismo est constituido por tres proposi


ciones. Cada una corresponde a uno de los cuatro esquemas A,
E, I, O. Consecuentemente, para cada silogismo posible existe un
triplo o terna de letras, por ejemplo EAE, AOO, AOI, Eli, AEO,
AAA, etc., que se denomina el modo de un silogismo. Natural
mente, al escribir el modo de un silogismo, las dos primeras le
tras corresponden a las premisas y la tercera a la conclusin. Para
ilustrar esta explicacin indicaremos que el modo de nuestro ejem
plo III, por citar slo un caso, es AAA.
De lo anterior se deduce que todos los modos posibles estn
dados por todas las temas que se pueden formar con los cuatro
esquemas, A, E, I, O. Sabemos que el nmero de ternas posibles
que se pueden formar con cuatro cosas es igual a 43, lo que signi
fica que son 64 modos silogsticos posibles.
Tambin podemos observar que el llamado trmino medio
M, que es comn a ambas premisas, puede cumplir la funcin de
sujeto en la primera premisa y de predicado en la segunda, o des
empearse como predicado en ambas premisas, o ser sujeto en
ambas premisas, o ser predicado en la primera y sujeto en la se
gunda. Cada uno de estos arreglos posibles del trmino medio en
las premisas se llama figura silogstica. Las cuatro figuras pue
den ser esquematizadas como sigue:

MP PM MP PM
SM SM MS MS
SP SP SP SP
(1 .a) (2 .a) (3.*) (4.*)
Figuras silogsticas
Teniendo en cuenta que cada modo puede ser presentado en
cada una de las cuatro figuras, ocurre que el nmero total de
silogismos posibles es igual a 64 x 4, esto es, hay 256 silogismos
posibles. De ellos solamente 15 son vlidos de acuerdo con las
exigencias establecidas por los diagramas de Venn y otros mto
dos modernos. Las reglas de Aristteles permiten establecer 19

218
silogismos vlidos, pero con instrumentos ms potentes, como
los aqu usados, cuatro de ellos son defectuosos y necesitan ser
subsanados.
A continuacin indicaremos los 15 silogismos lgicamente
vlidos.

Primera figura AAA, EAE.AII,EIO


Segunda figura EAE, AEE, EIO, AOO
Tercera figura IAI, An, OAO, EIO
Cuarta figura AEE, IAI, EIO

Los lgicos medievales usaron nombres artificioso para re


cordar los modos y figuras de los silogismos vlidos. Estas ayu
das nemotcnicas contenan como vocales en el mismo orden de
aparicin las letras que constituyen el modo. A continuacin de
tallamos los referidos nombres especificando la figura:

Primera figura : Barbara, Celarent, Darii y Ferio.


Segunda figura : Cesare, Camestres, Festino y Baroco.
Tercera figura : Datisi, Feriso, Disamis y Bocardo.
Cuarta figura : Calemes, Dimotis y Fresison.

12.6 T ip o s de generalid ad

Es frecuente encontrar en textos de lgica que la deduccin silo


gstica se ilustre mediante el siguiente ejemplo:

Todos los hombres son mortales


Scrates es hombre
Scrates es mortal
Una pregunta inmediata a propsito de este supuesto silo
gismo era la referente a cual era su modo. Estaba claro que la
primera premisa corresponda al esquema A pero no estuvo claro
a qu esquema corresponda la segunda premisa y la conclusin.
Decir que 'Scrates es hombre' es una afirmacin universal iba
contra el sentido comn ya que se estaba hablando de un indivi
duo especfico. Y decir que era particular, tampoco resolva el

219
problema, porque se puede afirmar que hay al menos un hombre
que es mortal sin que ste tenga que ser necesariamente Scrates.
Podra ser Alejandro Magno o Pericles, por citar dos ejemplos entre
millones.
Es claro que si no se conoca a que esquemas pertenecan la
segunda premisa y la conclusin, entonces el modo de esta infe
rencia quedaba indeterminado. La discusin ha tomado siglos y
condujo a detectar que la expresin 'proposicin particular' es
imprecisa y desorientadora. En principio pareca significar lo
opuesto a proposicin general. Sin embargo, si se admite que el
significado de 'algunos' se expresa con claridad a travs de 'exis
te al menos un objeto x' o 'hay al menos un x', entonces lo ms
adecuado era denominar a las proposiciones que comienzan con
'algunos' proposiciones existenciales, pues su sentido es afirmar
la existencia de al menos un objeto, lo que equivale a sentenciar
que el conjunto de objetos al que hacen referencia no es vaco. De
esta suerte, actualmente se prefiere calificar a las proposiciones I,
O de existenciales.
Sin embargo, el hecho de que una proposicin sea existencial
no es incompatible con que sea, tambin, general. As la afirma
cin 'Algunos gatos son negros' adecuadamente formulada se tra
duce por Existe al menos un objeto x que es gato y que es negro, la que
es de carcter general porque no se refiere especficamente a gato
alguno sino que habla de cualquier gato de un conjunto que pue
de ser infinito. Consecuentemente, las proposiciones que comien
zan con 'algunos' se denominan propiamente proposiciones ge
nerales existenciales para diferenciarlas de A,E que son genera
les universales. Lo que ambos tipos de proposiciones tienen en
comn es que se refieren a un conjunto de objetos y no a un objeto

Los resultados anteriores han conducido como conclusin a


establecer que la afirmacin 'Scrates es hombre' no corresponde a
ninguno de los cuatro esquemas que aparecen en el cuadro de
Boecio. Consecuentemente, ha quedado abierto el problema con
sistente en encontrar una interpretacin lgica adecuada a la pro
posicin anterior.

220
12.7 Los nombres propios

Un criterio para distinguir, 7Algunos griegos son hombres7 de 7S


crates es hombre' ha sido sealar que esta segunda proposicin per
mite identificar a un individuo especfico, mientras que la primera
no se refiere a persona alguna en especial, pues todo lo que nos
dice es que dentro del conjunto de los hombres hay al menos uno
que es griego; pero no nos dice quin ni nos asegura que sea el
nico. La segunda proposicin contiene en cambio el trmino 'S
crates7 que es un nombre propio y, por tanto, expresa un modo
individualizador de hablar, mientras que la primera contiene
nombres comunes y expresa una manera genrica de hablar.
Un nombre comn no es nombre de un objeto sino de un con
junto de objetos. As el nombre "griego7 no designa a persona
alguna sino al conjunto de individuos que tienen la propiedad de
ser griegos. Por ello, en los lenguajes lgicos a los nombres comu
nes no se los interpreta como nombres sino como propiedades de
conjuntos de individuos. Los nicos nombres lgicamente ad
misibles son los trminos que permiten identificar inequvoca
mente a un objeto o individuo, esto es, en los lenguajes lgicos
slo hay nombres propios.
A los nombres, en lenguaje lgico, se les denomina constan
tes individuales y se les designa por las letras minsculas inicia
les del alfabeto a, b, c, etc. Cuando se necesita muchos nombres se
utiliza nmeros enteros positivos como subndices de las referi
das letras y as se genera tantos nombres como se desee. Empero,
en lgica no slo las personas son nombradas mediante nombres
propios sino cualquier objeto. Puede ser un mueble, una muestra
de sangre para el laboratorio, un tomo o una persona, etc.
El uso de nombres propios permite construir proposiciones
que son descripciones de las propiedades de individuos concre
tos. Por ejemplo, 'a es azul' es la descripcin de un objeto que
tiene la propiedad de ser azul. Asimismo, en la medida que esta
descripcin es irreductible a otra ms simple; la proposicin an
terior es un ejemplo de proposicin atmica. Pero si escribo 'a es
azul y a es pesado', entonces tenemos una descripcin que tiene un
solo sujeto pero que es molecular porque contiene la conectiva de
conjuncin. El hecho de que a aparezca dos veces no significa en

221
modo alguno que tenga dos sujetos sino que el mismo nombre
tiene dos apariciones (ocurrencias) en la proposicin conjuntiva.
Al mismo tiempo, podemos tener la situacin inversa. La propo
sicin 7a ama a b' tiene don nombres pero es atmica porque no
contiene conectiva alguna. A las proposiciones anteriores las
denominaremos descripciones individuales.
Lo anterior significa que 'Scrates es hombre' es tina descrip
cin individual y como tal no tiene lugar en los esquemas del cua
dro de Boecio que son para proposiciones generales. Este resulta
do conduce a establecer que la deduccin de la seccin anterior
no es propiamente un silogismo sino otro tipo de inferencia que
examinaremos cuando estudiemos el uso de un lenguaje predica
tivo con cuantificadores. En esa leccin abordaremos el tratamien
to que se da a las descripciones individuales en la deduccin me
diante las reglas de ejemplificacin universal y ejemplificacin
existencial.

222
XIII
DIAGRAM AS DE VENN

Objetivos:
Conocer el lenguaje del lgebra de Boole.
Decidir la validez lgica de expresiones del lgebra de Boole
mediante los diagramas de Venn.
Aplicar los diagramas de Venn para determinar la validez de
inferencias de la lgica clsica aristotlica.
CUESTIONARIO 13
Los diagramas de Venn

Instrucciones
I. A continuacin se presenta un conjunto de afirmaciones en
forma esquemtica. El ejercicio consiste en traducirlas al len
guaje de Boole y construir en cada caso el correspondiente
diagrama de Venn.

1. Todo S es P

2. Ningn S es P

3. Algunos S son P

4. Algunos S no son P

[225]
5. Todo S es no P

6. Todo no S es P

7. Todo no S es no P

8. Ningn S es no P

9. Ningn no S es no P

1Q. Algn S no es no P

11.Algn no S es P

226
12. Algn no S no es no P

13. Algn n o S es no P

14. Algn n o S no es P

II. A continuacin se presenta un conjunto de afirmaciones para


cada una de las cuales se proponen tres traducciones a los es
quemas tradicionales A, E, I, O. El ejercicio consiste en: 1) deci
dir cul de las alternativas propuestas es la traduccin correcta
de la afirmacin respectiva; 2) escribir en el lugar en blanco la
frmula algebraica que corresponde a la traduccin antes elegi
da; y 3) construir el diagrama de Venn que corresponda a la
frmula antes mencionada. Como sugerencia proponemos las
letras A y B para representar predicados.

1. Nadie sino los becarios comern


en la mesa grande.
a) Todos los becarios comen en la mesa
grande.
b)Algunos becarios comen en la mesa
grande.
c) Todos los que comen en la mesa
grande son becarios.
A = becarios
B = personas que comen en la mesa grande

227
2. Nadie puede recordar la batalla de las
Carinas a menos que sea muy viejo.
a) Todo el que es muy viejo puede
recordar la batalla de las Cannas.
b) Todo el que puede recordar la batalla
de las Cannas es muy viejo.
c) Ningn joven estuvo en la batalla de
las Cannas.
A = personas muy viejas
B = personas que recuerdan
la batalla de las Cannas.

3. Los consumidores de alcohol pierden


el autodominio.
a) Todos los consumidores de alcohol pierden
el autodominio.
b) algunos consumidores de alcohol
pierden el autodominio.
c) Algunos de los que pierden el autodominio
son consumidores de alcohol
A = consumidores de alcohol;
B = personas que pierden el
autodominio.

4. Un hombre que mantiene su promesa


eshonesto.
a) Ningn hombre honesto mantiene
u promesa,
by Todo hombre que mantiene su
promesa es honesto.
c) Algunos hombres que mantienen su
promesa son honestos.
A = hombres que mantienen su promesa;
B = honestos.

228
5. Slo los becarios comern en la mesa
grande.
a) Todos los que comern en la mesa
grande son becarios.
b)Todos los becarios comern en la
mesa grande.
c) Algunos becarios comern en la
mesa grande.
A = personas qite comern en la mesa
grande;
B = becarios.

6. Nadie que va a una fiesta deja de


peinarse.
a)N7iguno de los que van a una fiesta
es un despeinado.
b)Todos los que se peinan van a una
fiesta.
c) Ningn peinado va la fiesta.
A = asistentes a una fiesta;
B = despeinados.

7. Cualquiera qne no objeta a los actores


admira el cine.
a) Todo el que admira el cine no objeta
a los actores.
b)Algunos que objetan a los actores no
admiran el cine.
c) Todo el que no objeta a los actores
admira el cine.
A = personas que objetan a los actores;
B = admiradores del cine.

8. Cada caballo tiene un jinete.


a) Todo caballo tiene un jinete.
b) Algunos caballos tienen jinete.
c) Algunos caballos no tienen jinete.
A = caballos
B = jinetes

229
9. Cada caballo no tiene un jinete.
a) Algunos caballos no tienen jinete.
b) Algunos caballos tienen jinete.
c) Ningn caballo tiene jinete.
A = caballos
B ~ jinetes.

10.No todos los caballos tienen jinete.


a) Algunos caballos tienen jinete.
b) Algunos caballos no tienen jinete.
c) Algunos jinetes no tienen caballo
A = caballos
B = jinetes

11. Todo caballo no tiene jinete.


a) Ningn caballo tiene jinete.
b)Ningn jinete tiene caballo.
c) Hay un caballo que no tiene jinete.
A = caballos
B = jinetes

12.Nadie es admitido en la sala a no ser


que sea bien educado.
a) Todo el que es admitido en la sala
es bien educado.
b)Todo el que es bien educado es
admitido en la sala.
c) Al menos uno de los admitidos en
la sala es bien educado.
A = admitidos en la sala
B = bien educados

13. Todos, inclusive el ms bravo,


saldrn corriendo.
a) Todos los bravos saldrn corriendo.
b)Todos los que saldrn corriendo
son bravos.
c) Algunos cobardes no saldrn
corriendo.
A = bravos.
B = personas que salen corriendo.

230
III. A continuacin presentamos un conjunto de silogismos cuya
validez debe ser decidida mediante los diagramas de Venn. El
ejercicio debe cumplirse a travs de los siguientes pasos: a) es
cribir en los lugares blanco respectivos las frmulas algebraicas
de las premisas y la de la conclusin; b) construir en el recuadro
el diagrama de Venn de las premisas; c) responder si el silogis
mo es vlido o invlido; d) indicar el modo y la figura del silo
gismo.

1. Ningn emperador es dentista y todos los dentistas son temi


dos por los nios. Luego, ningn emperador es temido por los
nios.

Premisas Conclusin

2. Todo vehculo popular es confortable y ninguna carretilla es


confortable. Luego, ninguna carretillas es popular.

Premisas Conclusin

231
3. Ningn ladrn es honesto y algunos honestos son gente lista.
Luego algunos ladrones son gente lista.

Premisas Conclusin

4. Todos estos bombones son de crema de chocolate y todos estos


bombones son deliciosos. Luego, todo lo que est hecho de cre
ma de chocolate es delicioso.

Premisas Conclusin

5. Alguna gente saludable es gorda y ninguna persona saludable


s dbil, algunos gordos no son fuertes.

Premisas Conclusin

232
6. Los diccionarios son valiosos y todos los libros valiosos son de
mucha ayuda. Por tanto, los diccionarios son de mucha ayuda.

Premisas Conclusin

7. Ningn pato es potico y algunas ranas no poticas. Luego, al


gunas ranas no son patos.

Premisas Conclusin

8. Al azcar es dulce y la sal no es dulce. Luego, la sal no es az


car. (Indicacin: el estudiante debe tener muy presente que
'sal7y 'azcar7no son nombres propios)

Premisas Conclusin

233
9. Algunos epicreos son no generosos y todos mis tos son gene
rosos. Por tanto, mis tos son no epicreos.

Premisas Conclusin

10.Todos los trabajadores son productores de la riqueza de un


pas. Ningn productor de la riqueza de un pas es antisocial.
Por tanto, ningn trabajador es antisocial.

Premisas Conclusin

234
LECCIN 13
Los diagramas de Venn

13.1 E l m todo de lo s diagram as de V en n

Al lgico y matemtico del siglo pasado John Venn se debe un


mtodo intuitivo para decidir la validez de argumentos del tipo de
silogismo. Aunque este mtodo desde el punto de vista lgico es
limitado, es mucho ms simple y ms potente que las numerosas
y tediosas reglas que formul Aristteles con el mismo fin. La su
perioridad de los llamados Diagramas de Venn en simplicidad y
claridad se debe, en gran medida, a la incorporacin de mtodos
del lgebra de conjuntos a la Lgica, que es una de las notas ms
saltantes de los desarrollos modernos de esta disciplina.
El mtodo de Venn consiste bsicamente en representar los
trminos de las inferencias mediante crculos. Se postula que uno
de tales crculos encierra al conjunto* de individuos que tienen
la propiedad denotada por un trmino dado. Esta postulacin
resulta correcta porque uno de los axiomas de la teora de conjun
tos, el de comprehensin, afirma que una propiedad determina
inequvocamente un conjunto.
En armona con lo anterior decimos que, por ejemplo, la pro
piedad de 'ser vertebrado' determina el conjunto de todos los
objetos que son vertebrados.

* Al nivel del curso podra parecer indiferente hablar de clases o de conjuntos.


Sin embargo, en la Lgica Matemtica moderna se diferencia ntidamente
ambos conceptos, razn por la que es decididamente mejor hablar de conjuntos
en relacin con los diagramas de Venn.

[235]
Esto se representa grficam ente as:

La letra mayscula S es el nombre del crculo que representa


al conjunto de objetos vertebrados, como lo muestra el diagrama 1.
Asimismo, de conformidad con los conceptos bsicos de la
teora de los conjuntos, se acepta que existe el conjunto vaco que
se caracteriza porque no tiene ni siquiera un objeto. Para repre
sentar grficamente el hecho de que un conjunto es igual a un
conjunto vaco se dibuja un crculo y se lo raya o sombrea. Las
rayas o sombras significa, que tal conjunto carece absolutamente
de miembros. Veamos el diagrama siguiente:

D iagram a 2

En el diagrama 2 las rayas indican que el conjunto S esta vez


carece de miembros y es igual al conjunto vaco, que lo denota
mos por la letra griega f que en espaol se llama 'phi'. Cabe sea
lar qu la convencin de sombrear, total o parcialmente, un crcu
lo sexisa en los textos de matemtica con un significado distinto
al establecido en este texto para los diagramas de Venn.
Tambin se asume que existe un conjunto muy grande en el
cual estn incluidos todos los conjuntos determinados por los
trminos de las inferencias que vamos a analizar. A este conjunto
lo llamaremos universo del discurso y lo representaremos por
un rectngulo (ver el diagrama 3) que contiene tantos crculos
como sea necesario dibujar. Algunos autores llaman a este con
junto clase universal, pero ello tiene el inconveniente, muy cono
cido, que conduce a contradicciones llamadas paradojas, las cua-

236
OO
les fueron sealadas particularmente por el lgico y filsofo
Bertrand Russell.

s p

Diagram a 3
Sin embargo, representar un conjunto con un crculo sin som
bras y sin ninguna otra seal, como en los diagramas 1 y 3, re
quiere de mayor precisin. Pues, por ejemplo, si el conjunto es
determinado por la propiedad de 'ser el hijo del Quijote', enton
ces hay que rayarlo porque es vaco, pero si es determinado por la
propiedad de 'ser peruano', entonces hay que escribir en su inte
rior una equis, como lo hacemos en el diagrama 4, para significar
con ello que sabemos que el conjunto no es vaco y tiene al menos
un miembro. Consecuentemente, cuando un conjunto es grafcado
slo por un crculo en blanco, diremos que no tenemos informa
cin sobre si es o no vaco tal conjunto.

S ^ <j>
(S diferente de <j>)
O

Diagram a 4

Por tanto, los diagramas nos plantean tres situaciones muy


claras:
El conjunto es vaco cuando est rayado; el conjunto es dife
rente del vaco si muestra en su interior una equis; el conjunto no
proporciona ninguna informacin sobre sus miembros si se trata
de un crculo en blanco.
Sealamos, adems, que para todo conjunto existe otro que
se llama su conjunto complemento, y est constituido por todos

237
los objetos que no pertenecen al primero. As, dado el conjunto A,
su complemento se escribe as: A y se lee no A. Esto se repre
senta grficamente en al diagrama 5. Asimismo, para todo objeto
x del que hablemos, este tendr exactamente dos posibilidades
con respecto a cualquier conjunto A. O el objeto x es miembro de
A o no lo es, pero no serlo equivale a que x es miembro de A.

Diagrama 5
Asimismo, podemos diagramar la operacin de interseccin
de conjuntos mediante dos crculos que se cortan o intersecan. El
conjunto interseccin estar constituido por la zona comn a
ambos crculos. De manera abreviada, dados dos conjuntos A y
B, su interseccin la denotaremos, usando el lenguaje algebraico,
mediante A n B que significa: el conjunto A se interseca con el
conjunto B. Tambin podemos decir que A n B es el conjunto de
interseccin de A y B. Lo anterior se grafica en el diagrama 6.

A
\ B
' 7 S '

( )

Diagrama 6
Si queremos representar grficamente que la interseccin de
A y B es vaca, entonces rayaremos la zona comn a ambos crcu
los. Si, en cambio, deseamos repre-sentar grficamente que la in
terseccin de A y B es diferente del conjunto vaco, en-tonces es
cribimos una x en la zona comn a ambos crculos para expresar
que en ella hay al menos un objeto x. Estas dos situaciones pue
den verse en los diagramas 7 y 8.

238
A n B = (j)

Diagram a 8 A n B*<j>

Todos los S
son P (A)
S n P = <j>

Algunos S no
sonP (0)
S n P ^<j)

Algunos S
nosonP(I)
S n P

239
El diagrama 9 tiene rayada la zona del crculo S que est fuera
de P. Ello significa que no hay ni siquiera un miembro de S fuera
de P, lo que equivale a afirmar que todos los miembros de S estn
dentro de P, que es lo que dice la proposicin 'Todos los S son P\
Consecuentemente, la proposicin A equivale, en los diagramas
de Venn a la afirmacin de que la interseccin entre S y P es vaca.
Por eso hemos rayado la zona comn al conjunto S y al conjunto P.
Esto lo denotamos algebraicamente mediante S n P = <j).
El diagrama 10 expresa que no existe ningn elemento en
comn entre los conjuntos S y P. Por esta razn hemos sombrea
do la zona comn a ambos crculos. Esto equivale claramente a la
proposicin 'Ningn S es Y , que en lenguaje algebraico de Venn
se escribe S n P = <j).
El diagrama 11 afirma que existe al menos un objeto x co
mn al conjunto S y al conjunto P, lo que significa que su intersec
cin es diferente al vaco. Ahora, como decir 'Al menos un S est
fuera de P' es lo mismo que afirmar la proposicin de forma O7.
'A l menos un S no es P7, entonces consideramos que la expresin
algebraica S n P * f es la traduccin de cualquier proposicin
predicativo-categrica que corresponde al esquema O.
El diagrama 12 afirma que hay un objeto x comn a los con
juntos S y P, razn por la que hemos escrito una equis en la zona
comn a ambos crculos. El sentido claramente corresponde al
del esquema I que afirma Algunos S son P' y que se expresa
algebraicamente mediante S n P ^ f
Es importante remarcar que los diagramas slo afirman
aquello que est claramente precisado mediante la presencia
de ifcyas o de una letra equis. Sobre las zonas del diagrama que
no contienen ni una equis ni rayas, no se puede afirmar nada
desde el punto de vista lgico y, como dijimos antes, ellas dela
tan ausencia de informacin.
Los diagramas nos dan una prueba visual muy sencilla de
que, en efecto, la negacin de una proposicin de forma A es una
proposicin de forma O y viceversa. Observando los diagramas
9 y 11 se encuentra que el primero est rayado en la zona en
donde el otro tiene una equis. Mientras que el primer diagrama
afirma que tal zona es igual al vaco, el segundo afirma que dicha
zona es diferente del vaco. Esto se traduce en sus expresiones

240
algebraicas que slo difieren en que donde una exhibe el signo
igual'= ' la otra exhibe el signo V .
Del mismo modo, los diagramas 10 y 12 prueban que la ne
gacin de una propo-sicin de forma E es otra de forma I y vice
versa. En efecto, respecto de la misma zona, el primero afirma
que es igual al conjunto vaco y el segundo afirma que es di
ferente del vaco. Esto tambin se refleja en sus expresiones alge
braicas que son una negacin muy clara de la otra. As, S n P =
es negada por S n P ^ ( | y viceversa.

13.2 Aplicacin de los diagramas de Venn a la decisin de


la validez de silogismos.

En lo que sigue daremos las reglas que nos permiten mediante


los diagramas de Venn decidir si un silogismo es o no lgicamente
vlido.
R15. Cada una de las tres proposiciones del silogismo debe
traducirse al lenguaje algebraico. Para ello al trmino medio hay
que representarlo por M, al trmino menor por S y al mayor por
P. As se obtiene lo que se llama el esquema algebraico del silo
gismo.
R16. A cada trmino se le asigna un crculo que se nombra
mediante una de las letras M , S o P segn convenga, procedindose
a dibujar los tres crculos de tal manera que ellos se corten o
intersequen entre s.
Si tomamos en cuenta la existencia de los conjuntos comple
mentos de S, de P y de M y los representamos como S, P, M,
entonces quedan determinadas ocho zonas de interseccin que
graficamos a continuacin.
R17. En el diagrama obtenido por la aplicacin de la regla
anterior se procede a representar las premisas de acuerdo con los
diagramas-modelo que hemos dado para las proposiciones A, E,
I, O. Si una premisa es universal y la otra particular, entonces
debe graficarse primero la premisa universal.
R18. Si al graficar una premisa particular (existencial) hay
ms de una zona donde se puede escribir la letra equis, entonces
sta debe ser escrita necesariamente en la zona que presupone
menos, que es aquella comn al menor nmero de crculos. (Esta

241
regla tiene limitaciones, como todo mtodo grfico, que no afec
tan a los ejercicios propuestos en este texto).
R19.B El silogismo es lgicamente vlido si y solamente si al
ser graficadas las premisas queda automticamente graficada, de
manera inequvoca, la conclusin. Si no queda graficada la con
clusin, entonces el silogismo es lgicamente invlido.
Veamos ahora la aplicacin de estas reglas a un ejemplo.
Tengamos el siguiente silogismo del modo OAO de la terce
ra figura:

Algunos nmeros no son transfinitos


Todos los nmeros son objetos ideales
Algunos objetos ideales no son transfinitos.
Al trmino medio, comn a ambas premisas, es 'nmeros' y
lo representamos por M. El trmino mayor es 'transfnito' y lo
representamos por P. El menor es 'objeto ideal' y lo representa
mos por S. De este modo, haciendo la traduccin ordenada por la
regla R l l , el esquema algebraico del silogismo es:

(O) M nP*<>
(A) M n S = ()
(O) SnP*<j>

242
Por la aplicacin de la regla R16 se obtiene el siguiente
diagrama:

Luego, aplicando la regla R17, procedemos a graficar las


premisas dentro del diagrama anterior y, como una es particular
y la otra es universal, entonces, conforme a lo indicado por la
misma regla, procederemos primero a graficar la que es univer
sal. As tenemos:

En el diagrama anterior slo est graficada la segunda pre


misa que es de forma A, la razn por la que hemos rayado toda la
parte de M que est fuera de S, como lo establece 'todos los M
son S'.
Completamos el diagrama procediendo a graficar la premisa
que resta escribiendo una equis en la nica parte no sombreada
de M , que est fuera de P , como lo exige la proposicin de forma
O.

243
M n S = (f>
El diagrama anterior ya est completo. Nosotros lo hemos
dibujado en tres pasos por razones pedaggicas, pero el lector se
habr dado cuenta que esto puede hacerse fcilmente en solamente
un paso. Corresponde aplicar ahora la regla R19. Esto se hace
observando atentamente si la conclusin ha quedado graficada
automticamente al graficarse las premisas. En efecto, ha ocurri
do as pues el diagrama nos dice con toda claridad que existe al
menos un objeto x fuera del crculo P y dentro del crculo S, lo
que significa que la interseccin entre S y P es diferente del con
junto vaco, que es lo que afirma la traduccin algebraica de la O.
Consecuentemente, el silogismo anterior es lgicamente vlido y
podemos decir que todo silogismo del modo OAO de la tercera
figura (OAO-3) es lgicamente vlido. Los medievales, para acor
darse del silogismo OAO-3 lo llamaban BOCARDO, que es una
ayuda nemotcnica cuyas vocales corresponden al modo del silo
gismo analizado.
En lo que sigue daremos dos ejemplos adicionales para ga
nar famiaridad con los mtodos de Venn:

(I) Ningn hombre es felin o


Todo felin o es un ser con garras
Luego, algunos seres con garras no son hombres.
(II) Todos los peruanos son americanos
Todos los limeos son peruanos
Luego, todos los limeos son americanos.

244
(III) Todos los hombres hablan
Todos los loros hablan
Luego, todos los loros son hombres.

El esquema algebraico del primer silogismo es el siguiente:

P n M = (j)
M n S = <|>

S n P * (j>

Graficando las premisas obtendremos el siguiente diagrama:

Como se observa, la conclusin no ha quedado autom


ticamente grafcada al diagramarse las premisas. Para q ello ocu
rra hace falta que, como la conclusin es de forma O, aparezca una
equis en alguna parte de la zona del crculo S que est fuera del
crculo P, lo que no sucede. Consecuentemente, un silogismo de la
forma EOA de la cuarta figura (EAO-4) no es lgicamente vlido.
Anotamos que una de las deficiencias de la lgica aristotlica y de
la medieval fue considerar a este silogismo vlido.
El esquema algebraico del segundo silogismo es el siguiente:

M n P _ = <j>
S n M = <j>

S n P =<|>

El diagrama resultante es el dibujado a continuacin:

245
Como se observa, la conclusin que es una proposicin de
forma A, ha quedado automticamente graficada al completarse
el diagrama de las premisas. Consecuentemente, un silogismo del
modo AAA de la primera figura (AAA-1) es lgicamente vlido.
Los medievales lo llamaban silogismo BARBARA.
El esquema algebraico del tercer ejemplo es como sigue:

P n M = <j>
S n M = <i>

S n P =4>

El grfico dibujado a continuacin muestra que este silogis


mo es lgicamente invlido porque, siendo la conclusin de for
ma A, toda la zona de S que est fuera de P debe quedar rayada y
esto, en efecto, obviamente no sucede.

246
Como se puede constatar, la conclusin no ha quedado
automticamente grafcada porque no aparece equis alguna en la
zona S n P , lo que significa que el silogismo examinado no es
vlido.
Aadiremos un ejemplo en el que se usa un predicado nega
tivo, lo que a menudo fuerza el uso del lenguaje natural pero pone
a prueba el uso de los diagramas de Venn para representar la
interseccin entre dos o ms conjuntos. El predicado que usare
mos es 'no-fumador':

Todos los no-fumadores son ahorradores


Ningn vegetariano es fum ador
Luegoy todo vegetariano es ahorrador.

Las frmulas de Boole correspondientes son:

(A ) M n P = <j)
(E ) S n M = <|>

(A ) S n P = *

Como puede observarse, la zona de S que est fuera de P ha


quedado totalmente sombreada al graficarse las dos premisas. Ello
significa que el silogismo anterior es vlido, aunque no corres
ponde directamente a ninguno de los 15 conocidos debido a que
usa un predicado negativo.

247
13.3 Silogismo en los que se establecen condiciones
necesarias

Intuitivamente W es condicin necesaria para Z cuando la reali


zacin o cumplimiento de Z presupone el cumplimiento de W.
En otros trminos, no es posible que se produzca Z y no W. Por
ejemplo, tener libreta electoral es condicin necesaria para poder
para poder sufragar. Ello nos permite construir el siguiente con
dicional verdadero: Si Juan ha sufragado en las elecciones de ayer en
tonces tiene libreta electoral. En efecto, no es posible dentro de nues
tro sistema legal que alguien sufrague y no tenga libreta electo
ral. En cambio, es posible que alguien tenga libreta electoral y no
sufrague. La causa de eso podra ser una enfermedad o un viaje
el da de las elecciones. Debido a lo anterior el condicional Si Juan
tiene libreta electoral entonces ha sufragado en las ltimas elecciones no
es necesariamente verdadero porque admite excepciones; el an
tecedente puede ser verdadero pero el consecuente falso en el caso
de que, por ejemplo, Juan haya estado enfermo.
Lo dicho demuestra que la condicin necesaria aparece siem
pre en el consecuente de un condicional que es verdadero porque no
admite excepciones. Sin embargo, en el lenguaje natural la condi
cin necesaria puede aparecer en la primera parte de una afirma
cin. Cuando se dice Nadie sino un mdico puede prescribir un
medicamento, lo que se est estableciendo es que ser mdico es con
dicin necesaria para legalmente prescribir medicamentos. Y ello nos
permite afirmar Si Wprescribe un medicamento entonces es mdico pero
no la proposicin recproca Si W es mdico entonces prescribe un medi
camento. Todo esto en el caso de que admitamos que ser mdico es
condicin necesaria para prescribir un medicamento. Otra manera
de decirlo sera Solamente los mdicos pueden prescribir medicamentos.
Este anlisis conduce a establecer que cuando estamos ante
una afirmacin que establece que W es condicin necesaria para Z
y deseamos traducirla a uno de los esquemas del cuadro de Boecio,
la solucin es recurrir al esquema A con la salvedad de que la con
dicin necesaria no debe figurar en el lugar del sujeto sino del pre
dicado, esto es, en el segundo lugar. Consecuentemente las afirma
ciones Nadie sino un mdico puede prescribir un medicamento y Sola
mente los mdicos pueden prescribir medicamentos deben ser traduci

248
das por Todos los que prescriben medicamentos son mdicos. Este expe
diente permite su representacin mediante los diagramas de Venn.
Un ejemplo ayudar a operativizar nuestra explicacin.
Este silogismo lo podemos traducir aplicando los criterios
expuestos de la siguiente manera:

N adie sino los ap robad os ingresarn a l aula


Solam ente los que ingresan a l aula recibirn prem io

Luego, solam ente los ap robad os recibirn p rem io.


El mtodo de los diagramas de Venn es completamente apli
cable a tales inferencias inmediatas para decidir su validez lgi
ca. En este caso tambin se cumple:

Todos los que Ingresarn a l au la sern los aprobados


Todos los que recibirn p rem io ingresarn a l aula
Luego, todos los que recibirn p rem io sern los aprobados.

Dejamos al lector, como ejercicio, la construccin del diagra


ma de Venn respectivo. Se trata de modos y figuras que deben
serle conocidos.

13.4 Inferencias inmediatas

La lgica tradicional trataba de un conjunto de inferencias a las


que llamaba inmediatas, porque estn constituidas slo por dos
proposiciones de las cuales una cumple la funcin de premisa y
la otra de conclusin. Asimismo, las proposiciones usadas tradi
cionalmente en tales inferencias son de una de las cuatro formas:
A, E, I, O. Ejemplos de este tipo de inferencias son:

I) Ningn p eru an o es australiano


L u ego, ningn australiano es p eru an o.
II) Todos los enteros p a r e s son nmeros
Luego, todos los nmeros son enteros p a res.

III) Algunos peru an os no son religiosos


L u ego, algunos religiosos no son peruanos.

249
lo establecido por la regla R19 para el caso de los silogismos.
Consecuentemente, una inferencia inmediata es lgicamente
vlida si slo si al graficarse su premisa queda automticamente
graficada su conclusin.
En lo que sigue nosotros decidiremos la validez slo del ter
cer ejemplo; los otros dos, que son muy sencillos, lo dejamos al
lector como parte de su prctica.

El grfico muestra claramente que la inferencia no es vlida,


pues para serlo debera haber una equis en la zona comn a S y P,
y eso no ocurre.

13.5 Falacias Lgicas y Retricas

Las inferencias lgicamente invlidas, de acuerdo con los diagra


mas de Venn, son falacias de la lgica de los predicados.
Las falacias as definidas son carcter estructural, formal,
sintctico o estrictamente lgico. A ellas se puede aadir lo que se
denomina falacias semnticas cuyo origen es el uso de trminos
cuyo significado es ambiguo. Un ejemplo es el siguiente silogis
mo: Todos los pases descritos por algn narrador famoso estn
situados en algunos de los cinco continentes. Alicia y sus per
sonajes vivieron en un pas descrito por un narrador famoso.
Por tanto, Alicia y sus personajes vivieron en un pas situado
en alguno de los cinco continentes. El error, intencional o no,
consiste en que el trmino medio, "pas descrito por algn narra
dor famoso", ha sido utilizado con dos significados distintos. En
el primer caso se refiere slo a pases geogrficamente existentes
y en el segundo caso se refiere a un pas de ficcin. De esta mane
ra un solo trmino encubre dos y, consecuentemente, el silogismo
anterior tiene realmente cuatro trminos, cuando por definicin

250
debe tener slo tres. Por ello a esta falacia, que generalmente se
comete en el uso del trmino medio, se le llama falacia de cuarto
trmino, en latn, Quatemio terminorum.
Hay otro tipo de falacias ajenas al campo de la lgica y se
denominan retricas. Se incluyen en algunos manuales porque
tienen frecuentemente semejanza con el uso de silogismos. Sin
embargo, la aplicacin de un mtodo como el de Venn, o cual
quier otro adecuado, permite mostrar la diferencia entre una de
ficiencia estructural o sintctica y una de carcter semntico. Las
falacias retricas se basan en la significacin difusa de los trmi
nos y en el impacto emocional que tienen en el auditorio. Los as
pectos sintcticos carecen, en este caso, de importancia. Un estu
dio detallado de este tema escapa a los objetivos de este texto.
Como orientacin general sealaremos que los argumentos
retricos se proponen exclusivamente convencer a un pblico o
auditorio. Ellos no pretenden probar la verdad como ocurre en la
lgica. Un ejemplo de falacia rtorica es la expresin Vox populi,
Vox dei, que significa literalmente que la voz del pueblo es la voz
de Dios. En breve, se asume que las mayoras siempre dicen la
verdad o tienen la razn. Esta falacia se denomina Argumento ad
populum. Su falsedad es clara, durante siglos el pueblo dijo que la
Tierra no se mova y se encarcel a Galileo y se persigui a
Coprnico por decir lo contrario. Este ejemplo histrico, y mu
chos como l, prueban concluyentemente que lo que dice el pue
blo no es necesariamente verdadero.
Existen muchos falacias retricas semejantes a la anterior.
Mencionaremos slo algunas de las ms conocidas. El recurso al
Argumentum ad Verecundiam consiste en apelar a una autoridad
muy prominente para apoyar la afirmacin que se sostiene. Por
ejemplo, en la Edad Media se deca "la Tierra no se mueve" es
indudable porque Aristteles as lo escribi. Y en efecto, Aristteles
fue un pensador, an hoy, de notable solvencia intelectual, pero
ninguna persona, por sabia que sea, es fuente de verdad. Desde
hace mucho tiempo todos conocemos que respecto de la posicin
y movimiento de la tierra Aristteles se equivoc. El Argumentum
ad hominem consiste en refutar una tesis descalificando a la perso
na que la sostiene. Un ejemplo de esta falacia sera afirmar que lo
que deca Winston Churchill era falso porque padeca de alcoho

251
lismo. El Argumenhim ad misericordiam consiste en apelar a los sen
timientos piadosos o humanitarios del auditorio para que se acepte
una conclusin. Por ejemplo, que un acusado no es culpable por
que su infancia fue muy dura y miserable. La falacia ignoratio
elenchi (ignorancia del asunto) consiste en pretender probar una
afirmacin con argumentos que carecen de pertinencia o antingen-
cia. Por ejemplo, pretender probar la verdad de un enunciado
sobre un experimento recurriendo al trabajo denodado de los
investigadores cientficos. La falacia de la Falsa generalizacin
es uno de los casos especiales porque es de carcter lgico y ret
rico. Consiste en atribuir una propiedad a todo un conjunto
cuando se ha constatado que la poseen slo algunos elementos de
ste. Su invalidez lgica se prueba demostrando que las frmulas
(3x) Px (Vx) Px y Fa (Vx) Fx son lgicamente invlidas. Ello
ocurre cuando se afirma que Todos los brasileos son buenos
futbolistas porque se conoce que algunos efectivamente lo son.
La comprensin del funcionamiento de las dos frmulas anterio
res requiere del estudio de la leccin 14. En fin, hay otras muchas
falacias retricas menos frecuentes. Lo comn a todas es que pre
tenden persuadir a un auditorio para obtener su apoyo sin que
interese la verdad. Los sofistas griegos fueron los que usaron con
especial maestra las falacias retricas cuando intervenin en el
gora de Atenas, por encargo de quienes tenan poder pero no el
don de la palabra, para influir en las decisiones de la ciudad to
madas mediante votacin. Como se conoce, el gora de Atenas
es uno de los lejanos antecedentes de los parlamentos modernos.

252
XIV
EL LENGUAJE PREDICATIVO

Objetivos:

Conocer y aplicar un sistema formal cuyos mecanismos de fun


cionamiento no se fundan en la observacin de diagramas sino
en reglas de deduccin.
Formalizar usando el lenguaje PMP expresiones mondicas y
relacinales cuantificadas.
Analizar la estructura interna de las proposiciones y ampliar
las reglas RDN.
Superar las limitaciones propias de los diagramas de Venn.
CUESTIONARIO 14
EL LENGUAJE PREDICATIVO PMP

Instrucciones

I. Determinar cules de las siguientes frmulas no son inter


pretables como proposiciones.
1. F ( a , b )
2. ( 3 y ) F ( x , y )
3. ( 3 y ) F ( y ) - > ( V y ) F ( x ) )
4. (V x ) (V y ) F ( x , y ) - R ( x , y )
5. P ( x ) a ~ Q ( x )
6. P ( a ) A ~ Q ( a )

II. Determinar cules de las siguientes frmulas estn en forma


normal prenex.
1. (V X ) F ( X ) V ( 3 y ) G ( y )
2 . F ( a , b ) - > ( 3 x ) ( F ( X/b) a ~ P ( x ) )
3. (Vx)( 3 y ) R ( x , y )
4. (Vx) F ( x ) - (V x ) Q( x )
5. (Vx)( 3 y ) ( R ( x , y ) - > S ( x ) )
6. (Vx) (Vy) (Vz) ( ( R ( x , y ) A R ( y , z ) ) - > R ( x , z ) )

III. Cules de las siguientes frmulas no son lgicamente vlidas?


1. ( V x ) F ( x ) ^ F a
2. ( 3 x ) F ( x ) - > F ( y )
3. F a ^ ( V y ) F ( y )
4. ( 3 x ) F ( x ) - > ( V y ) F ( y )
5. ( 3 y ) P ( y ) - > ( V x ) P ( x ) )
6. ( V x ) ( 3 y ) R ( x , y ) - ( 3 y ) ( V x ) R ( x , y )

IV. Formalizar, usando cuantifcadores, las siguientes afirmaciones:

1. No todos los nmeros son pares.


2. Algunos nmeros son primos.

[255]
3. Ningn electrn es un protn.
4. Cada alumno tiene nota aprobatoria.
5. Solamente los aprobados recibirn diploma.
6. Si todos son enemigos de todos, entonces cada uno es su propio
enemigo.
7. Hay una persona que ama a todas las personas.
8. Hay un nmero natural que es menor que todos los nmeros
naturales.
9. Hay al menos un alumno que se matricula en todos los cursos
10. Todas las cabezas de caballo son cabezas de animales.

V. Probar las siguientes equivalencias (Puede usarse la regla RAA)

l-(3y)(P(y)-*(Vx)P(x))*>((Vy)P(y)-(Vx)P(x))
2 . ( Vx ) ( P ( x ) a Q ( x ) ) o ( ( V x ) P ( x ) ) a ( V x ) Q ( x ) )
3.(Vx)(P(x)v(Vy)Q(y))H((Vx)P(x)v(Vy)Q(y))
4 . ( 3 x ) ( P ( x ) v Q ( x ) ) ( ( 3 x ) P ( x ) v ( 3 x ) Q ( x ))

VI. Construir una deduccin que pruebe que las premisas impli
can a la conclusin.

1. Todos los cuadros son rombos


Algunos rectngulos no son rombos
Lugo, algunos rectngulos no son cuadros.
2. Ningn nmero imaginario es un nmero real
Algunos nmeros complejos son nmeros reales
Luego, algunos nmeros com plejos no son imaginarios.
3. Ningn nmero imaginario es un nmero real
Todos los nmeros racionales son nmeros reales
Luego, ningn nmero racional es imaginario.
4. Todos los cuadros son polgonos regulares
Ningn trapezoide es un polgono regular
Luego, ningn trapezoide es un cuadrado
5. Hay un cisne que no es negro
Luego, no todos los cisnes son negros.

256
LECCIN 14
Un lenguaje predicativo PMP

En esta leccin, nos proponemos incrementar el repertorio de sm


bolos y de reglas del lenguaje proposicional PM extendindolo a
un lenguaje predicativo PMP, el mismo que se construye aadien
do medios expresivos que nos permitirn analizar la estructura
interna de las proposiciones y ampliar nuestras reglas de deduc
cin. Si consideramos un esquema de frmula como A B , ahora
estaremos en condiciones de decidir, en situaciones especficas,
en base al examen de la estructura interna de A y de B, si el con
secuente se deduce o no del antecedente.

14.1 Predicados lgicos

Hay una semejanza parcial entre los predicados del lenguaje na


tural y los predicados lgicos, en el sentido, de que palabras, que
denotan propiedades o cualidades como 'rojo', 'caliente', 'veloz7,
'peruano', etc. son predicados gramaticales y tambin son predi
cados lgicos de una posicin, porque se afirman de slo un nom
bre; por ejemplo, 'Juan es veloz'. La diferencia se produce con tr
minos como 'gato', 'len' u otros que son sustantivos comunes
pero que en lgica, en ningn caso, son nombres sino predicados.
La situacin se acenta ms con palabras como 'hermano', 'cu
ado', 'cabeza'que el lenguaje PMP considera predicados de dos
posiciones o predicados relacinales en el sentido de que se apli
can a dos nombres; por ejemplo, 'Juan es hermano de Magda' o
'Elena es cuada de Rosa'. En estos casos, de manera general, los

[257]
predicados son '...hermano de '...cuada de../, '...cabeza de...'.
Como para comprender el sentido de estos predicados no es ne
cesario determinar nombre alguno, en lugar de ello podemos usar
letras tales como x, y, z, etc. cuyo sentido es representar cualquier
nombre sin necesidad de especificarlo. Debido a lo dicho, tales
letras se conocen como variables de nombre o variables indivi
duales. De manera anloga, en lugar de 'hermano de', 'cuado
de' y 'cabeza de' podemos usar letras maysculas R, S, T, VV, etc.
De tal suerte que los ejemplos anteriores pueden quedar repre
sentados por las frmulas R( x , y ), S( x , y ), T( x , y ). A las
letras maysculas anteriores se les denomina variables predica
tivas o, simplemente, letras predicativas. Su funcin es represen
tar cualquier predicado de dos posiciones lo que no quiere decir
que sen las nicas variables predicativas que usemos. Podemos
usar cualquier otra letra mayscula, y, lo que define su sentido
en el lenguaje PMP, es el nmero de variable de individuo que
se escribe a su derecha.
Los predicados que se aplican a una sola variable de nombre
tambin se representan mediante variables predicativas. Por ejem
plo, el predicado 'nmero par' se escribe en PMP as: P ( x ).
Anlogamente, 'gato' se puede representar por G ( y).
En el lenguaje natural la frmula P ( x ) se lee x es P o x tiene
la propiedad P. Particularizando la expresin a la interpretacin
que hemos dado en estas exposicin, tendramos como lectura: x
es una nmero par. Y para G ( y ) la lectura sera y es un gato. En
circunstancias en las que el contexto est claro y se requiere ser
muy rpido y operativo las frmulas como P(x) se leen en trmi
nos eje P de x.
las frmulas de la forma R ( x , y ) se las lee: el par x, y
satisface la relacin R, o el par x , y cumple la relacin R. En algu
nos textos se propone la siguiente lectura: entre el par x, y existe
la relacin R. Esta propuesta es intituitiva pero es inexacta por
que equivale a cuantificar existencialmente a la variable predi
cativa R, lo que rebaza a un lenguaje de primer orden como PMP.
Sin embargo, el lector principiante slo podr comprender este
argumento, con exactitud, en un texto ms avanzado que este.
A las frmulas P ( x ) , G ( y ) etc. Se les denomina funciones
proposicionales mondicas o de grado 1 porque su variable

258
predicativas se aplican a una sola variable de nombre. A las fr
mulas R ( x , y ) , S ( w , z ) , etc. Se les llama fu nciones
proposicionales didicas porque sus variables predicativas se
aplican a un par de variables de nombre. Hay funciones
proposicionales tridicas porque sus variables predicativas se
aplican a tres variables. Y, en general, hay funciones proposi
cionales de grado n cuando sus variables predicativas se apli
can a n variables de nombre y n es un nmero tan grande como
se desee. La forma general de una funcin predicativa de grado
n es P ( x , xy ..., x ) en la que el exponente n slo indica el
grado de la variable"predicativa P y, por tanto, de la frmula en
su conjunto. Siempre que n 3 2, la variable predicativa es una
relacin.

14.2 Proposiciones en el lenguaje PMP

Debido a que el lenguaje PMP es de primer orden, se asume que


las variables predicativas se comportan como predicados o cons
tantes predicativas en el sentido de que siempre que se las escri
be, el contexto determina su significado especfico.
Sin embargo, aunque se asume que el significado de las va
riables predicativas es conocido, por lo que se les llama abrevia
damente predicados, en cambio el significado de una funcin
proposicional cualquiera P ( x ) se considera desconocido porque
como x no es un nombre no se sabe de qu objeto se predica P y,
consecuentemente, P ( x ) no es ni verdadera ni falsa. Sim ple
mente, las funciones proposicionales, desde el punto de vista
lgico, carecen de valores de verdad (verdadero-falso) llamado
tambin valores alticos o veritacionales.
Un primer mecanismo para lograr dotar a una frmula P ( x )
de un valor altico consiste en reemplazar todas sus variables de
nombre por nombres o constantes individuales que se represen
tan por las primeras letras minsculas del alfabeto a, b, c , ... En
caso necesario se usa subndices a , ay ..., ba, by ... Estas constan
tes individuales son, en este contexto nombres propios abre
viados . Si asumimos que P es el predicado 'peruano' y a una
manera abreviada de escribir Tupac Amaru' entonces desde P ( x )
podemos obtener P ( a ) como una proposicin verdadera porque

259
la variable x ha sido reemplazada por el nombre de un individuo
que tiene la propiedad de ser peruano. Si b es una manera abre
viada de escribir 'Leonardo de Vinci' entonces P ( b ) ser, evi
dentemente, una proposicin falsa.
Es importante sealar que, en el lenguaje PMP, los nmeros
naturales, enteros, racionales, reales y complejos son considera
dos nombres propios o constantes indi-viduales. De tal suerte,
que si Q es el predicado 'impar7, entonces Q ( y ) es una funcin
proposicional y Q ( 6 ) es una proposicin falsa. En general, a
frmulas como P ( b ), Q (6) etc. las llamaremos ejemplificadones,
las mismas que permiten deducir que para dotar de significado a
una funcin proposicional o interpretarla es necesario recurrir a
un conjunto, no vaco, de objetos o individuos nombrables al que
denominaremos dominio de interpretacin. Este puede ser un
conjunto muy pequeo o bastante grande; pero siempre limitado
a slo los objetos de los que pretendemos hablar: personas, n
meros, figuras geomtricas, etc. Los conjuntos que pretenden in
cluir todo tipo de objetos estn excluidos como dominios de in
terpretacin. Asimismo la nica manera de hablar de los objetos
de un conjunto es asignndoles nombres al7 a3, ...etc.

Definicin 19. Una funcin proposicional P(x^.,.xJ de grado n se convierte


en proposicin si, y slo s, cada una de sus variables de nombre es reemplazada
por una letra constante individual o nombre.

14.3 Trminos y frmulas

A las letras variables de nombre y a las constantes individuales


se le denomina, por convencin, trminos y nos referimos a ellos
mediante la letra minscula t. En cambio, a las letras P, Q, R, etc.,
como dijimos antes, las denominaremos abreviadamente predi
cados. Las variables de nombre son trminos que se refieren a in
dividuos u objetos no especificados del dominio de interpretacin
cuya identidad se desconoce. Las constantes individuales estn
asociadas con individuos especficos del dominio de interpreta
cin. Adicionalmente, aunque no las utilizaremos en este manual,
las letras que denotan funciones matemticas de la forma f (x) tam
bin son trminos en el lenguaje PMP. Asimismo, una letra pre-

260
dicativa del tipo P, Q, etc. No es una frmula de PMP y tampoco
lo es un trmino aislado. Pero una letra predicativa de grado n
seguida de n trminos si es una frmula. Es el caso de P( x ), P
( a ), R ( x, y ), R ( a , y ), R ( x , b ), R ( a, b ), que s son frmulas
de PMP bajo la suposicin de que P es un predicado de grado 1
y de que R es un predicado de grado 2. Las frmulas anteriores
son, excepto la segunda y la ltima, funciones proposicionales
porque en ellas hay al menos una variable que no ha sido susti
tuida por un nombre, la misma que se llama variable libre. Asi
mismo, las frmulas predicativas que tienen al menos una varia
ble de nombre libre se denominan frmulas abiertas. Todas las
frmulas anteriores son atmicas y la conjuncin, disyuncin in
clusiva, disyuncin exclusiva, el condicional, la equivalencia y la
incompatibilidad que las tienen como componentes son frmu
las predicativas moleculares de PMP. Ejemplo : P ( x ) v R ( x , b ) .

14.4 Cuantificadores

En el lenguaje natural hay trminos como 'todos7, 'cada uno',


'cualquiera', etc. que se usan para hacer referencia a la totalidad
de los miembros de un conjunto. Por ejemplo la afirmacin 'Cual
quier ciudadano puede defender la constitucionalidad de la Re
pblica' significa que todo ciudadano puede hacerlo. Si en el sa
ln de clases el profesor dice Cada uno tome su lpiz, se entiende
que la orden debe ser cumplida por todos los alumnos.
Todas estas expresiones del lenguaje natural son expresadas
en el lenguaje PMP mediante el operador ' V ' que se denomina
cuantificador universal. Tambin hay expresiones como 'hay un
objeto', 'existe al menos uno', 'alguna cosa', 'al menos una cosa'
etc. que en el lenguaje PMP son representadas por el operador' 3 '
denominado cuantificador existencial. En la medida que los
cuantificadores estn siempre asociados a variables de nombre
los escribiremos as ( V x) y ( 3 x ). Asimismo, anotamos que las
variables de nombre cumplen en PMP una funcin muy semejan
te a la cumplida por los pronombres personales en los lenguajes
naturales como el castellano, ingles, etc.
Supongamos que la funcin proposicional L ( x,a ) la inter
pretamos en el lenguaje natural como la expresin 'x aprende

261
Matemtica' donde a es el nombre de la asignatura de Matemti
ca y sealamos como dominio de interpretacin al conjunto de
los alumnos de un saln de clase D. Es posible que afirmemos la
proposicin 'Hay al menos un alumno que aprende Matemtica',
la misma que puede ser formalizada en el lenguaje PMP como
( 3 x ) L ( x , a ) . Igualmente, la proposicin Todos los alumnos de
la clase aprenden Matemtica' puede ser formalizada como (V x )
L ( x, a ). La primera frmula la leeremos Existe al menos un indivi
duo x tal que x aprende Matemtica y la segunda Para todo individuo
x, x aprende Matemtica.
De lo anterior se infiere que cuando anteponemos un cuanti-
ficador existencial a una funcin proposicional lo que hacemos es
afirmar que el conjunto de individuos que satisface un determi
nado predicado no es vaco.En cambio, cuando anteponemos un
cuantificador universal a una funcin proposicional afirmamos
que todos los miembros de un conjunto satisfacen un predicado.
Como sabemos, en ambos casos, ese conjunto se denomina domi
nio de interpretacin.

14.5 Frmulas cerradas

Si tenemos una funcin proposicional L ( x , y ), denominada tam


bin frmula abierta, y le anteponemos un cuantificador que afec
ta a cada una de sus variables libres, entonces obtenemos una fr
mula cerrada de la forma ( V x ) ( 3 y ) L ( x , y ) y a l a s letras x , y se
les denomina en este caso variables ligadas. Si asimismo mante
nemos la interpretacin anterior de tal manera que x vara sobre el
conjunto D de los alumnos e y vara sobre el conjunto D* de las
asignaturas del currculum, entonces la frmula anterior puede leer
se en lenguaje natural como Para todo alumno x, existe al menos una
asignatura y, tal que x estudia y. Esta proposicin equivale a la ora
cin coloquial Todo alumno estudia alguna asignatura'.
En este caso el dominio de interpretacin es un conjunto de
pares ordenados que constituyen el producto cartesiano D x D*.
Es respecto de este dominio que los cuantificadores adquieren
significado y que la frmula anterior es verdadera o falsa. Ser lo
primero si, en efecto, todo alumno estudia al menos una asigna
tura. Y ser lo segundo si existe algn alumno que no estudia

262
asignatura alguna. De lo dicho se deduce que una frmula ce
rrada s es una proposicin en el sentido de que es verdadera o
falsa respecto de un dominio de interpretacin.

14.6 Alcance de un cuantificador

Examinemos los siguientes ejemplos:

i) ( V x ) ( P ( x ) v Q ( x ) )
ii) ( V x ) L ( x , y) v ( 3 y ) R ( x , y )
m) (3 x ) L ( x,a) v Q ( x ) )

Se denomina alcance de un cuanfificador a la porcin de una


frmula, hacia su derecha, dentro de la cual liga las ocurrencias o
apariciones de una variable. Un cuantificador hacia su derecha slo
puede tener o un parntesis '('/ llamado de abre, o una letra
predicativa con n trminos de la forma P ( t t ). En el primer
caso, su alcance llega hasta el respectivo parntesis')', llamado de
cierre, y en el segundo caso, su alcance llega hasta el parntesis que
cierra al trmino t . En el ejemplo i) la variable x tiene dos ocurren
cias ligadas, porque el cuantificador es el operador de mayor jerar
qua debido a que es externo respecto de los parntesis que agru
pan a la disyuncin inclusiva. La primera ocurrencia de x en i) no la
contamos porque en ella forma parte del cuantificador. Con la ex
cepcin anterior, definimos las ocurrencias de una variable de
nombre v en una frmula F como el nmero de veces que aparece
v en F a la derecha de algn predicado.
En el ejemplo n) la disyuncin inclusiva es el operador de
mayor jerarqua, porque el alcance del primer cuantificador ter
mina antes de la primera conectiva a su derecha. El segundo
cuantificador no puede disputarle la jerarqua a 'v' porque, sim
plemente, no tiene alcance hacia la izquierda y 'v ' s tiene alcance
hacia la izquierda y hacia la derecha. En este ejemplo la primera
ocurrencia de y es libre porque el primer cuantificador slo liga
la primera ocurrencia de x y el segundo no tiene alcance alguno
hacia la izquierda. Asimismo, la segunda ocurrencia de x tam
bin es libre porque el segundo cuantificador no liga a x pero si la
segunda ocurrencia de y.

263
En el ejemplo m) las dos ocurrencias de la nica variable es
tn ligadas y a la letra a no se le puede aplicar un cuantificador
porque no es una variable sino una constante individual con sig
nificado especfico. Tanto el primer ejemplo como el tercero son
frmulas cerradas interpretables como proposiciones generales,
universal y existencial, respectivamente. El segundo ejemplo es
una funcin proposicional de forma disyuntiva a causa de que
tiene a y como variable libre, en su primera ocurrencia, y a x en la
misma situacin, en su segunda ocurrencia. Por tanto, este ejem
plo no es una proposicin en el lenguaje PMP.

14.7 Forma normal prenex

Si consideramos una funcin proposicional tal como L ( x , y ) en


contramos que hay ocho maneras distintas de cerrarla.

(Vx)(Vy)L(x,y)
( Vy)( Vx)L(x,y)
(3x)(3y)L(x,y)
(3y)(3x)L(x,y)
(Vx)(3y)L(x,y)
(3y)(Vx)L(x,y)
(Vy)(3x)L(x,y)
(3x)(Vy)L(x,y)

Si esta vez interpretamos L ( x, y ) como x ama a y, entonces el


dominio de interpretacin va a ser el conjunto de los pares o pa
rejas de seres humanos. Si denominamos al conjunto de los seres
humanos H, entonces el dominio de interpre-tacin estar consti-

264
2
tuidos por el producto cartesiano de H x H ( H ). En la medida
que todas las frmulas anteriores son cerradas, entonces cada una
2
de ellas ser verdadera o falsa respecto de H .
La interpretacin de las dos primeras frmulas afirma que
todo ser humano ama a todo ser humano lo que equivale a soste
ner que todos los pares ordenados que formemos con el conjunto
H satisfacen la funcin proposicional x ama a y. De ser asi, las dos
primeras frmulas son lgicamente equivalentes.
Las frmulas tercera y cuarta tambin son lgicamente equi
valentes. La interpretacin de ambas afirma hay al menos una
persona que ama a al menos una persona. Las cuatro primeras
frmulas establecen que el orden es irrelevante cuando los cuanti-
ficadores que estn antepuestos a una frmula, denominados en
conjunto prefijo, son todos universales o todos existenciales.
La quinta frmula se interpreta en funcin de que toda perso
na ama a al menos tina persona. Esta proposicin excluye la posibili
dad de que haya una persona que no ame a persona alguna.
La sexta frmula establece que hay al menos una persona
que es amada por todos, cuyo sentidos es distinto del de la ante
rior a causa slo del diferente orden en el que aparecen los cuanti-
ficadores. La sptima frmula afirma la totadad de las personas
es amada por al menos una persona. Y la ltima afirma que hay al
menos una persona que ama a todas las personas.
El sentido distinto, en cada caso, que muestran las interpre
taciones de las cuatro ltimas frmulas prueban que cuando los
cuantificadores que forman el prefijo son universales y existen
ciales el orden es relevante y si es alterado se altera tambin las
condiciones de verdad de las frmulas correspondientes.
Se dice que cuando una frmula exhibe todos los cuanti
ficadores adelante, independientemente del orden en que se en
cuentren, entonces est en forma normal prenex. Asimismo, si
ocurre que en el prefijo todos los cuantificadores existenciales
preceden a los cuantificadores universales, entonces est en for
ma normal de Skolem.
Los ocho ejemplos anteriores estn en forma normal prenex.
Los ejemplos sexto y octavo son ejemplos de la forma normal de
Skolem. Las frmulas tercera y cuarta ilustran, adems, el caso en
el que el nmero de cuantificadores universales es igual a cero.

265
14.8 Formalizacin del cuadro de Boecio en el lenguaje
PMP

Un ejemplo de la proposicin A del cuadro de Boecio es la afir


macin fsica 'Todos los cuerpos que se someten a la accin del calor
se dilatan. En esta afirmacin hay dos predicados mondicos. El
primero x es un cuerpo sometido a la accin del calor y el segundo x
se dilata. Al primero lo escribimos como P( x ) al segundo como
Q ( x ), siguiendo convenciones antes establecidas. Un aspecto
adicional a considerar es que toda afirmacin general universal
del tipo de A es interpretada como hipottica en el sentido de que
no es posible constatar que todos los individuos de un conjunto
tienen una propiedad cuando este conjunto es infinito. Y ocurre
que en la formalizacin de las teoras cientficas con frecuencia
hay que postular que los dominios de interpretacin son conjun
tos infinitos, como, por ejemplo, el conjunto de los nmeros na
turales. Ello conduce a que se interprete que la estructura interna
de una proposicin A es condicional y a que pueda ser adecua
damente parafraseada como sigue: Para todo objeto x, si x tiene la
propiedad P, entonces x tiene la propiedad Q. Ello nos conduce a la
siguiente frmula:

(A ) (Vx)(P(x)-Q(x)

Una afirmacin del tipo E tambin es en el lenguaje PMP una


frmla general universal de estructura interna condicional por
la misma razn que antes expusimos. Manteniendo la misma in
terpretacin de los predicados P y Q, la afirmacin Ningn cuerpo
sometido a la accin del calor se dilata puede ser parafraseada as:
Para todo objeto x, si x tiene la propiedad de ser sometido a la accin del
calor, entonces x no tiene la propiedad de dilatarse. Esto da lugar a la
siguiente frmula:

(E ) ( V x ) ( P ( x ) ~ Q ( x ) )

Una afirmacin del tipo I, que es una frmula general


existencial afirmativa, dentro de la misma interpretacin es
parafraseada as: Existe al menos un objeto x tal que x tiene al mismo

266
tiempo la propiedad de ser sometido a la accin del calor y de dilatarse.
La respectiva frmula es la siguiente:

(I) (3x)(P (x)aQ (x)

De manera anloga el parafraseo de una proposicin del tipo O


es: Existe al menos un objeto x, tal que x tiene la propiedad de ser sometido
a la accin del calor y x no se dilata. La frmula en este caso es :

(O ) ( 3 x ) ( P ( x ) a ~ Q ( x ))

14,9 Formalizacin de proposiciones con predicados de


grado 2

Una limitacin fundamental del cuadro de Boecio y de la teora


del silogismo de Aristteles, es que no permite traducir al lenguaje
lgico proposiciones cientficas, inclusive algunas muy elemen
tales. La razn es que las afirmaciones cientficas establecen re
laciones entre los objetos de un conjunto o dominio, lo que no pue
de ser formulada mediante predicados mondicos que son los
nicos que admiten el cuadro de Boecio. Por ejemplo, la afirma
cin que dice dos cosas iguales a una tercera son iguales entre s,
que se usa como axioma en los Elementos de Euclides, no puede
ser expresada usando los medios expresivos del cuadro de Boe
cio debido a que este viejo axioma contiene el predicado de igual
dad que slo puede ser formalizado usando un predicado de gra
do 2, denominado tambin una relacin.
S formalizamos el predicado x es igual a y mediante la fun
cin proposicional R ( x, y ), entonces la proposicin matemtica
antes mencionada da lugar a la siguiente formalizacin:

(V X ) (V y ) (V z ) ( ( R ( x , y ) a ( R ( y, z ) ) > R ( x , z ))

La lectura de esta frmula, que es tambin su parafraseo en


lenguaje natural es: Para toda terna de objetos x, y, z, si el primero
est con el segundo en la relacin R y si el segundo est con el tercero en
relacin R, entonces el primero est con el tercero en la relacin R.

267
Hay otras afirmaciones, comunes en aritmtica, que tampo
co pueden ser expresadas en el lenguaje de Aristteles y Boecio.
Por ejemplo la proposicin T ara todo tipo de nmero natural
existe siempre otro mayor que l" la misma que equivale a la pro
posicin negativa 'No existe el mayor nmero natural'. Si forma
lizamos el predicado relacional x es mayor que y mediante la fun
cin proposicional M ( x, y ), la formalizacin correspondiente es:

( Vy ) ( 3 x ) M ( x, y )

14.10 Reglas de equivalencia entre cuantificadores

Los esquemas de frmula que presentamos, en lo que sigue, tie


nen validez general sin que las limite el hecho de que el predica
do que escribimos sea presentado como mondico. Lo que ellas
autorizan es a intercambiar un cuantificador universal negado en
el flanco izquierdo (externo) por uno existencial negado en el flan
co derecho (interno) y un cuantificador existencial negado en el
flanco externo por uno universal negado en el flanco interno. La
tercera y la cuarta equivalencia son consecuencias lgicas de las
anteriores.

~(Vx)P(x)o(3x)~P(x)
~ ( 3,x ) P ( x ) < - ( V x ) ~ P ( x )
~(3x)~P(x)-o-(Vx)P(x)
~{ V x ) ~ P ( x ) o ( B x ) P ( x )

Si se supone que el dominio de interpretacin es un conjun


to finito de n objetos, la prueba de las equivalencias anteriores
es inmediata en relacin con los conocimientos brindados en este
manual. Ella se basa en la aplicacin de las reglas de De Morgan.
En tal caso una cuantificacin universal es definida como una
conjuncin de n compo-nentes y una cuantificacin existencial
como una disyuncin de tam bin n componentes. En la medi
da que estamos trabajando con cuantificadores negados, la fr
mulas ~ ( V x ) P ( x ) y ~ ( 3 x ) P ( x ) dan lugar a las siguientes
equivalencias:

268
~ ( V x ) P ( x ) ^ ~ ( P ( a i ) A P ( a 2) A ........A P ( a J )
- ( 3 x ) P ( x ) <-> - ( P ( a 1) v P ( a 2) v / . . . / v P ( a n) )

Si se obtiene, aplicando la regla de De Morgan, la equivalen


cia de la conjuncin negada, se tendr una disyuncin con cada
uno de sus n componentes negados que corresponde a la defini
cin de (3 x ) ~ P ( x ). Y si, anlogamente, se obtiene la equiva
lencias de la disyuncin negada, se obtendr una conjuncin de n
componentes, cada uno negado que es la definicin de ( V x ) ~ P
( x ). Los detalles los dejamos como ejercicio en la medida que el
lector atento, al estudiar esta parte del libro, ya tiene conocimien
tos y entrenamiento suficiente como para hacer las verificaciones
necesarias. La prueba de validez de la equivalencia mostrada por
la tercera y cuarta frmula, restringida a este contexto, tambin
se puede efectuar, de manera inmediata, siguiendo el mismo pro
cedimiento. Esta tarea la dejamos al lector.

14.11 R eg las de e lim in a ci n y re in tro d u cci n de


cuantificadores

La realizacin de deducciones con frmulas cuantificadas requiere


de una ampliacin de la versin que hemos dado antes de las re
glas RDN de Gentzen. De esta manera, construiremos un siste
ma de deduccin natural RDNP que est constituido por RDN
ms las reglas adicionales que a continuacin expondremos. Es
tas reglas tienen como funcin eliminar cuantificadores por me
dios lgicamente vlidos para permitir la aplicacin de las reglas
de deduccin a las conectivas que forman parte de la estructura
interna de las frmulas cuantificadas. Luego, posibilitan la rein
troduccin de los cuantificadores, all donde es posible, para ob
tener las conclusiones buscadas.

14.11.1 Ejemplificacin universal


Una frmula de la forma ( Vx ) P ( x ) es verdadera respecto de
un dominio D si todos (y cada uno) de los elementos de D per
miten la construccin de afirmaciones verdaderas de la forma Pa,
Pb, etc. (En este caso y en adelante, por razones de simplicidad,

269
escribiremos las constantes individuales sin parntesis). Supon
gamos que D = { 2, 4, 6..., 2n \y que la funcin proposicional P ( x )
es interpretada como x es par, luego la frmula cerrada ( Vx ) P ( x )
es verdadera en D debido a que cada una de las afirmaciones
P (2), P (4), etc. es verdadera. Como cada una de estas afirmacio
nes es un ejemplo o un caso particular respecto de ( Vx ) P ( x ),
podemos decidir que esta frmula es verdadera en D porque to
dos sus ejemplos son verdaderos. Lo anterior pone en claro que
la definicin de la verdad de una proposicin cuantificada debe
realizarse necesariamente respecto de un dominio, pues una mis
ma proposicin puede ser verdadera respecto de un dominio y
falsa respecto de otro. Por ejemplo, si tomamos como dominio de
interpretacin el conjunto de los nmeros naturales N = { 1, 2, 3,
..., n, n+1, ... \,debe resultar claro que la proposicin ( Vx ) P ( x )
no es verdadera respecto de N porque tendra infinitos ejemplos
falsos, tales como P ( 1 ) , P ( 3 ), etc. Lo dicho nos permite dedu
cir que si ( V x ) P ( x ) es verdadera, entonces cualquier ejemplo
de ella es verdadero o tambin un ejemplo concreto. La idea de
cualquier ejemplo la expresaremos a travs de P ( y ) pues la va
riable y no identifica individuo alguno. La idea de un ejemplo con
creto la expresaremos usando nombres propios en frmulas ta
les como Pa. El esquema de esta reglas es:

( Vx ) P ( x ) ( Vx ) P ( x )
----------
---------- --------------------- (EU) RDNP22
P(y) Pb

La postulacin es que desde una frmula cuantificada um


versalmente podemos derivar o un ejemplo arbitrario o un ejem
plo concreto debido a que damos por sabido que todos los miem
bros del dominio satisfacen la funcin proposicional P ( x ).
La idea de un ejemplo arbitrario es la de un ejemplo tomado
al azar como, por ejemplo, cuando se saca un bolo de un nfora
en un sorteo.

14,11.2 Generalizacin universal


Esta regla permite reintroducir el cuantificador universal bajo la
presuposicin de que la propiedad que es verdadera de un indi-

270
viduo arbitrariamente tomado de un dominio, es verdadera de
todo el dominio. Vale decir, si en una lnea de deduccin tene
mos el ejemplo arbitrario P ( y ) y sabemos que la variable y no
est libre en nin-guna de las premisas que estamos utilizando, en
tonces podemos deducir ( V x ) P( x ). El esquema es el siguiente:

P(y)
-------------------------( G U ) RDNP 23
( Vx ) P ( x ) siempre que y no aparezca al menos una vez
libre en alguna lnea anterior que es premisa.

Esta regla no nos permite deducir, por ejemplo, de la afirma


cin x es ladrn la conclusin Todos son ladrones porque la variable
x est libre en la nica premisa que hemos usado.

14.11.3 Ejemplificaciones existencial

Una frmula ( 3 x ) P ( x ) es verdadera, en un dominio D, si exis


te al menos un objeto del domino que permite construir un ejem
plo verdadero. Este objeto, que podra ser nico, lo designaremos
con un nombre propio o constante individual que debe satisfacer
la condicin de no haber aparecido antes en la deduccin para
evitar la presuposicin innecesaria de que se trata del mismo ob
jeto al que hacen referencia otras proposiciones. El esqueleto es
el siguiente:

(3x)P(x)
( E E . ) RDP 24
Pa siempre que a no aparezca antes en la deduccin

Esta regla nos impide deducir desde Hay un gato y Hay un


perro la conclusin Hay un animal que es a la vez perro y gato porque
tendramos que presuponer que las premisas hacen referencia al
mismo objeto, lo que se impide con la restriccin que establece
que el nombre propio que se introduzca por aplicacin de EE. no
debe aparecer antes en la deduccin.

271
14.11.4 Generalizacin existencial

Si en una lnea de deduccin tenemos un ejemplo arbitrario del


tipo P ( y ), entonces podemos deducir (3 x ) P ( x ). Esta frmula
tambin puede ser deducida de un ejemplo con constante indivi
dual tal como Pb. El esquema es como sigue:

P(y) P(a)
--------------------- --------------------- ( G E .) RDNP 25
(Bx)P(x) (3x)P(x)

Debe anotarse que en esta exposicin la frmula P ( y ) es


denominada funcin proposicional y tambin ejemplo arbitrario.
Ello introduce una dosis de ambigedad que no puede evitarse
sin complicar la simbologa que estamos utilizando a un nivel que
excede los alcances de este manual. Esta concesin al rigor es co
mn en los textos introductorios.

14.12 A plicacin de las reglas R D N P a la deduccin


silogstica
En esta seccin formalizaremos algunos silogismos presentados
en lenguaje natural y mostraremos que su conclusin se deduce
desde las premisas aplicando las reglas RDNP. Para el efecto re
curriremos directamente a un ejemplo.

Todas las criaturas hoscas son vistas con desconfianza


Todas las avispas son criaturas hoscas
lu e g o , todas las avispas son vistas con desconfianza

Si representamos con M ( x ), S ( x ) y P ( x ) los predicados x


es una criatura hosca, x es una avispa y x es una criatura vista con
desconfianza, respectivamente, entonces la formalizacin en el len
guaje PMP del silogismo anterior y la deduccin a que da lugar
aplicando las RDNP es la siguiente:

1. ( V x ) ( M ( x ) P ( x ) )
2. ( V x ) ( S ( x ) M ( x ) ) / ( V x ) ( S ( x ) -> P ( x ) )
3. M ( y ) -> P ( y ) EU. en ( 1 )

272
4. S (y )->M (y ) EU. en(2)
5. S ( y ) -> P ( y ) SH. en 3,4
6. ( Vx ) ( S ( x ) -> P ( x ) ) GU. en 5

El procedimiento anterior puede ser aplicado a la siguiente


deduccin que es frecuentemente presentada como s fuera un
silogismo:

Todos los hombres son mortales


Scrates es hombre
Luego, Scrates es mortal.
La correspondiente formalizacin en PMP y la deduccin a
que da lugar es:

(1)(Vx)(H(x)->M(x))
(2) Ha /M a /.\ M a
3. Ha -> Ma Eu. en 1
4. Ma MP. en (2), 3

Como se puede apreciar, no es propiamente un silogismo


porque solamente tiene, en lenguaje clsico, dos trminos ( H ( x ),
M ( x )). El nombre propio 'Scrates' lo hemos representado por
a. Como se comprende fcilmente, los diagramas de Venn no per
miten decidir la, validez de esta sencilla y antigua deduccin por
que no consideran el uso de nombres propios.

14.13 Deduccin con predicados relacinales

El lenguaje PMP posibilita formalizar deducciones que contienen


predicados relacinales. Examinemos el siguiente ejemplo:

Quienquiera que perdone a cualquier persona es un santo


No hay santos
Luego, nadie perdona a nadie.

273
Si representamos x perdona a y por F ( x, y ) y x es un santo por
S ( x ), entonces el razonamiento anterior da lugar a la siguiente
deduccin.

(1) ( V x ) ( Vy ) ( F ( x , y ) S ( x ) )
(2) ~ ( 3x) S ( x ) / :. ( V x ) ( V y ) ~ F ( x , y )
(3)(Vy)(F(w,y)->S(w)) EU. en (1)
(4) F ( w , z ) -> EU.
S (wen) 3.
(5) ( V x ) ~ S ( x ) Equiv. de cuant.
en 2.
(6) ~ S ( x ) EU. en 5.
(7) ~ F ( w , z ) MT. en 4,6
(8) ( V y ) ~ F ( w , y ) GU. en 7.
(9) ( V x ) ( V y ) ~ F ( x, y ) GU. en 8.

Para entender la formalizacin dada es importante conside


rar que las afirmaciones negativas, en el lenguaje natural, tienen
una traduccin lgica poco intuitiva pero precisa. Por ejemplo, si
deseo representar en PMP la afirmacin Nadie es matemtico, pue
do usar la funcin proposicional P ( x ) para representar x es mate
mtico y la frmula ( V x ) ~ P ( x ) para expresar la idea de que
todos los individuos del dominio de interpretacin no son mate
mticos lo que equivale a la afirmacin Nadie es matemtico. Este
criterio de traduccin debe ser considerado para comprender la
traduccin que le hemos dado a la conclusin del razonamiento
anterior que equivale a la afirmacin Para todo par de personas, no
se cumple que una perdone a la otra.
Asimismo, desde el punto de vista de la aplicacin de las
RDNP, es importante sealar que en la lnea 3, al hacer EU. no era
posible usar la variable y debido a que hubiera quedado ligada
por el cuantificador ( Vy ) que se mantiene, en esa lnea, dentro
del cuerpo de la frmula. Esa es la razn por la que necesaria
mente recurrimos a la variable w, pues de otro modo se incurrira
en incorreccin deductiva. Por tanto, cuando se aplica la regla EUen
lina determinada lnea de deduccin, la variable y utilizada no slo no
debe ser libre en premisa alguna usada en la deduccin sino, adems, no
debe quedar ligada por un cuantificador todava no eliminado en esa
lnea. sta es una restriccin que no hemos incluido en la seccin
14.11.1 por razones pedaggicas.

274
14.14 Mecanismo de refutacin de hiptesis

Las reglas RDNP posibilitan mostrar el mecanismo de refutacin


de hiptesis cientficas, bajo la suposicin bastante aceptada de
que toda hiptesis cientfica se formaliza mediante una frmula
A del cuadro de Boecio o mediante una versin relacional de esta
frmula, anloga a la estructura de la primera premisa del ejem
plo de la seccin anterior.
El mecanismo lgico a mostrarse aprovecha, como paso in
termedio, la relacin de contradiccin que existe en el cuadro de
Boecio entre una frmula A y una O, en el sentido de que la ver
dad de ambas no puede ser afirmada simultneamente debido a
que O es equivalente a la negacin de A y A es equivalente a la
negacin de O. Esto es, si se asume o se prueba la verdad de una
proposicin de la forma O, entonces se concluye la verdad de ~A,
lo que equivale a la falsedad de A que representa a una hiptesis
cualquiera. Este mecanismo que consiste en probar la verdad de
una proposicin O para demostrar la falsedad de una hiptesis
de forma A fue utilizado por el filsofo Karl Popper para susten
tar sus tesis epistemolgicas falsacionistas.
El argumento parte de la tesis que sostiene que toda observa
cin de hechos especficos se describe mediante enunciados sin
gulares que identifican a los objetos de los que estamos hablando.
As, supongamos, por ejemplo, que observamos un objeto a que es
cuervo y que no es negro. Esto dara lugar a la verdad del enuncia
do singular Pa a ~ Qa, el mismo que sera un refutador o falsador
de la hiptesis Todos los cuervos son negros, ( V x ) ( P ( x ) - Q ( x ) ) ,
en la medida que permitira deducir la verdad de su negacin,
como mostramos a continuacin.

(1) Pa a ~ Qa //.~(Vx)(P(x)->Q(x))
2. ( 3 x ) P ( x ) a ~ Q ( x ) GE en (1)
3.(3x)~(~P(x)vQ(x)) DM. En 2
4. (3x)~(P(x)-Q(x)) RDNP 1 6 e n 3
5. ~ ( V x ) ( P ( x ) - Q ( x ) ) Equiv. 14.10en4.

Es importante alcarar que en tanto todas las reglas de deduc


cin aplicadas al ejemplo anterior son equivalencias, no ha sido
necesario usar las reglas EE y GU.

275
XV
LA INDUCCIN CLSICA

Objetivos:

Extender la introduccin de Stuart Mili como un procedimien


to de investigacin de los casos de causacin.
Conocer la lgica subyacente en los manuales de metodologa
de la investigacin.
CUESTIONARIO 15
La induccin clsica

Instrucciones

I. Responda a las siguientes preguntas, respecto de la lgica de


Stuart Mili:

1. Cul es la obra de lgica principal de Stuart Mili y cul es su


temtica?
2. Qu es lo especfico de las inferencias estrictas segn S. Mili?
3. Es el silogismo una inferencia circular segn S. Mili?
4. Acepta S. Mili el criterio que establece que lo que legitima a
una inferencia es la transferencia de la verdad?
5. Es la induccin exhaustiva una genuina inferencia?
6. Es la induccin matemtica una genuina induccin?
7. En qu consisti el error de S. Mili respecto de la naturaleza de
la matemtica?
8. Cul es la limitacin de las leyes de Kepler segn S. Mili?
9. Cmo se define el principio de uniformidad de la naturaleza?
10. Cul sera el fundamento del principio de uniformidad de la
naturaleza?
11. Cmo se justifica lgicamente un silogismo inductivo?
12. Es el principio de uniformidad de la naturaleza evidente por
s mismo?
13. Cmo se define una ley de la naturaleza?
14. Son leyes de la naturaleza las regularidades derivadas?
15. Cmo se definen los fenmenos sicrnicos?

[279]
16. Cmo se definen los fenmenos diacrnicos?
17. Cul es la ley de los fenmenos diacrnicos?
18. Es lo mismo una relacin causal que una relacin de
invarianza?
19.. Sostiene S. Mili que la casualidad es siempre aditiva?
20. Por qu el mtodo de la concordancia no proporciona induc
cin completa?
21. Cul es el mtodo que S. Mili privilegia por su grado de fiabi
lidad ?
22. Se asocia la tesis de Laplace sobre probabilidad con los mto
dos deS. M ili?
23. Juega la enumeracin de casos un rol importante en los mto
dos de S. Mili?
24. Cul fue la propuesta de S. Mili sobre la oposicin tradicional
entre induccin y deduccin?

280
LECCIN 15
La induccin clsica

La presente leccin se propone hacer un estudio de los plantea


mientos fundamentales que sobre la induccin formul Stuart Mili
(1806-1873) en su conocido libro Sistema de lgica. Las referencias
que haremos se basan en la versin castellana de Daniel Jorro pu
blicada en Madrid en 1917. La naturaleza misma de nuestra ta
rea nos ha llevado a elegir aquello que consideramos sustantivo
y a desestimar lo que podra ser considerado accesorio o de ca
rcter reiterativo, como, por ejemplo, las diversas y abundantes
ejemplificaciones en las que entra con alguna frecuencia Stuart
Mili para probar la misma tesis.
Consideramos importante aclarar que en la literatura espe
cializada en lgica matemtica es normal que se omita como tema
la induccin clsica. Sin embargo, si se revisa los manuales de
metodologa de la investigacin de carcter operativo y de uso
actual generalizado, se encontrar que ellos se basan en la lgica
Stuart Mili ms que en los resultados especficos de la lgica con
tempornea. Por ejemplo, la lgica subyacente en los llamados
diseos de investigacin, ya clsicos, de Campbell y Stanley, no
es de manera directa un sistema de lgica matemtica sino las
reglas decimonnicas de Stuart Mili sobre los casos de causacin.
Debido a la consideracin anterior, hemos juzgado necesario
incluir este captulo que facilita la comprencin de los manuales
sobre investigacin cientfica y que apertura una forma distinta
de razonar que en nuestro siglo ha sido perfeccionada a travs de
los sistemas de lgica probabilitarios.

[281]
La presente exposicin la hemos dividido en dos partes: la
primera referente a la ubicacin y fundamentacin de la induc
cin dentro de lo que podramos considerar una teora general de
la inferencia; y la segunda versa sobre la induccin entendida
bsicamente como un procedimento de investigacin de los casos
de causacin. Asimismo, hacemos notar que la segunda parte es
la que usualmente se resume en algunos manuales de lgica y
metedologa bajo la denominacin de los mtodos de Mili.
En lo que sigue procedemos a desarrollar el plan antes descrito.

15.1 La induccin como inferencia amplificadora

Comenzaremos puntualizando que Struart Mili define a la lgi


ca como una disciplina cuyo objeto de estudio es la prueba de la
verdad de proposiciones o aserciones, lo que se desprende clara
mente de una lectura del numeral 1 del captulo primero del li
bro II de Sistema de lgica (SL). Asimismo, considera que la ac
cin de probar consiste principalmente en hacer inferencias, pero,
hasta donde hemos podido constatar, no se esmera en proporcio
nar inicialmente una definicin explcita, precisa y rigurosa de in
ferencia sino que prefiere proporcionar un conjunto de nociones
a partir de las cuales el lector puede derivar una caracterizacin
de ella.
Entre tales nociones podemos citar la siguiente: Inferir una
proposicin de otra proposicin previa o de otras; prestarle fe o
exigir que se le preste fe como conclusin de alguna otra, es razo
nar en el sentido ms amplio de la palabra (p. 182). De este texto
se sigue que inferir es la operacin por la que se desprende o se
deriva una conclusin a partir de otra u otras proposiciones, lo
cual, en gran medida, coincide con lo que clsicamente se ha di
cho al respecto.
Sin embargo, la lectura numeral 2 revela que Stuart Mili no
est pensando exactamente igual que los clsicos aristotlicos
porque, en esta parte de su exposicin, de manera sumaria, anali
za las tradicionales inferencias inmediatas sistematizadas en el
cuadro de Boecio y concluye que no son inferencias. La razn es
que se trata de equivalencias, de modos distintos de decir lo mis
mo, esto es, de meras traducciones que como tales slo compor

282
tan una modificacin en los medios expresivos; pero no en el es
tado del conocimiento que de esta manera no sufre incremento
alguno.
El hecho de que Stuart Mili descalifique como inferencias a
operaciones lgicas clsicas, por ser equivalencias, se compren
de, si se considera que sus planeamientos de manera clara exigen
que para que algo pueda ser denominado inferencia posea una
conclusin cuya verdad sea nueva con respecto a lo dicho en las
premisas. Vale decir, toda inferencia genuina debe producir un
proceso de real incremento del conocimiento y en ese sentido debe
ser amplificadora. Consecuentemente, de acuerdo con esta con
cepcin, las operaciones realizadas por la lgica no son meros
medios de explicitacin de lo ya dicho en las premisas sino genui-
nos instrumentos de descubrimiento de verdades que antes no se
conocan.

15.2 El fundamento del silogismo

Stuart Mili, inicialmente, distingue dos tipos de inferencias: las


que operan de lo particular a lo general, llamadas inductivas, y
las que operan de lo general a lo particular, llamadas silogismo o
argumentaciones. Al hacer esta clasificacin se aparta de las pro-
siciones inductivistas tradicionales, como la de Bacon, que consi
deraban al silogismo como un procedimento circular y como una
seudoinferencia. En cambio, para Stuart Mili, por definicin, el
silogismo es una inferencia en sentido estricto porque permite el
paso de lo conocido a lo desconocido.
Abundando en detalles argumentativos, afirma que la pre
misa mayor de un silogismo es una frmula cuya intencin con
densa un conjunto muy grande de inferencias que proceden de lo
particular a lo general, lo que significa que el fundamento de la
premisa mayor es su intencin inductiva. El silogismo resulta as
una inferencia en sentido estricto porque su conclusin no esta
contenida en la proporcin denominada premisa mayor, sino, que
es obtenida conforme a su intencin inductivas que no es lo
mismo.
La implicacin material, tal como lo sostuvo el obispo Whately
y otros filsofos, era de este modo insuficiente para justificar la

283
validez del silogismo, pues Mili rechaza el criterio que dice que
lo que legitima a una inferencia es la imposibilidad de que las
premisas sean verdaderas y la conclusin falsa. El fundamento
del silogismo, es en ltima instancia, la intencin inductiva de la
premisa mayor. En concordancia con este punto de vista, rechaza
la posibilidad de reducir la teora del silogismo a un formalismo
algebraico, lo que en su tiempo, sin embargo, ya haba sido logra
do por los trabajos de Boole y de Venn.
Con relacin a la induccin, Struat Mili es explcito en su
definicin como vemos en la siguiente cita: La induccin por
consiguiente, es la operacin del espritu por la cual inferimos
que lo que sabemos ser verdadero en uno o varios casos particu
lares ser verdadero en todos los casos que se parezcan a los pri
meros bajo ciertas relaciones asignables. En otros trminos, la in
duccin es el procedimiento por el cual concluimos que lo que es
verdadero de ciertos individuos de una clase es verdadero de toda
la clase, o que lo que es verdadero algunas veces, lo ser siempre
en circunstancias semejantes, (p. 292).
En esta definicin se aprecia claramente el carcter ampli-
ficatorio asignado a la induccin, pues la verdad de la conclusin
afecta a ms casos que los incluidos en las premisas. Pero esto no
debe llevamos a pensar en que el sentido de esta inferencia es
establecer una relacin que va del presente al futuro (predictiva),
en la medida que muchos de los casos referidos por la conclusin
puedan coexistir con los descritos por las premisas con la diferen
cia de que en un determinado momento todava no son conoci
dos. Consecuentemente, lo fundamental en todo proceso induc
tivo, como en cualquier inferencia, es proceder de lo conocido a
lo desconocido.

15.3 Las seudoinducciones

En armona con el planteamiento anterior, nuestro autor considera


necesario sealar tres tipos de seudoinducciones debido a que son
tenidas como inferencias genuinas por algunos tratadistas.

(i) El primer tipo de seudoinduccin es lo que hoy da podemos


llamar induccin exhaustiva, caracterizada porque en las

284
premisas se agotan los casos del universo del discurso o lo que
hemos denominado antes dominio de interpretacin y, por con
siguiente, la conclusin no puede agregar ninguno ms, convir
tindose as en una escritura abreviada de las premisas. Evi
dentemente, los procedimientos de este tipo no satisfacen la de
finicin de inferencia adoptada por Mili, pues la conclusin no
excede lo dicho en las premisas.
(ii)El segundo tipo de seudoinducciones lo constituyen las demos
traciones por induccin matemtica. Es oportuno anotar que en
este caso S. Mili da muestras de no conocer las diferentes for
mas que puede adoptar la induccin matemtica, a pesar de
que se trata de un mtodo conocido desde los trabajos de Pascal.
La base de su error consiste en que confunde las leyes de la
matemtica con las leyes de la naturaleza; as por ejemplo, iden
tifica las leyes de la geometra con las leyes del espacio real,
desconociendo que ya en su tiempo estaban desarrolladas las
geometra no-euclidianas de Lobachevski y Riemann, cuya co
rrespondencia con el espacio real no tena ningn significado
matemtico.
(m)El tercer tipo de seudoinduccin est constituido por las llama
das coligaciones que, segn S. Mili, no son propiamente infe
rencias sino solamente descripciones que se reducen a los casos
observados y que, consecuentemente, no dicen nada acerca de
los casos no observados. La coligaciones se limitan a ser des
cripciones de concordancia que se observan en la naturaleza,
como sera el caso de las llamadas leyes de Kepler sobre las
rbitas de los planetas. De acuerdo a Mili, estas leyes enuncian
relaciones constantes o uniformidades observadas; pero no per
miten un conocimiento mayor debido a que no establecen las
causas. En cambio, las inducciones hacen referencia a regulari
dades observadas, pero, tambin, hacen afirmaciones acerca de
hechos nuevos que exceden en contenido a las coligaciones.

Despus de examinar las seudoinducciones, sealaremos que


lo importante, para Mili, no es solamente hacer inducciones ge-
nuinas sino ser capaz de escoger aquellas que son relevantes para
probar algo con respecto a ciertos objetos. Las reglas de la lgica
nos sirven para determinar la validez de las inducciones una vez

285
que stas han sido creadas, sin embargo son de poca ayuda para
crear las inducciones que necesitamos. Esta tesis, con ligeras va
riantes, ha sido defendida en nuestro siglo por el positivismo l
gico y tambin por Popper, entre otros.

15.4 El principio de uniformidad de la naturaleza

El fundamento de la induccin, segn S. Mili, est dado por el


principio de uniformidad de la naturaleza, el mismo que est pre
supuesto en toda inferencia de este tipo. En efecto, la generali
dad inherente a toda conclusin obtenida por mtodos inducti
vos slo es posible si se asume como principio que en circunstan
cia semejantes la naturaleza se comporta de la misma manera. Mili
expresa esta idea en los siguientes trminos. Es preciso ante todo
observar que hay un principio implicado en el enunciado mismo
de lo que es la induccin, un postulado relativo al curso de la na
turaleza y al orden del universo, a saber: que hay en la naturale
za casos paralelos; que lo que sucede una vez suceder tambin
en circunstancias bastantes semejantes, y suceder siempre que
las dichas circunstancias se presenten.
Pero afirmar que el principio de uniformidad de la naturale
za es el fundamento de la induccin no significa en modo alguno,
para Mili, aceptar algunas tesis tradicionales que vean en l la
expresin de una especie de tendencia instintiva a inducir, que
supuestamente era inherente a la naturaleza humana. Sostiene,
que el fundamento de este principio tambin es inductivo en la
medida que lo adquirimos como una especie de generalizacin
suprema de nuestra experiencia. Por tanto no constituye una de
nuestras inducciones ms simples y ms primigenias, sino ms
bien se encuentra en el rango de ser la ms elaborada. Esto expli
ca, que el enunciado de dicho principio prcticamente haya sido
accesible slo al espritu entrenado de los filsofos, que han sido
capaces de derivarlo de otras generalizaciones de menor jerar
qua. En efecto, sera la constatacin de que en los diversos rde
nes de la naturaleza existen leyes cientficas que expresan relacio
nes uniformes y constantes respecto de fenmeno de un cierto
gnero, lo que habra conducido a los filsofos a pensar que las
diversas regularidades conocidas, slo son la manifestacin de

286
una regularidad y unifor-midad universal inherente al compor
tamiento de la naturaleza como un todo. El proceso del razona
miento ha sido inductivo, sostiene S. Mili, en tanto se ha inferido
que en todos los nuevos fenmenos que se conozcan, la naturale
za observar un comportamiento uniforme, consistente en que
condiciones semejantes sern seguidas por efectos semejantes.

15.5 Premisas mayor de un silogism o inductivo


Evidentemente el planteamiento anterior abre inmediatamente
una interrogante crucial, cmo es posible que un principio, que
ha sido obtenido presuponiendo un nmero muy grande de in
ducciones, sirva de fundamento a la induccin? La respuesta de
S. Mili es que el principio de uniformidad de la naturaleza es el
fundamento de la induccin, no en el sentido de que a partir de
l se construyan todas las inducciones, porque esto es claramen
te falso, sino en el sentido de que este principio constituye la jus
tificacin lgica que hace legtima a toda inferencia inductiva. Y
esta justificacin consiste en que l guarda con las inferencias in
ductivas la misma relacin que la premisa mayor con un silogis
mo. Por consiguiente, el mencionado principio es fundamento no
porque sirva para explicar las inducciones sino porque es condi
cin necesaria para la prueba de su validez lgica, puesto que de
acuerdo a reglas clsicamente conocidas no hay conclusin leg
tima si es que no hay una premisa mayor en la cual fundarla.
Este punto de vista, que luego no recibe un tratamiento deta
llado y sistemtico, est presente de manera implcita y a veces
explcita en el desarrollo de la induccin que hace S. Mili espe
cialmente en el libro III. Su importancia radica en que, nuestro
autor, no cay en la tradicin baconiana de oponer deduccin a
induccin pues, como resulta claro de lo dicho anteriormente, la
induccin es lgicamente vlida solamente porque puede ser for
mulada en los trminos de un silogismo vlido cuya premisa
mayor es el principio de uniformidad de la naturaleza. Aunque
Mili no lo afirma con suficiente claridad, su posicin implica lgi
camente que la induccin es un caso particular de silogismo en la
medida que, de acuerdo con sus tesis, toda induccin puede ser
traducida a la forma de un silogismo que tiene como premisa

287
mayor al principio de uniformidad de la naturaleza. Sin embargo
pero no todo silogismo tiene que ser necesariamente inductivo en
el sentido de que puede tener una premisa mayor distinta.

15.6 Leyes de la naturaleza y sistema axiomtico

Consecuentemente, para que la lgica inductiva sea posible de


ben existir previamente premisas mayores que permitan justifi
car las inferencias, y por tanto, es necesario para construir este
tipo de lgica postular la existencia de ciertas inducciones bsi
cas y universales, anlogas al principio de uniformidad de la na
turaleza, que ya no se justifican dentro del sistema de la lgica
inductiva sino que lo hacen posible. Asimismo, si somos ms exac
tos, el mencionado principio debe ser entendido como una plu
ralidad de uniformidades ms que como una sola. Se trata ms
bien de una conjuncin de las uniformidades que se dan separa
damente en los diferentes fenmenos que acaecen. De otra parte,
entre esta multitud de uniformidades puede distinguirse aquellas
que son primitivas e irreductibles y aquellas que resultan como
una combinacin de las premisas y que son, por tanto, reducibles
a sus componentes bsicos. Segn S. Mili, solamente las unifor
midades que son irreductibles pueden llamarse propiamente le
yes de la naturaleza y aquellas que son derivadas, como seran
las de Kepler, son meras concordancias que pueden ser deriva
das de proposiciones ms simples como las leyes de Newton. Con
secuentemente la investigacin de las leyes de la naturaleza con
siste en tratar de responder a la siguiente pregunta: Cul es el
njnero mnimo de proposiciones generales desde las cuales po
dran ser inferidas deductivamente todas las uniformidades de la
naturaleza?.
Como puede comprenderse, esta manera de plantear la in
vestigacin de las leyes de la naturaleza coincide con el proceso
de construccin de un sistema hipottico-deductivo de proposi
ciones, conocido tradicionalmente como sistema axiomtico, en
el que a partir de un nmero mnimo de axiomas se derivan las
otras como teoremas. Este modelo de investigacin cientfica, que
tienen su origen en los Elementos de Euclides, fue tradicionalmen
te usado para ejemplificar un mtodo deductivo que se entenda

288
como opuesto al inductivo. Sin embargo, en S. Mili encuentra una
especie de conciliacin con los planteamientos inductivistas, pues
aunque el desarrollo del sistema es deductivo y consiste en silo
gizar, la naturaleza de los axiomas es la de inducciones bsicas e
irreductibles que constituyen las uniformidades primarias de la
naturaleza. En el logro de esta comprensin no-baconiana del
proceso de desarrollo y organizacin de la ciencia, indudablemente
influy la posibilidad que tuvo S. Mili de conocer la versin
axiomtica de la mecnica debida a Newton. Esta presuncin re
sulta completamente plausible cuando se constata que a lo largo
de la obra estudiada abundan los ejemplos tomados de los Prin
cipios de matemtica aplicados a la filosofa natural.
Asimismo, aunque podra ofrecerse abundantes argumento
para apoyar la interpretacin de que Mili consider al modelo
hipottico-deductivo como el paradigma hacia el cual debera ten
der la organizacin de la ciencia, consideramos que ser suficien
te aportar la siguiente cita, cuya claridad y comprensin nos exi
me de hacer mayores comentarios: As no hay que desesperar
de elevar la Qumica y la Fisiologa al rango de las ciencias deduc
tivas, pues aunque sea imposible deducir todas las verdades qu
micas y fisiolgicas de las leyes o principios de las sustancias
simples o agentes elementarlos, podran ser deducidas de las le
yes que aparecen cuando estos elementos se renen en un peque
o nmero de combinaciones no muy complejas.

15.7 La ley de la causalidad

Mili divide en general los fenmenos, en sincrnicos y diacrni-


cos. Las leyes de los fenmenos sincrnicos son las referentes al
nmero y a la figura, esto es, son las leyes de la aritmtica y de la
geometra que, como dijimos antes, l les otorg el mismo esta
tuto que a las dems leyes de la naturaleza. Al hacer esta tipifi
cacin, sin embargo, no pudo dejar de notar que las leyes de la
aritmtica pueden ser intituivamente entendidas con referencia
a la sucesin de los nmeros naturales y, consecuentemente, pue
den fcilmente, por este hecho, ser caficadas de leyes de fen
menos diacrnicos. Debido a ello, concedi que las leyes del n
mero se cumplen tambin en los fenmenos sincrnicos adems

289
de en los diacrnicos, mientras que las de la geometra, que es
tn referidas a las propiedades del espacio, se limitaran a los fe
nmenos sincrnicos. Pero como la verdad de las leyes de lo sin
crnico puede ser establecida sin necesidad de presuponer el
transcurso de un intervalo de tiempo, las leyes de ambas ciencias
pueden ser entendidas como intemporales y como establecidas
universalmente de una sola vez para siempre.
De manera anloga, los fenmenos diacrnicos tienen a su
vez una ley fundamental que es privativa de ellos porque su ver
dad presupone necesariamente la nocin de sucesin temporal.
Esta ley, que es una de las pocas que gozan del mismo grado de
universalidad que las verdades matemticas, es la de causalidad.
S. Mili expresa esta idea de la manera siguiente: Esta ley, sin
embargo, es universal tambin en otro sentido; es coextensiva con
el campo entero de los fenmenos, pues todos los hechos de suce
sin son ejemplos de ella. Esta ley es la ley de causalidad. Es ver
dad de que todo lo que comienza a ser tiene una causa que es
coextensiva a toda la experiencia humana.
La cita anterior, adems de revelarnos la universalidad que
nuestro autor atribuy a la causalidad, nos proporciona una enun
ciacin muy general de dicho principio, al cual dedicaremos al
gunas lneas ms. Este proceder se justifica porque, segn el pen
samiento examinado, la nocin de causa es la raz de toda la
teora de la induccin (p.325). En consecuencia, una compren
sin correcta de la lgica inductiva clsica requiere que precise
mos cuatro aspectos de la nocin de casualidad a los que S. Mili
otorga especial atencin.

15.8 Aspectos de la causalidad

(i) En principio, para S. Mili, una causa no es simplemente en evento


sino una conjuncin de condiciones que producen un efecto o
que son seguidas por un efecto, de tal suerte que cada que tal
conjuncin de condiciones se repite, se seguir necesariamente
el mismo efecto.
(n) La referida conjuncin de condiciones, sin embargo, no tiene
que ser hecha explcita completamente siempre, pues en la prc
tica resulta suficiente enumerar slo las condiciones ms rele

290
vantes y muchas pueden ser omitidas en la formulacin sin
mayor dificultad.
Simultneamente, con la conjuncin de condiciones que son se
guidas de un cierto efecto, pueden darse algunas condiciones
contrarias. Para incluir este caso, la causa en sentido general es
la suma algebraica de las condiciones positivas y negativas el
total de las contingencias de toda naturaleza, que siendo rea
lizadas, hacen que siga el consiguiente de toda necesidad
(p. 332).
(m) Aunque una relacin de causalidad establece una relacin de
invarianza o uniformidad, la causalidad es mucho ms que esta
ltima, es adems una relacin incondicionada. Un ejemplo de
relacin invariante es la sucesin del da y la noche que se pro
duce uniformemente en el tiempo, sin embargo, no es una rela
cin de causalidad, porque todos sabemos que el da no es la
causa de la noche ni viceversa. Ms bien ambos fenmenos tie
nen una causa comn en el movimiento de la tierra alrededor
del sol. La relacin de causalidad segn S. Mili, es incondi
cionada en el sentido de que la relacin entre la causa y el efecto
es necesaria en todas las circunstancias imaginables y, por tan
to, para explicar el efecto siempre ser suficiente recurrir a su
antecedente y no a otro elemento ajeno a la sucesin, lo que es
necesario en el caso de la secuencia establecida por el da y la
noche. Consecuentemente, mientras toda relacin de causalidad
es una relacin de uniformidad no toda relacin de uniformi
dad o de invarianza es una relacin de causalidad, con lo que se
estatuye que la relacin de causalidad es un caso particular muy
importante, dentro del conjunto de las relaciones de invarianza.
A partir de la distincin anterior, S. Mili deduce los casos en los
que un mtodo establece una induccin completa y los casos en
los que esto no es posible. Un mtodo conduce al logro de una
induccin completa cuando permite la determinacin de una
relacin incondicionada. Si slo permite la determinacin de
una relacin de invarianza o concordancia, entonces no aporta
una induccin completa en la medida que no nos permite aso
ciar inequivocadamente una causa con su efecto y viceversa.
(iv) Segn S. Mili la relacin de causalidad es, en general, aditiva
en el sentido de que el efecto total de un conjunto de causas es

291
igual a la suma de los efectos que produciran stas por separa
do. Esta relacin es anloga a la de composicin de fuerzas en
la mecnica, en la cual la resultante de la aplicacin de un con
junto de fuerzas es igual a la suma de los efectos que stas pro
duciran por separado (la suma es aqu obviamente entendida
en sentido algebraico puestos que unas causas pueden actuar
en sentido distinto o contrario a las otras, tal como ocurre con
las fuerzas). Excepcionalmente, segn S. Mili la causalidad no
es aditiva, en el sentido de que el efecto total de un conjunto de
causas no es igual a la suma de los efectos que estas produci
ran por separado. En este caso, el todo es algo ms que la suma
de las partes y tiene leyes que difieren de las que gobiernan las
partes tomadas aisladamente. Es el caso de la Qumica y de la
Fisiologa que por esta razn estn constituidas por leyes que S.
Mili llama heteropticas. En la imposibilidad de reducir las le
yes de la qumica a las de sus componentes ms simples, radi
cara el origen de la dificultad para lograr una presentacin
axiomtica de esta disciplina.

15.9 Los mtodos de Stuart Mili

Realizadas las delimitaciones anteriores sobre la nocin de cau


salidad, en lo que sigue pasaremos a exponer los mtodos de S.
Mili cuyo objetivo es determinar cules son las leyes de causa
cin existentes en la naturaleza, determinar los efectos de cada
causa y las causas de todos los efectos (p. 355).
Para exponer los mtodos ser conveniente que previamente
establezcamos algunas convenciones de notacin. Las causas las
denotaremos con las letras maysculas A, B, C, D,...etc. y sus efec
tos respectivos con las correspondientes minsculas a, b, c, ...etc.
Usaremos una flecha que en este trabajo es slo una manera abre
viada de decir desde...se sigue... , lo cual no debe confundirse ni
con la nocin de implicacin lgica ni con el nexo casual. Cuando
la flecha debe expresar adems de desde...se sigue... un nexo cau
sal, lo indicaremos al costado entre parntesis.

292
15.9.1 El mtodo de la concordancia

El enunciado del mtodo de la concordancia propuesta por S. Mili


es el siguiente:
Si dos o ms fenmenos objeto de la investigacin tienen sola
mente una circunstancia comn, la circunstancia en la cual todos
los casos concuerdan es la causa (o el efecto) del fenmeno
(p.369).

El enunciado anterior nos conduce a una inferencia de la si


guiente forma:

A, B , C , -------------------------------------------------- ^ a , b, c.
A, D, E , ---------------------------------------------------" ^ a , d, e.
Luego A ^ a

La inferencia anterior nos indica que siempre que sucede A


tambin ocurre a, pero para ser vlida requiere que entre los con
juntos de circunstancias ABC y ADE deba haber solamente una
circunstancia comn porque si adems de A hubiera otra implci
ta, entonces cualquiera de las dos podra ser la causa. De haber
ms de una circunstancia en comn, A seguira siendo un antece
dente de c, como lo es el da de la noche, pero no necesariamente
la causa.
El mtodo de la concordancia no proporciona induccin com
pleta, en la medida de, que no existe ningn procedimiento segu
ro para decidir si dos conjuntos de circunstancias tienen solamente
un elemento en comn. Asimismo, al no permitirnos identificar
de manera unvoca la causa no es aparente para la experimenta
cin, pues ella exige que seamos capaces de producir un efecto lo
que no es posible sin el conocimiento unvoco de su causa. Conse
cuentemente, este mtodo slo permite el establecimiento de re
laciones de invarianza, lo que reconoce claramente S. Mili en el
siguiente texto: El mtodo de concordancia no conduce ms que
a las leyes de los fenmenos (como se les llama a veces, pero
impropiamente, pues que las leyes de causalidad son tambin le
yes de los fenmenos), es decir, a uniformidades que, o no son
leyes de causacin, o respecto de las cuales la cuestin de la
causalidad puede por el momento quedar (p. 373).

293
Sin embargo, es importante sealar que nuestro autor consi
dera que en ciertas condiciones que exigen reiteradas variaciones
de las circunstancias y el uso de mtodos adicionales, la concor
dancia puede permitirnos establecer relaciones de causalidad.
Aqu la cuestin radica en el nmero de variaciones de las cir
cunstancias necesitado y el nmero de concordancia que es signi
ficativo para pensar que un antecedente es adems una causa.
Esta problemtica puede ser respondida por el clculo de las pro
babilidades a travs de la nocin de frecuencia relativa. Para S.
Mili el gran auxilio en este caso es el desarrollo realizado por
Laplace en su Ensayo filosfico sobre la probabilidad, regla sexta, que
aparece ampliamente comentada en el Cap. XVIII, numeral 5, del
libro III de Sistema de lgica. Lo que aqu ha expuesto S. Mili no
es otra cosa que los fundamentos del conocido teorema de Bayes
para averiguar la probabilidad de una causa. El razonamiento que
justifica este recurso segn nuestro autor es que es mejor recurrir
a la probabilidad para determinar una causa que pensar que cier
tos acontecimientos se producen por una especie de coincidencia
maravillosa que supera en mucho las leyes del azar. Consecuen
temente, cuando se usa el mtodo de la concordancia variando
muchas veces las circunstancias y se encuentra una uniformidad
que excede mucho las reglas del azar, tal como fueron definidas
por Laplace, entonces hay que pensar que esta uniformidad o
invarianza es adems una relacin de causalidad.

15.92 Mtodo de la diferencia

La enunciacin que Mili da del mtodo de la diferencia es la si


guiente: Si un caso en el cual el fenmeno se presenta y un caso
en el que no se presenta tienen todas las circunstancias comunes,
fuera de una sola, presentndose sta solamente en el primer caso,
la circunstancia nica en la cual difieren los dos casos en el efecto
o la causa, o parte indispensable de la causa del fenmeno (p. 370).
Esta regla conduce a una inferencia del siguiente tipo:

A, B , C,
A, C,
Luego "K
causalmente a)

294
Esta inferencia segn Mili nos proporciona induccin com
pleta porque el hecho de que en dos circunstancias semejantes la
presencia de A da lugar al efecto a y la ausencia de A conlleve la
desaparicin del mencionado efecto, nos autoriza a concluir que
la causa del efecto a es la circunstancia A. Este resultado, asimis
mo, nos pone en condiciones de producir y suprimir el efecto a en
la medida que 01 icemos su causa, la razn por la que el mtodo
de la diferencia es considerado como el adecuado a la investiga
cin experimental mientras que el de la concordancia es restringi
do a los casos en lo que es factible slo la observacin.
S. Mili considera que este mtodo es conclusivo porque es
posible decidir con seguridad cuando dos situaciones se diferen
cian solamente en una condicin sin importar que tengan ms
puntos de coincidencia de los que ya tenemos enumerados. Des
de su punto de vista es ms fcil controlar las diferencias que las
semejanzas, pues nosotros podemos introducirlas experimental
mente en forma tal que, por ejemplo, el estado de un perro en el
momento M es el mismo que en el momento M' con la diferencia
nica de que en M' hemos puesto un trozo de carne ante su vista.
De esta manera, si se produce salivacin ella debe ser causada
por el factor introducido y esto se ratifica cuando comprobamos
que despus que retiramos la carne la salivacin cesa.
Actualmente se sabe que es ms fcil controlar las diferen
cias manteniendo constantes ciertas condiciones, sin embargo,
tampoco hay ninguna regla que nos asegure completamente que
la nica diferencia entre dos estados es la que nosotros hemos
introducido, pues bien podra haberse producido una sin que nos
percatemos de ella. Consecuentemente, aqu la inferencia tam
bin es de probabilidad, aunque nuestro autor no haya recomen
dado el clculo de las probabilidades en este caso. Sin embargo,
es evidente que este tipo de inferencia, cuando es posible, es el
ms compatible con los diseos experimentales que buscan un
control ptimo de las variables consideradas en una investiga
cin. Los metodlogos contemporneos han prestado especial
atencin a esta regla y a la circunstancia diferencial A la denomi
nan variable experimental.

295
15.9.3 Mtodo indirecto de la diferencia

El enunciado de este mtodo es el siguiente: Si dos casos o ms


en los cuales se efectan el fenmeno tienen una sola circunstan
cia en comn, mientras que dos casos o ms en los cuales no se
efectan no tienen ms en comn que la ausencia de esta circuns
tancia, la circunstancia por la cual nicamente difieren los dos gru
pos de casos es el efecto, o la causa, o una parte necesaria de la
causa del fenmeno (p. 376).
Este mtodo es una combinacin de los anteriores, se trata de
dos grupos de concordancias que pueden ser distinguidos por
que tienen diferente elemento comn. En el primer grupo el ele
mento comn es una circunstancia A y el segundo el elemento
comn es la circunstancia no-A, esto es la ausencia de la circuns
tancia anterior. Vale decir, al interior de los dos pares de grupos
hay concordancia y al exterior de ellos diferencia por un slo fac
tor. La forma de la inferencia es como sigue:

A, B , C , D,

A ,F ,G ,H ,

no - A , B , C , D

n o -A ,F ,G ,H

A a

Segn Mili este mtodo debera proporcionar induccin


completa pues los dos grupos al presentar slo una variante per
mitira]? aplicar el mtodo de la diferencia que es conclusivo.
Sin embargo, su debilidad estara en que cada grupo tomado
aisladamente es un caso de concordancia y este no es un mtodo
conclusivo.

15.9.4 Mtodo del residuo

El enunciado de este mtodo es el siguiente: Separad de un fe


nmeno la parte que se sabe, por inducciones anteriores, ser el
efecto de ciertos antecedentes restantes (p.379).

296
Esta regla nos conduce a una inferencia de la forma:

A, B, C, ----------------------------^ a, b, c,
Por inducciones anteriores se sabe que:

A ------------------------^ a
B ---------------------^ b
Luego, C --------------------------- c

Como es claro, esta es una variante del mtodo de la diferen


cia. Lo distintivo de este caso es que recurre a inducciones ante
riores como premisas, mientras que en todos los casos anteriores
se supone que las premisas son registros inmediatos de observa
ciones. Obviamente, la confiabilidad de este mtodo depende de
la confiabilidad con que hayan sido obtenidas sus premisas. Si
ellas han sido obtenidas por el mtodo de la diferencia y se est
seguro de que C es el nico antecedente de c, entonces segn Mili,
el mtodo es conclusivo. Sin embargo, l mismo reconoce que es
muy difcil llegar a estar seguro de que C es el nico antecedente
de c por lo que no afirma que en general el mtodo del residuo
aporte una induccin completa.
A este mtodo frecuentemente se le ha llamado deductivo
por su forma, pues presenta la estructura de una premisa mayor
general y premisas particulares de las que se concluye tambin
una proposicin particular. Esto, no afecta la posicin de Mili,
porque l no hace inconciliables induccin y deduccin y, por otro
lado, el hecho de que algo se concluya para un caso, segn S. Mili,
no significa que la conclusin necesariamente es particular, pues
si la inferencia ha sido correcta, lo que es verdad de un caso lo es
tambin de todos los de su gnero. Segn S. Mili la enumeracin
no siempre es necesaria en la induccin ni lo ms importante, pues
lo fundamental es como se eligen los casos. A veces puede ser
suficiente un caso, como lo indica en el siguiente texto: Cuando
un qumico anuncia la existencia de una sustancia nuevamente
descubierta y de sus propiedades, si tenemos confianza en su exac
titud estamos seguros de que sus conclusiones deben valer uni

297
versalmente, aunque su induccin no se funde ms que en un
solo hecho (p.313).

15.9.5 M todo de las variaciones concomitantes

El enunciado de este mtodo es el siguiente: Un fenmeno que


vara de cierta manera, siempre que otro fenmeno vara de la
misma manera, es, o una causa o un efecto de este fenmeno, o
est ligado a l por algn hecho de causacin (p. 382). Para re
presentar la inferencia a que da lugar esta regla, introduciremos
la notacin v (A) que es una abreviacin de la expresin varia
cin de A....

A ,B 9C ---------------------------^ a , b, c
v ( A ), B, C ----------------------- ^ v ( a ) , b , c
Luego~A V a ( De A se sigue
^ causalmente a ).

En la interpretacin que hemos hecho de la regla de Mili he


mos presupuesto, legtimamente, a nuestro juicio, que la intro
duccin de una variacin en una condicin se hace manteniendo
constantes todas las dems condiciones para poder sentirnos au
torizados a atribuir la variacin en el efecto a este hecho y no a
otro. SegrtS. Mili, la proposicin que sirve de base a este mtodo
es la siguiente: Una cosa cuyas modificaciones tienen siempre
por consecuentes las modificaciones de un efecto debe ser la cau
sa (o debe estar ligada a la causa) de este efecto (p. 384).
Nuestro esquema lo que expresa es que si A es un anteceden
te de a y una variacin de A es seguida por una variacin de a,
entonces A es la causa de a. La nica limitacin que seala nues
tro autor a esta inferencia es la mayor o menor posibilidad de
observar las variaciones; siempre que tal observacin pueda ha
cerse, la inferencia es conclusiva, esto es, da induccin completa.
Como puede apreciarse, es te mtodo para llevarse a la prc
tica necesita algn tipo de medida o de cuantificacin de las va
riaciones lo que propicia la matematizacin de la inferencia. Lo
que propiamente se establece aqu, es una relacin de funcio
nalidad que bien puede ser directa o inversa. Ser directa si a

298
mayores valores de A corresponden mayores valores de a y a
menores valores de A corresponden menores valores de a. Ser
inversa si a mayores valores de A corresponden menores valores
de a y a menos valores de A corresponden mayores valores de a.
En la investigacin en ciencias humanas de nuestros das se usa
frecuentemente un esquema parecido a ste que es expresado a
travs del mtodo estadstico de la correlacin, solamente que a
esta no se la considera siempre reveladora de causalidad, salvo
cuando relacin entre A y a es claramente asimtrica y tiene sen
tido, adems, afirmar que A produce o genera a.

15.10 La deduccin

Stuart Mili declara que los cuatro mtodos anteriores son los ni
cos mtodos inductivos que l conoce y puede imaginar. Cierta
mente no est contando al tercero que es una combinacin de los
dos primeros. Todo lo que queda fuera de ellos es la deduccin
de la cual ya dimos una idea en la parte introductoria. Sin em
bargo, puede encontrarse otra caracterizacin de la deduccin, un
tanto ms general, que la define como un mtodo que est cons
tituido por tres partes: la induccin, el razonamiento y la verifi
cacin. A la deduccin as definida se le asigna un rol preponde
rante en la ciencia. En sus palabras: Al mtodo deductivo as de
finido en sus tres partes constituyentes: la induccin, el razona
miento y la verificacin debe el espritu humano sus ms brillan
tes triunfos en la investigacin de la Naturaleza. Le debemos to
das las teoras que renen fenmenos numerosos y complicados
bajo algunas leyes simples, que, consideradas como leyes de es
tos fenmenos, no habran podido nunca ser descubiertas por el
estudio directo (p. 439).
Hemos credo pertinente citar el texto anterior, para indicar
cmo para S. Mili, con toda claridad, la induccin slo tena sen
tido dentro del marco de la deduccin, aunque los axiomas de los
sistemas deductivos sean obtenidos inductivamente, razn por la
que la induccin es la primera fase de la deduccin. De esta ma
nera, la lgica es entendida como una teora de la prueba a travs
de inferencias deductivas, las mismas, que cuando tienen como

299
premisa mayor una proposicin sobre uniformidades de la natu
raleza, se llaman inducciones.
Lo anterior corrobora fuertemente la afirmacin que hicimos
anteriormente en el sentido de que S. Mili constituye un antece
dente muy claro de la tesis contempornea que concibe a las teo
ras cientficas como sistemas hipotticos-deductivos de enun
ciados contrastables. En efecto, inscribir la induccin dentro de
un proceso deductivo ms complejo, como un momento de l, no
es otra cosa que asignarle en la prctica cientfica la fase constitui
da por la postulacin de hiptesis o conjeturas de las que hay que
obtener consecuencias observables (proceso de razonamiento)
para luego someterlas a contrastacin (verificacin).
Es ciertamente un mrito de S. Mili el haber establecido las
bases del llamado modelo hipottico-deductivo, el mismo que es
uno de los ejes sobre los que ha girado la discusin epistemolgica
en los ltimos sesenta aos. Evidentemente, el modelo hipottico
deductivo como mtodo de trabajo cientfico existe desde la po
ca de los Elementos de Euclides, en la forma de mtodo axiomti
co, pero su conceptualizacin filosfica y su uso como criterio de
cientificidad son logros recientes.
Es la conjuncin, del aporte de S. Mili y del desarrollo fron
doso de la lgica matemtica en nuestro siglo, lo que ha posibili
tado que la tradicional oposicin entre induccin y deduccin sea
resuelta en trminos de la teora de la probabilidad, recibiendo a
su vez un tratamiento hipottico deductivo que circunscribe la
discusin a la naturaleza de las probabilidades bsicas.

15.11 Resultados de la induccin clsica


1. La tesis de S. Mil que caracteriza la inferencia, en sentido estric
to, como la operacin lgica en la que el contenido cognoscitivo
de la conclusin excede al de las premisas remite la decisin, en
ltimas instancia, a la medida de la cantidad de informacin
que portan las premisas y la conclusin. Este recurso, aun con
siderando los aportes de la moderna teora de la informacin,
tiene grandes dificultades para lograr resultados precisos y la
mayor de ellas la nica definicin disponible de cantidad de
informacin es la de Shannon, pero no es aplicable a esta discu-

300
sin la pretensin parece ser que las reglas de deduccin en un
sistema lgico dado slo deban permitir el paso lgicamente
necesario desde un conjunto de frmulas P (consideradas como
premisas o axiomas) a un conjunto de frmulas C (consecuen
cias); pero no el proceso recproco. Sin embargo, esta propie
dad no parece autorizar de manera precisa ninguna estimacin
fundada sobre la cantidad de informacin de los referidos con
juntos de frmulas. Lo que s queda a salvo es la idea del desa
rrollo deductivo y, por tanto, de la superacin al menos de la
circularidad formal.
2. La tesis de nuestro autor, que afirma que una induccin slo es
fundada y vlida cuando puede ser formulada como un silogis
mo en el que la premisa mayor es el principio de uniformidad
de la naturaleza, privilegia definitivamente al silogismo como
modelo de inferencia vlida y es claramente incorrecta, Esto ex
cluira del tratamiento lgico de la ciencia a todas las propieda
des expresables como predicados de grado 2 o mayores que 2.
Afortunadamente, en este aspecto y en otros conexos, la evolu
cin de los hechos ha sido muy distinta a la que S. Mili previo.
Asimismo, esta tesis presupone que existen premisas mayores
o inducciones bsicas que propiamente no perteneceran al sis
tema de proposiciones inductivamente probadas sino que lo
hacen posible. La justificacin de stas, tambin llamadas uni
formidades bsicas, sigue siendo problemtica.
3. El planteamiento directo de la lgica inductiva como un conjun
to de mtodos para investigar los casos de causacin es proba
blemente uno de los aspectos ms rescatables del pensamiento
de Mili y que de hecho ha tenido una acusada influencia entre
los metodlogos de nuestro siglo. Sin embargo, dentro de la filo
sofa epistemolgica contempornea este aporte incidira direc
tamente en lo que se ha denominado contexto del descubrimien
to y gravitara menos sobre el llamado contexto de la justifica
cin. Asimismo, como en gran medida la discusin filosfica hasta
aproximadamente 1960 ha sido sobre el contexto de la justifica
cin, la atencin que en los crculos epistemolgicos han mereci
do los mtodos de S. Mili no ha sido de mayor significacin.
4. La relacin de causalidad es para S. Mili en general aditiva, pero
las excepciones que el seala para la Qumica, la Fisiologa as

301
como las ciencias humanas, que estaran constituidas por leyes
heteropticas, esto es, no aditivas, tienen especial significacin
porque tal anlisis es una interesante aproximacin a las difi
cultades, actuales para usar el modelo hipottico deductivo,
particularmente, en ciencias humanas, pues, en otras reas, como
la Qumica, la dificultad ha sido superada.
5. A pesar de que S. Mili estuvo dedicado al cultivo de la lgica, se
aprecia en su pensamiento grandes dificultades para reconocer
la peculiar naturaleza de la lgica y de la matemtica respecto de
las ciencias naturales. Asimismo parece que desconoci o no con
cedi la debida importancia a aportes de su poca como los de
Boole, Venn y las geometras no-euclidianas. Posiblemente ese
desconocimiento lo llevo a privilegiar al silogismo otorgndole
posibilidades que no tena o negndole otras que s tena, como
su expresin algebraica. Lo que s es rescatable, en este aspecto,
es su concepcin de la lgica como una disciplina dedicada al
estudio de la prueba de la verdad de las proposiciones.
6. Tal vez el aporte mayor de Sistema de Lgica es que echa las bases
para la construccin del modelo hipottico-deductivo como ca
tegora central para el anlisis de la ciencia. Esto permiti su
perar el tradicional antagonismo entre induccin y deduccin
dentro de un marco ms general e integrador que el de la teora
de la probabilidad. Tambin ha permitido resolver las insuficien
cias de la induccin enumerativa, en cualquiera de sus formas,
introduciendo el criterio de relevancia en la investigacin de los
casos de causacin. Podra afirmarse que el aporte del sector
neopositivista de la filosofa epistemolgica contempornea, con
algunas excepciones importantes, ha consistido bsicamente en
na profundizacin de la ptica hipottico-deductiva de S. Mili
a travs del uso de aparatos formales lgico matemticos y de
categoras e instrumentos de carcter semntico.

302
XVI
RESEA HISTRICA DE LA LGICA

Objetivos:
Comprender la evolucin histrica de los conceptos lgicos.
Adquirir informacin bsica sobre las etapas histricas princi
pales del desarrollo de la lgica.
Identificar a los principales investigadores en lgica a travs de
la historia.

303
CUESTIONARIO 16
Resea histrica de la lgica

1. Qu importancia histrica tiene el Organon y quin fue su


autor?
2. Cmo se denomina el tipo especial de deduccin a cuyo estu
dio dedic Aristteles sus mayores esfuerzos?
3. Aristteles estudi las deducciones desde el punto de vista de
su estructura o desde el punto de vista desde su contenido?
4. Por qu para Aristteles la lgica es una ciencia formal?
5. Existieron en la antigua Grecia, adems de Aristteles, otros
filsofos interesados en la lgica?
6. Cul es el primer sistema axiomtico de la historia?
7. Cul fue el aporte de Guillermo de Occam?
8. Por qu Leibniz es el primer precursor genuino de la Lgica
Matemtica?
9. Por qu la Lgica Matemtica creada por G. Boole es un len
guaje muy distinto al de Aristteles?
10. Por qu Leibniz quera convertir la lgica en un calculo?
11. Estuvo en lo correcto el filsofo Kant cuando consider a la
lgica un conocimiento cerrado y agotado por Aristteles?
12. Cal ha sido el aporte de Gottlob Frege?
13. Cul es el hito que marca el inicio de la Lgica Matemtica
contempornea ?
14. Cules fueron los hechos que crearon la necesidad de que
Hilbert inventara la Metamatemtica?
15. Cal ha sido el aporte de Kurt Gdel?
16. Cul fue la contribucin de Claudio Shannon?

[305]
17. Qu estudios han contribuido al surgimiento de la disciplina
de la Epistemologa?
18. Conoces los nombres de dos lgicos soviticos?
19. Conoces los nombres de dos lgicos latinoamericanos?
20. ; Qu diferencias existen entre la Lgica-Matemtica y las otras
"lgicas"?

306
LECCIN 16
Resea histrica de la lgica

16.1 Aristteles y los orgenes de la lgica

La disciplina cientfica conocida como lgica, en sentido ms pro


pio, se denomina Lgica Matemtica debido a que una de sus
principales caractersticas, a partir del siglo pasado, ha sido la
incorporacin a su campo de mtodos y smbolos algebraicos. El
desarrollo desbordante de esta disciplina durante el ltimo si
glo ha dado lugar a que influya decisivamente en la ciencia con
tempornea, tanto en sus proyecciones tericas como tecnolgi
cas. As, por ejemplo, puede afirmarse que la actual revolucin
electrnica debe su dinamismo y eficacia a las contribuciones del
lgebra de Boole, a las creaciones de Church y Turing y a la teo
ra lgica de circuitos elctricos de Claudio Shannon, entre otros
aportes.
Los orgenes de la lgica cientfica^se remontan al filsofo
griego Aristteles (384-322 a. C) quien en su trabajo conocido
como el Organo?i desarroll el primer estudio sistemtico de la
deduccin en la seccin denominada Primeros Analticos.
Aristteles examin en particular un tipo especial de deduccin:
el silogismo. Un ejemplo tpico de l nos lo proporciona el razo
namiento: Si todos los cuadrados son rombos y todos los rombos
son p aralelogram os, en to n ces todos los cuadrados son
paralelogramos.
El acierto de Aristteles radic principalmente en estudiar
estas deducciones considerando slo su forma o estructura con

[307]
independencia de su significado o contenido. De esta manera un
razonamiento como: Si todos los peruanos son americanos y to
dos los americanos son occidentales, entonces todos los perua
nos son occidentales es, desde el punto de vista lgico, igual al
anterior porque tienen exactamente la misma estructura o forma.
Desde el punto de vista de su significado, el primero habla de
figuras geomtricas y el segundo de seres humanos pero si se exa
mina las relaciones que existen entre sus trminos, se encontrar
que en ambos casos son las mismas. Los dos ejemplos correspon
den al esquema Si todo A es B y todo B es C, luego todo A es C.
Lo dicho anteriormente nos sirve para hacer comprensible
que la notable contribucin aristotlica fue desarrollar una teora
sobre la validez de los razonamientos o deducciones que no ten
gan en cuenta el contenido de los mismos, sino su forma o estruc
tura. Esta es la razn por la qu la lgica desde su creacin es una
ciencia formal o estructural y este carcter lo mantiene hasta nues
tros das despus de veinticuatro siglos. Asimismo, el tratamien
to estructural que hizo el estagirita (as se le llama a Aristteles
por haber nacido en Estagira) de la deduccin, le posibilit otro
aporte sustancial al desarrollo de la lgica y de la matemtica: el
mtodo axiomtico. Debido a que todos los razonamientos po
dan ser considerados como estructuras, Aristteles axomatiz
su teora del silogismo, seleccionado como puntos de partida cua
tro estructuras bsicas, a las que llam axiomas, y luego constru
y todas las dems como derivaciones de las bsicas. De esta
manera la teora del silogismo constituye el primer sistema axio-
mtico-de la historia de la ciencia.
Casi contemporneos con Aristteles fueron los lgicos es
toicos, quienes tuvieron el mrito de profundizar en algunos cam
pos a los que el autor del Organon no les haba concedido sufi
ciente atencin. Estos filsofos son los precursores ms lejanos de
la actual lgica proposicional y de las teoras que incluyen predi
cados relacinales que son indispensables para dotar a la mate
mtica de una lgica adecuada que el silogismo no proporciona.
Tambin los lgicos conocidos como megricos hicieron en po
cas, cercanas a Aristteles, aportes ingeniosos a la llamada lgica
modal. El ms importante de ellos Diodoro Cronos, se dedic a la
lgica de las modalidades temporales esclareciendo relaciones

308
importantes entre verdad y tiempo. Sin embargo, el influjo de
Aristteles fue avasallador y los estoicos y megricos fueron des
conocidos en la Edad Media durante la cual las investigaciones
lgicas se centraron en el silogismo y sus aplicaciones. Esta tem
tica acapar las preocupaciones de Boecio, Toms de Aquino,
Pedro Hispano y Juan Buridano. Escaparon a ella Abelardo, Lulio
y Occam que visualizaron otros horizontes, especialmente este
ltimo que trabaj apreciablemente la lgica proposicional y co
noci sus principales reglas de inferencia, a pesar de no manejar
un lenguaje simblico adecuado, lo que hizo muy difcil su tarea.
Por aadidura, su conocida concepcin nominalista de los uni
versales, que interpreta a los conceptos como nombres genricos,
es muy prxima a la nocin contempornea de predicado lgico.

16.2 Los precursores de la Lgica Matemtica


Los especialistas consideran al filsofo alemn Leibniz (1646-1716)
como el primer genuino precursor de la Lgica Matemtica, aun
que reconocen que esta idea ya estaba en germen en la obra Ars
Magna del espaol medieval Raimundo Lulio. Leibniz fue el pri
mero que sostuvo con claridad que el mtodo para convertir la teo
ra de la deduccin lgica en una ciencia estricta e infalible era con
vertirla en un clculo mediante la utilizacin de procedimientos
matemticos. Esta nueva ciencia sera una mathesis universalis
cuya funcin consistira en demostrar la verdad de las afirmacio
nes filosficas y cientficas sin tener en cuenta su significado sino
solamente su estructura expresada en smbolos de un lenguaje ar
tificial, construido especialmente para calcular. Leibniz deca que
calcular era operar con smbolos y, consecuentemente, as como se
poda calcular con smbolos aritmticos tambin ello era factible con
smbolos que representen estructuras deductivas. El ideal leibni-
ziano era lograr un instrumento lgico lo suficientemente podero
so como para traducir cualquier discusin significativa sobre la cp-
rreccin de las deducciones a una operacin en la que los oponen
tes se limiten a revisar los clculos para ubicar el error, de manera
parecida a como se corrige una suma cualquiera.
El proyecto de Leibniz era demasiado ambicioso y por ello
fracas. Aunque su intuicin fue grande, estuvo lejos de lo posi-

309
ble y de la construccin de un lenguaje simblico que supere
significativamente la vieja silogstica aristotlica. Fue la inexis
tencia de un lenguaje lgico-matemtico adecuado hasta media
dos del siglo XIX lo que llev al filsofo Kant (1724-1804), a pesar
de su genialidad, a afirmar errneamente que la lgica creada
por Aristteles era un conocimiento acabado, cerrado y comple
to, puesto que la investigacin post-aristtelica no haba ni refu
tado ni aportado nada nuevo en relacin con las enseanzas del
Organon. Este famoso error del filsofo de Knisberg se debi fun
damentalmente a que no conoci o no valor suficientemente los
avances de los estoicos, de los megricos y de Guillermo de Occam.
El creador indiscutible de la Lgica-matemtica fue el ingls
George Boole (1815-1864) a travs de sus obras Anlisis matemti
co de la lgica e Investigaciones de las leyes del pensamiento. Boole
utiliz el lenguaje del lgebra para atacar los problemas lgicos
tradicionales planteados por el silogismo aristotlico, los cuales
resolvi a travs de procedimientos mecnicos de clculo. Sin
embargo, este nuevo lenguaje, conocido como Algebra de Boole,
manifest su potencia resolviendo problemas que excedan los
alcances de la lgica aristotlica y poniendo por primera vez en
evidencia los errores del estagirita. El Algebra de Boole tambin
se conoce como lgebra de clases o lgebra de conjuntos que con
tinu investigando Augusto de Morgan (1806-1878). Posterior
mente el ingls Jevons, el alemn Schroeder y el sovitico Poretskiy
convirtieron el lgebra de clases en un lgebra de proposiciones;
y Gottlob Frege en su trabajo titulado Begriffsschrft (en espaol,
Ideografa), propuso un mtodo de clculo de matrices para la l
gica proposicional muy semejante al que se usa actualmente. Asi
mismo, Frege desarroll de manera importante la lgica predica
tiva con el fin de aplicar el mtodo axiomtico a la naciente teora
de conjuntos de G. Cantor.

16.3 La lgica matemtica contempornea

La lgica contempornea debe mucho de manera inmediata a las


enseanzas de Frege y el hito que marca su inicio es la obra mo
numental de Bertrand Russell y Alfred Whitehead titulada Prin
cipia Mathematica aparecida en 1910, editada en Inglaterra, en tres

310
tomos. El propsito de esta obra fue poner toda la matemtica co
nocida hasta entonces en estricto orden lgico, utilizando lo que
ahora se conoce como un lenguaje lgico de primer orden. Para
ello Russell y Whitehead aprovecharon los hallazgos del matem
tico italiano Peano expuestos en su libro Los principios de la arit
mtica presentados por un nuevo mtodo, en el que se aplica por pri
mera vez el mtodo axiomtico a la aritmtica. Debido a este he
cho, el simbolismo lgico ms usado actualmente (es el que se usa
en este manual) recibe el nombre de notacin Peano-Russell.
La aparicin de las geometras no euclidianas por creacin de
Lobachevski (1793-1856), Bolyai (1802-1860) y Riemann (1826-1866)
introdujo en la matemtica espacios hiperblicos y esfricos que
alteraban el espacio rectilneo trabajado por Euclides. Alteraciones
semejantes en el lgebra tradicional haban sido introducidas por
la creacin del lgebra abstracta por Evaristo Galois en 1832. Estos
hechos crearon la necesidad de estudiar a las teoras matemticas
mismas a fin de determinar sus propiedades. David Hilbert, en esta
lnea de trabajo, invent la Metamatemtica cuyo objetivo es el es
tudio de las teoras matemticas aplicando los lenguajes lgicos
que haban sido creados por Frege y Russell. Notables investigado
res han dedicado sus mejores esfuerzos a la Metamatemtica y a la
solucin de sus grandes problemas que fueron planteados por
Hilbert en un Congreso de Matemtica realizado en 1900. El ms
conspicuo de todos ha sido Kurt Godel, quien demostr alrededor
de 1930 el ms importante teorema de Lgica-Matemtica de este
siglo, conocido como Teorema de las proposiciones indecidibles.
En 1938, Claudio Shannon aplic el lgebra de las proposiciones al
diseo de circuitos elctricos a conmutadores y relays lo que cons
tituye el aporte ms importante a la construccin de las modernas
computadoras electrnicas digitales. De esta manera, la Lgica-
matemtica dej de ser un instrumento puramente terico para
convertirse en un instrumento que sirve de soporte a la tecnologa
ms sofisticada de nuestro siglo.
La diversificacin de las investigaciones en Lgca-Matemt-
ca, durante los ltimos sesenta aos, ha conducido al surgimiento
de ramas altamente especializadas. El polaco Lukasiewicz desa
rroll las lgicas polivalentes y Tarski, del mismo origen, cre la
semntica lgica con sus investigaciones sobre el concepto de ver-

311
dad en los lenguajes formalizados y demostr la necesidad inelu
dible de usar metalenguajes, reafirmando as los resultados de
Russell y Hilbert. A partir de estos resultados se'ha formulado la
moderna teora de modelos que tiene entre sus representantes a
Tarsky, Keisler, Kreisel, Morley y Robinson. De otra parte Hans
Reichenbah, Keynes, Carnap y Popper han desarrollado las lgi
cas probabilitarias y las han aplicado al anlisis de teoras fsicas y
del mtodo de investigacin cientfica. Estos estudios y sus resul
tados han contribuido al nacimiento y afianzamiento de una nue
va disciplina llamada Epistemologa, cuyo sentido es el anlisis
de la ciencia utilizando instrumentos proporcionados por la Lgi
ca-Matemtica a travs de sus diferentes ramas. Han destacado
como epistemlogos el mismo Popper, Hempel, Nagel, S. Barker,
Stegmller, Moulines y el argentino Mario Bunge, entre otros. En
Estados Unidos han descollado alrededor de la dcada del cin
cuenta los trabajos de Kleene y los de Church sobre funciones
recursivas, cuyos resultados han permitido esclarecer a nivel te
rico y prctico las limitaciones y los alcances de una computadora
electrnica cualquiera. Tambin son notables en este pas los tra
bajos del profesor W. O. Quine quien ha inventado lenguajes muy
complejos y potentes. Sin embargo, el mayor aporte de la lgica
norteamericana est dado por la demostracin que hizo Paul Cohn,
en la dcada del sesenta, de la independencia de la de la hiptesis
del continuo en la teora de conjuntos de Cantor. Este teorema que
al igual que el de Gdel constituye una respuesta a uno de los veinte
problemas de Hilbert, puede ser considerado el segundo en im
portancia en la Lgica-matemtica de nuestro siglo.
E la Unin Sovitica tambin ha habido aportes sustancia
les a travs de Malser, Kolmogorov, P.S. Novikov, A. Markov y
Shanin, entre otros. En la China se han destacado Wang Hsien
Chun, Hao Wang y Shih Hua. El segundo ha trabajado en Esta
dos unidos y ha aportado al mtodo de procesamiento de teore
mas lgicos a travs de computadoras.

16.4 La Lgica Matemtica en Amrica Latina

La Lgica-Matemtica ha ocupado la actividad de un nmero cre


ciente de investigadores latinoamericanos durante los ltimos

312
veinte aos. Tal vez el ncleo ms activo sea el ubicado en Brasil
en las universidades de Sao Paulo y Campias. Su representante
ms distinguido es Newton da Costa, quien es creador de lengua
jes lgicos especiales conocidos como paraconsistentes debido a
que hacen un uso muy especial del principio de no-contradiccin.
Otro sector importante de investigadores se agrupa alrededor de
la universidad de Baha Blanca en Argentina y entre ellos mere
ce especial mencin L. Monteiro, que con un grupo de profeso
res colaboradores ha hecho importantes publicaciones en lgica
algebraica. En Chile destac un residente alemn, Gerold Stahl,
quien hizo investigaciones metamatemticas. En las universida
des catlicas de Santiago y Valparaso existe un grupo de profe
sores que trabajan en lgica algebraica y lgica probabilitaria.
En el Per, la actividad en Lgica-Matemtica la inici el po
laco Rosenblat y la continu Francisco Mir Quesada C. Los tra
bajos estn orientados en su mayor parte a la divulgacin, espe
cialmente en niveles introductorios. Sin embargo, la actividad es
creciente y su ncleo ms activo se encuentra en la Universidad
de San Marcos y en la Universidad Catlica de Lima.

16.5 La lgica matemtica y otras lgicas


El filsofo Hegel es uno de los grandes responsables de las ambi
gedades producidas con el uso de la palabra lgica. En efecto,
escribi el libro titulado La ciencia de la Lgica que no es una obra
que trate de sistemas deductivos sino de filosofa metafsica. Con
las modificaciones adecuadas, C. Marx fund a partir de Hegel
el mtodo dialctico materialista al que algunos de sus seguido
res han denominado lgica dialctica. Este mtodo tambin es fi
losfico y no es, ni Marx pretendi que lo fuera, un procedimien
to altamente sofisticado de clculo que slo es posible si se usa
un lenguaje matemtico.
Sin embargo, debido a que a veces se ha utilizado la palabra
lgica libremente por el motivo antes explicado, se han cometido
confusiones adicionales al considerar como mtodos lgicos a for
mas del antiguo pensamiento oriental indio y chino que difieren
sustancialmente del racionalismo occidental. Es verdad que esas
formas del pensar tienen su propia estructura interna y que exis

313
ten leyes y reglas que las gobiernan, las cuales en un sentido es
pecial constituyen su lgica interna. Pero eso es hablar de lgica
en un sentido, muy lejano al de la Lgica-Matemtica.
Es difcil definir breve y elementalmente a la Lgica-mate-
mtica. Empero, para los fines que nos proponemos es suficiente
decir que es una ciencia formal dedicada a la construccin de
lenguajes especiales, llamados lenguajes formales, que sirven
para expresar o mostrar la estructura de la teoras cientficas y
para dar las reglas que permiten transformar una estructura dada
en otra. Como puede comprenderse a partir del estudio de su
historia y de sus problemas, esta disciplina se encuentra indisolu
blemente unida a la matemtica y sus principales esfuerzos y lo
gros han estado orientados hacia el esclarecimiento de los funda
mentos de la aritmtica, de la teora de conjuntos, de la geometra
y del lgebra abstracta. Es a partir de sus aportes en estos campos
que la Lgica-Matemtica ha aportado decisivamente a la cons
truccin de computadoras y ha hecho posible la elaboracin de
lenguajes para la comunicacin entre el hombre y la mquina.
Como se comprende, no existe dificultad en que alguien
hable de otras lgicas siempre y cuando no pierda de vista
la diametral diferencia que existe entre ellas y la Lgica-mate
mtica. En el caso de un manual introductorio, como ste, hacer
una distincin clara y fundada es tarea ineludible que esperamos
haber cumplido.

314
BIBLIOGRAFIA BSICA

AGAZZI, Evandro
1986 La lgica simblica. Barcelona; Ed. Herder.
BLANCH, Robert
1963 Introduccin a la lgica contempornea. Buenos Aires; Ed. Carlos Lohl.
COHEN, M. y NAGEL E.
1971 Introduccin a la lgica y al mtodo cientfico. Buenos Aires: Amorrortu
Ed. Tomo 1.
COPI, Irving y COHEN
1997 Introduccin a la lgica. Buenos Aires: Eudeba.
DA COSTA, Newton
1997 Logiques classiques et non classiques. Pars: Masn.
DEAO, Alfredo
1974 Introduccin a la lgica formal. Madrid: Alianza Editorial.
FERRATER, J. y LEBLANC, H.
1962 Lgica matemtica. Mxico: Fondo de Cultura Econmica.
FUCHS, Walter R.
1974 Los padres descubren la nueva lgica. Barcelona: Ediciones Omega
S.A.
GRANELL, M.
1949 Lgica. Madrid: Ed. Revista de Occidente.
HOPCROFT, John y Jeffrey ULLMAN
1969 Formal language and their relation to Autmata. Addison: Wesley Pu-
blishing Company.
MATES, Benson
1971 Lgica matemtica elemental. Madrid: Ed. Tecnos.

[315]
MENDELSON, E.
1964 Introduction to Mathematicl logic. Van Nostrand.
MIR QUESADA, Francisco
1964 Lgica. Lima: IPEM.
NEBENDAHL, Dieter (ed.)
1988 Sistemas expertos. Berln: Siemens Aktiengesellschaft.
QUINE, W.O.
1971 El sentido de la nueva lgica. Buenos Aires: Ed. Nueva Visin.
REINHART, Jacques
1961 Las etapas de la lgica. Buenos Aires: Ed. Marisol.
SACRISTN, Manuel
1964 Introduccin a la lgica y al anlisis fomial.Barcelona: Ed. Ariel.
STHAL, Gerald
1962 Introduccin a la lgica simblica. Santiago: Ed. De la Universidad
de Chile.
SUPPES, Patrick
1957 Introduction to logic. Van Nostrand.
SUPPES, P. y HILL, S.
1996 Introduccin a la lgica matemtica. Barcelona: Ed. Reverte.
TARSKI, Alfredo
1951 Introduccin a la lgica y a las metodologas de las ciencias deductivas.
Buenos Aires: Ed. Espasa Calpe.
VAN HEIJENOORT, J. (ed.)
1967 From Frege to Gdel Harvard: Harvard University Press.

316
Glosario

ABSTRACCIN: Tradicionalmente, abstraer es el proceso intelectual


mediante l cual se separa, en la mente, una propiedad que en la realidad
es indesligable de otras que constituyen la totalidad de un objeto. Por ejem
plo, el concepto de fisiologa de corazn Imce referencia especficamente
al funcionamiento del corazn, aspecto que se puede estudiar aislndolo
mentalmente de otros. Sin embargo; en la realidad el funcionamiento del
corazn es inseparable de sus componentes anatmicos. No existe un co
razn, sin componentes anatmicos, que funcione. En trminos actuales
podemos decir que la abstraccin es el proceso de formacin o construc
cin de conceptos (Ver la definicin correspondiente).
ABSURDO: Una afirmacin (o un conjunto de ellas) es absurda mando
es lgicamente contradictoria. La contradiccin puede estar constituida
por ella misma o puede ser deducible de dicha proposicin. De esta ma
nera, lo contradictorio es lo irrazonable o lo racionalmente injustifica
ble. Recientemente, en medios especializados, algunos investigadores han
definido grados de contradiccin para los sistemas lgicos, dejando abierta
la posibilidad de que lo que es absurdo en un determinado sistema lgi
co no lo sea del mismo modo en otro. A los sistemas lgicos, que de ma
nera demostrada carecen de contradiccin, se les llama sistem as con
sistentes.
ALETICIDAD: Es la propiedad que tiene cualquier proposicin por ser
susceptible de ser verdadera o falsa. A los valores verdadero-falso se
les denomina valores alticos o tambin valores veritativos.,
ALGORITMO: Es un conjunto finito de reglas, de aplicacin m ec
nica o au tom tica a la solucin concluyente de un cierto tipo de pro-

[317]
blemas. A los algoritmos tambin se les llama -procedimientos de deci
sin, y los mtodos para sumar, restar o sacar raz cuadrada son ejem
plos de algoritmos conocidos. Las tablas de verdad tambin son algort
micas pero no las reglas conocidas como de deduccin natural. Asimis
mo, cualquier conjunto de rdenes adecuadas para un computadores un
algoritmo.
CAUSALIDAD: Aristteles, en su obra M etafsica distingui cua
tros causas: 1) la causa m aterial, que es aquello de lo que est hecho
algo o aquello que lo constituye; 2) la causa form al, que es aquello que
define especficamente a una cosa o que es su esencia; 3) la causa efi
ciente, que es la fuerza que produce una cosa, la mueve o la modifica;
4) la causa fin a l, que es el fin hacia el cual tiende algo. Ellas responden
a las preguntas De qu...?, Qu...?, Por qu...? y Hacia qu...?
respectivamente. El principio de causalidad de la ciencia moderna, fun
dada por Galileo, asume slo la causa eficiente de Aristteles al afirmar
que todo hecho o acontecimiento que ocurre es efecto de otros que son
su causa y, a su vez, l mismo es causa de otros efectos . De esta mane
ra, se concibe a la naturaleza como una multiplicidad de hechos que
constituyen elementos de cadena causales, las cuales pueden ser enten
didas sin necesidad de recurrir a entidad sobrenatural alguna. Por con
siguiente, la explicacin de un hecho se convierte en la identificacin del
hecho o hechos que la causan. Con el surgimiento de la mecnica de
Newton se comienza a hablar de leyes causales que rigen la naturaleza
y, particularinente, el movimiento, dichas leyes eran ecuaciones que fue
ron perfeccionadas por Laplace. De este modo, si se conoca los valores
numricos del m om ento y de la p osicin de un mvil, por ejemplo la
Luna, aplicando las ecuaciones diferenciales de Laplace, se poda deter
minar ios valores de sus momentos y posiciones posteriores. Esto con
dujo a Laplace al m ecanicism o, que fue una corriente filosfica que con
cibi al universo como una gran mquina cuyo funcionamiento est
regido por las leyes causales de Newton. El mecanicismo tambin es co
nocido como determinismo y fue superado, en nuestro siglo, por la me
cnica cuntica de Max Planck y Wemer Heisenberg.
CIENCIA: La palabra 'ciencia' tiene como origen etimolgico la pala
bra griega episteme que fue usada por Aristteles para referirse al co
nocimiento de las causas que producen el movimiento de las cosas par
ticulares y el cambio de las mismas. Actualmente, se usa 'ciencia' para
hacer referencia al conocimiento que est constituido o conformado por

318
teoras cientficas que se caracterizan por ser conjuntos de proposicio
nes que muestran las relaciones ms generales que existen entre los ob
jetos y procesos que constituyen, un sector de la realidad. Las proposi
ciones que constituyen las teoras son usualmente llamadas leyes cien
tficas, de manera preferente en lo campos de la fsica, qumica, biologa
y, por extensin, en los de las ciencia sociales como la economa y la so
ciologa, o en el campo de las ciencias de la conducta, como es el caso de
la psicologa. Las leyes cientficas, en las reas ms desarrolladas del co
nocimiento, se expresan en lenguaje matemtico y cumplen la funcin
de explicar con precisin las causas de los hechos o procesos, del mbito
que les es propio, y de predecir con rigor matemtico el curso futuro ms
probable de los mismos.
CIENCIAS EMPRICAS: Son todas aquellas ciencias constituidas por
proposiciones cuya verdad se establece contrastndolas con los hechos
de la realidad natural o social, que pueden ser percibidos por un obser
vador. Consecuentemente, en estas ciencias es la experiencia la que nos
permite decidir la verdad o falsedad de una proposicin. A estas disci
plinas tambin se les denominan ciencias fcticas.
CIENCIAS FORMALES O ESTRUCTURALES: Son aquellas cien
cias constituidas por proposiciones, cuya verdad se establece mediante
la construccin de demostraciones que se ajustan a reglas lgicas de de
duccin o inferencia. En estas ciencias carece de sentido intentar esta
blecer la verdad de una proposicin por medio de la contrastacin con
los hechos, pues en ellas slo se admite una proposicin como verdadera
si es que existe una demostracin o prueba lgica para ella. Las ciencias
formales, son la Matemtica y la Lgica-Matemtica. Asimismo, la apli-
cabilidad de la matemtica a la realidad natural y social no es una prueba
de su verdad sino consecuencia de ella. Para distinguir las diferentes
maneras de establecer la verdad en ciencias empricas y en ciencias for
males .se dice que las primeras poseen proposiciones empricamente ver
daderas o a p osteriori y las segundas proposiciones lgicamente ver
daderas o vlidas a priori.
CLASE: En los niveles elementales se usa indistintamente los concep
tos de clase y conjunto, lo que da lugar a imprecisiones de magnitud
creciente. Desde Von Neumann, uno de los matemticos ms notables
de este siglo, se ha establecido que el concepto de clase es de mayor ex
tensin que el de conjunto. De este modo, intuitivamente, se admite que
una clase es cualquier coleccin de elementos. Un conjunto es una clase

319
que puede ser elemento de otras clases y una clase ltim a es una clase
que no puede ser elemento de clase alguna. As los objetos concretos son
elementos de los conjuntos. Y los conjuntos, que son un tipo especfico
de clase, son elementos de las clases ltimas. A grosso m odo se dice
que un conjunto es una clase que no es demasiado grande y esta distin
cin se ha hecho con el propsito de evitar paradojas que son contradic
ciones especiales. Una consecuencia de ello, es que se puede definir la
clase de todos los conjuntos que satisfacen la condicin P(x) . Pero no
la clase de todas las clases que satisfacen la condicin P(x) . La la
clase de todas las clases es simplemente una contradiccin en los tr
minos. (Los crculos de Venn usados en este texto representan conjun
tos; y el rectngulo, una clase ltima.)
CONCEPTO: Es un trmino que denota una propiedad que la posee o
satisface un conjunto de objetos o elementos. Un concepto, se refiere a
una pluralidad de individuos que constituyen el conjunto extensin del
concepto. Un nombre propio es desde el punto de insta lgico, lo opuesto
a un concepto en la medida que se refiere a un tnico elemento para iden
tificarlo. Los conceptos pueden tener diversos grados de abstraccin. Los
menos abstractos son aquellos cuyo conjunto extensin tiene como ele
mentos objetos macrofsicos como, por ejemplo, el concepto felin o. Los
ms abstractos tienen un conjunto extensin cuyos elementos o no son
directamente observables como el concepto microfsico de quark, o sim
plemente se trata de objetos slo entendibles como el de nmero com
p lejo. Lgicamente, los conceptos son predicados de la forma P(x).
CONJUNTO: (Ver CLASE)
CONTRASTACIN: Es el acto por el cual se coteja lo que dice una
proposicin con lo que constatamos que ocurre en la realidad natural o
social As se decide si la proposicin es empricamente verdadera o em
pricamente falsa. La contrastacin puede hacerse directamente, esto es,
estableciendo la correspondencia entre lo que dice la proposicin y la rea
lidad o indirectamente, esto es, cotejando con la realidad no la proposi
cin misma en cuestin sino las proposiciones que se deducen lgica
mente de ella. El concepto de contrastacin, en la bibliografa actual, ha
reemplazado prcticamente al de verificacin debido a la influencia del
filsofo Karl Popper.
CUANTIFICADOR: En lgica se denomina cuantificador, en gene
ral, a un operdador que nos permite referimos a todos los objetos de una
clase, que se escribe ( V x ) , y a otro que nos permita afirmar que una

320
clase o universo dado tiene al menos un objeto, pudiendo tener muchos
ms. Este segundo se escribe as: (3x). Al primero se le denomina cuan-
tificador universal y al segundo cuantificador existencia!. El cuantifica
dor (\fx) se lee: para todo objeto x , es el caso que y el cuantificador
(3x) se lee: existe al menos un objeto x tal que. De este modo, la fr
mula Ox) P(x) se lee: Existe al menos un objeto x tal que este x tiene
la propiedad P y la frmula (Vx)(P(x)>Q(x)) se lee: Para todo obje
to x, si x tiene la propiedad P, entonces x, tiene la propiedad Q. Usan
do el operador de negacin los cuantificadores son interdefinibles y sus-
titibles entre s. As, la frmula (\fx)P(x) es definible y sustitidble por
~ (3x) ~ P(x) y, de la misma manera, (3x) P(x) es reemplazada por
~ (Vx) ~ P(x). Esto significa que nn sistema lgico puede prescindir de
tino de ellos . Para algunos casos especiales se usa al denominado cuan
tificador iota, introducido por B. Russell que se escribe (3xi) y se lee:
Existe un nico objeto x tal que. Este cuantificador es existencial y
su uso es muy limitado porque se puede prescindir de l mediante el
uso adecuado de los otros dos . Los cuantificadores tambin se utilizan
con predicados relacinales (ver PREDICADO). Por ejemplo la frmula
( V x ) ( 3 y ) R ( x , y ) s e lee: Para todo objeto x existe al menos un
objeto tal que es el caso que x est en la relacin R con y. El cuantifi
cador de mayor jerarqua es el que aparece al comienzo de la frmula y
as en orden de jerarqua decreciente.
DEDUCCION: Es una operacin lgica que consiste en obtener, me
diante la aplicacin de reglas lgicas, llamadas de deduccin, a partir
de un conjunto de proposiciones, conocidas como premisas, una nueva
proposicin conocida como conclusin .
Lo que garantizan, en lo fundamental, las reglas lgicas de deduccin
es que la conclusin se deriva de las premisas sin contradiccin. Asi
mismo, una demostracin en matemtica es estrictamente un ejemplo
de deduccin.
DEFINICIN; Es una operacin metodolgica que sirve para explicar
de manera precisa el significado de un trmino. Consta de un Definien-
dum y de un Definiens. El Definiendum, es el trmino cuyo signifi
cado se pretende explicar o determinar y el D efiniens, es el conjunto de
palabras de uso conocido que nos permite explicar el significado del De
finiendum. Una definicin se escribe a manera de una igualdad cuyo
primer miembro es el Definiendum y cuyo segundo miembro es el De
finiens. El requisito fundamental que debe cumplir toda buena defini-

321
cin es que el Definiendum no sea elemento del Definiens. Si esto ocu
rre, la definicin es defectuosa y se le llama circular. Existen muchos
tipos de definiciones, tales como lexicogrficas, estipulativas, operado-
nales, etc.
DEMOSTRACIN POR REDUCCIN AL ABSURDO (RAA):
Una proposicin P se demuestra lgicamente por Reduccin al absurdo
cuando el primer paso deductivo consiste en suponer que P es falsa (lo
cual se formaliza mediante no-P), y a partir de esta suposicin se de
duce una contradiccin de la forma A y no-A. A si se dice, como la ne
gacin de P conduce a contradiccin, entonces P es una proposicin ne
cesariamente verdadera. A esta demostracin se les llama indirectas o
apaggicas.
HIPTESIS: Es una conjetura que pretende dar respuesta a una inte
rrogacin que expresa un problema cientfico. Tambin puede definirse
como una solucin tentativa a un problema cientfico. Una caractersti
ca fundamental de las hiptesis es que deben ser decidibles como verda
deras o como falsas, razn por la que son expresadas a travs de propo
siciones o enunciados.
INDUCCIN: Tradicionalmente se considera la induccin como la in
ferencia que, a partir de premisas que describen hechos singulares o par-
ticidares, derivan una conclusin de carcter universal o general. Sin
embargo, como las premisas slo se fundan en constataciones hechas en
algunos de los objetos del universo estudiado, esto es, en lo que se lla
ma una muestra, y la conclusin es tina proposicin general que hace
una afirmacin sobre la totalidad de los objetos del universo, entonces
no se le considera verdadera sino probable. En breve, los clsicos como
Aristteles y Francisco Bacon concibieron a la induccin como una in
ferencia, opuesta a la deduccin, que opera de lo particular a lo general
En et siglo pasado John Stuart Mili precis que la induccin no es una
inferencia que estrictamente obtenga una conclusin general a partir de
premisas particulares, debido a que contiene siempre como premisa im
plcita una proposicin general, denominado por l principio de uni
fo rm id a d de la naturaleza. De este modo, para Stuart Mili la induc
cin era una deduccin que se distingua de las dems por tener como
premisa mayor al principio de uniformidad de la naturaleza. Actualmen
te, la induccin se define como una inferencia no conclusiva que esta
blece el grado de confirmacin de una proposicin general a partir de la
verdad de las proposiciones de observacin que de ella se deducen. Por

322
inferencia no conclusiva se entiende que la conclusin no se sigue nece
sariamente de las premisas y el concepto de grado de confirmacin es
traducible por un valor detenninable dentro del clculo de probabilida
des, el mismo que fue utilizado por R. Carnap y C. Hempel para definir
matemticamente dicho concepto. Asimismo, la tendencia actual enca
bezada por el filsofo Karl Popper sostiene que la induccin carece de
fundamento y no forma parte del razonamiento cientfico.
INDUCCIN MATEMTICA: El matemtico italiano Guiseppe
Peono fiie el primero en presentar la aritmtica como una teora riguro
sa, esto es, como un sistema hipottico-deductivo de enunciados. Esta
tarea la cumpli en su obra P rincipios de aritm tica publicada en
1889. El quinto axioma de su teora es conocido como principio de de
duccin matemtica o principio de induccin completa, que es un enun
ciado implicativo de naturaleza muy distinta a lo que se denomina in
duccin en la filosofa tradicional (Ver IN D U C C I N ). Dicho princi
pio afirma que si una propiedad cualquiera se cumple para el cero,
y se cum ple adem s para un nm ero natural cualquiera bajo la
presuposicin de que se cum ple para su antecesor, entonces to
dos los nm eros naturales tienen la propiedad en cuestin . Usan
do la simbologa introducida en este glosario (Ver P R E D IC A D O y
C U ANTIFIC AD O R), una formulacin aceptable del quinto postidado
de Peano es la siguiente:

(P(0)&( Vn)(P(n)->P(n+l)))->("x)P(x)

En este caso la letra n representa al nmero anterior al denota


do por 'n+1' y la variable individual V hace referencia a cual
quier nmero natural. La formulacin anterior se conoce como
principio de induccin matemtica dbil. Existe la versin fuerte
que se deduce de la primera. Intuitivamente se enuncia as: Si del
hecho de que todos los nmeros anteriores a un nmero n ten
gan la propiedad P se deduce que tambin el nmero n tiene
la propiedad P, entonces todos los nmeros naturales tiene la
propiedad P. Utilizando la expresin 'z < x para denotar que un
nmero z cualquiera es anterior a otro x, la formula lgica de la
llamada induccin fuerte es la siguiente:

(Vx)[(Vz)(z<x^P(z))^P(x) ( Vx )P ( x )

323
LENGUAJES NATURALES U ORDINARIOS: Son aquellos len
guajes que se usan en la vida cotidiana, como el espaol, el ingls, el
alemn, etc. Esta denominacin se usa para distinguirlos de los lengua-
jes formales como los de la lgica y de la matemtica.
LEY CIENTFICA: Es una hiptesis cuya verdad ha sido establecida
a travs de la observacin de hechos o acontecimientos que pertenecen
al sector de la realidad al cual se hace referencia. En la prctica cientfi
ca se acepta como una limitacin a esta definicin el hecho de que exis
ten proposiciones cientficas que tienen la condicin de leyes pero que
no son verdaderas en el sentido que lo son las leyes macrofsicas, como
las de la mecnica de Newton. Ello se debe a que estas proposiciones ex
presan relaciones estadsticas lo que las ubica dentro del mbito de la
probabilidad ms que de la verdad. Las leyes de la termodinmica y de
la microfsica en general son probabilsticas y tambin las de la econo
ma o las de la psicologa, entre otras ciencias.
METALENGUAJE: Se dice que tm lenguaje L es un metalenguaje
cuando se lo usa para describir a otro lenguaje L que se denomina len
guaje objeto. Esto significa que, por ejemplo, el lenguaje L no tiene nom
bres que denoten cosas u objetos del mundo real sino que los nombres
de L denotan o hacen referencia a los signos de L, tambin a sus propo
siciones y a sus nombres. En breve, L no habla de cosas o hechos sino de
objetos lingsticos. As, si en L existe la palabra gato como nombre de
un animal domstico, en L existir la expresin 'gato' que no es el nom
bre de un animal sino la palabra gato. Puede resultar raro que una pa
labra tenga nombre; pero desde el punto de vista lgico, cada palabra de
L tiene un nombre en L que se construye ponindole comillas simples a
las palabras de L. Tarski demostr que si con el lenguaje L construi
mos ua teora cientfica cualquiera, entonces la palabra 'verdadero'no
puedf ser parte de L sino L, que es el metalenguaje de L.
METATEORA: Es la disciplina que estudia las propiedades de las teo
ras cientficas que son tomadas como lenguajes-objetos. La metateo-
ra hace uso de metalenguajes y algunos la llaman m etaciencia.
MODELO DE CAJA NEGRA: Es un diagrama explicativo y de si
mulacin que consiste en un rectngulo con una lnea de entrada y otra
de salida (en ingls: input y ontput). Pretende representar el compor
tamiento de un organismo o de un artefacto, de estructura indetermi
nada, en interaccin con su ambiente recibiendo estimulaciones a travs
de sus entradas y emitiendo respuestas mediante sus salidas. Tiene ade

324
ms un circuito d efeed back (retroalimentacin o autorregidacin) que
representa la forma como el organismo o el artefacto representado va
modificando su comportamiento en funcin del modo como el ambiente
acepta o no sus respuestas.
ORGANON: Las obras de Aristteles sobre lgica son conocidas bajo
el ttulo de Organon, palabra de origen griego que tambin es conocida
en su versin latina de Organum. Los discpulos de Aristteles, con la
palabra Organon quisieron enfatizar el sentido instrumental de las re
glas de la lgica aristotlica para probar la verdad. En la filosofa pos
terior Organon ha mantenido este sentido metedolgico enfatizando la
idea de conjunto sistemtico de reglas para pensar correctamente. Cier
tamente, la pretensin tambin ha sido que tal conjunto de reglas es com
pleto. Una ms sera superfina y una menos dara lugar a insuficiencia.
Francisco Bacon, duro crtico de Aristteles, escribi el Novum Org
anum propugnando que deba dejarse atrs definitivamente la lgica y
metodologa aristotlica expuesta en el Organon.
PARADOJA: Se conoce como paradoja a un tipo especial de contra
diccin constituida por una proposicin P cuya verdad implica su fal
sedad y cuya falsedad implica su verdad. Recurriendo a un ejemplo cl
sico supondremos que P es la proposicin Yo miento. Luego, si es
verdad que miento, entonces hago afirmaciones falsas y, como yo afir
mo P, entonces P es falsa. Recprocamente, si es falso que miento, en
tonces digo la verdad y, como yo digo P, entonces P es verdadera. Una
contradiccin normal del tipo L a rosa es roja y la rosa no es roja,
de la forma P y no-P, no tiene este comportamiento. Por ello es insufi
ciente decir, simplemente, que una paradoja es una contradiccin. Asi
mismo, las paradojas se clasifican en lgicas y semnticas. La que he
mos expuesto se conoce desde la antigedad como paradoja de Epimni-
des o del mentiroso.
PARALOGISMO: Es una falacia que generalmente consiste en alte
rar ilegtimamente la estructura del silogismo. Los diagramas de Venn
permiten descartar la falacias que los antiguos conocieron como paralo
gismos y otras que desconocieron por falta de instrumentos adecuados.
POSTULADO: En matemtica, un postulado es una proposicin cuya
verdad se acepta o se supone en la condicin de punto de partida para
la demostracin de la verdad de otras proposiciones, que son sus conse
cuencias lgicas. Para algunas corrientes lgicas y filosficas lo que con
vierte a una proposicin en postulado no es su verdad, que es irrelevan-

325
te, sino el hecho de que ofrezca caractersticas que la conviertan en un
punto de partida-productivo. Se considera que si las consecuencias de
un conjunto de postidados son verdaderas, entonces tales postulados son
verdaderos.
El gemetra griego Euclides (siglo III a. C.) distingui entre axioma y
postulado en su obra Elem entos. Para la matemtica contempornea
carece de importancia tal distincin y adolece de sustento el criterio de
evidencia que us Euclides para definir dichos conceptos.
PREDICADO: (En lgica) A las propiedades y cualidades de los obje
tos, individuos, o procesos se denota en lgica usando predicados a lo
que se representa mediante frmulas del tipo P ( x ) , Q ( x ) , S ( x ) ,
etc. las mismas que se leen en castellano como el objeto x tiene la pro
piedad P, El objeto x tiene la propiedad Q, y as sucesivamente. Las
letras maysculas P, Q, S... se denominan letras predicativas y deno
tan propiedades como las de 'ser azV, 'ser nmero par', 'ser hombre'
o 'ser un polinomio' por citar slo algunos ejemplos. La letra minscu
la x se denomina variable individual y denota de manera general a cual
quier objeto o individuo que tiene la propiedad que la letra predicativa
respectiva representa. En lgica no se acepta los nombres comunes de la
gramtica como tales sino como predicados. As, el nombre comn 'gato'
para efectos lgicos es el predicado 'x tiene la propiedad de ser un gato'
y se escribe G ( x ). Las frmulas P ( x ), Q ( x ), etc. se conocen como
funciones proposicionales o predicativas. Las que tienen una sola varia
ble individual se llaman mondicas, que es el caso de los ejemplos da
dos. Existen tambin las didicas de la forma R ( x , y ) , las tridicas de
la forma R (x, y, z), y as sucesivamente. Las funciones proposicionales
que tienen dos o ms variables individuales se conocen genricamente
como predicados relacinales o, simplemente, relaciones. Por ejemplo la
fnnulh W ( x, y, z ) se lee: Los objetos x, y, z se encuentran en la
relacin W. Se dice en general, que un predicado tiene grado n si po
see un nmero n de lugares para variables individuales. En el caso de
W ( x, y, z ) se trata evidentemente de un predicado de grado 3, y en el
de R ( x, z ) de un predicado de grado 2. Debido a lo anterior, la fonna
general de escribir un predicado en lgica es: P ( x , x^..., x j .
PROPOSICIN NECESARIAMENTE VERDADERA: Una pro
posicin P es necesariamente verdadera si desde su negacin no-P es
posible deducir una contradiccin lgica de la forma A y no-A. A estas
proposiciones tambin se les llama apodcticas.

326
RELACIN DE FUNCIONALIDAD: Se dice que existe una rela
cin de funcionalidad entre dos hechos o propiedades cuando sta puede
ser expresada mediante una proposicin matemtica de la forma y - f
(x). Sin embargo, esto no significa necesariamente que la propiedad o
hecho denotado por y es producido por el hecho denotado por x. Si esto
ltimo ocurre, entonces, la relacin de funcionalidad es adems una re
lacin de causalidad.
TRMINO OBSERVACIONAL: Es aquel que denota o hace referen
cia a objetos, propiedades o procesos directamente observables. Son tr
minos observacionales, por ejemplo, 'rojo', 'azul', 'estornudo', 'columna
de mercurio de 37 grados centgrados', etc. En particular se afirma que
hay observacin directa cuando el investigador tiene como dato inme
diato no un efecto del objeto estudiado sino al objeto mismo. La presen
cia o ausencia de instrumentos de observacin en este proceso es irrele
vante para su caracterizacin.
VERDAD: El concepto de verdad se usa en la ciencia y en la filosofa
como una propiedad que pueden tener slo las proposiciones. De acuer
do con esto no existen cosas verdaderas sino slo proposiciones verda
deras. A lo largo de la historia de la filosofa se ha dado al menos tres
tipos de definiciones para el concepto de verdad. La ontolgica, sostiene
que una proposicin es verdadera cuando lo que afirma corresponde a lo
que ocurre en realidad. La formal o de la coherencia sostiene que una
proposicin es verdadera cuando es demostrable o deducible sin contra-
diccin. La pragmtica sostiene que una proposicin es verdadera cuan
do lo que afirma es til o produce acciones exitosas. Resultados debidos
principalmente al polaco Alfredo Tarski han puesto en evidencia, en este
siglo, que una definicin adecuada del concepto de verdad requiere el uso
de un metalengnaje para evitar incurrir en paradojas (contradicciones)
como la del mentiroso.

327
SOLUCIONARIO

En lo que signe no se proporciona soluciones para todos los ejer


cicios propuestos sino slo para los ms representativos.

CUESTIONARIO 1
Proposiciones

1. No 12. No 23. No 34. No


2. S 13. S 24. No 35. Si
3. No 14. No 25. S 36. No
4. No 15. No 26. No 37. S
5. No 16. No 27. S 38. No
6. No 17. No 28. No 39. S
7. No 18. S 29. S 40. S
8. S 19. No 30. S 41. No
9. No 20. S 31. No
10. No 21. No 32. No
11. S 22. S 33. S

CUESTIONARIO 2

1.pAq 5. p Aq 9.pAq 13, pAq


2. p A q 6. pAq 10. p A q 14. p a q
3.pA~q 7. p A q ll.pA ~q 15.pAq
4. pAq 8.-pA~q 12. p

329
CUESTIONARIO 3

I.
1. D isyuncin inclusiva (DI) 6. DF 11. DF
2. D isyuncin exclusiva (DF) 7. DI 12. DI
3 .D F 8. DI 13. DI
4. DI 9. DI 14. DI
5. DI 10. DF 15. DI

l.~(~p vq) 5. ~ ( P V ~ q )
2.~(~p vq) 6. ( p v q )
3~(~pAq) 7. ~ ( ~ p v - q )
4*~(pA~q) 8. ~ p v q
. l.F 2. V 3. V 4. F

CUESTIONARIO 4
II.
1. p q 5. q p
2. q -> p 6. p solam ente si q: p q
3. q p 7. Si p, entonces q: p -> q
4. q -> p 8. N o es posible p porque no q: ~ q
V.
l . p - > ~ ~p
2. ~ ~p p
3- (P-v ~ p) p
4. pT-> ~p
5. ~ ~ p -* p

CUESTIONARIO 5
I. -

1.p<H>q
2.~ p<H>~p
3.~p->~q
4. ~ q -> p
5. ~ p o- q

330
II.
1. p <-> q
2. p<H>p
3. p -> q
4. q - > p
5.p^q
III. Las proposiciones equivalentes son: 3 ,4 y 5
IV. Las afirm aciones falsas son : 2 y 3.
V I. p: Est lloviendo
q: Est nevando
r: Est corriendo viento
2. p: U sted se casar
q: U sted se convertir en actriz
r: U sted ser fam osa
3. p: La gente piensa en la crisis
q : A lguien distrae a la gente

r: El ftbol es una distraccin de masas


s: El ftbol puede ser propiciado por los beneficios por la
crisis.
VI. Todos los ejem plos (1-5) representan proposiciones lgicam ente
equivalentes

CUESTIONARIO 6

I. Solamente son frmulas de PM:


l . ( p - > (q a r ) ) - > ~ ( ~ q v ~ ( ~ r ) )
4 . ( p v q ) -> ( p A q)
6.p (~ (~ (~q ) ) )
II.

1. (p -o -q ) a (p <->q)
2- (p<-> q ) - K q v ~ p )
3-(P *q ) v (rs)

331
4. ( p - > q )-> r
5 . ( p > q ) > ( ( r v p ) ( r v q ) )
6 - ( p > q ) > ( ( q - > r ) - > ( p - > r ) )
7. ( p - > ~ q ) o ( q - > ~ p )
S .((p -> q )A r) A ((p -> q )v (p -> r))
III.
1. p V q A r v p

2. p v q a rv ~ p

3. p ^ q a r a~ s

4. p > q . v . r s a svp

5. p - q v s . A . ~ r
V.
1 . ~ ( p >q )
2.~ (p* q)
3.~(p v~ q )
4. ~ ( p ~ q )
5. ~ ( ( P -> q ) -( q ->P ) )
6.~((p-> q )- ((p -> q )v r))

CUESTIONARIO 7
I.
1. Tautologa.
2 . Consistente.
3. Tautologa.
4. Tautologa.
5. Contradictoria.
6. Consistente
II.
1.p-(q vp)
2 . ( p ^ q ) v ( q ^ p )
3.p-(q-p)

332
4. (p->(qA~q))-~p
5- ( P -> ~ P ) -> ~ P
III.
1. A firm acin verdadera pero no tautolgica
2. Es tautolgica
3. Es tautolgica
4. Afirm acin verdadera pero no tautolgica
5. Es tautolgica
6. A firm acin verdadera pero no tautolgica
IV Solam ente los ejem plos 2,3 son proposiciones equivalentes
V.
1. V 3. F 5. F 7. V 9. V

2. V 4. F 6. V 8* V 10. F

C U E S T IO N A R IO 8

II.
1. Vlida
2. Invlida
3. Invlida: Justificacin:
H iptesis H: F(((p v q ) v r ) K P A S))
F(V ((p v q) v r) F (p a s))
F(V ((p v q) v V (r)) - > F(F(p) a F(s)))
H iptesis H aceptada. La formula es
necesariam ente F cuando r es V, p es F y S es F
sin que importe el valor de q.
4. Vlida
5. Invlida
III.
1. [ ( p -> ~ q ) a p J - ~ q
2. A
[ ( P ->q ) ( ~ q -> ~ r ) ] ^ ( p ^ ~ r )
3. [(pvq)A~p]->cI
4. A
l ( P ->q ) ~ q ] -> ~ P
5. [ ( p ~ q ) A ( q - > r ) ] - ( ~ P -* r)
6. {1 ( P -q ) A ( r '- q ) ] ( q v ~ q ) } >(
A
7. [ ~ ( P A q ) A p ] - > ~ q

333
8 - [ ( p > q ) / \ q ] > p

9. [ ( p - q ) A ( q - r ) ] - ( ~ r - ~ p )

10. { [ ( q ^ p ) A ( ~ r ^ s ) ] A ( - s v ~ p ) } ^ ( r v ^ q )

V.
1. V lido 2. Las premisas 3. falsas 4. Verdaderas - falsa.

C U ESTIO N A RIO 9
Deduccin natural

I. Justificacin de deducciones
(1)(0 -> ~ P )a (~Q ->R )
(2) ( S - T ) a ( ~ U - > ~ Z )
(3) ( ~ P S ) a ( R - > ~ U )
(4) ( T v - Z ) ( W a X )
(5) O v ~ Q / W a X
6.~PvR DC. (1), (5)
7. S v ~ U DC. (3), 6
8. T v - Z DC. (2), 7
9. W a X MP. (4), 8
2.

0) [ ( A v ~ B ) v C ] [ D - ( E F)]
(2) ( A v ~ B ) [ ( F < - > G ) > H ]
(3) A - [ ( E < -> F ) - > ( F G)]
(4) A / .. D - > H
5. A v~B Adic. 4
6. (Av~B)v C Adic. 5
7. D -> ( E F) MP. ( 1) , 6
8. (E ^ F) -> ( F o G ) MP. (3), (4)
9. D > ( F <h > G ) SH. 7,8
10. (F G ) > H MP. (2) 5
11. D -> H SH. 9,10

334
u> 0) A -> B
(2) C - D
(3) ~ B v ~D
(4) ~~A
(5) (Ea F) C/ ~(E a F)
6. (A B ) a ( C D ) Adj. (1), (2)
7. ~ Av ~ C DC. (3), 6
8. ~C SD. (4), 8
9. ~(E a F) MT. (5), 8

4.

O) (G - H ) - K I <->J)
(2) K v ~ ( L M )
(3) ( G H ) v ~ K
(4) N -> ( L -> M)
(5) ~(I .-. ~N
6. ~(G H) MT. (1), (5)
7. ~K SD. (3), 6
8. ~ ( L - M ) SD. (2), 7
9. ~N MT. (4)
5.

(1)- H -> ( I -> J )


(2). K > ( I > J )
(3). ( ~ H a ~ K ) > ( ~ L v ~ M)
(4). ( ~ L > ~ N ) a ( ~ M > - O )
(5)- (P-N)a(Q-0)
(6). ~(I->J) /.'. ~ P v ~ Q
7. ~H MT. (1), (6)
8. ~K MT. (2), (6)
9. ~ Ha ~ K Adj. 7, 8.
10. ~Lv~M MP. (3), 9
11. ~ Nv ~ O DC. (4), 10
12. ~Pv~Q DD. (5), 11

335
II. EJECUCIN DE DEDUCCIONES

(1) Fv(GvH)
(2) ( G I ) a ( H > J )
(3) ( Iv J ) - > ( F v H )
(4) ~F /:. H
5) G vH SD. (1), (4)
6) IvJ DC. (2), 5
7) FvH MP. (3), 6
8) H SD. (4), 7
2.

(O K L
(2) M >N
(3) ( 0 -> M )a(P->L)
(4) ( ~ N v ~ L ) a ( ~ M v ~ O ) /.: . ( - O v ~ P )
v ~ K)
5) ~ Nv ~ L Simp (4)
6) ~Ov~P DD. (3), 5
7) ( M N ) a ( KL ) Conj. (1),(2)
8) ~Mv~K DD 5,7
9) (~Ov ~ P )a (~ M v ~ K) Adj. 6,8
J.
1) Q ( R - > S )
.2) ( R S ) - T
3) (SaU )^-P
4) ~ P ( R ~ W)
5) ~ T v ~ ( R o ~ W ) /:.~ Q v ~(SAU)
6) Q -> T SH. (1), (2)
7) (S a U )->(R o ~W) SH. (3), (4)
8) ~(R<-~W)v~T CONM. (5)
9) ( R < - ~ W ) ~ T RDN. 16,
10) (S a U)-~T SH. 7,9
11) T - > ~ ( S a U) Transp. 10
12) Q->~(S a U) SH. 6,11
13) ~ Q v ~ ( S a U) RDN. 16,

336
4.

(1) ( 0 - ~ P ) a(P-Q)
(2) Q - N
(3) ~ R - P / . . R
4) ~R P.A.
5) o->~p Simp. (1)
6) P >Q Simp. (1)
7) P MP. (3), 4
8) Q MP. 6,7
9) ~o MT. 5,7
10) ~Q MT. (2),9
11) Q a ~ Q Conj. 8,10
12) ~ R - > ( Q a ~ Q ) PC. 4, 11
13) ~ ~ R RAA. 12
14) R DN. 13

(1) X ( Y Z )
(2) X > ( A > B )
(3) X a ( Y v A)
(4) ~Z /.. B
5) ~ B RA.
6) X Simp. (3)
7) Y Z MP. (1),6
8) A B MP. (2),6
9) Y v A Simp. (3)
10) ~ A MT. 5,8
11) Y SD. 9,10
12) ~Y MT. (4),7
13)
>

Conj. 11, i:
>*
<
l

14) ~ B -> (Y a ~Y) PC. 5, 13


15) ~~B RAA. 14
16) B DN. 15

1) C - ( D ~ C )
2) C <-> D /.*. ~ C a ~ D

337
3) ( C a D) v (' ~ C A ~ ) DEF. BC (2)
4) ~ C v (-- D v - - C) DEF. 16. (1)
5) (~D v ~-C) v-~ c C0M .4
6) ~D v (~-C v ~ C) AS.5
7) (~C v ~' C) v -~ D C0M.6
8) ~Cv~D IDEMP. 7
6) ~ ( C a D) DM. 8
7) ~ Ca ~ D SD. 3,6
7.

1) Jv(~KvJ)
2) K v ( ~ J v K ) /.*. ( J a K ) v ( ~ J a ~ K )
3) (J v ~ K) v J Asoc. (1)
4) (K v ~ J) v K Asoc. (2)
5) ~JvK Asoc. + IDEMP. 4
6) ~K vJ Asoc. + IDEMP. 3
7) J-> K DEF. Asoc. + RDN.
8) K-> J DEF.
Asoc.+RDN.l,
9) ( J > K ) a ( K > J ) Conj. 7,8
10 J <h>K DEF. BC. 9
11) ( J a K ) v ( - J a - K ) DEF. BC. 10
8.
1) : ( L v M ) v ( N a O )
2) (-L aO )a ~ (-L a M ) /. ~ L a N
3) ~ LaO Simp. (2)
4) Simp. 3
5) ~ (~ L a M ) Simp. (2)
6) Lv~M DM. 5
7) - M SD. 4,6
8) ~ L a ~M Conj. 4,7
9) - (LvM ) DM. 8
10) N a O SD. (1), 9
11) N Simp. 10
12) - L a N Conj. 4,11

338
III. PRUEBA CONDICIONAL

1.
(1) ( A v B ) ^ ( C a D)
(2) ( D v E ) - > F /.'. A - > F
3) A PA.
4) A vB Adic. 3
5) CaD MP. (1),4
6) D Simp. 5
7) D vE Adic. 6
8) F MP. (2),8
9) A - F PC. 3, 8
2.

(1) (EvF)->G
(2) ( J ~ G ) a ~ H
(3) JvK /.\ E - K
4) E PA.
5) EvF Adic. 4
6) G MP. (1),5
7) J > ~ G Simp. (2)
8) ~J MT. 6,7
9) K SD. (3),8
10) E->K PC. 4, 9
o

(1) Q > P
(2) T vS
(3) Qv~S ~(PvR)^T
4) ~ (Pv R) PA.
5) ~PA~R DM. 4
6) ~p Simp. 5
7) ~Q MT. (1),6
8) ~s SD. (3),7
9) T SD. (2), 8
10) ~ ( P v R ) - T PC. 4, 9
4.
(1) A->(B->C)
(2) B > ( C D ) A->(B->D)

339
3) A PA.
4) B PA.
5) B^C MP. (1),3
6) C^D MP. (2),4
7) C MP. 4, 5
8) D MP. 6, 7
9) B^D PC. 4, 8
10) A ^ ( B ^ D ) PC. 3, 9

a) Premisas.
1) pv~q
2) r ~p

C onclusiones que se siguen lgicam ente.


C, (1) q ~ r
C2 ( 5 ) ~ p v ~ r
C7 (7) q ( ~ r v p )

C U ESTIO N A R IO 10
II.
1) (A/B)/(D/E)
2) ((A/B)/(B/B))/((B/B)/C)
3) ((N/N)(B/B))/((N/C)/(D/D))
4) ((C/D)/(E/E))/((A/A)/B)
5) (A/B)/(C/D)
6) (A/B)/(C/D)
7) [ A / ( B / B ) ] / [ B / ( A / A ) ]
8) [ ( A / A ) / ( B / B ) ] / [ A / ( B / B ) ]
9) { [ A / (B/B)]/[(A/A)/B]}/[(C/C)
/(D/D)]
10) { [ A / ( B / B ) ] / A . / . [ A / ( B / B ) ] / A } / ( B / B )
11) [ ( A / A ) / ( B / B ) ] / { [ ( B / B ) / ( A / A ) ] / [ ( B /
B)(A/A)]}
12) A / { [ ( B / B ) / ( A / A ) ] / [ ( B / B ) / ( A / A ) ] }

340
CUESTIONARIO 11

II. Construccin de circuitos para las frmulas dadas:

S / + ------- cr o------ 1

R ^ /c _________c /o SO /' 0------

Vo_
_v<
V o --------&/V-

X / o ------i / *

4. ( p A q A r ) v ( p A q A ~ r ) v ( ~ p A ~ q )

p J a___s u / oo--------------
____io l J/ o--------
, 1

_____ i / ro / "

u /c ___ / o-

341
5. { ( p A q ) v [ p A ( ~ q v ~ r ) ] v ( r A s ) } A( q v r v s )
III.

l.(p A q ) v ( p v q )
( AB ) + ( A+B )

AB
B.
(AB)+(A+B)

A+B
B

2 . ( p v ( q A r ) ) v ( r v s ) = ( A + ( B C ) ) + (C + D )

A+(BC)
BC
(A+(BC)) + (C+D)
C+D
D

3 . ( p A q ' ) v ( ~ q A r ) v ( ~ p A ~ s ) = ( AB ) + ( BC ) + ( AD )

__A_ A.B
Bf
B.C (AB)(BC) (AD)
B k B

A. A.D
H ^D

342
4. ( p A q A r ) v ( p A q A ~ r _ ) v ( ~ p A ~ q ) s
( ABC ) + ( ABC ) ( A B )

5.((pAq)v(pA(~qv~r))v(rAs))A(qvrvs)
( ( AB ) + ( A ( B+C ) ) + ( CD ) ) . ( B+C+D )
A
AB

(AB) +(A.(B + C))+(CD)

((AB)+(A(B+C))+(CD)).(B+C+D)

343
IV.
La frmula es:
( pa ~ q)v (~pAq)v(~pA~q)

CUESTIONARIO 12
Io Debe saber:

Algunos Gatos son cuadrpedos


Ningn cuadrpedo sabe silbar
Luego, algunos animales que saben silbar no son gatos.

II.
6. EAE. Segunda figura
7. AII. Tercera figura
8. AAA. Tercera figura
10. AEE. Primera figura

CUESTIONARIO 13

I. 9. S h P = <)>

10. SnP* <) >

344
12. SnP*<j>

13. SnP*<|>

14. S n P ^ (| )

II.
1. c) B n A = <)> 8. a) A n B = <)>
2. b) B n A = <|> 9. a ) A n B i |i
3. a) A n B = <j> 1 0 . b ) A n B ^ <)>
4. b) A n B = <j> 11. a) A n B =<)>
5. a) A n B = <)> 12.a) A n B = <)>
6. a) A n B = cf) 13.a) A n B = <>
7. c)A n B =(()

III.
6. Premisas Conclusin
S n M = <|) M n P = <|) S n P = ^>

Vlido AAA-1

345
7. Premisas Conclusin
P n M = (j) S n M ^ SnP^cj)

Conclusin
PnM^I S n M -(| ) S n P = (|)

9. Premisas Conclusin
P n M ^ (| ) S n M = <) S n P -< | )

10. Premisas Conclusin


S n M = (j> M n P = (j> S n P = (j)

346
CUESTIONARIO 14

1.- Determinar cules de las siguientes frmulas no son interpretables


com o proposiciones:
1- F(a,b) S
2. (3y) F(x,y) No
3- (3y) ( Fy > (V y) F x ) No
4. (V x) (V y) F(x,y) - R (x,y) No
5. Px A ~Qx No
6. Pa A ~Qa S

2.- Determinar cules de las siguientes frmulas estn en forma normal


Prenex:
1. (V x) Fx V (3y) Gy No
2. F (a,b) - (3x) ( F(x,b) A -P x ) No
3- (V x) (3y) R (x,y) S
4. (V x) Fx - (V x) Qx No
5- (V x) (3y) ( R (x,y) > Sx) S
6. (V x) (V y) (V z) ( ( R (x,y) A R (y ,z)) -> R (x ,z)) S
3.- Cules de las siguientes frmulas no son lgicam ente vlidas?
1. (V x) Fx Fa : Lg. vlida
2. (3x) Fx > Fy : N o es vlida
3. Fa - (V y) Fy : N o es vlida
4. (3x) Fx - (V y) Fy : N o es vlida
5. (3y) ( Py-> (V x) P x ) : Lg. vlida
6. (V x) (3y) R (x,y) - (3y) (V x) R (x,y) : N o es vlida
4.- Fonnalizar, usando cuantificadores, las siguientes afirmaciones:
1. N o todos los nmeros son pares.
~(V x) (N x -> Px)
2. A lgunos nmeros son primos.
(3 x )(N x A Px)
3. Ningn electrn es un protn.
(V x) (Ex - -P x )
4. Cada alumno tiene nota aprobatoria.
(V x) (Ex > A x)
5. Solam ente los aprobados recibirn diploma.
(V x) (D x - A x)

347
6. Si todos son enem igos de todos, entonces cada uno es su propio
enem igo.
(V x) (V y) E( x ,y )-> ((V x) (3y) E(x,y) .A .(x=y))
7. Hay una persona que ama a todas las personas.
(3x) (V y) A ( y ,x )
8. Hay un nmero natural que es menor que todos los nmeros
naturales.
(3y) (V x) M ( x,y)
9. Hay al m enos un alumno que se matricula en todos los cursos.
(3y) (V x) M ( x,y)
10. Todas las cabezas de caballo son cabezas de anim ales.
(V x) (Cx A x) (V y)[(3x)( Cx A H(y,x)) -+(3x) (A x A H(y,x))]
5.- Probar las siguientes equivalencias:

1. (3 y ) { Py (V x) P x ) ((V y) Py - (V x) Px).
1.1 (3y) ( Py -> (V x) P x ) - ((V y) Py - (V x) Px).

P1 (3y) ( Py -> (V x) P x ) ((V y) Py > (V x) Px)


2. (V y) Py P.A.
3. P a -> (V x) Px E.E. 1
4. Pa E .U .2
5. (V x) Px M.P. (3,4)
6. (V y) Py - (V x) Px P.C. (2-5)

1.2 ((V y) Py - (V x) Px)) - (3y) ( Py - (V x) Px)


P V (V y) Py > (V x) Px // (3y) ( Py -> (V x) Px)
2. ~(3y)(P y > (V x) Px) RAA
3 . ( V y y (Py (V x) Px) R 14.10,2.
4. -(P y - (V x) Px) R 1 4 . l l . 1,3
5. Py a ~ (V x) Px R D N. 16+D M ,4
6. Py Sim p. 5
7* (Vy) P P 1 4 .1 1.2,6
8. K V x)P x Sim p. 5
9. ~(V y)P y M T en P i, 8
10. (V y)Py a --<Vy) Py A dj.7,9

348
1 l.~ (3 y) (Py (V x)Px)(V y)Py a ~(V y) Py PC. 2,10
12. (3y)(P y -> (V x) Px) RAA, 11

II. (V x) ( Px a Q x ) <- ((V x) Px a (V x) Qx)


2.1 (Vx) ( Px a Q x ) - > ((V x) Px a (V x) Qx)
P l (V x) ( Px a Q x ) (V x) Px a (V x) Qx.
2. Py A Q y EU. 1
3. Py Simp. 2
4. Qy Simp. 2
5. (V x) Px GU. 3
6. (V x) Qx GU. 4
7. (V x) Px a (V x) Qx Conj. (5,6)
2.2 ((V x) Px a (V x) Qx) -> (V x) ( Px a Q x )
P l (V x) Px a (V x) Qx //.*. (V x) ( Px a Q x )
2. (V x) Px Simp. 1
3. (V x) Qx Simp. 1
4. Pz E .U .2
5. Q z EU. 3
6. Pz a Qz Conj. (4,5)
7. (V x) ( Px a Qx) GU. 6
3. (V x) ( Px V (V y) Qy ) ((V x) Px V (V y) Qy ).
1. (V x) ( Px V (Vy) Q y ) / / . . ((V x) Px V (V y) Q y )
2. ~((V x) Px V (V y) Q y ) RAA.
3. -(V x ) Px A ~(V y) Qy DM.2
4. (3x) -P x A (3y)~Q y R. 14 J0
5. Px V (V y)Q y EU,1
6. -(V y ) Qy Simp. 3
7. Px SD .5,6
8. (V x) Px GU.7
9. (3x) -P x Simp.4
10. ~Pa EE.9
11. Pa EU.8
12. Pa A -P a Adj. 11,10
13. - ( ( Vx)Px V (Vy)Qy) ^ (Pa A -P a) PC. 2,12
14. (V x)Px V (V y )Q y R A A ,13

349
1, (Vx) Px V (Vy) Qy) II:. (Vx) ( Px V (Vy) Qy )
2. ~(V x) ( Px V (V y) Qy ) RAA
3. (3 x ) ~ ( Px V (V y) Qy ) R. 14.10,2
4. (3x) (~P x A ~(V y) Qy ) DM .3
5. ~Pa A ~ (V y) Qy EE. 4
6. ~Pa Simp. 5
7. ~(V y) Qy Simp. 5
8. (V x) Px SD. (1,7)
9. Pa EU. 8
10. Pa A ~Pa Adj. 6,9
11. ~(V x) (Px V (V x)Q y) (Pa A ~Pa) PC 2,10
12. (V x) (Px V (V y) Qy ) R A A . 11
4. (3x) ( Px V Q x ) ^ (3x) Px V (3x) Qx.
1. (3 x ) ( Px V Q x ) //.-. (3x) Px V (3x) Qx.
2. ~ ((3 x ) Px V (3x) Qx) RAA.
3. ~ (3 x ) Px A ~ (3 x ) Qx DM.2
4. ( Vx) ~Px A (V x) ~Q x R. 14:10.3
5. (V x) ~P x Sim p. 4
6. (V x) ~Q x Simp. 4
7. Pa V Qa EE. 1
8. ~Pa EU. 5
9. Qa SD .7,8
10. ~Q a EU. 6
11. Q A~Q a Adj. 9,10
12. ~ ((3 x ) p x V (3x)Q x) -> (Q a A ~Q a) PC. 2,11
13. (3 x ) Px V (3x) Qx RAA.
1. ((3 x ) Px V (3x) Qx) //.-. (3x) ( Px V Q x )
2. ~ (3 x ) (Px V Qx) RAA.
3. (V x) ~ ( Px V Qx ) RR. 14.10
4. (V x) (~ Px A ~ Q x ) DM . 3
5. ~ Px A ~ Qx EU. 4
6. ~Px Simp. 5
7. (V x) ~P x GU. 6
8. ~ (3 x )P x R .14.10,7
9. (3x)Q x SD . 1,8
10. ~Q x Sim p. 5

350
11. (Vx)~Qx GU.10
12. Qa EE. 9
13. -Qa EU.ll
14. Qa A ~ Qa Adj. 14
15. ~(3x)(Px V Qx) -> (Qa A ~Qa) PC. 2,14
16. (3x)(Px V Qx) RAA. 15

5. (3y) ( Py ->(Vx) Px ) ^ (Vy) ( Py) -> (Vx)(Px)


1. (3y) ( Py ->(Vx) P x ) //.*. (Vy) (Py) -> (Vx)(Px)
2. (Vy)Py PA.
3. Pa - (Vx) Px EE.l
4. Pa EU.2
5. (Vx)Px MP. 3,4
6. (Vy) Py - (Vx) Px PC. 2,5

1. (Vy) ( Py) - (Vx) (Px) // ,\(3y) ( Py -(Vx) Px )


2. M3y)(Py -> (Vx)Px) RAA.
3. (Vy) ~(Py -> (Vx)Px) R. 14.10,2
4. -(Py - (Vx)Px) EU. 3
5. PyA ~(Vx)Px RDN.16+DM,4
6. -(Vx) Px Simp. 5
7. Py Simp. 5
8. (Vx)Px GU.7
9. (Vx)Px A ~(Vx)Px Adj. 6,8
10. ~(3y)(Py - (Vx)Px) -> [(Vx)Px A ~(Vx)Px] PC. 2,9
11. (3y)(Py - (Vx)Px) RAA.10
6.- Construir una Deduccin que pruebe que las premisas implican a la
conclusin:
1.- Todos los cuadrados son rombos.
Algunos rectngulos no son rombos.
Luego, algunos rectngulos no son cuadrados.
1. (Vx) (Cx - Rx)
2. (3x) (Tx A -R x ) // (3x) (Tx A ~Cx)
3. Ta A ~Ra EE. 2

351
4. Ca - > Ra EU. 1
5. ~Ra Simp. 3
6. ~Ca MT. (4,5)
7. Ta Simp. 3
8. Ta A ~Ca Conj. (6,7)
9. (3x) (Tx A ~Cx) GU. 8

2.- Ningn nmero imaginario es un nmero real.


Algunos nmeros com plejos son nmeros reales.
Luego, algunos nmeros com plejos no son imaginarios.
1. (Vx) (Ix ~Rx)
2. (3x) (Cx A Rx) //.-. (3x) (Cx A ~Ix)
3. Cb A Rb EE.2
4. Ib ~Rb EU. 1
5. Rb Simp. 3
6. -Ib MT. (4,5)
7. Cb Simp. 3
8. Cb A -Ib Conj. (6,7)
9. (3x) (Cx A ~Ix) GE. 8

3.- N ingn nmero imaginario es un nmero real.


Todos los nmeros racionales son nmeros reales.
Luego, ningn nmero racional es imaginario.
1. (Vx) (Ix - ~Rx)
2. (Vx) (Cx - > Rx) (Vx) (Cx - > ~Ix)
3. Iy - > ~Ry EU. 1
4. Cy> Ry EU. 2
5. Ry-> ~Iy Transp. 3
6. C y-> ~Iy SH. (4,5)
7. (Vx) (Cx - > ~Ix) GU. 6

4.- Todos los cuadrados son polgonos regulares.


Ningn trapezoide es un polgono regular.
Luego, ningn trapezoide es un cuadrado.

352
1. (Vx) (Cx Px)
2. (Vx) (T x >~Px) (Vx) (Tx ->
3. Cy - Py EU. 1
4. Ty - ~Py EU. 2
5. ~Py ~Cy Transp. 3
6. Ty - > ~Cy SH. (4,5)
7. (Vx) (T x - > ~Cx) GU. 6
5.- Hay un cisne que no es negro.
Luego, no todos los cisnes son negros.
1. (3x) (Cx A~Nx) / / ~(Vx) (Cx ->
2. ~(V x)~ (Cx A ~Nx) R. 14. 10,1
3. ~(V x) (~Cx V Nx) DM. 2
4. ~(V x) (Cx - > Nx) Def. Cond. 3
ACTIVIDAD
MAPAS CONCEPTUALES

A continuacin presentamos una sugerencia de cmo podra es


tudiar y provocar en usted un autoaprendizaje significativo, uti
lizando la tcnica de los mapas conceptuales. Esta tcnica fue crea
da por Joseph Novak quien la presenta como estrategia esquem
tica que pretende ayudar a los estudiantes a aprender represen
tando grficamente un conjunto de relaciones conceptuales.
Construya un resumen esquemtico de cada leccin estudia
da, ordenado los conceptos de manera jerarquizada. Situ cada
concepto dentro de un recuadro y relacinelo con los conceptos
de jerarqua semejante mediante lneas horizontales en forma de
flechas. Con los conceptos de menor jerarqua la relacin se esta
blece mediante flechas descendentes que nacen del recuadro co
rrespondiente al concepto al que asignamos mayor jerarquia. Es
tas flechas, con frecuencia, se bifurcan en ramas que deben
correponder a las distinciones conceptuales que consideramos
necesarias.
Veamos ahora algunos ejemplos de mapas que se han cons
truido en base a algunas lecciones que figuran en el texto. Ud.
puede construir los suyos y as crear esquemas y estructuras rele
vantes que le facilitarn la comprensin del sistema conceptual y
operativo que ofrece este libro.

354
MAPA CONCEPTUAL N. 1: DISYUNCIN

355
MAPA CONCEPTUAL N. 2: BICONDICIONAL

El Bicondicional
es un
conectivo proposicional
que relaciona
Proposiciones
a travs de
expresiones

formalizando

Condicin necesaria Condicin necesaria


y suficiente y suficiente

de de

ejemplo [ e j e mp l o
Juan viaja a Jauja si, y slo Juan toma tren si, y slo si
' si si toma tren viaja a Jauja
I____________
se expresa como
/
poq

que es la forma de
una proposicion
bicondicional

356
MAPA CONCEPTUAL N. 3: DEDUCCIN NATURAL

357
MAPA CONCEPTUAL N. 4 SILOGISMO CLSICO

Algunos S Algunos S Todos los Ninguno


Son P Son P S Son P S es P9

358
ltimas publicaciones
del Fondo Editorial de la UNMSM

Serie Hum anidades


Normalmente se cree que existe una nica manera de
Melisa Moore
En la encrucijada. Las ciencias sociales y la novela en el
pensar lgicamente que correspondera a la estructura
Per. Lecturas paralelas de Todas las sangres
Nanda Leonardini
, >
profunda de la mente de la razn o del cerebro segn el
El grabado en el Per republicano. caso. Esto conduce a suponer que la Lgica se descubre de
Diccionario histrico
la manera anloga a cmo se habra descubierto la
Gonzalo Espino Reluc (comp.)
Tradicin oral, culturas peruanas estructura de la clula o del tomo. Por tanto, la Lgica
una invitacin al debate
Martha Barriga Tello existira ya hecha en algn lugar y la tarea del profesor -ya
Influencia de la Ilustracin borbnica en el arte limeo:
siglo xvm sea a travs de las clases o de un libro- consistira en
Carlos Garca-Bedoya M. ensearnos a descubrirla. Sin embargo, el desarrollo no
Para una periodizacin de la literatura peruana
(segunda edicin corregida y aumentada) slo de la Lgica sino de lo que actualmente se conoce
S erie C o e d ic io n e s como ciencias cognitivas, nos conduce a pensar que lo

Thomas Cummins
anterior es un error: En efecto, en sentido estricto no existe ,
Brindis con el Inca. La abstraccin andina y las
imgenes coloniales de los queros
dentro de la comunidad cientfica y filosfica, la Lgica
UNMSM-UMSA (Bolivia)-Embajada de los EE.UU.
como una unidad sino un conjunto diversificado de
Manuel Burga
Nacimiento de una utopa.
Muerte y resurreccin de los incas
sistemas lgicos o, en trminos ms descriptivos de ,
UNMSM-Universidad de Guadalajara lenguajes lgicos que no siempre son equivalentes entre s.
Cristbal Aljovn de Losada J Eduardo Cavieres (eds.)
Per-Chile/Chile-Per: 1820-1920
Convenio Andrs Bello-Universidad de Valparaso
Miguel ngel Zapata (editor)
Mario Vargas Llosa and the persistence of memory
UNMSM-Hofstra University
Liliana Regalado de Hurtado
Clo y Mnemsine. Estudios sobre historia, memoria y
pasado reciente
UNMSM-Fondo Editorial PUCP
Francisco Mir Quesada
Del gora ateniense al gora electrnica.
UNI-UNMSM
Jos Enrique Briceo Berr
En busca del hombre.
Consejo Hispanoamericano de Artes y Letras-UNMSM
Carlos Caldern Fajardo
La segunda visita ae William Burroughs
Miguel ngel Zapata (editor)
Asir la forma que se va. Nuevgos asedios a Carlos
Germn Bell i
Roco Quispe-Agnoli
La Fe andina en la escritura: Resistencia e identidad de
Guamn Poma de Ayala
Biaggio D'Angeh
Borges en el centro del infinito
UNMSM-Universidad Catlica Sedes Sapientae
Csar Ferreira (editor) UNIVERSIDAD NACIONAL
Edgardo Rivera Martnez: Nuevas lecturas
MAYOR DE SAN MARCOS
S erie C l s ic o s Sa n m a r q u in o s
La universidad es lo que publica
Jos Antonio Russo Delgado
La tica en Demcrito
ltimas publicaciones
del Fondo Editorial de la UNMSM

Serie Hum anidades


Normalmente se cree que existe una nica manera de
Melisa Moore
En la encrucijada. Las ciencias sociales y la novela en el
pensar lgicamente que correspondera a la estructura
Per. Lecturas paralelas de Todas las sangres
Nanda Leonardini
, >
profunda de la mente de la razn o del cerebro segn el
El grabado en el Per republicano. caso. Esto conduce a suponer que la Lgica se descubre de
Diccionario histrico
la manera anloga a cmo se habra descubierto la
Gonzalo Espino Reluc (comp.)
Tradicin oral, culturas peruanas estructura de la clula o del tomo. Por tanto, la Lgica
una invitacin al debate
Martha Barriga Tello existira ya hecha en algn lugar y la tarea del profesor -ya
Influencia de la Ilustracin borbnica en el arte limeo:
siglo xvm sea a travs de las clases o de un libro- consistira en
Carlos Garca-Bedoya M. ensearnos a descubrirla. Sin embargo, el desarrollo no
Para una periodizacin de la literatura peruana
(segunda edicin corregida y aumentada) slo de la Lgica sino de lo que actualmente se conoce
S erie C o e d ic io n e s como ciencias cognitivas, nos conduce a pensar que lo

Thomas Cummins
anterior es un error: En efecto, en sentido estricto no existe ,
Brindis con el Inca. La abstraccin andina y las
imgenes coloniales de los queros
dentro de la comunidad cientfica y filosfica, la Lgica
UNMSM-UMSA (Bolivia)-Embajada de los EE.UU.
como una unidad sino un conjunto diversificado de
Manuel Burga
Nacimiento de una utopa.
Muerte y resurreccin de los incas
sistemas lgicos o, en trminos ms descriptivos de ,
UNMSM-Universidad de Guadalajara lenguajes lgicos que no siempre son equivalentes entre s.
Cristbal Aljovn de Losada J Eduardo Cavieres (eds.)
Per-Chile/Chile-Per: 1820-1920
Convenio Andrs Bello-Universidad de Valparaso
Miguel ngel Zapata (editor)
Mario Vargas Llosa and the persistence of memory
UNMSM-Hofstra University
Liliana Regalado de Hurtado
Clo y Mnemsine. Estudios sobre historia, memoria y
pasado reciente
UNMSM-Fondo Editorial PUCP
Francisco Mir Quesada
Del gora ateniense al gora electrnica.
UNI-UNMSM
Jos Enrique Briceo Berr
En busca del hombre.
Consejo Hispanoamericano de Artes y Letras-UNMSM
Carlos Caldern Fajardo
La segunda visita ae William Burroughs
Miguel ngel Zapata (editor)
Asir la forma que se va. Nuevgos asedios a Carlos
Germn Bell i
Roco Quispe-Agnoli
La Fe andina en la escritura: Resistencia e identidad de
Guamn Poma de Ayala
Biaggio D'Angeh
Borges en el centro del infinito
UNMSM-Universidad Catlica Sedes Sapientae
Csar Ferreira (editor) UNIVERSIDAD NACIONAL
Edgardo Rivera Martnez: Nuevas lecturas
MAYOR DE SAN MARCOS
S erie C l s ic o s Sa n m a r q u in o s
La universidad es lo que publica
Jos Antonio Russo Delgado
La tica en Demcrito

Vous aimerez peut-être aussi